Skip to main content

Full text of "Constructive text-book of practical mathematics"

See other formats


This is a digital copy of a book that was preserved for generations on library shelves before it was carefully scanned by Google as part of a project 
to make the world's books discoverable online. 

It has survived long enough for the copyright to expire and the book to enter the public domain. A public domain book is one that was never subject 
to copyright or whose legal copyright term has expired. Whether a book is in the public domain may vary country to country. Public domain books 
are our gateways to the past, representing a wealth of history, culture and knowledge that's often difficult to discover. 

Marks, notations and other marginalia present in the original volume will appear in this file - a reminder of this book's long journey from the 
publisher to a library and finally to you. 

Usage guidelines 

Google is proud to partner with libraries to digitize public domain materials and make them widely accessible. Public domain books belong to the 
public and we are merely their custodians. Nevertheless, this work is expensive, so in order to keep providing this resource, we have taken steps to 
prevent abuse by commercial parties, including placing technical restrictions on automated querying. 

We also ask that you: 

+ Make non-commercial use of the files We designed Google Book Search for use by individuals, and we request that you use these files for 
personal, non-commercial purposes. 

+ Refrain from automated querying Do not send automated queries of any sort to Google's system: If you are conducting research on machine 
translation, optical character recognition or other areas where access to a large amount of text is helpful, please contact us. We encourage the 
use of public domain materials for these purposes and may be able to help. 

+ Maintain attribution The Google "watermark" you see on each file is essential for informing people about this project and helping them find 
additional materials through Google Book Search. Please do not remove it. 

+ Keep it legal Whatever your use, remember that you are responsible for ensuring that what you are doing is legal. Do not assume that just 
because we believe a book is in the public domain for users in the United States, that the work is also in the public domain for users in other 
countries. Whether a book is still in copyright varies from country to country, and we can't offer guidance on whether any specific use of 
any specific book is allowed. Please do not assume that a book's appearance in Google Book Search means it can be used in any manner 
anywhere in the world. Copyright infringement liability can be quite severe. 

About Google Book Search 

Google's mission is to organize the world's information and to make it universally accessible and useful. Google Book Search helps readers 
discover the world's books while helping authors and publishers reach new audiences. You can search through the full text of this book on the web 



at |http : //books . google . com/ 



TECHNICAL 
ALGEBRA 

PART I. 
HORACE WILMER MARSH 







i-vi tH HI iiH I m mM u I iHU '^ "* . 7iiinm»iH 1 1 iim H I itiimui mi>= 




Dolizfid-teV^Lj O OQ IC 



Library 

/lip 



Digitized by 



Google 



Digitized by 



Google 



Digitized by 



Google 



MARSH'S CONSTRUCTIVE TEXT-BOOK 

OF 

PRACTICAL MATHEMATICS 

For Use in Industrial, Manual Training, Tech- 
nical Ilion Schools and Colleges, and 
Apprentice and Evening Classes 

THE PRACTICAL ESSENTIALS OF ARITHMBTIC, ALGEBRA, OBOH- 
ETRY, TRIOONOMBTRir, ANALYTICS, AND CALCULUS; IN- 
CLUDING THE EXTENSIVE USE OF LOGARITHMS 
AND THE aLIDE-RULB« WITH THOUSANDS 
OF EXAMPLES AND APPLIED PROBLEMS 
BASED ON INDUSTRIAL DATA 



Vul. I. 
Vol. II. 
Vol. III. 
Vol. IV. 



NOW READY 

Industrial Mathematics, with tables... ne< 
Technical Algebra, with tables, Part I. ,ndt 

Technical Geometry net 

Technical Trigonometry net 

Interpolated Siz-place Tables nd 

Mathematical Work-book for stu- 
dents' use, removable blank sheets, 
^ with instructions for use net 



$2.00 
$2.00 
$1.25 
$1.50 
$1.25 



.65 



Each volume complete in itself. The entire course a unity. As 
constructive, developing, and creative as shop work. Eklucates 
tbroui!h self-activity, affords self-realization through self-expression, 
and gives the knowledge and the use of the Mathematics of modem 
Industries. 



Digitized by 



Google 



CONSTRUCTIVE TEXT-BOOK 

OF 

PRACTICAL MATHEMATICS 



BY 

HORACE WILMER MARSH 

Head of Department of Mathematiea, School of Science 
and TecJinology, Pratt Institute 



Volume II 

TECHNICAL ALGEBRA 

Part I 



FIRST EDITION 



NEW YORK 

JOHN WILEY & SONS, Ino. 

London: CHAPMAN & HALL, LiMrria) 



Digitized by 



Google 



Copyright, 1913, 

BY 

HORACE WILMER MARSH 



PRESS OF 

BRAUNWORTH h. CO. 

BOOKBINDERS AND PRINTERS 

BROOKLYN, N. V. 



Digitized by 



Google 



PREFACE 



Professor John Dewey long ago emphasized the idea 
that the school is truly educative only when it represents 
actual, social conditions. In the trade and technical school 
of secondary rank, such as the School of Science and Tech- 
nology of Pratt Institute, we have been forced to recognize 
and prove the truth of Professor Dewey's assertion. The 
graduates of a school of this kind, although requiring an 
acquaintance with books, must be able to do and to direct 
others. Their mathematics must be of the sort which they 
can use and apply in their daily work after graduation. 
It must be presented in such form and manner during the 
entire course that its use and application shall be an 
acquired art the same as the use of any other tool. 

This text in form and method is the result of an attempt 
to solve the problem of the teaching of mathematics, not 
from the point of view, therefore, of the mathematician 
but from the necessities of the student and the demands 
of the environment in which he is and into which he is to go. 
The mathematician sees mathematics only; the student sees 
the unknown and is estranged; the true teacher sees active 
boys and girls or young men and women whose study must 
be so directed that they shall become genuinely interested 
in mathematics and shall have a feeling of confidence and 
satisfaction in its use. 

This direction is possible only when mathematics is 
regarded and taught, not primarily as a means to mental 



n33i5r; 

Digitized by VjOOQIC 



vi PREFACE 

discipline, but as an instrument and a tool whose use can be 
acquired only by continued practice on concrete pieces of 
work. The student will so regard mathematics the moment 
he feels its intimate relationship to his other studies and to 
life. The things profoundly educative in the Uves of all 
of us are the worthy things we love to do. What teacher 
but would have boys and girls study mathematics for the 
same reason that they play— because they love to? What 
teacher but would have them experience something of the 
same pleasure in preparing a lesson in algebra as in a whole- 
some, lively game? 

The mathematician has this pleasure as he follows the 
unblazed trail which his genius illumines. This trail, still 
seen dimly, I have endeavored to blaze in such manner in 
this and in the other constructive and developing texts in 
this course, through a study of the educational masters and 
thirteen years' experiment and test of the mimeographed 
text with upward of 2000 students, that the students who 
use,^this book shall become genuinely enthusiastic and pro- / 
ficient, for " He who sees without loving is only straining 
his eyes in the dark." 

The conventional, x-y-z mathematics — artificial in 
subject-matter, notation, and method, the embodiment 
and the outcome of the scholastic demand for mental dis- 
cipline and of the educational ideal that the more difficult 
the subject-matter the greater the discipline — has rarely 
produced this result for the reason that it has few apparent 
points of contact with the activities of the shop and labora- 
tory or of industrial, scientific, engineering, and professional 
pursuits. 

To the beginner no study is significant whose use is not 
apparent and which is not presented in a way to stimulate 
creative effort. Algebra in its conventional form is a 
striking illustration of this truth because it is the first branch 
of mathematics to deal with generalizations. These can 
have meaning only as they are the expression of individually 



Digitized by 



Google 



PREFACE vii 

investigated particulars which are intimately associated 
with the student's interests. Students are interested tem- 
peramentally in things which appeal to their activities; 
hence their interest in machinery and in physics, chemistry, 
or domestic science laboratories. Their interest in mathe- 
matics is an unknown quantity, and he who likes it, is the 
exception to whom a teacher and a special method are not 
a necessity. 

A serious disadvantage, therefore, confronts the teacher 
of algebra. His success is conditioned by the interest of 
his students, yet natural interest on their part is wanting. 
In this emergency I know of but one interest to which the 
teacher can appeal. This is the universal, human interest 
in doing and in the creation of something worth while. 
This text is therefore written in such form that from the 
first, each student shall be busy on his own individual problem 
with increasing interest and pleasure, because the con- 
structive form of the text, the practical character of the 
subject-matter, and its correlation with his other studies 
give him an instinctive feeling that the problem, the form 
in which the work is required, and the result to be found, 
are worth the effort. 

Even an unskilful and careless workman takes pride 
in the results of his attempts to do a really good piece 
of work. An indifferent student likewise becomes interested 
and appreciative, the instant an enforced excellence of 
form results in a piece of work which he is proud to own 
and to have his friends see. If this be true, as I am 
compelled to believe by repeated experiment, the advantage 
and the necessity of a standard form for each student's 
work is evident. The mathematics' work-book is an endeavor 
to meet this requirement. This book consists of a note- 
book cover with removable sheets, in which every student 
writes in ink the work required in the text. It is desirable 
that most of this work be done in the classroom and a pro- 
vision for it by double periods for mathematics, is ideal. 



Digitized by 



Google 



Tiu PREFACE 

The educational purpose of the mathematics work-book 
is two-fold. It demands a daily, finished product and the 
written expression of the student's individual thought on 
the topics assigned. Its incidental results are steady 
improvement in spelling, penmanship, and lettering, the 
use of correct English, a growing sense of method and order, 
and an increasing ability to plan and work in a rational, 
economic, and therefore businesslike manner. 

One of the minor details of the method employed in this 
text, which is of great value in explanation, is the require- 
ment that in all work with equations every equation shall 
be numbered. If not nmnbered or otherwise named, no 
equation can be indicated in explanation at the blackboard, 
except by reading. This not only lessens interest and wastes 
time but imavoidably emphasizes symbols, instead of 
operations and reasons and authorities for operations. 
What the weak student must hear repeatedly and invariably, 
is what kind of equation is given, by what operation each 
subsequent equation is obtained, and by what mathematical 
authorities. He can hear these, only when he hears nothing 
else. 

Students in industrial courses, many of whom have been 
employed in technical industries, while fairly mature in 
their thought, have usually a varied and sometimes inade- 
quate school preparation. Others have taken advanced 
courses in mathematics. In this text provision has been 
made by examples and problems ranging from the simplest 
to those of considerable difficulty, so that every student 
whether beginner or advanced, with whatever accomplish- 
ment or experience, shall have the work he is fitted to do, 
and which shall be of most service both in his major, tech- 
nical studies and subsequent employment. 

In this connection attention is called to the absence of 
the conventional, artificial problem whose very statement 
necessitates a knowledge of the answer, whose data and 
solution have absolutely nothing to do with science or 



Digitized by 



Google 



PREFACE ix 

mental development and whose principles have no applica- 
tion to any industry in the universe. 

Each chapter of the book contains specific instructions 
for the order of procedure in all operations. Notable 
instances are the chapters on complex fractions, factoring, 
logarithms, and the slide-rule, for which there have com- 
monly been no definite methods. 

One of the most valuable chapters to the student is the 
chapter on Transformation of Formulas, names imknown at 
the time of preparation of this manuscripts Generations of 
university graduates have had occasion to remark in their 
first position that their first task was to forget all they 
knew. The reason in so far as it concerns algebra, is 
obvious. They studied books of one language, the conven- 
tional. The mathematics of industry is written in an 
apparently different language — the significant. The student 
who knows this chapter needs not to learn a new mathe- 
matical tongue in his employment or in his technical reading, 
for he will find it " parent speech." In the conventional 
treatment of algebra everything is labeled so that one 
must never ask what is this, before attempting to solve. 
In the chapter on transformation the formulas are arranged 
so that the kind of equation must invariably be deter- 
mined before operation is possible. By this means the 
student gains the same power of analysis and classification 
that is required in technical positions. 

The author is confident that in other schools, as in his 
own classes, this text will prove an integral part of a 
student's industrial and technical courses, and an adequate 
and unfaiUng interpreter of his technical studies. Since 
one cannot write imtil he knows the alphabet, this text 
will render greatest service if the principles of operation 
required in any part of an applied study are presented 
previous to their employment in the technical work. 

Some of the most helpful suggestions for this text were 
obtained from the shops, laboratories, and drafting rooms 



Digitized by 



Google 



X PREFACE 

t 

of Pratt Institute. The men directing these kinds of work 
do not " hear lessons "; their rooms are not " recitation " 
rooms in which " classes " assemble to repeat what they have 
learned from a book. The teachers of these useful crafts 
do not assign for memorizing — definitions, directions, and 
descriptions. The student who has once made a hexagonal 
nut, or a milling cutter, or a propeller blade pattern, or 
has worked out a hysteresis curve, knows them, not from 
memorized definition or description which are vaguely or 
entirely misunderstood, but from intimate acquaintance 
and individual, creative work. He can define, make, and 
direct the making of these things, not because he has memo- 
rized another's thought, but because he has made them, 
himself. 

In the industrial or technical school it is inevitable that 
the mathematics be no less constructive and creative than 
shop-work and I am certain that in no school will the study 
of mathematics be truly educative unless its conventional 
study is deferred until its need is felt, and unless fundamental 
propositions and principles are developed in a form to make 
every student experience the joy of individual discovery 
and creation. 

This book aims to make the student proficient in the 
fundamental, algebraic processes and in their application 
to the numerous computations in technical industries. 
That a student may have the best means of securing an 
accurate result, the chapters on Logarithms and the Slide- 
rule are the most complete yet written concerning every 
use of these wonderful instruments with which no student 
in this busy, industrial, twentieth century can afford to be 
unfamiliar. 

The ideals which have been the final authority in the 
preparation of the Constructive Text-Book of which this 
is the second volume, are perhaps best shown by contrast 
with the old, as outlined *from a lecture by Professor F. M. 
McMurry: 



Digitized by 



Google 



PREFACE 



XI 



The Old Ideal: 
Knowledge, 
Stillness, 
Machine, 
Group, 
Theory, 

External Authority, 
Restraint, 
Self-consciousness, 
Scholarship, 

Motive: 

Competition, marks, prizes. 

Stimulus; 
Compulsion, 

Comparison with others. 



The individual fact. 



The Modern Ideal: 
Development. 
Motor Activity. 
Germ. 
Individual. 
Executive Ability. 
Inner control. 
Freedom. 

Unconscious effort. 
Enlistment of interest. 



Love for the subject. 



The created thing in its 
gradual creation. 

Comparison of the student's 
work with his own previous 
work. 

Unified knowledge, social ser- 
vice and eflficiency. 



In furtherance of these ideals which now dominate educa- 
tion, the world over, mathematics work should be planned 
with the same care and in much the same maimer as a man 
must plan a piece of work which he is to do, or whose con- 
struction he is to superintend in shop, factory, or other 
enterprise. When so studied, mathematics is of the greatest 
service in the development of those habits of thought which 
qualify one for positions of responsibility. Thus consid- 
ered, it is not a question whether one will use this or 
that branch of mathematics in a certain position, but 
whether one knows sufficient mathematics so that promo- 
tion shall not be impossible because of the lack of it, and 
whether it has been studied in such a way as to make one 
increasingly valuable in any position. 

Thoreau revealed a truth of tremendous import in educa- 
tion, when he wrote: 



Digitized by 



Google 



xii PREFACE 

" The true poem is not what the public reads, 
but is what the poet has become through his work." 

So in the study of mathematics, the actual, supreme 
result is not mathematical attainment, ability, or skill; it 
is not the student's created product as represented in the 
work-book, but is the student himself , as represented in what 
he has become through the self-directed, systematic exer- 
cise of neatness, accuracy, patience, seriousness, judgment, 
native conmion sense, sincerity, and love cf work, which 
are incidental and, I trust, inevitable in the method and 
content of this text. 

In conclusion I gratefully acknowledge my indebtedness 
to my wife, Annie Griswold Fordycc Marsh, my constant 
critic, co-worker, and counsellor without whose aid this 
volume would not have been possible. 

Horace Wilmer Marsh. 

Brooklyn, New York, 
August 19, 1913. 



Digitized by 



Google 



CONTENTS 



PAGE 

The Work-book 3 

CHAPTER I 

Introduction 

Section 1. Algebraic Notation. Exercise 1. Lettering. Exer- 
cise 2. Ratio. Exercise 3. Compound Ratio. Exercise 4. 
The Equation. Exercise 5. Coefficient and Exponent. 
Section 2. Solution of a Simple Equation. Section 3. For- 
mulation of Mathematical Laws. Section 4. Formulation 
and Computation 11 

CHAPTER II 

Resolution and Composition op Forces 

Section 1. Projection. Section 2. Graphical Resolution. Sec- 
tion 3. Resolution by Computation. Section 4. Functions 
of an Obtuse Angle, and Examples. Section 5. Resultant 60 

CHAPTER III 

Variation •» 77 

CHAPTER IV 

The Four Fundamental Operations 

Section 1. Addition. Section 2. Subtraction. Section 3. Mul- 
tiplication. Section 4. Division 101 



Digitized by 



Google 



xiv CONTENTS 

CHAPTER V 

Laws of Numbers 

PAoa 

Section 1. The Square of the Sum of Two Numbers. Section 2. 

The Square of the Difference of Two Numbers. Section 3. 

The Product of the Sum and the Difference of Two Numbers. 

Section 4. The Difference of Two Cubes. Section 6. The 

Sum of Two Cubes. Section 6. The Square of any Polynomial 

Section 7. The Exact Divisor of a Polynomial 120 

CHAPTER VI 

Factoring 

Section 1. A Common Factor. Section 2. Grouping. Section 3. 
The Difference of Two Squares. Section 4. The Difference of 
Two Cubes. Section 6. The Sum of Two Cubes. Section 
6. The Trinomial. Section 7. The Polynomial. Section 8. 
Various Expressions 128 

CHAPTER VII 

Fractions 

Section 1. Reduction. Section 2. Addition and Subtraction. 
Section 3. Multiplication and Division. Section 4. The Com- 
plex Fraction 144 

CHAPTER VIII 

The Quadratic Equation 

Section 1. Introduction. Section 2. Solution by Factoring 
Section 3. Solution by Completing the Square. Section 4. 
Equations in Quadratic Form 155 

CHAPTER IX 

The Fractional Simple Equation 

Section 1. Denominators Numerical. Section 2. Some De- 
nominators Literal 166 



Digitized by 



Google 



CONTENTS X? 

CHAPTER X 

SlBHTLTANEOUS SiMPLE EQUATIONS 

PAQB 

Section 1. Two Unknowns. Section 2. Three or More Un- 
knowns 173 

CHAPTER XI 

Exponents 

Section 1. A Zero Exponent. Section 2. A Negative Exponent. 

Section 3. A Fractional Exponent 180 

CHAPTER XII 
The Binomial Theorem 191 

CHAPTER XIII 

Powers and Rooxs 

Section 1. Powers. Section 2. Square Root. Section 3. Cube 

Root. Section 4. Other Roots 200 

CHAPTER XIV 

Radicals f 

Section 1. Reduction. Section 2. Addition and Subtraction. 
Section 3. Multiplication and Division. Section 4. Ra- 
tionalization. Section 5. Powers and Roots. Section 6. Rad- 
ical Equations 207 

CHAPTER XV 

Logarithms 

Section 1. Logarithm of a Number Greater than Unity. Section 
2. Logarithm of a Number Less than Unity. Section 3. 
Naperian or Hjrperbolic Logarithms. Section 4. Logarithm 



Digitized by 



Google 



xvi CONTENTS 

PAGS 

of a Product. Section 5. Logarithm of a Quotient. Section 
6. Logarithm of a Power. Section 7. Logarithm of a Root. 
Section 8. Solution of an Exponential Equation. Section 
9. Model Solutions. Section 10. Logarithmic Computa- 
tion 221 



CHAPTER XVI 
The Slide Rule 

Section 1. Introduction. Section 2. Sines. Section 3. Tangents. * 
Section 4. Multiplication. Section 5. Division. Section 6. 
Proportion. Section 7. Logarithms. Section 8. Powers 
and Roots. Section 9. Gage-Points. Section 10. The Log 
Log Rule 279 

CHAPTER XVII 
Tbanspormation op Formulas 350 



Digitized by 



Google 



REFERENCE TABLES 



PAGB 

I. Measures of Length 391 

II. Measures of Area 392 

III. Measures of Volume 393 

IV. Measures of Weight 394 

V. Decimal Equivalents of fractions of an Inch 395 

VI. U. S. and Metric Equivalents 396 

VII. International Atomic Weights, 1913 397 

VIII. Period Arrangement of the Elements, 1913 398 

IX. Specific Gravities and Weights of Materials of Construction. 399 

X. Wire Gage Sizes 401 

XI. Four-place Logarithms of Numbers 402 

XII. Four-place Trigonometric Functions 405 

XIII. Three-place Trigonometric Functions 412 

xvii 



Digitized by 



Google 



Digitized by 



Google 



TECHNICAL ALGEBRA 



Digitized by 



Google 



THE WORK-BOOK 

Note. The pages immediately following contain the detailed 
instructions which govern the use of the mathematics' work-book 
in the author's classes where one hour and a quarter of outside 
preparation per class period is the maximum requirement. Even 
in schools where written work does not seem feasible these pages 
may perhaps suggest ways of making mathematics of the same 
significance as shop and laboratory work in the student's prepara- 
tion for life, and at the same time pleasurable and useful both to 
student and teacher. 




A student's work-book containing the first year's work. 



Digitized by 



Google 



TECHNICAL ALGEBRA 



THE WORK-BOOK 

1. Description. The constructive form of this text 
requires that each student shall prepare the work daily 
in accordance with the suggestions, questions, and direc- 
tions in numerous developing exercises, which force a student 
not only to do his own thinking but to express his thought 
in written form. 

For this purpose the mathematics' work-book is used, 
which both in size and form is the result of sixteen years 
of experiment with over 2000 students. It consists of a 
note-book cover, the daily record sheet described in a 
subsequent paragraph, and 250 removable sheets of 
16-pound, unruled, linen paper measuring 5^X8| inches, 
with fasteners for attaching the sheets to the back cover 
a,nd the student's written work to the front cover. 

2. Instruments. In order to prepare the work in a 
satisfactory manner and to secure the greatest educational 
benefit each student will require the following equipment: 

12-inch triangular scale with U. S. and metric gradu- 
ations, 
medium lead pencil, 
ink and pencil erasers, 
fountain pen, 
ruling pen, 
compasses, 

3 



Digitized by 



Google 



4 TECHNICAL ALGEBRA 8 

protractor, 

red, black, and India ink, 

slide-rule. 

3. Value of Careful Work. It is obvious that skill is 
never acquired by careless, indifferent effort. Therefore, 
as in manual training, the desideratum in every exercise, 
example, and problem in this text, is perfection in the 
finished work. 

The instructions in the two subsequent paragraphs and 
throughout the text have accordingly been written to 
stimulate each student to a serviceable ideal of excellence 
and efficiency and to give him an increasing ability and 
enthusiasm for its realization. 

4. Instructions for Work-book Entries. (1) What to 
DO First. On the inside of the front cover of the work- 
book write your full name, home and rooming address, 
and name of school and course. 

Attach all record sheets except one, to the back cover 
under the blank sheets. 

(2) Use of Ink. With the exception of the drawing 
and the first exercise in lettering, all work is to be done 
directly with pen and ink whether in the classroom or 
outside. 

(3) Date, and Page Number. Enter date on whict 
work is prepared, in the upper right corner of the page 
about one and one-half inches from the top. 

Number each page in the lower right corner when 
finished. 

(4) Headings. Enter all work under the same heading 
as in the text or as otherwise specified. 

Begin paragraph numbers and headings about one-half 
inch from the left margin. 

(5) Lettering. The greater legibility of lettered head- 
ings and the practical value of the ability to letter neatly 
and rapidly, justify the requirement that title pages, 



Digitized by 



Google 



4 THE WORK-BOOK 5 

chapter, section, and paragraph headings, and problem 
titles shall be lettered. The best style of letter for this 
purpose, because the simplest and the most easily made, is 
the Engineering News alphabet shown below and used 
quite generally in drafting rooms throughout the country. 
The only principle involved in learning to use this 
alphabet is that each letter consists of straight lines, or 
arcs of circles, or both, and that the width and the height 
are the same. 



ABCDEFGHIJKLMNOPQRSTUVWXYZ ft 
1234567890 li 2131411 5i6& 7.039 

ABCDEFGHIJKLMNOPQRSTUVWXYZ 1234667690 

- — iM — s — HK^Vilf-^ 

Be sure to observe that W is not an inverted il/, nor M 
an inverted W. Observe also particularly how R and 
G are made. 

(6) Title Pages. Letter title pages in India ink 
without punctuation, for the subject and for each chapter. 
Insert these in the work-book preceded by a blank sheet. 

(7) Spacing. Indicate a new topic both by heading 
and by extra space. Keep all work in straight lines with 
no irregular spacing between words or lines. 

If straight lines are difficult without a guide, rule a 
page of the work-book in India ink with lines from three- 
sixteenths to one-fourth of an inch apart, and place it 
under the page when writing. 

Examples and problems are best separated by extra 
space only, but if preferred the separation may be em- 
phasized by a hair-line not over two inches long, lightly 
drawn with straight-edge and ruling pea. 



Digitized by 



Google 



6 TECHNICAL ALGEBRA 5 

(8) Drawing. Drawings which are to be measured 
should be drawn lightly and carefully with pencil and straight- 
edge, and after measurement should be inked in with India 
ink and a ruling pen. Other drawings may be done directly 
in India ink. 

As soon as possible learn to draw a lights smooth, drafts- 
man's line. 

(9) Symbols. Make parentheses, equality signs, and 
other sjnnbols carefully: parentheses with regular curves 
of the same height as the quantities inclosed; the lines 
of the equality sign exactly the same length and about 
one-eighth of an inch long. 

Learn to make comparatively small, neat figures and 
to draw free-hand, smooth, light, straight lines when 
performing the four fundamental operations. 

Work slowly, seriously, and steadily, and thereby 
become expert by avoiding careless mistakes. 

6. Instructions for the Record Sheet. Six daily record 
sheets, a half year's supply, are furnished with the work- 
book. These have columns for the instructor's stamp 
and for the daily entry by the student, of date, paragraph 
and problem numbers, and number of hours spent in 
outside preparation of studies. 

Submit Work for Inspection as Follows: On the 
first day prepare a record sheet by filling in the blanks as 
indicated. At the top of the time columns letter the 
names of the studies in which outside preparation is 
required, as Math., Phys., C. L. (chemistry laboratory), etc. 

Observe that the record sheet provides for a complete 
record of the mathematics work and is a time sheet for 
all studies. 

Whenever work is to be submitted attach it to the 
record sheet on which fill in the entries denoted by the 
column headings, making no entries in the remarks column. 

When an additional record sheet is needed place it on 
top of those already filled. 



Digitized by 



Google 



9 THE WORK-BOOK 7 

6. Excuse for Non-performance and Absence. If 

unable to do assigned work, present to the instructor at 
the beginning of the period a written excuse with date, 
assignment, exact reason for failure, and signature. Enter 
the date on the record sheet and write the word " Excuse " 
in the remarks column. 

In case of absence enter the date of each day's absence 
and write " Absent '' in the remarks column. 

7. Collection and Distribution of Work-books, (a) 
Collection. On the stroke of the bell at the beginning 
of the period each student will pass his work-book along 
the row in reverse order from which the chairs or desks 
are numbered, being sure to place it on top of the books 
passed to him. 

The student receiving the books of the last row will 
collect each row's books and will place them in the file. 

Work-books may be taken from the classroom only when 
permission is noted on the record sheet by the instructor. 

(6) Distribution. At the beginning of the mathe- 
matics period the collector will place the books at the 
end of the rows so that each student may remove his book 
from the pile as it is passed. 

Books of absentees will be reported ^ directly to the 
instructor's desk by the collector. In the collector's absence 
the next student in the row will attend to the books. 

8. Inspection. The remarks column on the record 
sheet is for the instructor's stamp. When the dater is 
used instead of the " accepted " stamp it signifies that 
the work is incomplete, or unsatisfactory, or incorrect. 
Changes in such work unless obvious or indicated in the 
book, must be arranged with the instructor before the 
close of the period. 

9. Corrected Work. Incorrect or rejected work is due in 
correct form at the beginning of the next mathematics period. 

Make corrections in red ink on the same page with the 
incorrect work. If numerous mistakes have been made 



Digitized by 



Google 



8 TECHNICAL ALGEBRA 10 

prepare a new page and insert it following the incorrect 
one on which write in red ink " Corrected on next page/' 
with date of correction. 

10. Solution of Equations. Number equations at the 
left in a vertical column with Arabic numerals inclosed 
in a parenthesis. 

Separate equations from specified operations by a 
horizontal rippled line not shorter than half an inch. It 
must follow each equation except those taken from the 
text or formed from the conditions specified in the problem. 

The following model solution shows how to number, 
how to use the rippled line, and how to abbreviate. 

LCDS. 

(1) 5x-24x=2— Mul (a) by 8 

(2) -19x = 2 Col in (1) 

(3) a:=-^— Div(2)by-19 

As illustrated, equations from the text or from the 
conditions of the problem are numbered with the first 
letters of the alphabet. 

Equations resulting from operation are numbered in 
succession in Arabic numerals. 

11. Indication of Results. Indicate final results in all 
problems by double imderlining with parallel hair-lines 
not over one-sixteenth of an inch apart. Do not write the 
word " Answer.'* 

On the same line with the result write a statement in 
initial capitals specifying exactly what the result repre- 
sents, whether niunber of revolutions per minute, horse- 
power, speed, etc. 

12. Index. At the end of the school year arrange all 
the work-books of the year in order, make an alphabetic 
index for your combined book, cut the work-book cover 



Digitized by 



Google 



12 THE WORK-BOOK 9 

in two through the back, and bind all together with one 
pair of fasteners. 




This shows a stuaent's work-book at the end of the second year. 

On the front cover attach a label about 4"X5" with 
a line border and lettered title enumerating the subjects 
covered as suggested in the facsimile label below: 



MATHEMATICS WORK-BOOK 

FIRST YEAR 

ALGEBRA 

GEOMETRY 

TRIGONOMETRY 

WRITTEN BY 




PRATT INSTITUTE 

1911-1912 S.M.D. 



Digitized by 



Google 



10 TECHNICAL ALGEBRA 13 

13. The Axioms. An axiom is a self-evident truth 
which does not depend on a simpler truth by means of 
which it can be proved. 

The axioms below are the basis of algebra and are 
the sole authority for the operations necessary to the solu- 
tion of an equation. 

(1) Equality Axiom. ( = ity Ax.) Numbers equal to 
the same number or to equal numbers, are equal to each 
other. 

(2) Addition Axiom. (Add Ax.) If equations are 
added together, or if the same number is added to both 
members of an equation, the result is an equation. 

(3) Subtraction Axiom. (Sub Ax.) If one equation 
is subtracted from another or if the same number is sub- 
tracted from both members of an equation, the result is an 
equation. 

(4) Multiplication Axiom. (Mul Ax.) If equations 
are multiplied together, or if both members of an equation 
are multipUed by the same number, the result is an equation. 

(5) Division Axiom. (Div Ax.) If one equation is 
divided by another or if both members of an equation are 
divided by the same number, the result is an equation. 

(6) Power Axiom. (Power Ax.) If both members of 
an equation are raised to the same power, the result is an 
equation. 

(7) Root Axiom. (Root Ax.) If the same root of both 
members of an equation is indicated or extracted, the result 
is an equation. 

(8) Sum of Parts Axiom. (Sum Pts Ax.) The whole 
of any quantity equals the sum of its parts and is therefore 
greater than any of its parts when all the parts are positive. 

Make no effort to memorize these axioms as they will 
become familiar through repeated appUcation. 



Digitized by 



Google 



CHAPTER I 

INTRODUCTION 

Section 1, Algebraic Notation. Exercise 1, Lettering. 
Exercise 2, Ratio. Exercise 3, Compound Ratio. Exer- 
cise 4, The Equation. Exercise 5, Coefficient and 
Exponent. Section 2, Solution of a Simple Equation. 
Section 3, Formulation of Mathematical Laws. Sec- 
tion 4, Formulation and Computation. 

§ 1. ALGEBRAIC NOTATION 
Exercise 1. Lettering. 

14. Lettering of LabeL* With pencil and straight- 
edge rule ofif three spaces on the first page of the work- 
book, the size of the gummed label to be attached to the 
front cover, about IJ by 3^ inches. 

Carefully letter with pencil each of the spaces in the 
style of letter shown on page 5, in the following order: 



Term 


School Year 




Subject 




Student's name 


Volume 


Class Year of course and section 



* See 4, page 4. 

11 



Digitized by 



Google 



12 TECHNICAL ALGEBRA 16 

16. Lettering of Title Page. When the lettering has 
been approved, lightly letter the label itself in pencil; 
also letter the title page Technical Algebra. 

Before the next class period in mathematics letter the 
title page a second time with greater care than before 
and attach the label to the front cover of the book. Also 
write your name, home and rooming address on the 
inside of the front cover. 

Exercise 2. Ratio. 

16.. Measurement. Carefully draw a straight line A 
about two-thirds the width of the page. With the scale 
measure it to the nearest sixty-fourth of an inch and to 
the nearest millimeter. 

Express the result as follows: 

A = inches. 
A= mm. 

How was the length of A determined? * 
A equals how many sixty-fourths of an inch? 
When a line is measured in sixty-fourths of an inch, 
what is the unit of measure? 

A contains the unit of measure how many times? 

When a line is measured in millimeters, what is the 
miit of measure? 

A contains this unit how many times? 

Ratio is the quotient relation of one quantity to another, 
expressed as a fraction. 

Denoting the unit of measure by S, express the ratio 
of Aio B. 

Express the ratio of H to S; of £ to jB. 

The ratio of two quantities of the same kind always 
equals a fraction whose numerator is the iium15er~of times 

* In answering all questions in the text include the question in the 
answer. Thus: The length of A was determined, etc. 



Digitized by 



Google 



le 



INTRODUCTION 



13 



the first quantity contains the unit of measure, and whose 

denominator is the mmiber of times the second contains 

the unit of measure. 

A 
To what fraction is — equal? (Express with equality 
B 

sign.) 

Draw a line x about the same length as A, but longer. 

Measure it with the scale and write what it equals to 
the nearest sixty-fourth of an inch and to the nearest 
millimeter. 

Draw a second line y considerably shorter than A. 

Write what y equals to the nearest sixty-fourth of an 
inch and millimeter. 

Express the ratio oi x to y and indicate what it equals 

X 

in inches; also express what - equals in millimeters. 

y 




Fig. 1. 

Carefully draw this figure in the work-book making 
h and h' perpendicular to OD, 

Determine and enter with an equality sign the follow- 
ing measurements and ratios, both in U. S. and metric 
units: 

A, OD, W, 0D\ OV, OT, 

Jl JL Jl JdL 9L 9IL 

OD' OD" or OV" OD' OD'' 



Digitized by 



Google 



14 TECHNICAL ALGEBEA 17 



Exercise 3. Compound Ratio. 

17. Definition. A compound ratio is a product resulting 
from the multiplication of two or more ratios. 

A product is expressed in algebra by writing one quantity 
following another; as, 5q/, lahx. Observe that the numer- 
ical factor always precedes the literal, and that literal 
factors are arranged in alphabetic order. 

By the law of multipUcation of fractions, multiply 

gby-; by^; by-^; by-j^; byy; by 3^; byj^. 

Each of the results you have written is what kind of a 
ratio? Why? 

18. Factors. Factoring is the process of finding the 
numbers whose product equals a given number. These 
mmibers are called factors. 

Thus, 

18 = 2 times 9, or 3 times 6, or 3 times 2 times 3. 
Factor the compound ratios, 

llA 34ax ,m 
12B' 50y ' .09r 

Exercise 4. The Equation. 

19. Definition. An equation is an expression of equality 
between quantities. 

Denoting by z the unit one sixty-fourth by which the 

nr 

line X was measured, to what fraction is -- equal? 

oZ 

y 

To what fraction is — equal? 
z 

x = how many times 2? 

2/ = how many times 2? 



Digitized by 



Google 



21 INTEODUCnON 15 

Why are your answers to the last four questions equa- 
tions? 

Number the equations in succession at the left, inclosing 
the numbers in a small circle or parenthesis. (See para- 
graph 10.) 

20. Members. The members of an equation are the parts 
separated by the equality sign. The first member is the 
part to the left of the equality sign; the second member 
is the part to the right of the equality sign. 

An addition to an equation must always he made to both 
memberSj otherwise the equality is destroyed. 

An addition is indicated by a plies sign; a subtradionhy 
a minus sign. 

Add 5 to the first equation. 
To how many members did you add it? 
Add z to the second equation. 
Subtract 13 from the third equation. 
Both add 8 and subtract y from the fourth equation. 
Number each of the resulting equations in succession 
with the first four of paragraph 19. 

21. Terms. The terms of an equation are the parts 
separated by the signs, plus, minus, or equality, but no 
other signs. 

How many terms has the eighth equation? 
Give reason for your answer. 

22. Multiplication of an Equation. An equation is 
multiplied by multiplying all of its terms. A numerical 
multiplier is multiplied into the numerical factors of the 
term; if a term has no numerical factor, the numerical 
multiplier is written in front of the literal factors with 
no sign between. 

For example, 5 times 4fe = 206, the numerical multiplier 
5 being multiplied into the numerical factor 4. 



Digitized by 



Google 



16 TECHNICAL ALGEBRA 23 

5 times a6 = 5afe, the numerical multiplier 5 being written 
in front of the Uteral factors ab, 

A literal multiplier is written in alphabetic order in the 
term into which it is multiplied. 

For example, d times 5acx = 5acdXy the literal multiplier 
d being written in alphabetic order in the term 5acx. 

Multiply the fifth equation by 7. 

Multiply the sixth equation by z, the seventh by 3fe, 
and the eighth by 5a. 

Number the resulting four equations successively with 
those previously written. 

By what mathematical law do you know that when 
an equation is multiplied, the result is an equation? (See 
page 10.) 

Exercise 6. Coefficient and Exponent. 

23. Coefficient. A coefficient is a multiplier or a factor. 

For example, in SaJ^c^y, 8 is the numerical coefficient 
and is called the coefficient of the term. 
8a2 is the coefficient of c^y. Why? 
What is the coefficient of a^y? Of a^? 

When no numerical coefficient is written, 1 is under- 
stood. 

In the third equation of paragraph 22, what is the 
coefficient of the first term? 
Of the second term? 

24. Exponent. An exponent is a number, which, when 
a positive integer, shows how many times a number affected 
by it is used as a factor. 

Exponents are written at the right of and slightly above 
the numbers which they affect. When no exponent is 
indicated exponent 1 is understood. 



Digitized by 



Google 



25 INTRODUCTION 17 

In a^, a is used how many times as a factor? 
3 is what? What does it show? 
In 5a^€?y, 5 is what? 2 is what? 
In the same term, what is the coefficient of a^yt 
What is the exponent of y1 
a * times a times a = what? 
Express a^ times a'* without exponents. 
Does a^ times a^ = a^ or a®? Why? 
Therefore when the same letters are multiplied together 
should their exponents be multiplied, or added? 
3x2 times y^ times x times y^ times 2/ = what? 
5fe times fe times b^ times 4c6~^ = what? 
r times r times r^ times 7r^ times 8r~^^=what? 
7'-Himes y+Swhat? 

26. Marks of Parenthesis. When more than one term 
of an expression or when more than one of several factors 
is affected by the same operation or exponent, the terms 
and factors affected are inclosed by marks of parenthesis: 
These are: 

( ) the parenthesis, 
{ } the brace, 
[ ] the bracket, 
the vinculum. 

Q 

— (5— 3i) indicates that the difference between 5 and 

8 
3j is to be multiplied by jq. This may be done in two 

ways: Each term within the parenthesis may be mul- 

g 
tiplied by 75 ^^^ ^^® second product then subtracted from 

the first, or the terms may first be subtracted and the 

Q 

remainder multiplied by — . 

* In your answer use the multiplication symbol. 



Digitized by 



Google 



18 TECHNICAL ALGEBRA ^6 

In the work-book perform the operation in both ways. 
Which way is preferable? Why? 

Copy the following and perform the indicated opera- 
tions: 



1, 



ll(^t-|). 



2. ^(5J+2f)+i 

4. 3a;(8+2x). 

§ 2. SOLUTION OF A SIMPLE EQUATION 

26. Definitions. A simple equation is an equation 
having only the first power of the unknown quantity. 

An unknown quantity is a quantity whose value maj 
be determined by solution. 

Unknown quantities are denoted conventionally by the 
last letters of the alphabet. 

In the application of mathematics any letter may 
denote an unknown quantity, usually the initial letter of 
the word by which the quantity is named, as V for velocity, 
A for area, C for circumference, etc. 

Copy the following equations in the work-book: 

1. x+2x-15=0. 2. 7x-4x-6 = 12. 

3. 6T^+x-S = i. 4. 5x+8-x=48. 

Which of the four equations are simple? Why? 
In the first equation what terms contain unknown quan- 
tities? 

What terms are unknown in the second? 
What terms are unknown in the third? 
What terms are unknown in the fourth? 



Digitized by 



Google I 



28 INTRODUCTION 19 

27. How Solved. Simple equations like three of 
those in paragraph 26 are solved as follows: 

(1) Collect the unknown terms into the first member. 

(2) Collect the known terms into the second member. 

(3) Divide both members of the resulting equation by 
the coefficient of the unknown quantity. 

For example, in the first equation if the unknown terms 
X and 2x are collected in the first member, we have 3x. 

In collecting — 15 from the first member into the second, 
the sign is * changed and we have 15 for the second member. 

Therefore after collecting the unknown terms in the 
first member and the known terms into the second member, 
the equation becomes 

3x = 15. 

The value of x may now be determined by dividing 
both members by the coefficient of x. 

Therefore, 

x = b. 

28. Examples. Solve the following examples according 
to the directions and form given on page 8, and as explained 
in paragraph 27. If a result is not integral carry it to two 
decimal places. 

Those who have never studied algebra will work from 
1 to 35 only and from 86 to 105. All others will begin 
with example 30. 

1. 5a:+8a;-7a:-a:=25. 2. 2x-x+^-^z-llx^U, 

3. Sx+x- 12+9 -1=0. 4. 2a:+14-6a:+12a;=110. 

5. 3a;-5+2a;-9a:=-6x+15. 6. 18+7a;-10=3^x+57-8. 

•Whenever a quantity is trans poied from one member of an 
equation to the other member, the sign of the quantity must be changed. 
Actually there is no transposition. The word expresses what happens 
when a quantity is eliminated from a member of an equation by 
subtracting it from both members. 



Digitized by 



Google 



20 TECBNICAL ALGEBRA ^$ 

7. |a:-132+8Jx=x-12. 8. 4(2x+7) -46=0. 

o 

9. 2x+9(2+3a:) =76. 10. 19+10x-5ix=3(28-2x). ^ 

11. 5(3a:+l)+2(2x+8)=42+4x. 

12. 3j(4x-2)=3i(x+14). 

13. ^x-3x+15(4-2x)=31Jx+80. 

14. |(y^+64x) =41. 15. nh+3h+U^+lb\ -25=0. 

16. 12r+|(^+^) =124. 17. 300+|c+lic+. 125c =425. 

18. 2(18.5-30.2^+34.30=2^-16.6. 

In the following examples divide both members by the coeffi- 
cient of the unknown quantity, transposing when necessary. 

A fractional coefficient must be inverted and used as a mul- 
tiplier. 

19. —V* -gs. Solve for v* and for v. 

20. 2s -gV^, Solve for s and for g. 

21. li =Q. Solve for i and for 7. 

22. IK = y. Solve for 7 and for K, 

23. Eff(ir+w;) =w;. Solve for EfT. 

24. Ir—E- —p. Solve for 7 and for r. 

26. — =7. Solve for E and 7^. 
it 

26. ^ = .0009477^. Solve for H. 

27. 9C =5(/^ -32°). Solve for C and for F. 

28. >S(n-l)=G. Solve for 5. 

29. Ar =/?(Ai -il). Solve for R. 

30. 7(r ns+/?) = J^ ns. Solve for 7. 

31. RRi = {R+Ri)J. Solve for J. 

32. Sn ^G+S. Solve for n and for G. 



Digitized by 



Google 



28 INTRODUCTION 21 



.7854 



=4a6. Solve for A. 



34. ^ = .5236. Solve for V and for d. 

4 
36. Fr* =r-x. Solve for V and for r. 
o 

36. 2|x =t{ ^+8 ) . Solve for x. 



5. 2|x =1(1+8) 



37. X =— ~2^. Solve for D. 

Ji 

38. — =-7-. Solve for E and for A, 

e A 

39. 2Kg = Aft;2. Solve for K and for t;. 

40. 8 =— (r+0. Solve for p and for r. 

41. FLD =9600000<2-'^ Solve for P and for D. 

42. R^ = A«+52. Solve for 7^, for A, and for 5. 

43. Q(a+6) =a(P+Q). Solve for Q and for 6. 

44. 7 = — — — , Solve for t and for s. 
46. A =^+6(5+n). Solve for 6 and for n. 
46. A = jW^ -di^). Solve for rf. 



'-ii(--f) 



47. I =77^^3i22 -- ) . Solve for R and for 8, 

48. i2 =\/t' Solve for 7 and for A. 

49_R = ZiL__EL. Solve for d. 
4 

60. iV =L — -rrr: - 777. Solvc for a and for L. 
100 144 

61.* P =^^^- Solve for D and for iV. , 

*The equations for examples 51 to 70 are from formulas in 
"Gearing," published by Brown & Sharpe Mfg. Co. 



Digitized by 



Google 



22 



TECHNICAL ALGEBRA 



28 



52. s = 



N+2' 



Solve for D and for N. 



t^^P'. Solve for P'. 



63. 

64. s+f = .3683P'. Solve for s and for P\ 
66. 



F =|+y. Solve for P and for A. 



66. r' = — -2s. Solve for d' and for s. 

67. r" =r'+D"+/. Solve for D" . 

68. 0=^^'+^. Solve for P'. 

69. Z =Z)"+2/+^. Solve for D" and for /. 

o 

C= — T- — s. Solve for D' and for s. 



60. 

61. 6 =7r(d -2s). Solve for d and for s 
62. 



iV 



D =— +2s. Solve for s and for A/". 



63. 
64. 
66. 
66. 
67. 



P' = 



iV+2* 



Solve for N and for D. 



937iV 
D = ' „^ +4s. Solve for A'' and for s. 



P' = 



frC 



70. 



KA^a + iVi,)* 




2)"+/ = 2s+--. Solve for « and for /. 
1,2 =— 7^77— ^ Solve for W and for Gi. 
2)" = f:i^ _/. Solve for £>" and 
«+/=^(l+J). Solve for 



28 



INTRODUCTION 



23 



71. P =-^. Solve for S and for r. 

r+l 

72. jtD^p =TDtS. Solve for p and for D. 

73. -=-^. Solve for /and fore. 

C 04 



/ 64 
74. — = ■■ . Solve for / and for d. 
c id 

76. P =KN^^^^. Solve for AT. 



76. J =— (d« -d,«). Solve for d. 

S^d' Si,r(d,'-d,*) 

~16~ iCdi ■ ^'veford. 



78. B 



\ V3.2n/ 



Solve for D, 



79. .7CP=Sb 



Solve for P and for b. 



80. 'S = .. . ^ . Solve for F. 




Digitized by VjOOQ IC 



24 TECHNICAL ALGEBRA 28 

The following are examples in solution by substitution: 

jD I r\ -Iff -Iff 

^- ^="^57;;ir+T^+V- Compute the value of « when B=4", 
^Uu lb o 

2> = 16". 

[87. In example 73 compute the value of — when Af = 19000, 

/ = 15, ^4=9000. 

88. W=VA+B+C+D-E, Compute the value of W when 

A=2A, B = li, C=0, D~ E~ 

89. In example 76 compute the value of / when d =8", di =6". 

90. S =-| (2t-l). Compute the value of t when S =530, a =32.2. 

91. In example 71 compute the value of S when 

r=5f, t~ P = 172. 

W L 

92. -Tr=77. Compute the value of F when T7 = 196, L=8, 

t H 

if=4i. 

93. G(<-ii)x =(?(«! -<2). Compute the value of x when <=90, 
fi=25, <2 = 18, G = 125. 

94. In example 68 compute the value of / to four decimal places 
when s = . 5570, P = 1.7952. 

96. In example 69 compute the value of P' when (7=7.25, 

iVa=28, iV6=72. 

96. H.P.= ^^'~^^^ . Compute H.P. when !ri = 100, T^^^.ZTi, 

7=700. 

W 2R 

97. 17 = ^ — . Compute the value of Pwhen 72=3.75, r=2.4, 

T7 = 1250. 

S8. A ^ /7r • Compute the vialue of h when g =32.2 iV =90. 
47rW* 

99. t* =7r2 — . Compute the value of t when L =39.1, ^ =32.2. 



Digitized by 



Google 



2d INTRODUCTION 25 

100. Kw = — . Compute the value of E when 

ouOOO 

G = .07, R = 1320, T7 =25000, Kw =65. 

101. /S = y r-. Compute the value of S when 

L=3400, P' = 1.2, /=5, C = .08, r=600, F = 1200. 



._!H) 



102. P' = — ^ — ^. Compute the value of P' when 

a 

P = 2i, d = 14.1, /=5i 

103. W = .0357/P'(6.25n+.04n«). Compute the value of W when 

/=3i, P'=2J, n=21. 

' 104. Vi = — .^. , "L . Compute the value of Vi when 
n{t +/) 

F = 125, <'=95, n = 120, /=115, <=85. 

2d 
106. 2> =^ . Compute the value of D when 

^tanA+1 
Jtf 

d =8, Af =6, 72/ =2, tan A = .5774. 



§3. FORMULATION OF MATHEMATICAL LAWS 

To THE Teacher. The aim in this section is to have the 
student learn to formulate. Explanation of laws is therefore 
unnecessary, will leave less time for formulation, and may con- 
fuse rather than instruct. 

29. Illustration. How would you compute the distance 
traveled by a train which runs for a given number of hours 
at a given rate per hour? 

Distance = what times what? 
If d = the distance in miles, 
r=the rate in miles per hour, 



Digitized by 



Google 



26 TECHNICAL ALGEBRA 80 

and <=the time of running in hours, 
then d=what in terms of rate and time? 

(Express your answer entirely in the above symbols.) 

Your answer to the last question, if correct, is a statement 
in symbols, of the exact mathematical relation of distance to 
time and rate. It is therefore a formula. 

To be more definite it is a significant formula because the 
symbols in which it is expressed are the first or initial letters 
of the names of the quantities involved, as d for distance, 
r for rate, and t for time. 

The same law might be formulated conventionally by 
denoting distance by x, rate by y, and time by z. 

Then a: = what in terms of y and 2? 
Of these two ways of writing a formula, do you prefer 
the conventional or the significant? Why? 
Write the definition of a formula. 

30. Weight. In significant notation write a formula for 
the law that Ws, the weight of any substance, equals its 
volume F, times its specific gravity sp.gr., times the weight 
TF of a unit volume of water. 

Solve the weight formula for volume. 

Solve it for specific gravity. 

Solve the distance formula of paragraph 29, for rate. 

Solve it for time. 

31. Expression of Ratio. There are three ways of denot- 
ing a ratio; one by the colon, one by the division symbol, 
and the other by a fraction line. 

Thus the ratio of 3 to 16 is expressed 

3: 16, ;3-M6, ^, or 3/16. 

The fraction form with a horizontal fraction line is always 
to be preferred and should be used in all mathematical 
calculations involving fractions. 



Digitized by 



Google 



33 INTRODUCTION 27 

Thus it is suggested that the ratio of a to 6 be indicated 

by 

T- and not by alb, or a 4- 6, or a/6. 

The ratio of energy to momentum should be indicated by 

enercv 

^ — and not by energy : momentum, 

momentmn '' ^'^ ' 

or energy -^ momentum, 

or energy/momentum, 

E 
In symbols it may be indicated by — . 

32. Specific Gravity. The specific gravity of a solid 
substance equals the ratio of its weight to the weight of an 
equal volume of W3,ter. 

Express this law as an equation, omitting as many words 
as possible but not using symbols. 

Write a significant formula for specific gravity. 
Write a conventional formula. 
In both instances specify the notation employed. 
Solve the significant formula for TF, and W. 

33. Problems. Under proper heading and in significant 
notation write the formulas for the following laws and solve 
for each of the literal quantities in the second member. 

When a figure is shown in the text, draw it in the work- 
book. 

1. Force. The force F in pounds, imparted by a moving 
body, equals its mass M times its acceleration a. 

2. Mass. The mass of a substance equals its weight W in 
pounds, divided by 32.2. 



3. Area of a Rectangle. The area of 

a rectangle equals the length or base h 

times the width or altitude h. ^ ^i 



Digitized by 



Google 



28 



TECHNICAL ALGEBRA 




b 
FiQ. 3. 



• 4. Area of a Parallelogram. 

The area of a parallelogram 
equals the base b times the alti- 
tude h. 



6. Area of a Triangle. The area 
of a triangle equals one-half the base b 
times the altitude h. 





7. Area of a Trapezoid. 

two sides parallel and two 
non-parallel. 

The area of the trape- 
zoid equals one-half the 
sum of the parallel sides, 
times the perpendicular 
distance between them. 

8. Area of a Trapezium. 



6. Square of the Hypotenuse. 

The hypotenuse of a right tri- 
angle is the side opposite the 
right angle. 

The square of the hypotenuse 
equals the sum of the squares of 
the other two sides. 

A trapezoid is a quadrilateral having 





Fig. 7. 



6 
Fig. 6. 

A trapezium is a quadrilateral 
having none of its sides 
parallel. 

The area of a trape- 
zium equals one-half 
times one of its diag- 
onals times the sum of 
the perpendiculars to 
that diagonal from the 
vertices of the opposite 
angles. 



Digitized by 



Google 



INTRODUCTION 



29 



9. Area of a Regular Polygon. A 
regular polygon is one which has equal 
sides and equal angles. 

The apothem is the perpendicular 
distance from the center to any side. 

The area of a regular polygon equals 
one-half the perimeter times the apothem. 



>9 y V 



Fig. 8. 




10. Angle of a Regular Polygon. An angle 
6f of a regular polygon having n sides, equals 
180° multiplied by n-2 and divided by n. 



11. Central Angle of a Regular 
Polygon. A central angle of a regular 
polygon of n sides, equals 360° divided 
by n. 




Fig. 10. 




12. Ratio of Circumference to Diam- 
eter. The ratio of the circumference of 
a circle to its diameter, equals 3.1416 
approximately. 

In the formula substitute t for 3.1416. 

Formulate also in terms of radius. 



Fig. 11. 



13. Area of a Circle. The area of a 
circle equals t timies the square of the 
radius. 

Formulate area also in terms of diam- 
eter. 




Fig. 12. 



Digitized by 



Google 



30 



TECHNICAL ALGEBRA 



33 




14. Area of a Ring. The area of a 
ring included between the circumferences 
of two concentric circles equals t times 
the difference between the squares of the 
two radii. 



Fig. 13. 



In the remaining problems in this paragraph, do not 
solve for the literal quantities in the second member, but 
do as specified in each problem. 

16. Length of Arc of Sector of a 
Circle. A sector of a circle is a portion of 
it bounded by two radii and the inter- 
cepted arc. 

The length of the arc of a sector of 

Q 

6° equals -— times the circumference of 
360 

the circle. 

In the formula substitute for circum- 
ference in terms of radius, and simplify. 

16. Area of a Sector of a Circle. The area of a sector of a 
circle equals one-half its radius times its arc. 

In the formula, substitute for arc from the second formula 
of problem 15. 




Fig. 14. 




Fig. 15. 



17. Area of an Ellipse. The area of 
an ellipse equals t times the product of 
the two semi-axes. 

In the formula denote the semi-axes 
by a and 6, a being half the long axis 
and b half the short axis. 



18. Perimeter of an Ellipse. The perimeter of an ellipse 
equals approximately 1.82 times the long axis, plus 1.315 times 
the short axis 



Digitized by 



Google 



INTRODUCTION 



31 



19. Total Area of a Cylinder. The total area 
or surface of a cylinder, equals its circumference 
times its length, plus twice the area of the base. 

In the formula, substitute for circumference 
and area of base in terms of radius of the base. 

Formulate, also, the curved surface of a 
cylinder. 



Fig. 16. 




20. Volume of a Cylinder. The volume of a cylinder equals 
the area of the base times the length of the cylinder. 

In the formula substitute for area of the base in terms of radius; 
also in teems of diameter. 

21. Total Area of a Regular Pjrramid. 

A regular pyramid is one whose apex is 
directly over the center of its base which 
is a regular polygon. 

The total area of a regular pyramid 
equals the area of the base Ai,j plus the 
area of the sides Ai^. 

In the formula substitute for At from 
problem 9, and for Ax, from problem 5. 

22. Volume of a Regular Pyramid. The volume of a regular 
pyramid equals one-third the area of the base times the altitude. 
In the formula substitute from problem 9. 



23. Total Area of a Cone. The total 
surface or area of a cone equals the area of 
the base, plus the lateral area. 

In the formula substitute for area of the 
base, in terms of radius of the base, and for 
lateral area substitute one-half the circum- 
ference of the base times the slant height. 

In the formula thus obtained, substitute 
for circumference in terms of radius. 




Fig. 18. 



24. Volume of a Cone. The volume of a cone equals one- 
third the area of the base times the altitude. 

In the formula substitute for area of base in terms of radius. 



Digitized by 



Google 



32 



TECHNICAL ALGEBRA 



S3 



26. Volumes of the Frustum of a Cone and a Pyramid. A 

frustum of a cone and of a pyramid is the part which reniains when 
the top is cut off parallel to the base. 






Fig. 19. 



The voliune of a frustum of a cone and of a pyramid equals 
one-third the altitude times the following: 

Area lower base Ab, plus area upper base Aj, plus the square 
root of the product of the areas of the two bases. 

26. Area of a Sphere. The area of a 
sphere equals tt times the square of its 
diameter. 

Formulate also in terms of radius. 

27. Volume of a Sphere. The volume 
of a sphere equals two-thirds tt times the 
square of the radius, times the diameter. 

In the formula substitute for radius 
in terms of diameter, and simplify; also 
substitute for diameter in terms of radius, and simplify. 




Fig. 20. 



28. Volume of a Rectangu- 
lar Solid. Write a law for the 
volume of the solid here shown, 
and formulate the law. 




Fig. 21. 



29. Volume of a Cylindrical Ring. The volume of a ring 
with a circular cross-section, equals 2.4674 times the square 
of the thickness, times the ^sum of the thickness and the inner 
diameter. 



Digitized by 



Google 



33 



INTRODUCTION 



33 



30. Area of a Cylindrical Ring. The 
surface or area of a cylindrical ring 
equals 9.8696 times the thickness, times 
the sum of the thickness and the inner 
diameter. 

By thickness is meant the diameter 
of the materikl of which the ring is made. 




Fig. 22. 




Fig. 23. 



31. Volume of a Spherical 
Segment. The volume of a 
spherical segment which is less 
than a hemisphere equals .52367r 
times the depth d, times the 
following sum: Square of depth, 
plus three times the square of 
the radius of the base of the 
segment. 



32. Change in Velocity of a Falling Body. The change in 
the velocity Vc of a falling body, equals its acceleration a, multiplied 
by the time of motion. 

33. Final Velocity of a Falling Body. The final velocity V/ 
equals the initial velocity F/, plus the change in velocity. 

For the change in velocity substitute from the formula of prob- 
lem 32, and for initial velocity substitute zero. 

34. Average Velocity of a Falling Body. The average 
velocity Va equals one-half the siun of the initial velocity and the 
final velocity. 

For initial velocity substitute zero and for final velocity sub- 
stitute from the last formula of problem 33. 

36. Space Traversed. The distance s equals the average 
velocity multiplied by the time. 

In this formula substitute for average velocity from the last 
formula of problem 34, and simplify. 

36. Velocity of Falling Body. The velocity 7 of a falling body 
under the influence of gravity equals the force of gravity gr, mul- 
tiplied by the time. 

Solve the formula for time. 



Digitized by 



Google 



34 TECHNICAL ALGEBRA S3 

37. Velocity of a Falling Body. In the last formula of problem 

35 substitute for time from problem 33 and solve the resulting 
formula for velocity. 

38. Law for Velocity. By reference to the last formula of 
problem 37 write the law for velocity of a body falling freely from 
rest. 

39. Moment of Inertia. The moment of inertia of a particle 
equals its mass times the square of the distance of the particle 
from the center of rotation or the axis. 

Denote moment of inertia by / and distance from center by r. 
Solve for mass and r. 

40. Centrifugal Force. The centrifugal force Fe of a rotating 
body equals its mass times the square of its velocity in feet per 
second, divided by the radius of revolution. 

Solve for 7. 

41. Centrifugal Force. In the first formula of problem 40 
substitute for mass from the first formula of problem 2. 

42. Work. The work done by a force which acts on a body 
equals the force in pounds times the distance in feet through which 
the force acts in the direction of displacement of the body; or 
briefly, work equals force times distance. 

43. Horse-Power. Horse-power, H.P., equals the force in 
pounds times the distance in feet, divided by 33,000 * into the 
time in minutes. 

44. Horse-Power of a Steam Engine. This formula may 
be written by substituting the following in the formula of problem 43 : 

For force substitute PA, mean effective pressure of steam in 
pounds per square inch times the area of the piston in square inches. 

For distance substitute LN, the length of the stroke in feet 
times the number of strokes per minute. 

For time substitute 1 minute. 

State the notation before writing the formula. 

1000 

45. A Kilowatt, Kw. One kilowatt equals ^— - horse-power. 

1 kilowatt equals approximately how many thirds of a horse-power? 
* In mathematics "into" is frequently used to denote times. 



Digitized by 



Google 



INTRODUCTION 



35 



46. Elinetic Energy. The kinetic energ>' of a moving body 
equals one-half its mass times the square of its velocity in feet per 
second. 

In the fonnula substitute for mass from problem 2. 

47. Kinetic Energy. The kinetic energy of a rotating body 
equals one-half its moment of inertia times the square of its angular 
velocity a, 

48. Momentum. The momentum m of a moving body equals 

its weight in pounds times its velocity, divided by 32.2. 

W 
For -TT^ substitute from a preceding problem. 

49. Coefficient of Friction. The force required to slide one 
body on another equals the weight of the body times the coefficient 
of friction /. Solve for /. 

60. Law of Moments. In a 

lever of whatever class, the force 
P, times the force arm Pa^ equals 
the weight times the weight arm 

Solve this formula for each 
quantity in both members. . 

61. Ohm's Law. In an elec- 
tric conductor through which a 
current is flowing, the current / 
in amperes equals the electro- 
motive force E in volts, divided 
by the resistance R in ohms. 

Solve for E and /. 



i<-W7r- 



W 



-Wa 



-Ws 



W 



X: 



w 



FiQ. 24. 



62. Current from a Series Battery. 




Fig. 25. 



In a battery arranged in 
+ series, the current / 
_ equals the number 
of cells ns times the 
electromotive force, 
divided by the sum 
of the internal and 
external resistances, 
nr and R. 



Digitized by 



Google 



36 



TECHNICAL ALGEBRA 



33 





63. Period of a Pendulum. The time T of oscil- 
lation of a pendulum in seconds equals x times the 
square root of the quotient of the length in feet and 
the force of gravity g. 

64. Current from a Multiple Battery. The cur- 
rent from a battery arranged in parallel equals the 
voltage (electromotive force) divided by the sum of 
the external resistance and the ratio of the internal 
resistance of one cell to the mmiber of cells np, 

O 4 > 



Fig. 26. 



Fig. 27. 



56. Current from Multiple-Series Battery. The current from 
any multiple-series combination equals the voltage times the num- 
ber of cells in series ns, divided by the external resistance plus the 
ratio of the product of the number in series and the resistance of 
each cell to the number in multiple arrangement np. 

56. Current through a Shunted Galvanometer. The cur- 
rent Is flowing through a galvanometer in a shunt circuit equals the 
current / in the joint circuit, divided by 1 plus the ratio of the 
galvanometer resistance Rgj to the shunt resistance Ra, 

57. Electrical Efficiency. The electrical efficiency Eff of a 
djmamo equals the number of kilowatts Kw which it delivers to 
the circuit, divided by the number of kilowatts lost in the arma- 
ture Kwa plus the number of kilowatts lost in the field coils Kwe 
plus the number of kilowatts delivered to the circuit. 

58. Determination of High Resistance. The resistance of a 
circuit R equals the resistance of the voltmeter /2p, *into the 
difference between the ratio of the two deflections 5i and 62, and 1. 

* In mathematics and its applications the word "into" is often 
used instead of times or multiplied by» 



Digitized by 



Google 



34 INTRODUCTION 37 

69. Power Required. The number of watts required for the 

746 
propulsion of a trolley car on a level road equals 5^7^ of the 

weight in tons of the car as loaded, times the tractive force T 
in pounds, times the speed of the car in feet per minute, divided 
by the efficiency of the motor. 



§ 4. FORMULATION AND COMPUTATION 

To The Teacher. The sole purpose of this section is for- 
mulation and computation. A student's work primarily is there- 
fore the translation- of words into mathematical symbols, and the 
more closely the attention is fixed on formulation to the exclusion 
of externals, the better. 

One of the means of securing this concentration is by the use 
of laws regarding things of which the student has little or no knowl- 
edge, .but with which he will become familiar in subsequent 
mdustrial studies. At times the mathematics' instruction may 
be made more significant by the use of a machine or by analjrsis of 
mechanical principles in connection with students' work at the 
blackboard. 

In some instances it may be advisable to stimulate interest by 
the use of the picture of the machine in a trade catalogue or tech- 
nical periodical, when the machine itself is not available. In a 
technical school arrangements may also be made from time to time 
so that a class or section may visit the school shops or laboratories 
and have a machine demonstrated by some advanced student or a 
shop instructor. 

None of these things is necessary, however, in teaching this 
book, nor will they aid in formulation except indirectly. The 
entire book may be well taught without them. 

34. Definitions. A proportion is an equality of ratios. 

Which of the equations written in your work-book, from exer- 
cises 2 and 4, are proportions? Why? 

Can a proportion be written which is not an equation? Give 
reason. 

J 



Digitized by 



Google 



38 TECHNICAL ALGEBRA 35 

A simple proportion is a proportion having two ratios. 
A continued proportion is a proportion having more than 
two ratios. 

The proportions you have previously written are what 
kind? 
Why? 
Write two more proportions of the same kmd. 

The terms of a proportion are the quantities forming the 
proportion. 

How many terms has a simple proportion? 

The extremes of a proportion are the first and last terms. 
The means are the second and third terms. 

What quantities are the extremes in your first proportion? 
What quantities are the means in your last proportion? 

36. Law of Proportion. In Chapter XXII, Part II, are 
the various laws of proportion. Only one of these is neces- 
sary in this part of algebra. You will recognize it as the 
same law that was applied in the solution of problems in 
proportion in your study of arithmetic. 

This law is as follows: 

The product of the means equals the product of the extremes. 

Write the equations which are obtained by applying this 
law to three proportions in your work-book, as written in 
exercise 2, page 13. 

36. A Simple-Geared Lathe. The drawing shows the 
arrangement of gears on a simple-geared lathe. Through 
the train of gears the revolution of the spindle to which the 
work is attached, is transmitted to the lead-screw which 
causes the carriage holding the cutting tool to move along 
the lathe-bed. 

When a lathe is simple-geared the gears required to cut a 



Digitized by 



Google 



37 



INTRODUCTION 



39 



given number of threads per inch may be determined from 
the following law: 

The pitch of the lead-screw is to the pitch of the screw 
to be cut, as the number of teeth in the gear on the spindle 
is to the number of teeth in the gear on the lead-screw. 

Notation: L = pitch* of lead -screw, 

5 = pitch of screw to be cut, 
^5 = number of teeth in gear on spindle, 
Tl = number of teeth in gear on lead-screw. 



Spinflle gear 




gear 



Fig. 28. 

In the work-book enter paragraph number and title, 
and the notation. 
Write the formula. 

37. Problems. The first problems which follow illus- 
trate the use of proportion in determining the gears which 
are required on a screw-cutting lathe under various condi- 

♦ Pitch as here used, means the number of threads to the inch. 



Digitized by 



Google 



40 TECHNICAL ALGEBRA 87 

tions. They are here introduced to show the simplicity 
of solution by formulas and to help you to understand the 
difference between arithmetic and algebra and the advantage 
of both letters and figures in representing quantity. 

In computation be satisfied with a correct result only 
when obtained by the simplest and quickest method of 
solution; in other words, by a method which minimizes 
mistakes and saves time and energy. 

In the solution of the following problems enter the work 
under problem number and title with formula and solution 
near the left margin of the page and data near the right. 

1. Lead-screw Gear. A machinist desires to cut a 12-pitcii 
screw with a 6-pitch lead-screw. If he uses a 24-gear on the spindle, 
what gear must he use on the lead-screw? 



D 



miii 




12-pitch screw. 24 spindle-gear. 

Fig. 29. 

Determine the lead-screw gear by substituting directly in the 
formula of paragraph 36. 

Cancel if possible. 

Double underline the final result as specified in paragraph 11, 
page 8. 

Check. To test the correctness of a result by solution by another 
method is called checking. Check your result as follows: 

Before substituting the numerical values from the data, solve 
the formula of paragraph 36 for the unknown quantity in two 
ways: 

(1) Multiply the formula by the * least common denominator 
and divide the resulting equation by the coefficient of the imknown. 

* The least common denominator is the smallest number in which 
the given denominators are evenly contained. 



Digitized by 



Google 



37 



INTRODUCTION 



41 



(2) Apply the law of proportion regarding the product of the 
means. 

Substitute the numerical values, cancel if possible, and indicate 
the result in the same form as before. 

2. Spindle-Gear. A lathe is equipped as follows: 

L=4, 
5=10, 

What gear must be used on the spindle? 

3. Pitch of Screw. A lathe has a 28-gear on the spindle and 
a lead-screw of 50-gear and 6-pitch. 

It will cut a screw with what pitch, without changing the gears? 

4. Standard Gears. In standard gears the number of teeth 
ranges from 21 to 105 by intervals of 7, or from 24 to 120 by intervals 
of 4. 

Write down all the gears of both standards. 

If the pitch of the lead-screw is 8, and 12 threads per inch are 
to be cut, what must be the ratio of spindle and lead-screw gears? 

If all the gears of both standards are in the shop equipment, 
what three pairs of gears could be used under the ^ven conditions? 

6. Cutting Speed. The speed S in feet per minute with which 
a piece of work revolves under the cutting tool, equals ir times 
the diameter of the work in inches, times the number of revolu- 
tions per minute divided by 12. 

A shaft 8J inches in diameter is turned with a cutting speed 
of 18 feet per minute. 

Compute the niunber of revolutions. 



ji 



^ 




<L 



Fia. 30. 



Digitized by 



Google 



42 



TECHNICAL ALGEBRA 



87 



6. Time Formtila. The time in minutes required to turn 
a revolving piece of work equals the length of the work in inches 
divided by the product of the feed in inches per revolution and the 
number of revolutions R per minute. Write the formula, sub- 
stitute for R from the formula of problem 5, and simplify. 

7. Time Required. With a feed of J of an inch and a cutting 
speed of 21 inches per minute, a piece 10 feet long and 29 inches 
in diameter is to be turned. 

Find the time required in hours. 

8.* Weight of a Pulley. 

If R =the radius of a pulley in inches, 

W =the width of the belt in inches, 
and P =the approximate weight in pounds, 

P = I7'[.163|+.015(|) +.00309(1)']. 



then 



Compute the approximate weight in pounds of a 29-inch pulley 

carrying a 4-inch belt. 

9. Diameter of a Pulley 
Arm. 

Z)=the approximate diameter in 
inches of arm at rim, 

N =the number of arms, 

Tr=the width of the belt in 
inches, 

72= the radius of the pulley in 
inches. 




Fio. 31. 



3 V 4 lOAT/ 



A draftsman is to design a pulley 18 inches in diameter and 
having 4 arms, to carry a 3-inch belt. 

Find the approximate diameter of arm at rim. 

* The formulas in problems 8, 9, and 12 are from Cromwell's 
"Belts and PuUeys." 



Digitized by 



Google 



37 



INTRODUCTION 



43 



10. Velocity of a Pulley. The circumferential velocity of a 
pulley in feet per minute equals ir times the diameter of the pulley 
in feet, times the number of revolutions per minute. 

By the formula compute the number of revolutions per minute 
of a 28-inch pulley whose velocity is 2100 feet per minute. 

11. Circumferential Ve- 
locity. A shaft carries two 
fixed pulleys having diameters 
of 9 J inches and 16| inches. 

Compute the circumfer- 
ential velocity of each pulley 
when the shaft makes 125 
revolutions per minute. 

12. Diameter of a Shaft. The diameter of a pulley shaft in 
inches equals the cube root of the product of the radius of the 
pulley m inches and the number of pounds of force transmitted 
by the pulley, provided the cube root is multiplied by the following 
numbers for shafts of the material specified: 




Fig. 32. 



Material: Steel, 

Cast-iron, 
Wrought-iron, 



Multiplier: .075 
.108 



Write the three formulas. 

Compute the diameter of a shaft of each material to the nearest 
64th of an inch for a pulley 12f inches in diameter and transmitting 
a force of 900 pounds. 

13. Horse-Power of 
a Steam Engine. The 
horse-power of an engine 
equals the mean effective 
pressure in pounds per 
square inch times the 
length of the stroke in 
feet times the piston area 
in square inches times 
the number of strokes 
per minute, divided by 
Fig. 33. 33,000. 




Digitized by 



Google 



44 



TECHNICAL ALGEBRA 



87 



Notation: H.P.= horse-power, 

N = number of strokes per minute =2 times num- 
ber of revolutions, 

A =area of piston in square inches, 

P =mean effective pressure in pounds per square 
inch, 

L = length of stroke in feet. 

An engine has a piston area of 212.9 square inches, the stroke 
is 2J feet, and the mean effective pressure is 95 lbs. per square 
inch. How many strokes per minute will be necessary in order 
to develop 175 horse-power? 

14. Radius of a Gear Wheel. The 

radius of the pitch circle of a gear wheel 
equals the number of teeth times the pitch * 
divided by 2 tt. 

Write the formula, using significant capital 
letters. 

A gear wheel of 3J inches pitch and 
having 76 teeth, is to be drawn to a scale 
of I inch = l foot. 

What will be the length of the radius in 
the drawing? 




Fig. 34. 



16. Taper of Keys. 

Notation : T = taper per foot in inches, 
L = length in inches, 

a = large diameter or thickness in inches, 
b = small diameter or thickness in inches. 

Formula: ,^=»i 

12, . f— ni 



T=^(a-6). 



Fig. 35. 
Write the law. 

A key 16 inches long, is 2} inches in diameter at one end and 
and 3| inches at the other. 
Required the taper per foot. 

* Pitch is the distance between tooth centers, measured on the 
pitch circle. 



Digitized by 



Google 



S7 



INTRODUCTION 



45 





. 












1 


w 1 








1 


1 


i \ 






















*1 


(i 


di 




' 



Fig. 36. 



16. Center of Gravity. The distance of the common center 
of gravity from the center of gravity of the larger of two bodies, 
equals the distance be- 
tween the centers of 
gravity of the two bodies 
times the weight of the 
smaller body, divided 
by the sum of the 
weights of the two 
bodies. 

Illustrate by a dia- 
gram with lettered dis- 
tances denoting the quantities named, and write the formula. 

Two bodies weighed 150 and 95 pounds respectively and their 
centers of gravity were 46 inches apart. 

What was the distance of their com- 
mon center of gravity from the center 
of gravity of the larger body? 

17. Law for Theoretical Weight. 

If there were no friction, the weight Wt 
which could be lifted by a screw-jack, is 
to the power, as the circumference (in 
inches) of the circle in which the power 
moves, is to the pitch of the screw. 
Write the formula and solve for Wt . 

18. Law for Actual Weight. The 

approximate actual weight TTa, which 
can be lifted by a screw-jack equals 
the theoretical weight multiplied by the 
fraction whose numerator is the pitch 
of the screw and whose denominator is 
the sum of the pitch and the diameter 
(in inches). 
Write the formula, substitute for Wt from problem 17, and 
simplify. 

19. A Screw-jack. A screw-jack has a screw 1| inches in 
diameter with a pitch of \ of an inch. 

What approximate actual weight can be raised by it with a 




Fig. 37. 



Digitized by 



Google 



4i5 TECHNICAL ALGEBRA 87 

force of 125 pounds applied at the end of a lever 14^ inches long? 
(Pitch as used in problems 17, 18, and 19 is the distance between 
the threads.) 

20. Joint Resistance. Formula: 

K 7*1 7*2 • 7*8 7*4 

Find the value of Ry when n =12, r2 =20, rs = 18, r^ = \, 

21. Electrical Transmission. The size of copper wire in 
circular mils required to transmit a given horse-power, equals 
746 times 10.79 times the horse-power times the length of the 
circuit, divided by the efficiency of the motor times the voltage 
times the drop. 

Compute the size of copper wire required for the transmission 
of 260 horse-power through a circuit of 300 feet with a voltage of 
190 volts, the drop being 9 volts and the efficiency of the motor 
83 per cent. 

22. Volume of a Hollow Column. The volume of the material 
in a hollow column equals tt times the length times 

the difference between the squares of the outer and the 
inner radius. 

The outside diameter of a hollow steel colunm is 
12| inches, and the length Sf feet. 

Compute the volume when the inside diameter is 
\0\ inches. 

23. Weight of a Hollow Column. The weight 
of a hollow column in pounds equals 62.5 times the 

specific gravity of the material times the volume in cubic feet. 
Compute the weight of the colunm in problem 22, the specific 
gravity of steel being 7.85. 

24. Volume of a Sphere. The volume of a sphere equals 
f times TT times the cube of the radius. 

Compute the volume of a sphere whose radius is 2.5 centimeters. 
Check by problem 27, paragraph 33. 

25. Radius of a Sphere. By the formula of problem 24 
compute the radius of a sphere whose volume is 5.64 cubic 
centimeters. 




Digitized by 



Google 




37 INTRODUCTION 47 

26. Flux of an Electrical Field. 

= flux or total number of lines of force in an electrical field, 
Zf= field strength or number of lines of electrical force per 

square centimeter, 
A =area of field in square centimeters. 

Write a formula for <t> in terms of A and H, 
Compute when A = 125 and H =3000. 

27. Law of Flux. By reference to the formula of problem 26 
write a law for flux. 

28. Ratio of Velocities. If two pulleys are connected by a 
belt the number of revolutions of the first pulley is to the number 
of revolutions of the second, 
as the diameter of the second 
is to the diameter of the 
first. 

A 13J-inch pulley is joined 
by a belt to an 8A-inch Fig. 39. 

pulley. 

Compute the number of revolutions of the 8 A-inch pulley when 
the.l3^-inch pulley is making 115 revolutions per minute. 

29. Radius from Chord and Rise. The rise of an arc is the 

perpendicular distance to the arc from 
the center of its chord. 

When the length of a chord of a circle 
and the rise are known, the radius may 
be determined by the following law: 

The radius of the circle equals the 
sum of the squares of the rise and the 
half chord, divided by twice the rise. 

By the formula determine the radius 
Fig. 40. when the chord of a circle measures SJi 

inches and the rise is 2^ inches. 

30. Volume of a Spherical Segment. The volume of a segment 
of a sphere equals .5236 times the square of the depth or height 
of the segment times the difference between three times the diameter 
of the sphere and twice the height of the segment. 




Digitized by 



Google 



48 



TECHNICxVL ALGEBRA 



37 



Compute the volume when the sphere is 10 inches in diameter 
r.nd the height of the segment is ^ inches. 

31. Elevation of Outside Rail. On a railway curve the eleva- 

tion of the outside rail in feet 
equals the distance between rail 
centers in feet times the square of 
the maximum velocity of the train 
in feet per second divided by 
32.2 times the radius of the curve 
in feet. 

Compute the elevation on a 
curve having a radius of 1750 feet 
and a distance of 4f feet between 
rail centers, allowing a maximum velocity of 75 miles per hour. 

32. Deflection of a Beam. The approximate deflection in 
inches of a rectangular white oak beam supporting a central load 
equals .00023 times the 




Fig. 41. 



S" 



1 



Fig. 42. 



load in pounds times the 
cube of the length in feety 
divided by the width in 
inches times the cube of 
the depth in inches. 

Determine the deflection of a white oak beam 36i feet long 
whose width is 9 inches and depth 17 inches, when loaded at the 
center with IJ tons. 

33. Deflection of a Wrought-Iron Beam. The deflection in 
inches of a rectangular wrought-iron beam, supported at the ends 
and centrally loaded, approximately equals the load in pounds times 

the cube of the length 
in feet, divided by 
H 104,000,000 times the 
lH width in inches times 
the cube of the depth 
in inches. 

Determine the de- , 

flection in a wrought- , 

iron beam (rectangular) 31 feet 8 inches long with a width of 4 

inches and a depth of 16 J inches, under a central load of 2480 pounds. 



D. 



Fig. 43. 



Digitized by 



Google 



37 



INTRODUCTION 



49 



34. Safe Load on an I-Beam. The greatest safe, distributed 
load on an I-beam which is supported at the ends, equals 3390 
into the sectional area in square inches into the depth in inches, 
divided by the length in feet between end supports. 



TZL 



HIE 



OOOOOOOO 



III 



II I 



]Z~L 



I 



TZI 



n 




Fig. 45. 



Fig. 44. 

Find the maximum safe load when the cross-sectional area is 19 
square inches, depth 15 inches, the total length of the beam being 
25 feet 5 inches, and the supports extending a distance of 5 inches 
under the beam at each end. 

36. Equilibrium on an Inclined Plane. A load of 1750 

poimds rests on rollers on an 
inclined plane 900 feet long, 
the height of the plane being 
198 feet. 

Determine the force neces- 
sary to prevent the load from 
rolling down the plane, friction being disregarded. 

Law: The force is to the load, as the height of the plane is 
to the length of the plane. 

36. Weight of Spur-Gear Blank. The weight of the blank 
for a cut, cast-iron spur gear under 3J inches circular pitch, may 
be computed approximately by the following formula from the 
American Machinist Gear Book: 

in which W = weight of gear blank in pounds, 

pi = circular pitch = — , 

p= diametral pitch, 
N =number of teeth, 
/= width of face in inches. 




Fig. 46. 



Digitized by 



Google 



50 TECHNICAL ALGEBRA 37 

Compute the weight of the blank for a gear of 50 teeth with a 
4-iiich face and 2|-inch diametral pitch. 

37. Resistance of a Copper Wire. The electrical resistance 
of a copper wire 1 millimeter in diameter and 1 meter in length may 
be determined by the following formula in which 

R =the resistance of the wire in ohms, 
<=the temperature of the wire in degrees Centigrade, 
/e =.0203 (1 + . 00410. 

Determine the resistance when the temperature is 23|° C. 

38. Resistance of a Nickel Wire. The electrical resistance 
of a nickel wire 1 millimeter in diameter and 1 meter in length is 
expressed by the formula 

/e = .1568(1 +.0062 
Determine the resistance when the temperature is 18 J ° C. 

39. Specific Heat of Mercury. The specific heat of mercury 
in terms of temperature is expressed by the formula 

H =0.033266 -0.0000092 t. 

What is the specific heat of mercury when the temperature is 
100° C? 

40. Tension in a Rope. When a rope is suspended from 
two supports the tension in pounds on the rope at the supports 
approximately equals the weight of the rope per foot times the 
square of the distance in feet between the supports, divided by 
8 times the deflection of the rope in feet. 




Fig. 47. 

A rope weighing 2\ pounds per foot is suspended from two 
supports 150 feet apart. The deflection is 12 J feet. 
Determine the tension at the supports. 



Digitized by 



Google 



87 INTRODUCTION 51 

41. Diameter of a Piston-Rod. Th^ diameter in inches of a 
long piston-rod equals the fourth root of the fraction whose nu- 
merator is 640 times the square of the diameter of the cylinder in 
inches times the square of the length of the rod in inches times 
the maximum pressure on the pistion per square inch and whose 
denominator is the coefficient of' elasticity times the square of t. 

Determine the diameter of a steel piston-rod 42 inches long, 
the coefficient of elasticity of steel being 30,000,000, the diameter 
of the cylinder 18§ inches, and the maximum steam pressure on the 
piston per square inch 140 pounds. 

42. Safe Transmission by Hollow Shafts. The horse-power 
which may be safely transmitted by a hollow shaft equals a con- 
stant into the number of revolutions per minute into the fraction 
whose numerator is the difference between 

the fourth powers of the external and 
internal diameters in inches, and whose 
denominator is the external diameter in 
inches. 

Compute the horse-power which can Fig. 48. 

be transmitted safely by a hollow steel 

shaft majdng 50 revolutions per minute, whose diameters are 11 
and 8§ inches. 

For steel the constant is .028. 

43. Volume of a Wedge. The volume of a wedge equals J 
into the thickness into the length measured 
perpendicular to the base, into the sum of 
the length of the edge and twice the width of 
the wedge. 

The width of a wedge is 5 inches, the 
thickness is 2f inches, the length of the edge 
is 3 A inches, and the length of the wedge is 21 inches. 
Compute the volume in cubic inches. 

44. Horse-Power of a Shunt Motor. The horse-power of 
a shimt motor may be determined from the formula 





746H.P. = [/-^][/-H(/-f)J. 



Digitized by 



Google 



52 TECHNICAL ALGEBRA 87 

Compute the horse-power when 

7=80, e = 100, r=30, R=—. 

50 

46. Resistance of a Shunt Winding. The resistance of 
the shunt winding of a dynamo at a temperature of 15° was 
45 ohms. 

Compute the resistance when the temperature reached 58 J® 
C, using the formula 



R 



^ / l + .0042r \ 
~^\1+. 0042^,7' 



in which <o** =the lower temperature, 

t° =the highdr temperature, 
R =the resistance at t° C, 
r=the resistance at to° C. 

46.* Efficiency Formula of a Shunt Motor. The efficiency 
of a shunt motor is expressed by the equation 

. Power intake —losses 

Power intake 

in which Power intake = amperes volts, 

and Losses =stray loss+shunt field loss+armature loss. 

Copy the three equations and in the efficiency equation sub- 
stitute for power intake and losses. 

The formulas for the quantities in your last equation are as 

follows: 



Shunt field loss 






Armature loss 
Stray loss ==8, 

Copy the formulas and by substitution write a complete formula 
for efficiency. 

♦Formulas in 46, 53, and 58 are from Franklin and Espy's "Direct 

Currents." 



Digitized by 



Google 



37 INTRODUCTION 53 

47. Shunt Field Loss. Compute the shunt field loss when 
iS? = 120, /e,=45. 

48. Armature Loss. Compute the armature loss when • / =60, 
^ = 120, /e,=45, /2a = .15 

49. Efficiency of a Shunt Motor. Determine the efficiency 
when the stray loss is 800 and the other losses are as computed 
in problem 47 and 48. 

60. Efficiency Formula of a Series Motor. In a series motor 
the formula for efficiency is the same as for a shunt motor, as in 
problem 46, except that shunt field loss becomes 

Series field loss ^RJ^ 
and 

Armature loss =RalK 
Write the formula for efficiency. 

61. Series Field and Armature Losses. Compute the series 
field and armature losses when Re ^.10, / = 60, /2a = .15. 

62. Efficiency of a Series Motor. Determine the efficiency 
when the stray loss is the same as in problem 49 and the other 
losses are as computed in problem 51, ^ being 120. 

63. Efficiency Formtila of a Compound Motor (long-shunt). 
This formula differs from that of problem 50, as follows: 



Shunt field loss 
Series field loss =i?« 



Armature loss 






Write the complete formula for efficiency. 

64. Shunt Field Loss. Compute the shunt field loss when 
& = 60, ^ = 120. 

66. Series Field Loss. Compute the series field loss when 
Rt and E have the same values as in problem 54, Re = .082, I = 60. 

66. Armature Loss. Compute the armature loss with the 
same data as in problems 54 and 55, and Ra=.l. 



Digitized by 



Google 



54 TECHNICAL ALGEBRA 37 

57. Efficiency Formula of a Compound Motor (long-shunt). 
Determine the efficiency when the stray loss is the same as in the 
preceding problems and the other losses as in problems 54, 55, 
and 56. 

58. Efficiency Formula of a Compound Motor (short-shunt). 
This formula is the same as that of problem 50, with the following 
exceptions: 

Shunt field loss =rA — jr-^ j , 

Armature loss =/2a / - f — ^-^ j . 

Write the complete formula for efficiency. 

59. Armature Loss. Compute the armature loss with the 
data of problems 54, 55, and 56. 

60. Shunt Field Loss. Compute the shunt field loss with 
the same data as in problems 54 and 55. 

61. Efficiency of a Compound Motor (short-shunt). Com- 
pute the efficiency when the stray loss is 800 and the other losses 
arc as computed in problems 59 and 60. 

62. A Compotmd-Geared Lathe. The law for the gears 
required on a lathe when compound-geared, is as follows: 

The ratio of the threads to be cut, to the threads on the lead- 
screw, times the ratio of the number of teeth on the spindle-gear 
to the number of teeth on the fixed stud-gear, equals the number 
of teeth in the outer sleeve-gear times the number of teeth on the 
leadscrew-gear, divided by the number of teeth on the change 
stud-gear times the number of teeth on the inner sleeve-gear. 

Notation: Ts = number of threads to be cut, 

Tx,= number of threads on the lead-screw, 
S = number of teeth on the spindle-gear, 
F= number of teeth on the fixed stud-gear, 
0= number of teeth on the outer sleeve-gear, 
L= number of teeth on the leadscrew-gear, 
C= number of teeth on the change stud-gear, 
E =nmnber of teeth on the inner sleeve-gear. 

Write the formula. 



Digitized by 



Google 



87 



INTRODUCTION 



55 




Fia. 50. 




Leadscrew gear 



FiQ. 51. 



Digitized by 



Google 



66 



TECHNICAL ALGEBRA 



37 



63. Ntunber of Threads per Inch. 




Fig. 52. — Calorimeter. 



Compute the number 
of threads which may 
be cut per inch when 
rx,=6, F=48, iS =24, 
L=36, C = 18, E^12, 
0=36. 

64. Weight of 
Gaseous Steam. The 
weight of gaseous steam 
per cubic foot equals 
2.7074 times the total 
pressure in pounds per 
square inch times the 
specific density, di- 
vided by 461 plus the 
temperature Fahren- 
heit. 

Compute the weight 
when the pressure is 580 
pounds per square inch, 
the specific density .622, 
and the temperature 
482^ F. 



65. Per Cent of Moisture in Steam. 

Notation: M =the per cent of moisture, 

H = total heat of 1 pound of steam at boiler pressure, 
Hi = the latent heat at boiler pressure, 
/i =the total heat at reduced pressure, 
Hs =the specific heat of saturated steam, 
T= average calorimeter temperature, 
t = temperature of steam at reduced pressure. 



Formula; 



_, ^H^h^H»iT-t) 



M=ioa 



Hl 



Determine M when H = 1195.9, h = 1150.4, F, = .47, T =370.8° R, 
Hi, = 852.7, ^=212° F. 



Digitized by 



Google 



37 



INTRODUCTION 



57 



66. Percentage Composition. The per cent of an element 
in any compound equals 100 times the weight * of the element, 
divided by the molecular weight of the compound. 

The molecular weight of a compound equals the sum of the 
weights of all the elements in the compound. 

Formulate both laws, and in the first, substitute for moleculai 
weight from the second. 

67. Compute the per cent of copper, Cu, in crystallized copper 
sulphate, CUSO4+5H2O, arranging the work as follows under 
the problem heading and formula: 

fk 

Weight tCu = 

04= 

" . 5H2 = 
50 = 



Molecular weight = 

68. Formulate, compute, and tabulate 
in the work-book the percentage composi- 
tion of such substances as may be 
assigned in Table I., page 58. 

Tabulate both problems and results. 

69. Specific Gravity by Twaddell's 
Hydrometer. The specific grayity of a 
liquid heavier than water, equals .005 
times the reading on Twaddell's hydrom- 
eter Tw, plus 1. 

Compute the specific gravity of a 
liquid in which Twaddell's hydrometer 
stands at 56. 

70. A liquid is to be prepared having 
Fig. 53. a specific gravity of 1.30. Fig. 54. 
Twaddell What will be the Twaddell reading Baum6 

Hydrometer, when this point is reached? Hydrometer. 

* The weight of an element in a compound equals the weight of the 
atomB of the element in the compound. 

t Refer to the latest list of International Atomic Weights. 




Digitized by 



Google 



68 



TECHNICAL ALGEBRA 



87 



Table I. 
PERCENTAGE COMPOSITION 

Computed from International Atomic Weights for Year.. 



No. 



Substance. 



Name. 



Per Cent of Element. 



El. % El. % El, 



MnOa 

HgO 

KCIO, 

K,S04 

KMnO* 

MgS04 

H2O 

KOH 

HCl 

H,S04 

ZnSO* 

ZnCl, 

HNO, 

NH, 

KNO, 

CaCO, 

C,H204 

NaCl 

KjCfjOt 

AgNO, 



Manganese peroxide . . . . 

Mercuric oxide 

Potassium chlorate 

Potassium sulphate 

Potassium permanganate 
Magnesium sulphate . . . . 

Water 

Caustic potash 

Hydrochloric acid 

Sulphuric acid 

Zinc sulphate 

Zinc chloride 

Nitric acid 

Ammonia 

Potassium nitrate , 

Calcium carbonate. ... 

Oxahc acid 

Sodium chloride 

Potassium bichromate. . 
Silver nitrate 



O 



Mn 

Hg 

K 

K 

K 

M 

H 

H 

H 

H 

Zn 

Zn 

H 

H 

K 

Ca 

H 

Na 

K- 

Ag 



CI 

S 
Mn 

S 

K 

CI 

s 
s 

CI 

N 
N 
N 
C 

c 

CI 
Cr 

N 



71. A solution is to be evaporated until the reading is 55 Tw. 
Compute the specific gravity when this point is reached. 

72. Specific Gravity by Baum6's Hydrometer. 

(a) The specific gravity of a liquid heavier than water at 15.55** 
C. equals 145 divided by the difference between 145 and the Baum6 
reading. 

(6) The specific gravity of a liquid lighter than water at 15.55** 
C. equals 140 divided by the sum of 130 and the Baum6 reading. 

Write both formulas. 

73. Compute the specific gravity of a solution heavier than 
water, which reads 55 B6. 

74. When sulphuric acid has a specific gravity of 1.842 what 
will be the Baum6 reading? 



Digitized by 



Google 




37 INTRODUCTION 59 

75. Compute the Baum6 reading for a liquid whose specific 
gravity is .95. 

76. In a liquid lighter than water at 17.5** C, 
the reading is 45 B6. 

Compute the specific gravity of the liquid. 

77. Specific Gravity by Pyknometer. The 
specific gravity of a liquid by the pyknometer may 
be computed from the following law: 

Weight of bottle and liquid —weight of bottle 
Weight of bottle and water —weight of bottle* 

Simplify the second member of the equation, and j, -- 

formulate both the original equation and the simpli- ^ ' 

^ J ,. Pyknometer 

fied equation. or Specific 

78. Volume of Dry Air. Mendel6eff gives the Gravity 
following formula for the volume of dry air at any Bottle, 
temperature and pressure, when saturated with watery vapor: 

^D= p , 

in which V^ = volume of dry air in cubic centimeters, 

Vs = saturated volume of air in cubic centimeters, 
P = barometric pressure of saturated air in millimeters. 
Pa =. barometric pressure of the aqueous vapor in milli- 
meters at the given temperature, Centigrade. 
Copy the formula and write the law. 

79. Compute the volume of dry air in a- saturated volume of 
52.4 cc at 15.3° C under a pressure of 748.5 millimeters, the pres- 
sure of aqueous vapor at this temperature being 12.9 millimeters. 

80. Dry Volume at 760 mm. and 0**. The volume of dry 
air in cubic centimeters under a barometric pressure of 760 milli- 
meters at a temperature of 0° C, equals the moist voliune 
times the given pressure in millimeters divided by 760, times 273 
divided by the sum of 273 and the given temperature. 

Formulate and compute the dry volume in problem 79 at 760 
millimeters and O''. 



Digitized by 



Google 



CHAPTER II 

RESOLUTION AND COMPOSITION OF FORCES 

Section 1, Projection. Section 2, Graphical Resolution. 
Section 3, Resolution by Computation. Section 4, Func- 
tions OP AN Obtuse Angle, and Examples. Section 5, 
Resultant. 

" Whenever an idea is constantly recurring the best which can be done for the 
perfection of language and consequent advancement of knowledge is to shorten 
as much as possible the sign which is- used to stand for that idea." 

Db Morqan. 

§ 1. PROJECTION 

38. Definition. Under headings draw this figure to an 
enlarged scale in the work-book. 




Fia. 66. 

From R draw a perpendicular to OA^ terminating in 
OA at V. Then OV is the right-angled or orthogonal 
projection of OR on OA. 

Specify this under the figure. 

Draw the same figure again in the work-book and from 
A draw a perpendicular to OR, tenninating at C 

What then is OC? (Write answer immediately under 
the figure.) 

60 



Digitized by 



Google 



38 RESOLUTION AND COMPOSITION OF FORCES 61 
Draw Fig. 57 to an enlarged scale in the work-book. 



Fia. 57. 

From R and L draw perpendiculars to AB. 
The segment of AB lying between the feet of these 
perpendiculars is the projection of LR on AB. 
Specify this under the figure. 

Observe that in orthogonal projection perpendiculars 
are dravm to the line on which projection is to be made. 

Draw the figure again in the work-book and project AB 
on RL. 

Specify the projection. 

Draw Fig. 58 to an enlarged scale in the work-book and 
draw perpendiculars to AB from and R, terminating in 
AB at V and T. 



Flq. 58. 

Then VT is the projection of OR on AB. 
Specify the projection. 



Digitized by 



Google 



62 



TECHNICAL ALGEBRA 



39 



Draw Fig. 59 to an enlarged scale in the work-book and from 

R draw a perpendicular to OA 
produced, terminating at V. 

Then OV is the projection 
oiORonOA. 

Specify the projection. 

Write the definition of the 
orthogonal projection of one 
line on another. 

39. Examples. In the following examples draw each 
figure to an enlarged scale and make both projections as 
in the work required for Fig. 56, specifying the projection 
under each. 




\ 




Digitized by 



Google 




41 RESOLUTION AND COMPOSITION OF FORCES 63 



§ 2. GRAPHICAL RESOLUTION OF FORCES 

40. Components. A force is represented graphically 
by a straight line whose length equals the magnitude of the 
force and whose direction is the same as the direction of 
the force. 

In the figure / represents a force of 8 pounds acting on 
W in the direction shown 
by the arrow at the angle 
of 30° with the hori- 
zontal. 

Copy the figure to 
an enlarged scale in the 
work-book, making / 8 
units in length, and ' 
project / on a horizontal Fia. go. 

line through the point 
of application of the force on TF. 

The projection is the horizontal component of / and the 
perpendicular is the vertical component of /. In technical 
language, / has been resolved into horizontal and vertical 
components. 

By the scale determine the magnitude of each component 
in pounds. 

What therefore is the vertical pull on Wi 

What is the horizontal pull? 

41. Axes of Reference. In the graphical determination 
of components and of the resultant effect of forces acting at 
the same point, it is customary to denote the horizontal 
component by x and the vertical component by y. This 
S3mabolisnx ori^nated in the consideration of a force as 
acting at the point of intersection of a vertical and a horizontal 
line, called axes of reference. The horizontal is denoted 
conventionally by a capital X at the right extremity and 



Digitized by 



Google 



G4 



TECHNICAL ALGEBRA 



42 



+ 



Fig. 61. 



the vertical by a capital Y at the upper extremity as shown 
in the figure. The point of intersection is called the origin. 
The projections of forces on the X axis are called X com- 
ponents and the pro- 
Y jections of forces on 

the Y axis are called Y 
components. 

X components are 
denoted by a;; Y com- 

X ponents are denoted 

by 2/. 

As shown on the 
figure, X components 
are positive * to the 
right of the Y axis. 

Y components are 
positive above theX axis. 
X components are negative * to the left of the Y axis. 
Y components are negative below the X axis. 

42. Examples. As assigned, draw axes of reference, rep- 
rezent graphically the forces in the following table, draw 
vertical and horizontal components, measure with the scale, 
determine and fill in the omitted entries in the table in 
which 

F = the force in pounds, 
^l=rthe angle at which it acts with the horizontal, 
y = the vertical component or projection on the Y axis, 
a: = the horizontal component or projection on the X 
axis. 

♦ Positive numbers are indicated by a plus sign; negative, by a 
minus sign. 

t All angles are read counter-clockwise from the right segment of 
the X axis. 



Digitized by 



Google 



43 EESOLUTION AND COMPOSITION OF FOECES 65 

Table II 
RESOLUTION AND COMPOSITION OF FORCES 





F 


A 


X 


V 


No. 


F 




X 


V 


No. 


+ 


- 


+ 




A 


+ 


- 


+ 


- 


1 


5 


20 










11 


130 


85 










2 


10 


40 










12 


140 


95 










3 


15 


45 










13 


150 


115 










4 


20 


50 










14 


165 


125 










5 


40 


55 










15 


170 


145 










6 


60 


60 










16 


180 


235 










7 
8 


70 
90 


65 
70 










17 
18 






3 


6 


4 


8 






9 


100 


75 










19 






100 




50 




10 


120 


80 










20 






80 






40 











§ 3. RESOLUTION OF FORCES BY COMPUTATION 

43. The Ratios of a Triangle. Suppose the Une OY to 
revolve counter-clockwise about the point 0, in the plane 
of the pdper, and let 07' be the original position of OY^ 
6 * being the angle of revolution in degrees. 




If OV is projected as shown, a triangle is formed whose 
sides are named as follows: 

± perpendicular, 
Proj projection, 
RY radius vector. 
♦ A letter of the Greek alphabet, pronounced theta. 



Digitized by 



Google 



66 



TECHNICAL ALGEBRA 



44 



The relations of these sides to each other are commonly 
expressed in six ratios, of which only three will be considered 
here. 

These are named, sine dj cosine 9^ and tangent 6. 

They are defined and abbreviated as follows: 



(1) sin0 = 



RV 



(2) cos0=^ 



(3) tan^= 



Proj 



or 



or 



or 



opp 

byp' 

adj 

hyp' 

opp^ 
adj* 



Their value is always the same for a given angle, regard- 
less of the length of RV. 

44. Examples. Rule the following table in the work- 
book. By reference to the table of natural trigonometric 
functions determine and enter the sine, cosine, and tangent 
of the angles to three decimal places: 



Table III 
NATURAL TRIGONOMETRIC FUNCTIONS 



No. 


d 


sin 


cos 


tan 


No. 


e 


Bin 


COB 


tan 


1 


10° 








11 


18° 








2 


20° 








12 


23° 








3 


30° 








13 


31° 








4 


40° 








14 


42° 








5 


50° 








15 


54° 








6 


60° 








16 


35° 








7 


70° 








17 


65° 








8 


75° 








18 


61° 








9 


45° 








19 


12° 








10 


25° 








20 


27° 









Digitized by 



Google 



46 RESOLUTION AND COMPOSITION OF FORCES C7 

46. Law of Components. Copy each equation in 
paragraph 43 and clear it of fractions, observing the con- 
vention that a multiplier of a sine, cosine, or tangent, is 
written preceding and not following these functions. 

The equations obtained by clearing (1) and (2) of fractions 
may be used as formulas for computing the x and y com- 
ponents of a force of unknown magnitude and direction. 
If so used 

The perpendicular represents what component? 
The projection represents what component? 
The radius vector represents what? 
The Y component = the force times what? 
The X component = the force times what? 

Write a formula for the Y component; also a formula 
for the X component, denoting the force by F and the compo- 
nents by y and x. 

46. Examples. By reference to the 4-place table of 
natural trigonometric functions compute and fill in the 
omitted entries in the following table ruled in the work- 
book, in which 

F denotes the force, 
d the angle at which it acts to the horizontal (read 

counter-clockwise) , 
y the vertical component, 
X the horizontal component. 

Enter sines and cosines to three decimal places, with 
the third decimal figure increased by unity whenever the 
fourth figure is 5 or greater. 



Digitized by 



Google 



68 



TECHNICAL ALGEBRA 



47 



Table IV 
RESOLUTION BY COMPUTATION 













V 


X 


No. 


F 


B 


sin 


cos 




















+ 


- 


. + 


- 


1 


22 


32° 














2 


25 


30° 














3 


75 


35° 














4 


110 


38° 














5 


115 


41° 














6 


122 


50° 














7 


214 


54° 














8 


250 


56° 














9 


300 


90° 














10 


1050 


45° 














11 


55.4 


31.5° 














12 


GO. 5 


36.4° 














13 


12.5 


43.2° 














14 


11.75 


51.1° 














15 


450 


62° 














16 


67.2 


29° 














17 


98.3 


33° 














IS 


160 


51° 














19 


78.4 


63° 














20 


36.6 


25° 















I 



§ 4. FUNCTIONS OF AN OBTUSE ANGLE 

47. Signs. Suppose the line OV to have rotated from 

its original position OT 
until 6 the angle of rota- 
tion is obtuse. 

Draw the figure and 
project OV on OT pro- 
duced through the point 
of revolution. 
Fia. 63. Is the projection posi- 

tive or negative? Why? 

Is the perpendicular positive or negative? 

(The radius vector is positive in any position.) 




Digitized by 



Google 



48 RESOLUTION AND COMPOSITION OF FORCES G9 

On the figure in the work-book prefix the proper signs 
to the symbols for projection, perpendicular, and radius 
vector. 

Then sin d=what? 

cos d=what? 
tan d=what? 

The law of signs for fractions is as follows: 

When the numerator and denominator have unlike signs, 
both may be written + provided the sign before the fraction 
is changed. 

Apply this law to two of the preceding equations. 

Therefore is the sine of an obtuse angle, positive or 
negative? 

Is the cosine of an obtuse angle, positive or negative? 

Is the tangent of an obtuse angle, positive or negative? 

Explain fully each of your three answers. 

Therefore when the sine of an obtuse angle is used in 
computation, what sign is understood? 

When the cosine of an obtuse angle is used from the 
table, what sign should be prefixed? 

When the tangent of an obtuse angle is used from the 
table, what sign should be prefixed? 

Will you remember this when using these functions? 

48. How to Read Functions of an Obtuse Angle. The 
3Upplement of an angle is the difference between ISO*' and 
the angle. 

Therefore the supplement of 110^ is 180^-110^=70^. 

Draw a figure in your work-book like that in paragraph 
47 and denote by S the angle which is the supplement of 0. 

Under the figure state the relation of S to d. 

Is S an acute or an obtuse angle? 

sin S = what? sin ^= what? 

cos S=what? *cos ^=what? 

tan S = what? *tan = what? 

* Don't forget what you promised to remember. 



Digitized by 



Google 



70 



TECHNICAL ALGEBRA 



49 



Therefore the functions of an obtuse angle equal nu- 
merically the corresponding functions of an angle in what 
relation to the obtuse angle? 

49. Examples. Determine and record the omitted entries 
in the following table: 

Table V 
NATURAL FUNCTIONS OF AN OBTUSE ANGLE 



No. 


A 


sin 


COB 


tan 


No. 


A 


sin 


cos 


tan 


1 


100** 








21 


96** 








2 


105** 








22 


107** 








3 


98** 








23 


118** 








4 


110** 








24 


123** 








5 


115** 








25 


180** 








6 


120** 








26 


111** 








7 


125** 








27 


126** 








8 


130** 








28 


136** 








9 


135** 








29 


124** 








10 


140** 








30 


144** 








11 


145 ** 








31 


134** 








12 


148** 








32 


149** 








13 


150** 








33 


119** 








14 


152** 








34 


129** 








15 


155** 








35 


159** 








16 


160** 








36 


146** 








17 


162** 








37 


154** 








18 


165** 








38 


169** 








19 


168** 








39 


173** 








20 


94** 








40 


170** 


i 







50. Examples. Solve the following equations: 



1. c = 



118 
tan 51° 



2. sinL = 



18 
3. sin 50°=—. 

6. cos50°=~. 

X 



1728 
1893* 



4. 63 sin 40° = 



32.04' 



6. cosL = 



1728 
1893' 



Digitized by 



Google 



63 RESOLUTION AND COMPOSITION OF FORCES 71 

7. 63 cos 40° =r:^. 8. tan 50° =— . 

32.04 X 

9. tan L =^. 10. 63 tan 40° = - ^ 



1893' * 32.04* 

- .oo 14 ^^ 396 tan 51° , 

11. 638=-^ ', 12. -— =5c. 

tan B 792 

13. sin il tan il = Jc. il = 76°. 

^^si^^ 27806 ^^^^„ 
tan P 5 cos F 

16. 535.7c =|(cos2* /2+sin« R), 
o 

16. 98 cos S =.8476 sin S. S =30'. 

17. «iE4 =2.724. 

COS 5 

18. 34.1 tan 7 = 129 times ^. 

lb 

19. ^^=429.8c. «=60«. 

COS d 

20. 17.46 sin 29° = 1.7966 tan 84° 

^^ 200 tan 104° ^^^^ ^^^^ 

21. — . ,^,^ =5606 cos 104°. 

sin 104° 

22. -608sinl22°=-|-tanl22°. 

o 

^ sin 130° 

23. rrTB = -3.14c. 

cos 130 

24. -5.29 tan 114i°=2.5(l+cos 130°)+c. 

6 tnn 40° 

26. 3 sin« 121°+3 cos« 121° = ^,^^ . 

342 

cos 34°+sin 56° 



tan 20° 



=6 tan 134°. 



* Determine the value of this sum from a right triangle by writing 
the value of sin* and cos*, and adding the two equations. 



Digitized by 



Google 



72 TECHNICAL ALGEBRA 60 

27. 3.21 sin 123° ^~ tan 34^ 

o 

„„ -7 tan 141° „„, 
^- sin 82° ''•^'- 

29. 12.9cos34J° = -atanlllJ°. 

^^ -34.6 cos 100° ,^^, . ^,« 

30. jy^ =1726 sin 671°. 

31. -81.2c cot Mer^^^'""^' 



4 cos 84°' 

32. -8(cos« ^+sin2 6) =64c tan 112J°. 

„„ 7.25 tan 81° ,_ 

33. -— — = 190c tan 81°. 

1.25 cos 52° 

56 1 

34. j-^cotl54°=-116i2. 

QQ 1 

36. -78.2(sin2 a+cos* «)c =^ tan 115°. 

it 

36. -450(cos« i2+sin2 R) J- f ^'"^ ^^° 



8 2 cos 98°* 

37. 41.8 = -129c cot 171° sin 81°. 

38. 5 tan 24J° sin 42^° = 125c. 

39. 12.6 cot 114° tan 28J° = -.63 h ^^^ ^f i° 

2 cos 28J 

^^ 6 tan 108° 
^•l8^^U02-°=^^^«^^- 

41. 200 cos 48J° = - 16f c tan 94i°. 

12 cos 24° h 



lo' 



42. -122 cos 118° tan 118° = 

43. -31.8 tan 123° = 



4 tan 112°* 
tan 12° sin 29°+c 



cot 78° 

♦ cot=— . 
tan 



Digitized by 



Google 



50 RESOLUTION AND COMPOSITION OF FORCES 73 
18.4 tan 44° cos 81° 



.135c 



44. -7.9 cot 141° = - 

^^ 8 cos 85° „^^ . ^„ 
46. T7- — —3 =8.26 sin 5°. 
12 tan 45 

46. -94.6a+12co8l8r= " ' > 

cos IIOJ 

47. y =7.22 cos 74° tan 118°+78.4 sin 85°. 

^^ 155 sin 45*^ ,„ ^ , ^^o . ^^o 

48. "^ --^=17.5 tan 75° cot 50°. 

S cos 45 

49. 18x =9.62 tan 134° cos 18°+ 104 tan 30° 
18.75 



60. 



9.875(sin»^+cos«^)'"''*^''^^i°: 



61. 146(cos« ^+sin2 ^)+.1896 -^^ =3468.8. 

.u4o 

5.867 sin 39° 14- 96.785 

■ .6985 cos 39° 14' sin' a+cos« a" 

7Qfi 5 S 
64. 348.5 tan 81° 33'+sin 119° 38' = ' ,^^ . 

9 tan 45° 

^^ 8.283 cos 125° 41' 1267.005 tan 142° 5' 
• .00466+68.21 ~ 15 

12.785sin68°24'-1.005 
^^- .0812 tan 15° 16' -63.08 = 185.0096. 

^„ 34.56 sin 165° 19'+5 tan 83° 4' ,„^, / ^^ 

57. .fc^t:/ • ^ . o ^N = 1296.4 cos 6°. 

.1575(sin2 <^+cos2 <t>)a 

68. .054(17.5 sin 115° 29'+8Z) cos 43°) ^-^ f'^^^^lTo - 

24.2 cos 75 

,^ 39.604 tan 143° 31'+5.0954 cos 12° 19' , . ,,„,, 
^^- 73.81+43.9 tan 128° 10' =^^^^^^ ^^ 

785.91 sin« 28° -51 cos 140° 25' 

8A tan 71° 18' -1.8005 "l'-^^- 



Digitized by 



Google 



74 TECHNICAL ALGEBRA 61 



§ 6. RESULTANT 

61. Square of the Hypotenuse. As accurately as pos- 
sible draw a right triangle with the sides forming the right 
angle equal to 3 and 4 inches respectively. 

Measure the hypotenuse and determine and state as 
an equation, the relation of the square of the hypotenuse 
to the sum of the squares of the other two sides of the tri- 
angle. 

Draw another right triangle having one side extending 
neariy the full width of the page. 

Denote its sides on the figure by c, a, and 6, c being the 
hypotenuse. 

By measurement and computation, determine and formu- 
late the relation of <? to a^+ft^ 

Therefore, what seems to be the law for the square of 
the hypotenuse of a right triangle? 

62. Law of Resultant. The resultant of two or more 
forces is the amount of their combined effect in pounds. 

When two forces act at the same point on a body at an 
angle of 90® with each other, their resultant is the hypotenuse 
of the right triangle whose other two sides are the two given 
forces. 

To what therefore, is the square of their resultant equal? 

To what is their resultant equal? 

Illustrate by a dimensioned diagram. 

If two forces act on a body at any angle with each other, 
their resultant may be represented by the diagonal of a 
parallelogram constructed on the forces as adjacent sides, 
as shown in figure 64. 

Draw the figure in the work-book and by the law for 
square of the hypotenuse write the value of R^ in terms 
of h and {b+v). 



Digitized by 



Google 



63 RESOLUTION AND COMPOSITION OF FORCES 75 

But v = a times what? 

In the formula for R^ substitute this value of v and 
square the quantity within the parenthesis. 
By the law for square of hypothenuse, 

/i2 = what in terms of a and v? 

In the formula for R^ substitute this value for h^. Collect 
in the second member after substituting for ir^ in terms of a 
and 6. 




Fig. 64. 

If you have made no mistake in your work the formula 
will now read, 

R2=a^+V^+2ab cos 6. 

In this formula, R is the symbol for what? 
a is the symbol for what? 
b is the symbol for what? 
d is the symbol for what? 

By reference to the formula write the law for the square 
of the resultant of two forces acting at any angle with each 
other at the same point on a body. 

63. Examples. Compute and record the omitted entries 
in the following table in which the forces specified are sup- 
posed to act as indicated in paragraph 52. 



Digitized by 



Google 



78 TECHNICAL ALGEBRA 66 

66. Direct Variation. Direct variation is a relation of 
quantities in which the ratio of any two values of one quan- 
tity equals the direct ratio of the two corresponding values 
of the other quantity. 

In other words it is a relation of quantities in which the 
value of one of the quantities over a second value of the 
same quantity equals the value of the second quantity 
corresponding to the first value of the first, over the value 
of the second corresponding to the second value of the first. 

If BocP, by the definition of direct variation 

Bi* Pi 



B2 P2' 

in which Bi and B2 are any two values of JB, and Pi and P2 
are the two corresponding values of P. 

Observe that no variation expression can be used for 
purposes of computation until transformed by the definition 
of variation into a proportion and therefore into an 
equation. 

Apply the definition to the following: 

1. LocC. 3. Xocy. 

2. QocS. 4. Ao^R^. 

In expressing direct variation the word direct or directly 
i^ omitted as here illustrated. 

66. Indirect Variation. Indirect or inverse variation is a 
relation of quantities in which the ratio of any two values 
of one quantity equals the inverted ratio of the two cor- 
responding values of the other quantity. 

When two quantities vary inversely y therefore, one varies 
as the reciprocal of the other, 

* Read B one over B two. 



Digitized by 



Google 



67 VARIATION 79 

If V varies inversely as P, we write Foe—-, and by the 
definition of inverse variation, obtain the equation 

7l^P2 

F2 Pi' 

Observe that inverse variation is denoted by inverting 
one of the ratios. The proportion may be stated in words 
as follows: 

The ratio of any two values of V equals the inverse 
ratio of the two corresponding values of P. 

Apply the definition of inverse variation to the following: 

1. Ta^?- 2. /ioc-l. 3. Foci-. 

W A H 

4. loc-^. 5. Cocij. 6. Fcc^. 

67. Arrangement of Work. The following illustrates the 
arrangement of all work in variation: 

The thickness of materials of the same rectangular 
section area varies inversely as their width. 

A piece of rectangular section is 2f inches wide and f 
inch thick. 

Compute the thickness of a piece 3| inches wide and of 
the same rectangular section area. 

(a) reel. T^^^^. 

\^) ^2 Wi ^^ % 2, • 

* The preceding equation was solved for Tj, because Tj is unknown, 
as indicated by the data. 



Digitized by 



Google 



80 


TECHNICAL ALGEBRA 




(3) T2=^. W2^3h"~ 




W 12 8^^^^^-4Q • 




(5) T2 = A75" Thickness of Second Piece. 



68 



68. Problems, Solve the following problems in the 
work-book under problem number and lettered title with 
formulas and solution to the left and data near the right 
margin as shown in the preceding model. 

Reduce mixed numbers in the data to fractional form as 
indicated. 

In substituting in the formulas write fractional divisors 
as such; then write them as inverted multipliers. 

The order of work should be as follows: 

At the left enter problem number and title, formula of 
variation, and the proportion. 

At the right, enter in a vertical column each quantity 
of the proportion followed by an equality symbol and the 
data. 

1. Weight. The weight of a substance varies as its volume. 
A steel bar containing 104 cubic inches weighs 466 ounces. 
Compute the weight of a bar of the same material containing 

500 cubic inches. 

2. Circumference. A circle SJ inches in diameter has a circum- 
ference of 26.7 inches. 

Compute the circumference when the diameter is 14 J inches, 
the law of variation being that the circumference varies as the 
diameter. 

3. Velocity of a Falling Body. The velocity of a falling body 
varies as the time during which it is falling. 

When a body falls from rest, its velocity at the end of 1 second 
is approximately 32 feet per second. 

Compute its velocity at the end of 12 J seconds. 



Digitized by 



Google 



68 



VARIATION 



81 



4. Space Traversed. The distance through which a body 
falls from rest varies as the square of the time diuing which the 
body is falling. 

In one second a body falls from rest a distance of 16 feet. 

In how many seconds will it fall 185 feet? 

6. Time and Velocity. In the sunmier of 1908 a baseball 
dropped from a window of the Washington monument was caught 
at the base of the monument about 504 feet below. 

In how many seconds did the ball strike the catcher's hands 
4 feet above the ground? 
, What was its velocity in feet per second when caught? 

6. Relation of Time to Distance. When the speed is un- 
changed, the time required for a train to cover any distance, varies 
as the distance. 

The Pennsylvania Special and the Twentieth Century Limited 
for several years made the run between New York and Chicago 
in 18 hours, the distance being 908.7 miles by the Pennsylvania 
Road and 978 by the New York Central. 

If this rate were maintained across the continent, in what time 
would these trains make the run from New York to San Francisco 
if both used the shortest rail route of 2275 miles from Chicago? 

7. Velocity of a Pulley. The velocity of the ,rim of a pulley 
varies as its diameter. 

A 16-inch pulley has a rim velocity at a certain moment, of 155 
feet per minute. 

What is the rim velocity at the same moment, of a 9i inch 
pulley which is keyed to the same shaft? 

8. Shearing Stress and Resist- 
ance of Rivets. The shearing stress 
of a rivet varies as the square of its 
diameter. 

Solve the formula for di. 

The resistance of a rivet to crush- 
ing varies as its diameter. 

Solve the formula for Ri and 
for (fc. Fig. 66. 




Digitized by 



Google 



82 



TECHNICAL ALGEBRA 



58 



9. Pitch of Rivets. The number of rivets required for a 
boiler seam varies inversely as the pitch (the distance between 
rivet centers). 




Fig. 67. 

If 40 rivets are required when the pitch is 2| inches, determine 
the pitch when 41 rivets are required for a boiler of the same size. 

10. The Volume of a Gas. The volume of a gas varies in- 
versely as the height of the mercury in the barometer. 

The volume is 23^ cubic inches when the barometer registers 
29.4 inches. 

What is the volume when the barometer registers 30.7 inches? 

11. Weight. The weight of a body varies inversely as the 
square of its distance from the center of the earth. 

If a substance weighs 40.8 pounds at sea level (3960 miles 
from the center), compute its weight when on top of Mt. Everest, 
29,000 feet* above sea level. 

12. Generation of Hydrogen. In the generation of hydrogen 
the amount of sulphuric acid (H2SO4) varies as the amount of 
zinc (Zn). 

* See National Geographic Magazine, June, 1909, p. 497. 



Digitized by 



Google 



68 



VARIATION 



83 



For complete action 65 grams of Zn require 98 grams of H2SO4. 
How many grams of H2SO4 will be required for 100 grams of Zn? 




Fig. 68. 

13. Generation of Hydrogen. The amoimt of hydrogen 
produced varies as the amount of zinc. 

If 65.4 grams of zinc produce 2.016 grams of hydrogen, find the 
amount of zinc required to liberate 75 grams of hydrogen. 

14. Area of a Circle. The area of a circle varies as the square 
of its diameter. ^^ni^ji^ ' 

When the diameter is 6f inches the area is (approximately) 32 
square inches. 

What is the radius when the area is 56 square inches? 

15. Volume of a Sphere. The volume of a sphere varies as 
the cube of its diameter. 1^ 

The volume is 1437.3 cubic inches when the radius is 7 inches. 
Compute the volume when the radius is doubled. 

16. The Resistance of a Wire. The resistance of a wire varies 
inversely as the square of its diameter. 

The resistance of a coil of copper wire A of an inch in diameter, 
was 2.5 ohms. 

What is the resistance of a copper wire of the same length, with 
a diameter of f of an inch? 

17. Generation of Oxygen. In the preparation of oxygen 13} 
grams of mercuric oxide yield 1 gram of oxygen. 

If the amount of oxygen produced varies as the amount of 



Digitized by 



Google 



84 



TECHNICAL AiXiEBRA 



mercuric oxide, compute the number of grams of mercuric oxide 
which will produce 150 grams of oxygen. 






a w (^ o ^ 




Fia. 69. 

18. The Hydratilic Press. In a hydraulic press the distances 
through which the pistons move vary inversely as the areas of the 
pistons. 

Through what distance does the power piston 2J inches in 
diameter, move, when the weight piston 2 feet in diameter moves 
through a distance of 2.8 inches. 

19. Power of a Hydraulic Press. In a hydrauUc press the 
ratio of the weight to the power applied equals the ratio of the 
piston areas. 




Fig. 70. 



Digitized by 



Google 



68 



VARIATION 



85 



If the weight piston has an area of 214 square inches and the 
power piston an area of 4A square inches, what power will be 
required to lift a weight of 760 pounds? 

20. Penetration of Armor. The depth of penetration of armor 
by a projectile varies as the caliber of the gun. 

If an 8-inch gun penetrates to a depth of 4} inches, compute 
the penetration of a 14-inch 
gun in the same armor at 
the same distance. 

21. Strength of the Cur- 
rent. The current varies as 
the tangent of the angle of 
deflection (tan 5). 

The current from a bat- 
tery deflects the needle of 
a tangent galvanometer 10°; 
with a second battery, the 
deflection is 25°. 

Compute the ratio of the 
strengths of the two cur- 
rents. 

22. Law of the Lever. 

In a lever of whatever class 
the power is to the weight, 
as the weight's distance 
from the fulcrum is to the 
power's distance from the 
fulcrum. 

A lever 8^ feet long has the fulcrum 9 inches from one end. 

What force in pounds applied at the long end will balance a 
weight of 480 pounds, 2 inches from the short end? 

23. Height in Communicating Tubes. The heights of liquids 
at equilibrium in a U-tube vary inversely as their densities. 

The height of the water in one of two communicating tubes is 
14i inches. 

Compute the height of the mercury in the other tube, mercury 
being 13.6 times as heavy as water. 




Fia. 71. 
Tangent Galvanometer. 



Digitized by 



Google 



86 



TECHNICAL ALGEBRA 



58 



24. Weight of a Gun. The weight of a gun varies approxi- 
inately as the cube of its caliber. 
An 8-inch gun weighs 14.2 tons. 
Determine the weight of a 12-inch gun of the same tjrpe. 

25. Charles' Law. Under constant pres- 
sure, the volume of a gas varies as the abso- 
lute temperature.* 

A gas with a volume of 90 cc. at 10° C. 
is heated to 35° C. 

Compute its volume at the higher tem- 
perature, the pressure being the same at both 
temperatures. 

26. Osmotic Pressure. The osmotic 
pressure of a solution varies as the molec- 
ular mass M of the substance dissolved. 

Solve the formula for Mi. 




Fig. 72. 
Osmosis. 



27. A Falling Body. The distance 
through which a body falls from rest varies 
as the square of the time during which it is falling. 



A body falls 175.6 meters in 6 seconds. 
How far will it fall in J of a second ? 

28. The Lever. The distances from the fulcrum of a lever, of 
two weights which are in equilibrium, vary inversely as the weights; 
that is, the distance varies inversely as the weight. 

At the ends of a lever 10 feet long are two weights of 120 and 
84 pounds respectively. 

At what distance from the center must the fulcrum be placed in 
order that the weights may balance? 

See paragraph 37, problem 16. 

29. Oscillation of a Pendulum. The number of oscillations 
of a pendulum per time unit varies inversely as the square root of 
its length. 

♦Absolute temperature =273°-}- the recorded temperature Centi- 
grade. 



Digitized by 



Google 



68 VARIATION 87 

The length of a pendulum which beats seconds at the sear 
level, is 39.1 inches. 

How long is a pendulum which oscillates 3 times a second? 

30. Apparent Size. The apparent size of an object varies 
inversely as its distance. 

Solve the formula for Di. 

31. Flow of Gas in Pipes. The volume of gas discharged from 
a horizontal pipe under constant pressure and specific gravity, 
varies inversely as the square root of the length of the pipe. 

The volume in 3 J hours from a pipe 600 feet long was 1750 
cubic feet? 

Find the volume discharged per hour from a mile length of the 
same pipe. 

32. Vibration of Strings. The number of vibrations per time 
unit, of a string under constant tension, varies inversely as the 
length of the string. 



^i^ 



^ 




Fig. 73. 

When the length is 4 feet 8 inches, the string vibrates 256 times 
per second. 

. What must be the length of a string of the same material and 
diameter, in order that the vibrations may be doubled? 

33. Relation of Mass * to Velocity. The velocity imparted 
to a body varies inversely as its mass. 

A given force imparts a velocity of 75 feet per second to a 
body whose weight is 644 pounds. 

What velocity will the same force impart to a body whose mass 
is 120? 

34. Linear Velocity. Linear velocity varies directly as areal f 

* » * weight in lbs. 

* Mass = — . 

32 

t Areal velocity equals the rate at which an area increases. 



Digitized by 



Google 



88 



TECHNICAL ALGEBRA 



68 



velocity and inversely as the distance from the center of force to 
the line of motion. 

Solve the formula for D2. 

35. Linear Velocity. The linear velocity of a particle per 
time imit varies directly as the radius of revolution and inversely 

as the time of one revolution. 
Solve the formula for Ri. 

36. Angular Velocity. The 

angular velocity (co) of any 
point in a revolving body 
varies directly as the areal 
velocity and inversely as the 
square of the radius vector (R). 
Solve for R2, 

37. Centrifugal Force. 

The centrifugal force at any 
point in a revolving body 
varies as the square of the 
linear velocity. 

Solve the formula for Fx,j. 

38. Vibration of Light. 
The number of vibrations of 
light per second varies in- 
versely as the wave length L. 

Solve the formula for L2. 

39. Intensity of Heat. 
The intensity of heat H re- 
ceived from a radiating body 
varies inversely as the square 
of the distance from the source 
of heat. 

Solve the formula for H2. 




Fig. 74. 
Boyle's Law Apparatus. 



40. Boyle's Law. Under 
constant temperature the vol- 
ume of a gas varies inversely as the pressure. 

If a gas have a volume of 106 cubic inches under a pres- 



Digitized by 



Google 



5d VARIATION 89 

sure of 14.7 pounds, compute the volume when the pressure 
is 15.3 pounds more, 

41. Bending of a Bar. The amount a bar will bend varies 
inversely as the width when the thickness and the length 
remain the same. 

If a bar 3.5 inches wide bend through5° under theapphcationof a 
given force, compute the amount of bending of a bar 5.1 inches wide, 
of the same material, length, and thickness, imder the same force. 

42. Centrifugal Force. The centrifugal force at any point 
of a revolving body varies as the radius of revolution of the point. 

The centrifugal force is 900 pounds when the radius is 24 inches. 
Compute the force when the radius is 18 J inches. 

43. Density and Volume. TJie density of a given weight of a 
gas varies inversely as its volume. 

If the density is .108 when the volume is 4.5 cubic feet, find the 
density when the volume is reduced by pressure to 3.8 cubic feet. 

44. Internal Resistance of a Bat- 
tery. The total resistance varies ^s 
the cotangent of the angle of deflection. 

When the resistance is 9.8, the 
angle of deflection is 45°. 

Find the resistance when the angle 
is 24°. 

45. Area on a Globe. The area of 
a country on a terrestrial globe varies as 
the square of the diameter of the globe. ^^' 

On a globe 28 inches in diameter, the area of a certain island 
is 2.6' square inches. 

Compute the area of the same island on a globe 36 inches in 
diameter. 

46. Period of a Pendulum. The time of vibration (period) of 
a pendulum varies inversely as the square root of the force causing 
it to vibrate. 

Solve the formula for Fu 

47. Force of a Moving Body. The force imparted by a moving 
body varies as the mass of the body times its acceleration. 

Solve the formula for Af 2. 




Digitized by 



Google 



90 



TECHNICAL ALGEBRA 



68 



48. Area of a Triangle. The area of a triangle varies jointly* 
as its base and altitude. 

The area of a triangle whose base is 19 feet and whose altitude 
is 10 feet is 95 square feet. 

Compute the altitude when the base is 24 feet and the area is 
132 square feet. 

Check by problem 5, paragraph 33. 

49. Volume of a Cone. The volume of a cone varies jointly 
as its altitude and the square of the diameter of its base. 

The volume is 392.7 cubic inches when the altitude is 15 inches 
and the diameter of the base is 10 inches. 

Compute the diameter when the altitude is 10 feet and the 
volume 28,160 cubic inches. 

50. Weight of a Beam. The weight of a beam varies jointly as 
its length, cross-sectional area, and material. 

An iron bar 36 inches long and having a cross-sectional area 
of 1 square inch weighs 10 pounds. 

What is the weight of a wooden beam 18 feet long, whose material 
is 1^5 as heavy, with a cross-sectional area of 64 square inches? 

51. Wind Pressure on a Surface. The pressure of the wind 

perpendicular to a 
plane surface varies 
jointly as the area of 
the surface and the 
square of the wind's 
velocity. 

Under a velocity 
of 16 miles per hour, 
the pressure on 1 
square foot is 1 pound. 

What is the veloc- 
ity when the pres- 
sure on 2 square 
yards is 50 pounds? 

52. Volume of a Pyramid. The volume of a pyramid varies 
jointly as its height and the area of its ])ase. 

* " Varies jointly as " is a conventional expression meaning 
" as the product of." 




Fig. 76. 



Digitized by 



Google 



68 



VARIATION 



91 



The height- of the great p3Tamid is 481 feet. Each side of its 
base is approximately 756 feet. 

What is its vokime if the volume of a pjo-amid whose height 
is 20 feet and whose base is 9 feet square, is 540 cubic feet? 

53. Weight of an Engine Pier. The weight of a concrete 
pier varies jointly as its dimensions. 

A pier 7' by 4^' by 3f ' weighs 15400 pounds. 
What is the weight of a pier of the same concrete measuring 
10rby5rby4r? 

54. Friction. The friction between two surfaces varies j ointly as 
the perpendicular pressure between them, the coefficient of friction, 
and the area of the surface of contact. 

The perpendicular pressure between 
a journal arid its bearing is 12,000 
pounds. The coefficient of friction is .3 
and the area is 5 square inches. 

If the area were twice as great howmany 

times would the friction be increased? 

Fig 77 

55. Torque of a Magnetic Needle. 

The torque (F) acting on a needle varies jointly as its magnetic 



^'^t^:^'^^-vW'^j^ 



wmmmmM 





Fig. 78. 

moment (mZ), the horizontal intensity of the earth's magnetism 
(H), and the sine of the angle of deflection (sin 5). 
Express this law as an equation and solve for Fi. 

56. Joint Variation, z varies jointly as x and y. When a: = 1 
and2/=2, 0=4. 

Compute the value of x when 2 =30 and y =3. 

57. Inverse Variation. If x varies inversely as 2/^—2 and is 
equal to 24 when y = 10, compute x when y=5. 



Digitized by 



Google 



92 



TECHNICAL ALGEBRA 



68 



58. Velocity of Sound in any Medium. The velocity of 
sound in any medium varies directly as the square root of the elas- 
ticity of the medium and inversely as the square root of its density. 

Express this law as an equation and solve for E^ 

59. Transverse Vibration of Strings. The number of 
vibrations (n) varies directly as the square root of the tension (T), 
and inversely as the length (L) times the diameter {d) times the 
square root of the density (D) . 

Express this law as an equation and write the second member 
in its simplest form. 

60. Tensional Strength of 
Shafting. The tensional strength 
of a shaft varies as the cube of its 
diameter Z). 

Solve the formula for Di. 



D i 

1" 




Fig. 79. 



Fig. 80.— Tension Test. 



61. Volume of Hydrogen. The volume of hydrogen varies 
jointly as its weight and absolute temperature * and inversely as 
the pressure. 

Under 760 mm pressure at 0° C the volume of .08973 gram of 
hydrogen is 1 liter. 

What will be the volume of 18 grams under 1000 mm pressure 
at 50°? 

62. Illumination. The amount of illumination received by a 
body varies directly as the intensity of the light and inversely 
as the square of the distance from the light. 

From a light of 16 candle-power the illumination is 6 at a dis- 
tance of 5 feet. 

Compute the illumination at a distance of 12 feet from a light 
of 50 candle-power. 

* Absolute temperature =273°+ recorded temperature, Centigrade. 



Digitized by 



Google 



68 



VARIATION 



93 



63. Volume of a Gas. The volume of a gas varies as the 
absolute temperature and inversely as the pressure. 

Under a pressure of 1 atmosphere and temperature of 260° C the 
volume is 150 cubic inches. 

Required the temperature when the pressure is 45 pounds per 
square inch and the volume the same as in the first instance. 

64. Pressure of a Gas. The pressure of a gas varies jointly 
as the density and the absolute temperature. 

At a recorded temperature of 25° C a given volume of a gas whose 
density is .972, exerts a pressure of 2 pounds to the square inch. 
Determine the pressure when the recorded temperature is 125°. 

65. Offing at Sea. The distance of the offing at sea varies as 
the square root of the height of the eye of the observer above the 
sea level. 

The offing is distant three miles when the eye r^\ 

is 6 feet above sea level. 

Compute the distance of the offing when the 
eye is 560 feet above sea level. 

66. Elongation of a Spring. Within the elastic 
limit the elongation of a spring varies as the load. 

Under a load of 7 grams the elongation of a 
spring was 23.15 millimeters. 

Compute the load when the elongation of the 
same spring is 31.53 millimeters. 

67. Relation of Flow to Diameter. The amount of water that 
will flow through a pipe varies as the square of the diameter of 
the pipe. 

How many gallons will flow through a 12-inch pipe if 40 gallons 

flow through a 2-inch pipe 
in the same time? 



Fia. 81. 




68. Resistance of a 
Wire. The resistance of 
a wire varies directly as 
Fig. 82. ^^® length and inversely 

as the cross-sectional area. 
The resistance of 390 feet of i^^-inch copper wire is 1 ohm. 
Compute the resistance of 2 miles of J-inch copper wire. 



Digitized by 



Google 



94 



TECHNICAL ALGEBRA 



68 



69. Intensity of a Magnetic Field. The intensity (H) of the 
field varies directly as the number of vibrations and inversely 
as the square of the distance of the magnet. 

The intensity is .16 when the number of oscillations is 20 and 
the distance .28 inch. Compute the intensity when the number 
of vibrations is 100 and the distance is .0987 inch. 

70. The Thompson Ammeter. The effect upon the needle 
varies as the cube of the radius of the coil and inversely as the cube 
of the mean distance between the coil and the needle. 

Solve for Ri. 



^mmvmvvvmm vvmmvmmvM^^^ 




Fia. 83. — ^Thompson Ammeter. 

71. Visual Angle. The visual angle of a sphere varies directly 
as the diameter and inversely as the distance of the sphere from 
the observer. 

Solve the formula for ft. 

72. The Inclined Plane. The force acting parallel to the base 
of an inclined plane varies directly as the height of the plane and 
inversely as the length of the base. 

Solve the formula for Hi, 

73. Force of Gravity. The force of gravity at the surface of 
any planetary body varies directly as the relative mass of the 
body and inversely as the square of the radius of the body. 

At the surface of the earth, whose radius is about 3960 miles 
and whose relative mass is unity, the force of gravity is 32. 

Compute the force of gravity at the surface of Jupiter whose 
relative mass is estimated as 31L95 and whose radius is 43,000 
miles. 



Digitized by 



Google 




68 VARIATION 95 

74. Intensity of lUiunination. The intensity of illumination 
from a light varies directly as the sine of the angle at which the 
rays fall (sin 6)y and inversely as the square of the distance of 
the light. 

Solve the formula for D%, 

75. Law of Resistance. The resistance of a conductor varies 
directly as its length into the 
specific resistance of the material 
and inversely as the square of its 
diameter. 

The resistance of a Ruhmkorff 
coil wound with No. 12 copper 
wire (Brown & Sharpe wire- 
gage) was found to be 2.6 -pm. 84. 
ohms. 

The resistance of 2 meters length of the same wire was .006 
ohm. Find the length of wire in the coil. 

76. Thickness of a Hollow Cylinder. The thickness of a hollow 
cylinder varies directly as the amount of material and inversely 
as the length of the cylinder times the sum of the internal and 
external radii. 

Solve the formula for L2. 

77. Elongation of a Wire. The elongation of a wire varies 
jointly as the length and the force applied and inversely as the 
square of its diameter. 

The results of an experiment on the stretching of No. 12 iron 
wire were as follows: 





F L E 


D 


1st test 


2 40 1.5 


.26 


2d test 


2 30 


.26 



Find the value of E in the second test. 

78. Torsion. The angle of torsion of a rod varies directly 
as the twisting force in pounds times the length in inches times the 
reciprocal of the fourth power of the diameter in inches. 



Digitized by 



Google 



96 



TECHNICAL ALGEBRA 



68 



A rod 36 inches long and an inch in diameter is twisted through 
5** by a force of 10 pounds. 

Find the force which would twist through 11** a rod of the 
same material, 45 inches long and 2.3 inches in diameter. 




Fia. 85. 

79.^ Deflection of a Beam. The amount of deflection of a beam 
when supported at both ends and loaded at the center varies directly 
as the load times the cube of the length in feet, and inversely 
as the width in inches times the cube of the depth in inches. 

A beam 20' by 5" by 4" supported at the ends and loaded at the 
center with 4 tons is bent downward 2i inches. 

What must be the depth of a beam of the same material, 
length, and width, that shall bend only J inch under the same load? 

80. Stiffness of Shafting. The stiffness of a shaft varies directly 
as the fourth power of its diameter and inversely as the load and 
the cube of the length. 

Solve the formula for dt, 

81. Horse-Power of a Steamer. In steamers of the same type 
the horse-power varies jointly as the cube root of the square of 
the displacement in tons and the cube of the velocity. 

If the horse-power is 1500 when the displacement is 1650 tons 
and the velocity 14.4 knots per hour, determine the horse-power 
developed in a vessel of the same type, whose displacement is 
* 20,000 tons and whose speed is 23.2 knots per hour. 

82. Heat Due to Current. The heat developed in a conductor 
\aries jointly as the resistance of the conductor, the time the 
current flows, and the square of the current. 



Digitized by 



Google 



68 



VABIATION 



97 



In 2i minutes a current of 3| amperes develc^d 1300 imits 
of heat in a wire having 10^ ohms resistance. 

Compute the resistance of a wire of the same size in which 
22,000 units of heat were developed by a current of 7.11 amperes 
in 3J minutes. 

83. Weight of Cylinder. The weight of a cylinder varies 
jointly as its length and the square of its diameter. 

The weight of a cylinder 10.5 inches long and 3J inches in 
diameter is 30.6 pounds. 

Compute the diameter of a cylinder of the same material 
and thickness, 12 inches long and weighing 82 pounds. 

84. Rupture of a Beam. Under a central load the breaking 
strength of beams of the same depth varies jointly as the width 
in inches and the reciprocal of the length in feet. 





Fig. 86. 

A beam 8 feet long and 4 inches wide, supported at the ends, 
breaks under a weight of 3000 pounds acting at the middle. 

Under what weight will rupture occur in a beam of the same 
material whose dimensions are 10' by 5"? 

85. Resistance of Air. If a ball have a velocity greater 
than 1100 feet per second the resistance of the air approximately 
varies as the square of the diameter of the ball times the difference 
between the velocity and 800. • 

A ball .42 inch in diameter with a velocity of 2100 feet per 
second meets an air resistance of 1.6 poimds. 

Determine the resistance offered to the same ball when the 
velocity is 1850 feet per second. 



Digitized by 



Google 



98 



TECHNICAL ALGEBRA 



68 



86. Horse-Power Transmitted. The horse-power transmitted 
by a rope varies directly as the cube of the square root of the 
tension and inversely as the square root of the weight of the rope 
per foot. 

Forty-five horse-power is transmitted when the tension is 1000 
pounds and the weight per foot is .4 pound. 

Compute the horse-power transmitted by the same rope when 
the tension is reduced 200 poimds. 

87. Variation of the Current. The 

current varies inversely as the resist- 
ance and the resistance varies directly 
as the cotangent of the angle of deflec- 
tion (cot 8), 

Prove that the current varies in- 
versely as the cotangent of the angle 
of deflection. 

88. Coulomb's Law. The force (/) exerted between two charges 
of electricity varies as their product {qq') and inversely as the 
square of the distance (r) between them. 

Find the value of /2 when 




HI 11^ — AWVV 



Fig. 87. 



/l= 1 


n=15 


g. = 12 


g.'=9,' 


<?»= 1 


r,=2 



89. Range of a Jet. The 

range of a jet varies jointly 
as the square root of the 
head and the square root of 
the vertical height of the orifice 
above the level where the jet 
strikes. 

Solve for R2, 







^^ 



90. Flow from an Orifice. 

The amount of flow from an 

orifice varies jointly as the area of the orifice and the velocity of 

the flow. 

Solve the formula for Fj. 



Digitized by 



Google 



6d 



VARIATION 



99 



91. An Inclined Plane. The force on an inclined plane varies 
as the height of the plane, and the height of the plane varies as 
the sine of the angle of inclination of the plane to the horizontal. 

Prove that the force varies as the sine of the angle of inclination. 

92. Lateral Surface of a Right Cylinder. In a 

right cyUnder of constant altitude the lateral area 
varies as the circumference of the base, and the cir- 
cumference of the base varies as the radius. 

Prove that the lateral surface varies as the radius. 



Fia. 89. 



93. Velocity of a Falling Body. The velocity of 
a body falling freely under gravity varies as the 
square root of the space traversed from rest. 

When a body has fallen a distance of 16 feet its velocity is 
approximately 32 feet per second. 

Compute the velocity when the body has fallen 540 feet. 



\ 
\ 
\ 
\ 
\ 
\ 



Fig. 90. 



94. Diagonal of a Cube. The edge of a 
cube varies as the diagonal of the face, 
and the diagonal of the face varies as the 
diagonal of the cube. 

Prove that the diagonal of the cube varies 
as the edge. 



95. Diffusive Power of a Gas. The 

rates of diffusion of gases are inversely proportional to the square 
roots of their specific gravities. 

Formulate as a proportion and compute the ratio of the rates 
of diffusion of two gases whoso specific gravities are .97 and .0694. 

96. Revolution of a Planet. The square of the time of the 
revolution of a planet about the sun varies as the cube of its dis- 
tance from the sun. 

The earth at a mean distance of 92,000,000 miles from the 
sun, makes a complete revolution about the sun in 365J days. 

Compute the period of revolution of Neptune whose distance 
from the sim is approximately 2,775,000,000 miles. 

97. Period of a Pendulum. The time in which a pendulum 
makes one vibration varies as the square root of the quotient 
of the length and the force of gravity. 



Digitized by 



Google 



100 



TECHNICAL ALGEBRA 



68 



When the force of gravity is 32.14, a pendulum 39.1 inches 
long vibrates in 1 second. 

Compute the time of vibration of the same pendu- 
lum on the moon where the force of gravity is .165. 

98. Velocity of Sound. The velocity of sound in 
gases varies directly as the square root of their elasticity 
and inversely as the square root of their density. 

Sound travels through air at the rate of 1080 
feet per second. 

The elasticity of air is 1.6X10* and its relative 
density is 1. Fig. 91. 

Compute the velocity of sound in hydrogen 
whose relative density is .0694 and whose elasticity is 1.6X10*. 

99. Indicated Horse-Power. The coal consumed in tons by 
vessels of the same type varies jointly as the indicated horse- 
power and the time of passage in days. 

The displacement in tons varies as the coal consumed. 
Show that the indicated horse-power varies as the displacement, 
and inversely as the time of passage. 

100. Initial Velocity of a Projectile. The square of the 
initial velocity of a projectile in feet per second varies directly 
as the charge of powder in pounds, and inversely as the weight 
of the projectile in pounds. 

If 21 pounds of powder will give a 40-pound projectile an 
initial velocity of 2000 feet per second, compute the charge 
required to hurl a 50-pound projectile with an initial velocity of 
1800 feet per second. 



Digitized by 



Google 



CHAPTER IV 

THE FOUR FUNDAMENTAL OPERATIONS 

Secton 1, Addition. Section 2, Subtraction. Section 3, 
Multiplication. Section 4, Division. 

69. Classification. The four fundamental operations are: 

Addition, 
Subtraction, 
Multiplication, 
Division. 

They are called fundamental because they are basal in 
mathematics and other seemingly different operations are 
only these in various combinations. Their importance 
and the necessity of speed and accuracy in their application 
are therefore obvious. 

60. Kinds of Terms. The terms of an algebraic expres- 
sion are of two kinds, like and unlike. 

Like terms are those having one or more letters the same 
with the same exponents. 

Unlike terms are those which do not have the same letters 
with the same exponents. 

What is meant by like terms can best be shown by an 
illustration: 

ox, to, rx, 5x, ex, and 2a;, 

rioi 



Digitized by 



Google 



102 TECHNICAL ALGEBRA 61 

are like terms with respect to x, all having the same letter x, 
affected by the same exponent 1. 

5c2, 4a2c2, 10b3c2, and (u?, 

are like terms with respect to c^. 

Therefore to be like, terms must have one or more letters 
the same with the same exponents, 

ac and bc^, b(?, and 8c*, 

although having the same letter c, are unlike because the 
exponents of c are unlike. 

§ 1. ADDITION 

61. Definition. Addition is the process of finding the 
sum of the terms of an expression. 

To perform the addition it is convenient to write like 
terms in the same vertical column, making as many columns 
as there are unlike terms. The excess of plus or minus in 
each column is then determined and the result is written 
underneath. 

62. Illustration. Suppose the following polynomials * 
are to be added: 

\27?-%x^+l^\ and6x4-llx-12x3__8^ 

Writing Uke terms in the same vertical column and 
finding the excess of plus or minus in each column, we have 

3x4+ 5a:3__6a.2^ ^^^ 1 
-9x4_|_ a:3_|_53.2_i32.+ 1 

l23?-%7?+ 16 

6x4_i2a:3 -iix- 8 



6x3-8x2-20x+ 8 
* A polynomial is an algebraic expression of more than two terms. 



Digitized by 



Google 



66 THE FOUR FUNDAMENTAL OPERATIONS 103 

63. Terms with Literal Coefficients. Like terms witii 
literal coefficients or with both literal and numerical are 
added by inclosing the coefficients in a parenthesis followed 
or preceded by the letter with respect to which the terms 
were classified as like terms. 

For example, if the smn of ^x—ax+cx—h^x is desired, 
the result is written (5 — a+c — b^)^. 

64. Law of Addition. 

(1) Write like terms in the same vertical column.* 

(2) Find the excess of plus or minus in each column. 

65. Examples in Addition. Find the sum of the follow- 
ing as specified by the law: 

1. Zx'-6x^+2x*-Sx^y 3x'-6x*+2x2-x, Tx^+Oaj^-x^+lOa;, 

- 12x2+ llx- 16x3. 

2. y^+3y^-2y^+5y, 8y^-7y+ny^-my*, '-y^+2y*+l0y -Sy^, 

7y^+15y^-y^+y. 

3. 422-62+122^-202^+8, 62-1123-422+62^-9, 

22*+323+9x2 -302+1, 823 -72+92* -322+6. 

4. -34^+18^2 _7^4+8^3_9^ 7 _ 16^2+^ .8^3+9^4. 

lU2_9^+14_7^3+3^^ ldt-ti+U*-l6+t\ 

6. ax+hx+cx+dx+ex. 

6. 5x+3x-6x+cx+dx, 

7. 6a-56+7a-d, 36 -a+d-c, d+2c+a-26, c-4d+9a-l, 

l+8d+c-7a, 

8. 5x2-4x2-6x2+3x+8x-x+cx. 

9. a+5, 3a+8, -5a -7, -lla+16. 

10. by+cy+dy+8y, 

11. 4i/2+6i/2+ct/2. 

12. 8x2+0x2+6x2-6x2-10x2+8.' 

13. 3x2 -cx2+16x2-te2 -4x2+7. 

* Usually unnecessary, since in algebraic work the addition of 
like terms can be determined from the expression as written. 



Digitized by 



Google 



104 TECHNICAL ALGEBRA 65 

14. 5z^+dx^-kx^-3x^+7x-Zx+ax. 
16. bx+cx-Sx+9x^+fx-cx+3x^+x. 

16. a^x+9x^-b^-ax^+5x-dx. 

17. sVx-^Vx+aVx-sVx. 

18. sVbc+Vbc-ay/bc-sVbc. 

19. 5x^-ax^+5bx^'-2x^+icx^. 

20. ax^+3x^-x*+6x, 3x-hx^+x^+2x*. 

21. (a+h)\/x-c\/x+dVx-5Vx. 

22. 3(x+y) -3x^+i{x+y)+mx^-c{x+7/), 

23. WbTc-Sy+6--2Vb+c+15y+hVb+c-5, 

^b+c+ (r+s)y - uVb+c -dy+2^. 



2 

24. |aa;2-^aa;«-29+dr+3i/-^ax2, 

^x+30 -Siy+Y^ax^ - 14+ (a+6)y. 

25. 5x"-16x"+8Jx'+19aV+7a;"-10aV, 

bx^'+cbf-la^* - 17a«2/» -6x^ -ax^ 

26. Safy" -8x^y^+xY+7x'y'+Six*y*, 

-bafy'+12x^*-gxY+tx^*-y-K 

27. 2462c»x2-30aa;2+12%'+1662c»x*, 

-9by^+l5b^c^x^+33ax^+2lby^ -daxK 

28. rx2-da:*+5x*-te«+3x*-5x2+cte2-x». 

29. 5x^+Sx*-{a+b)x^+10x*-ax+5x+cx. 

30. 0^x2 -ft'x* - Ax2+5x -Sx+kx+Sx^ -5x+18. 

31. 24.8x2 -16.29x»+7.05x+24, 

-16+3.17X -32.2x2+1.48x8 -18.36, 
17.6x2+12.38x»+16.76-73.52x» -40.17. 

32. a^+Zab*c-7a*bc, a^bc-Zab*c+a*y 15a^bc -Sab^c. 

33. by--cy^+-ax\ ay+—cy*-—bx^, dy+—ay*-—dx\ 

34. r^x+s^—tr^x—ay+c^. 



Digitized by 



Google 



67 THE FOUR FUNDAMENTAL OPERATIONS 105 

36. 4.2a;»-3.3x«+1.5x+8.3, 2.62a;»-8.4a;«+3.17x-1.9, 
2x*+ix^-7x+l, 5x»+1.33x2+21.6x2-1.91x, and 
33.51x»+7.25x2 -6.5x+2.81. 

36. 5c"'+.3a+86c, .4c"+7.3a"+6.q56c, c~+3.07a"~56c, and 

3 13 1 

37. 5ab'-2x+-rxy, .3x+.03ax-4.05a6, -ax+-xy—-ab, 

4 o 4 6 

7x+2.0ax+.2x, —xy+—ax ——xy, 

5 4 o 

38. .3(m-3x)»-|(m-3x)», 38(fn-3xr, -2.13(m-3x)». 

39. x*-/-.4a^ .5x»-4/-.5a, .38x•+.5V-a^ 

40. j-ax«+|a«+-x»y+56», 4ax«+|x»y-7.5a«+|6», 

.8ax«+-x»y -;^a»+-6», — ax2+.25x»y --a« -— 6». 

41. 8y-3(a+6), 5t/+8(a+6), 102/-.9(a+6), -.2y-3(a+6). 



§2. SUBTRACTION 

66. Definition. Subtraction is the process of taking one 
expression from another. 

Subtraction is indicated by the minus sign placed between 
the quantities which are to be subtracted. This sign 
indicates that the quantity following it is to be taken from 
the quantity preceding it. 

In the expression R—SXj the minus sign indicates that 
Sx is to be subtracted from R, In bc—{x+2y), the minus 
indicates that x+2y is to be subtracted from 6c. 

67. The Subtrahend. The subtrahend is the quantity 
which follows a minus sign. 

In R—SXf what is the subtrahend? Why? 

In bc-'{x+2y)j what is the subtrahend? Why? 



Digitized by 



Google 



106 TECHNICAL ALGEBRA 68 

What does the parenthesis indicate? 

If written without the parenthesis what sign would be 
necessary preceding 2y in order to show that it also is to be 
taken from 6c? 

Write the expression without the parenthesis so as to 
indicate that both x and 2y are to be subtracted from be. 

68. Laws of Subtraction. 

(1) Write like terms in the same vertical column. 

(2) Change signs of all terms in the subtrahend and add 

it to the minuend. 

69. Mental Change of Signs. The signs in the subtra- 
hend should be marked changed only so long as may be 
necessary for certainty of result. As soon as possible this 
change should be effected mentally and the changed signs 
should not be written. 

70. Illustration. The following shows the application 
of the laws of subtraction: 

From ax^+c?y—4:by take 5by — 3ax^+2c^y 

Applying (1) of the preceding law we have three vertical 
columns of like terms. 

By the law of subtrahends the signs of all the terms 
in the subtrahend must be changed. The changed signs 
are therefore entered under those in the example, as shown. 

Observe that the changed signs are placed under the 
original signs. 

ax^+ c?y — 4by 
zpSax^^2c^y^5by 

4ax^— (?y — %y 

71. Examples in Subtraction. 

1. From 7a2-5624-8c, take4a2+462-9c. 

2. From la^hc-%ah'^c+llahc^-c^ 
take 8c3+21a62c -Za''hc+I2abc^. 

3. From ^x^-bxy-{-\%xHj''-2\xy^+^ 
take 6 -20xy^ -Sxy -6x^^ -9x\ 



Digitized by 



Google 



71 THE FOUR FUNDAMENTAL OPERATIONS 107 

4. From Uacx-19'ad^+6a%^-r^+l 
take - 19+42a262 - 17ad^ -40acx, 

6. From 21z^+l3x*-x+l4: -bc+t 
take 5t+x -60 -20x^+8x^ -be. 

6. From 5d^x -Sgx^ -2lky^+lSx^y* 
take 17^x2+20%3+5d2x -30. 

7. From ISa^x-Ub'^+llc^^+Sld* 
take Ud*-21a^x+%^y+l0c^K 

8. From 314a: -722/2+18x^-1960 
take 186c+ IQx*!/^ -300x+602/*. 

9. From {a+b)x-(c+d)y+{e+f)z 
take 2(a+6)x - (c+d)?/ -5(e+f)z. 

10. From 3\/ a;-y +2\ /a;g-y 2-13aa;»+l 
take e\/x-y-Vx^-y^+18+17ax\ 

11. From 16(r+s)«-20(6+c)»+21(d+c)* 
take 32(d+e)*-19(r+s)2+52(6+c)3. 

12. From ax2!/2+cx8+dx+92ar2 
take 5ar^+3dx - 10ax22/24-4cx3. 

13. From 23a^b^+30c^x*y -29b^xy^+U 
take -51 +7263x2/' -22a^b^+55c^x% 

14. From &r«+7x<-9x»-14ca:+15x2/-9 
take -x2/+34x»+17x*+19c2;-100x2. 

16. From inx^+.5ax^-.12xy^+73x 
take 9.6aa;«+1.42x?/2 -15.4x22/ -1.8x. 

16. From 5.9r2+3.4x22/2- 17.51/22+60 
take -72+13.21/22 -5.9x22/2 -11.2r2. 

17. From .61x2-9062x»-186x2+c* 
take 1.61x2+8c*+10.562x'- 196x2. 

3 11 3 11 

18. From -5»--ar2--6x2 take jr6x2 -— s' -— ar«. 

4 o o J lb o2 

13 5 

19. From — x*-— x»+j^-82/2+9cx«+6 

3 3 3 11 

take -ex* - 15+ j^ -^2 _ 2/+x». 

20. From 24ar» -86c -42/2+ 16x2/+32 
take 40 -xy+ 17ar» -96c+52/2. 



Digitized by 



Google 



108 TECHNICAL ALGEBRA 71 

21. From 12Ax^-31.5y^+ax+hx*+cy^ 
take - lQ,5y^+ax -6x« -cy« - 172.3x«. 

22. From 32.34r«-17.25s»-12.84(«+1.9 
take 3.42s^-7,9i^-d.79+15,Sr\ 

23. From 16.63m2~12.8n+42.54r«»2+7.4 
take 1 1 .09m+ 12.7 -3.7m«+ l9.7r*vK 

24. From l-L^-^V^'-^R^+iSW* 

o 10 d4 

take 15.9TF»+:^L«-7«+^i2«. 
16 64 

26. From 3.72x»-9.07x«+13.29x+41.6 
take 5.18x2-4x5+12.8 -40.1a;. 

26. From .7625-.9x*- 16.43+ 12.62y*+ 17.8 
take -42.2+41.042/2+1.3x*-12.022». 

27. From 10d*w;*-.llm»r+.78p5*-.9 
take mm^'-.15dHD*+5pg*+5.09, 

28. From 16.05x2 -.9^2+2x2/ -3x?/2+4.8 

take -12.92xy+ 15.55x2 -3x2/.2+8.l2/2+7.92. 

29. From 10.96c2-19.1acx2+32.76x»+6 
take -9.26x3+1.846c2x-18.2acx2. 

30. From -76.4^2x2 - 9.03A;x +4.752/2 -17^ 
take 54.82/2+l6.282/+15.5/i2a;2+ii.04ikx. 

31. From |+^x»-|-x2»+4.1x22/« 

take ——=r-x»+.25x2» -8.39x22/2. 
lo 7J 

32. From -12.18ar2-11.0862/2+.34x2/-9x* 
take 9.17x*+5.92ar2 -.69X2/+1.9862/2. 

33. From 1.08»2s2-4.19s22/2+8.84-7.296c«2_9.i 
take -3.86 - .3956c<2 _ 297s^^+ Mv^sK 

34. From dy/b^-c^+Wr^-s^-A^yz+y-M 
take aV62-c2 - dVr^-s*+ .03 \^x+y - .96. 

36. From 1 take 5y* -82/2+42/ and add the result to 8+5y Sy^+y*. 
36. From l+3Vx+3x+Vx» take l-3Vx"+3x-Vxr 



Digiti 



led by Google 



73 THE FOUR FUNDAMENTAL OPERATIONS 109 

37. From a+bVl-c* take a-6Vl-c«. 

38. Subtract hy/x+y—ay/x-y from aVx+2/+6V x -y. 

72. Meaning of a Minus Parenthesis. A minus pre- 
ceding a parenthesis indicates that each term within the 
parenthesis is to be subtracted from the quantity preceding 
the parenthesis. 

8a— (6a;+!/— 6a) is the mathematical way of stating 
that 6x+2/— 6a is to be subtracted from 8a. In this expres- 
sion, therefore, 8a is the minuend and 6a;+!/— 6a is the sub- 
trahend. 

By a minus parenthesis indicate that 3r— 5d+6 is to be 
subtracted from 2r^, 

Indicate the subtraction of 5ar+6y from 2x\ indicate the 
subtraction of 7x2 _ le + 2y'^ from Sx^ _ g _ 42^2^ 

73. Examples in Removal of Parenthesis. The follow- 
ing examples in removal of a negative parenthesis are merely 
examples in subtraction and are therefore numbered con- 
secutively with the preceding list. 

39. 7aa;+46-c-8-(19aa;+46-c+2). 
Copy this example in the work-book. 

The minus sign preceding the parenthesis indicates that the 
terms inclosed are what part of an example in subtraction? 

Each of the inclosed terms is therefore subject to what law? 

Therefore the parenthesis can be removed only by making 
what change? 

Rewrite the example with this change and add the terms. 

40. 7aa;+46-c-8-(19ax+46-*c+2). 

Copy this example. Write it with the parenthesis removed 
but with the vinculum retained. 

Write the result with the vinculum removed. 

In the filial expression, which of the inclosed terms have signs 
different from what they had in the example? 

* The sign of c is -f- understood, the minus being the sign of the 
culum only. 



vinculum only, 



Digitized by 



Google 



110 



TECHNICAL ALGEBRA 



73 



Which of the inclosed terms have signs the same as they had in 
the example? 

The terms whose signs were changed, were effected by how many 
minuses? 

The terms whose signs were not changed were effected by how 
many minuses? 

Rewrite the example with the vinculum removed and the paren- 
thesis retained. 

Write the result with the parenthesis removed. 

Is the final result the same as the result obtained when you 
removed the parenthesis first and then the vinculum or is it 
different? 



41. IQxy -5x^+6r -5+t-{7x^ -z^ -ir). 

Remove the marks of parenthesis by the two methods employed 
in example 40, first removing from without inward, then from 
within outward. 

Are the results the same or different? 

42. Rule the following table in India ink in the work-book 
and make the entries indicated. 

SOLUTION OF EXAMPLES 39 AND 40 



Examples. 


1 


2 


Number of 
Minuses Affect- 
ing Terms in 
Column I. 


Number of 
Minuses Affect- 
ing Terms in 
Column 2. 


Terms Having 
Signs Changed. 


Terms Having 
Signs Unchanged. 


39 
40 











In examples 39 and 40 were the signs of terms changed when 
affected by an even number of minus signs or when affected by 
an odd number? 

All marks of parenthesis, therefore, may be removed in one 
operation provided the signs are changed of all terms affected by 
what number of minus signs? 

Under a heading Law for Removal of Parenthesis, write a law 
for the removal of negative parentheses. 



Digitized by 



Google 



73 THE FOUR FUNDAMENTAL OPERATIONS 111 

Remove the marks of parenthesis from the following examples 
by one operation, applying the law regarding terms affected by 
an odd or an even number of minuses, and then add the terms. 

43. x+y-(2x+y) + {x-y)-(3x-2y+m). 

44. 3y-(x2+32/)-2(3x-2/+8)-(x-^). 
46. 6-(a+c)+a(c+6)-(a+c). 

46. 5(a-6-c)-(5a+6+c)-3(x-y+2)-[3-(x+y-l)]. 

47. 2d-(3d-2e+/)-2d+5-(c-6d+3/). 

48. 4a-{2a+c-l5a-(iS-b-c+d+30)]]. 

49. |--|a-(36+c)-|--(!;+s-?f^)J. 



61. -4-{-5-[-2-(-6+5c+2a-8)l}+3-[6-(-7a)]. 

1 



62. ix-— -Ih+Bc-iSb+s-Sa+c)] 

8 4 



53. ---^-b-j^-lb+c-(9c+ab-S)] 

64. 3x-.2-(3a+2/-.2x)-|.08-^^-^j •. 

66. 22/+5(x-32/+4)-[-2-(-x-22/-3)]. 

67. -3x-2(^-^+24)-(-3+2x). 

58. -2-{ -a;-[3+x-(3x+8-2x+8)]}. 
69. 5a+b{c-x+d-3+x+d)^[5a-{bc-bx+d)l 
1 



«.-i- 



+5a:+l -[-5a:-3(a:-4x-3+22/)] 



61. -(2x+y-3+j/-2a;)+5j/-{-2-I3-(6x+5)]}. 

62. 6x»+3x-(8y+2V)-[32-5(x»-3)(5-x)]+82/+a;V. 



Digitized by 



Google 



112 TECHNICAL ALGEBRA 73 

63 - • ......2-6C 4+7c 






66. 3x*-6(x8+5x«-8x+3) -(3a;*+9-6x8-2x«+7x). 



66. - ( -2a+6 - -3a+5) -6x+7 -a(2 ~x -6+2). 
3 



67. — - 2a-[6-(2c-12+8a),-26] -4c 

68. ^-j -2a:-[3x-(4a-c--x-2a)]-3a;-(2c+l)j. 



^3 . 5x 



^- -{7+-6-2-8a;)-{-3-[-2x-(6+5x-2+3x)]}. 



70. 2a;-(3x-22/)-[-l+6x-(-2x-oj;4-5-d)+2x]. 



71. 5ax^-{-2ax^+c-[-Zax^-(2c+16-2ax^-7b-c-2)]}, 



72. -24^2- {3<-t3+7<*-[-2^+8-(3<8+6^*- -^24.^3)]}. 



73. 3a;*-{ -3x2+x»^(-x« -x* -2x^+5) -8x- (9x2 -x)}. 



74. -9y-{-y-l-y-(-y-y-y^-Sy-'-y+y^)]}. 



76. 26^-(62+36-63-6-62)-{-6»-[-62-(62+6)]}. 

77. -5(^-^-2+45^) -{-4j,-[-3y-(2v+9)]}. 



Digitized by 



Google 



77 THE FOUR FUNDAMENTAL OPERATIONS 113 



§3. MULTIPLICATION 

74. Definition. Multiplication is the process of finding 
the product of algebraic expressions. 

Although not necessary it is a convenience to arrange 
the terms of both multiplicand and multiplier in the order 
of the ascending or descending powers of the same letter. 

The first term of the multiplier is written under the first 
term of the multiplicand and the others follow without 
regard to like or unlike. 

Each term of the multiplicand is multiplied in succession 
by the terms of the multipUer, and the like terms of the 
resulting products are written in the same vertical column 
in order that they may be added for the total product. 

76. Laws of Multiplication. 

(1) Law of Signs: Like signs, plus; unlike signs, minus. 

(2) Law of CoeflScients : Multiply numerical coefficients. 

(3) Law of Letters: Bring down letters. 

(4) Law of Exponents: Add exponents of the same 

letters. 

76. Illustration. 

Multiply ir3+|r2+2r-i 
by ir-2 



|r*+ir3+|r2- uV (1st product) 

- T^-JT^-^r+i (2d product) 

ir4-|r3-fr2-4^V+i (total product). 

77. ExplanatioYi. The first term of the first product is 
|r*, obtained as follows: 

The sign is plus; Like signs, plus. 

The coefficient is J; Multiply numerical coefficients. 

The letter is r; Bring down letter. 

The exponent of r is 4; Add exponents of the same 
letters. 



Digitized by 



Google 



114 



TECHNICAL ALGEBRA 



78 



Each of the terms may be explained in the same way. 
Observe that before multiplying, the terms of both 
expressions were arranged in descending order. 

78. Examples in Multiplication. In the following exam- 
ples enter the work as in the illustration given in paragraph 
76 but do not name the products. 

1. Multiply 2a+3b by 3a. 

2. Multiply 3x^+x+Q by 2x+3. 

3. Multiply 5x^-^^+x by x*+Sx. 

4. Multiply ax^-2bx-dO by x*-xK 

6. Multiply Qx*-5x+x*-Sx^ by 2x^-5x. 

6. Multiply 2d2-3c»+8 by 6d-5c+l. 

7. Multiply 2x^-3y^+x-2y^ by 2x2-4. 

8. Multiply x+y by x+y, 

9. Multiply x-y by x-y, 

10. Multiply x+y by x-y. 

11. Multiply 26c«+5e'+3a by 36c -8. 

12. Multiply 3ax«-26x+24 by 5a*+10. 

13. Multiply %«-16+3y by 6y^+2y+3. 

14. Multiply 7x^-2x^+ixy^''Sy* by 2x-3y«. 
16. Multiply 24x»+15x«-7x-8 by 5x-l. 

16. Multiply 8x*-llx»-3x*+10x+6 by 7x-4. 

17. Multiply 96*-156»+126«-26+6 by Gx^+x. 

18. Multiply 7x*-5x+4x»+x2 by 5x«-2x. 

19. Multiply 15bx-12cy+S by 2x-y-l. 

20. Multiply 30d«--9c2-ll/« by 2d*-e*+Sf. 

21. Multiply 2a+36+5c«+ll by 3a -6 -2c. 

22. Multiply Zr^+2s-6v^ by 2r^+2s-6v*, 

23. Multiply l2x+4y+9 by l2x+iy+9. 

24. Multiply 8x»-52/+32/«-6y» by 2x«-j/«. 

25. Multiply 3acx-2by+y* by 2ac-3j/-y. 



Digitized by 



Google 



80 



THE FOUR FUNDAMENTAL OPERATIONS 115 



26. Multiply 2x»-9+8a;2-12x by Q-Sz+SzK 

27. Multiply 2x'-3x*+4a;'; by Tx^-S. 

28. Multiply 5ac2+126"-6c"-^ by 8a' -4. 

29. Multiply 32z^-7z^-9z-6 by 2z^+4z-l. 

30. Multiply x^-^+x^-^+x" by x"-^-5. 

31. Multiply h'-'^-h^'+h by h'-2+h. 

32. Multiply a^+a^-a^+a by a*. 

33. Multiply x^-'+y*-3y-^+8y^-^' by x'-t/'. 

34. Multiply 31x*-3ax2-5x~^-8 by 36x-7x«. 

35. Multiply 21x*-20x'-^"'-15x"-H8x3 by 4x-*-2x-x-\ 

36. Multiply 5V^-aVy-hV7-9 by 2Vx-cVy+U, 

37. Multiply 3y/x+y-5Vx-y-7Vx^-y* by Vx+y-Vx-y. 

38. Multiply ax^+ay^'+l by ax^'-ay^'-l. 



§4. DIVISION 

79. Definition and Arrangement. Division is the process 
of finding how many times one algebraic expression is con- 
tained in another. It is the process of finding by what 
multiplier one of two expressions must be multiplied to 
give the other. 

To perform division the terms of dividend and divisor 
must be arranged in the order of the ascending or descend- 
ing powers of the same letter. The divisor should be written 
at the right of the dividend and the quotient should be 
written under the divisor. This position of divisor and 
quotient is an advantage both in writing the divisor and in 
multiplying it by the successive terms of the quotient. 

80. Laws of Division. 

(1) Arrange terms of both dividend and divisor according 
to the ascending or the descending powers of the same letter, 

(2) Law of Signs: Like signs, plus; unlike signs, minus. 



Digitized by 



Google 



116 TECHNICAL ALGEBRA 81 

(3) Law of Coefficients: Divide numerical coefficients. 

(4) Law of Letters: Bring down letters. 

(5) Law of Exponents: Subtract the exponents of 
letters in the divisor from the exponents of same letters in 
the dividend. 

81. Illustration. Divide 23x2-48+6x*-2x-31a:3 by 
6+3x2«5a. 

' 6x*-31a?+23ar^- 2x-48 (3rc2-5a:+6 (divisor) 

-6x^=Fl0x^=i=12x2 2x2-7x-8 (quotient) 

-21x3+lla:2« 2x 
( -21x3+35r2-42a: 



-24x2+40x-48 
-24x2+40x-48 



82. Explanation. 3x2 ^^^ gj^^ term of the divisor is 
contained in 6ar* the first term of the dividend, 2x^ times, 
obtained as follows: 

The sign is plus; like signs, plus. 

The coefficient is 2; divide numerical coefficients. 

The letter is a:; bring down letters. 

The exponent is 2; exponent of x in dividend minus 
exponent of x in divisor. 

Multiplying (all the terms of) the divisor by 2x^ we 
obtain the first product. 

Subtracting we obtain the first remainder. 

3rc2 is contained in— 21x3, — 7a: times, obtained as follows: 

The sign is minus; unlike signs minus. 

The coefficient is 7; divide coefficients 

The letter is x; bring down letters. 

The exponent is 1 (understood); exponent of x in 
dividend (first remainder) minus exponent of x in divisor. 



Digitized by 



Google 



83 THE FOUR FUNDAMENTAL OPERATIONS 117 

83. Examples in Division. 

1. Divide 4x* -16x*+23z -20 by 2a; -5. 

2. Divide 54x»+18a;«-6x-20 by 3a; -2, 

3. Divide 3a;»+7ar«-2x+12 by 2a;+6. 

4. Divide x*+3x*y+3xy^+y^ by x+y. 
6. Divide 58a+7a» -21 -24a* by 7a -3. 

6. Divide 21a;»-26a;»-12a;+32-51a;«+22x* by 3a;+4. 

7. Divide 15a;*-32x»+50a;«-32a;+15 by 3a;«+5-4a;. 

8. Divide x^—y* by x-y. 

9. Divide x*+2xy+y* by x+y. 

10. Divide x*-2xy+y* by x—y. 

11. Divide x*+y* by x+y, 

12. Divide h^—g* by h—g. 

13. Divide 23a;«-48+6a;*-2a;-31a;» by 6+3a;«-5a;. 

14. Divide a*— 6* by a— 6. 
16. Divide a*— 6* by a+6. 

16. Divide a«-&« by a+b. 

17. Divide a«— &" by a— 6. 

18. Divide a8-6« by a -6. 

19. Divide a*— 6* by a+b. 

20. Divide a^+fe* by a+6. 

21. Divide a'^+b'^ by a+6. 

22. Divide a^+¥ by a -6. 

23. Divide a^+6^ by a -6. 

24. Divide 625a* -816* by 5a+36. 
26. Divide a+y by x^+yK 

26. Divide x—y by x^—yK 

27. Divide x8-816» by a;«-36. 

28. Divide 2x*-xy*+6x^^'-Sx^+Qy* by x*-'2xy+3y\ 

29. Divide r7-3r«+l+2r»+7<-2r«+6r*+l-4r 

by r«+l-r-r»+r«. 

30. Divide 60a;+9x*+24-67a;« by a;-6+a;«. 



Digitized by 



Google 



118 TECHNICAL ALGEBRA 84 

31. Divide ^h^"" -h+21h!' -2bh^'' by 2A»-5. 

32. Divide ^+2t*-7<*-l+4<-e*+4/» by t-\+t\ 

33. Divide a«+6« -6a«6+15a*6« -6a!>«+15a«6* --20o»6». 

by a3-3a26+3a6»-6». 

o^ T^ -^ 9 . 7 3 , ,4 , 16 , 3 , 8 

34. Divide — x*—x^ — x'H — x-\ — bv —x*—x — . 

16 4 4 3 9 -^ 2 3 

or T^- ,1 « . 5 . 13 , 1 , 37 17 , 7 ^ ,.12 
36. Divide a«+-a^--a»--a«+-a«-^+- by a^+^^-a. 

36. Divide A:«-5ifc«-24A;»+15A;*+27A;»-13A;+5 

by A;*-2A;+H-4A;2-2A;». 

2 8 3 132 

37. Divide -x^+-x^^+--xy^--—x^y^-7:^ 

o o 10 7o 

2 14 

by -x^+-x^--xHj. 

38. Divide c'+^+c^A;+cA;^+Af +^ by d+l^. 

39. Divide c2''--s2''+2sV-c2' by d'+s'-d. 

40. Divide x«~l - x^-3a;2 by -2x^-x+x^-l. 

41. Divide l+4«34.3^4 by {t-\-\)\ 

42. Divide r'^-l by r-"-r-'+r-5-l. 

43. Divide a2*-62"+26V-c2'' by a'+U'-if. 

44. Divide TF''+^+TF^F-Try^-r+^ by W-V, 
46. Divide a^^+^-p^"^ by dF'^. 

46. Divide dr—dir—diR by dii2. 

47. Divide (a+6)3+l by (a+b) + h 

84. Summary of Laws of the Fundamental Operations. 

I. Addition. 

(1) Write like terms in the same vertical column. 

(2) Find excess of plus or minus in each column 

II. Subtraction. 

(1) Write like terms in the same vertical coliunn. 

(2) Law of subtrahends: 

(a) Change signs of all terms in the subtrahend. 
(6) Add. 



Digitized by 



Google 



84 THE FOUR FUNDAMENTAL OPERATIONS 119 

Removal of Parenthesis. 

Law: All marks of parenthesis may be removed from 
any algebraic expression, provided: 

(1) The signs are changed, of all terms affected by 

an odd number of minus signs. 

(2) The signs are not changed, of all terms aflfected 

by an even number of minus signs. 

(3) Other operations indicated by parenthesis, are 

performed. 
Abridged form of law: odd, change: even, don't. 

in. Multiplication. 

(1) Law of signs: like signs, plus; imlike, minus. 

(2) Law of coefficients: multiply numerical coef- 

ficients. 

(3) Law of letters: bring down letters. 

(4) Law of exponents: add exponents. 

IV. Division. 

(1) Arrange terms of both dividend and divisor 

according to the ascending or the descending 
powers of the same letter. 

(2) Law of signs: like signs, plus; unlike, minus. 

(3) Law of coefficients: divide numerical coefficients. 

(4) Law of letters: bring down letters. 

(5) Law of exponents: subtract exponents of letters 

in the divisor from the exponents of the same 
letters in the dividend. 



Digitized by 



Google 



CHAPTER V 

LAWS OF NUMBERS 

Section 1, The Square op the Sum op Two Numbers. Section 
2, The Square op the Difference of Two Numbers. 
Section 3, The Product of the Sum and the Difference 
of Two Numbers. Section 4, The Difference op Two 
Cubes. Section 5, The Sum of Two Cubes. Section 6, 
The Square of any Polynomial. Section 7, The Exact 
Divisor of a Polynomial. 

§ 1. THE SQUARE OF THE SUM OF TWO NUMBERS 

85. Law. In (a+fe)^, what is denoted by the exponent 
2? 

Perform this operation. 

Calling a the first and b the second, write the law for the 
square of the sum of two numbers as determined by your 
multiplication. 

86. Examples for Sight Work. Copy the following 
examples and by inspection write what each equals. 

1. (r+a)\ 2. (3x+4y)\ 

3. (a6'+^')«. 4. (12+7s)K 

5. (a^+9x-^)K 6. (5c^+Qb-^)\ 

7. (7r-'+9^)\ 8. (ll»«-i+12w;i-«)2. 



d^'+r-O' 



9. U"2/'+3-^"M • 10- (xV+13a-26-2)2. 



11. (x-''+y-^-')\ 



12. Uxy+h^x'-^^y. 



120 



Digitized by 



Google 



88 


T.AWS OF NUMBERS 


13. 


(c+sin x)'. 


14. (6+4cosx)». 


16. 


(|B-+7iC-.)'. 


16. (.8D-«+.05£?»-»)». 


17. 


(a;'-8+y»+»)«. 


18. (0f-i+A'+2).. 


19. 


(a;+Ax*)«. 


20. KL+U)+iP+Pi)]' 



121 



§ 2. THE SQUARE OF THE DIFFERENCE OF TWO 
NUMBERS 

87. Law. Perform the operation denoted by (a— 6)^. 
How does the result differ from that obtained by squar- 
ing (a+6)? 

Write the law for the square of the difference of two 
quantities. 

88. Examples for Sight Work. Copy the following 
examples and by inspection write what each equals. 

1. (r-s)«. 2. (3x-42/)2. 

3. (ab^-y^y. 4. (12-7s)«. 

6. (a^-Qx-O*. 6. (8c*-116-2)«. 

7. (Qr-^-Ms*)'. 8. {&tf''-12h^-^')K 

9. (o^^-t^-m'- 10. (a363-20a-»6)2. 



11. ((i-'-/-V-*)'. 



12. (sab-j^^r+^y. 
13. (3m-2sina;)«. 14. (46-5 cos a;)«. 

16. (Ui-^-^isLnxj. 16. (.07a;-^~.62/«+2)2. 

17. (x+2-.y-«-3)2. 18. (C^-i-6t+2)2. 

19. (x-Ax)«. 20. [(L+Li)-(P+P01*. 

* Read delta-x. A is the Greek capital D. Ax denotes one quantity 
and its square is therefore Ax* and not A^x*. 



Digitized by 



Google 



122 TECHNICAL ALGEBRA 



§ 3. THE PRODUCT OF THE SUM AND THE DIF- 
FERENCE OF TWO NUMBERS 

89. Law. Perform the operation denoted by (a+b) (a — 6) . 
Write the law. 

90. Examples for Sight Work. Copy the following 
examples and by inspection write what each equals. 

1. {x+y){x-y). 2. (2r+3s)(2r-3s). 

3. {ab+y)(ah-y). 4. (13+6r)(13-6r). 

6. (8s-2+10r3)(8s-»-10r»). 6. (5aa;2+4y-»)(5aa:2-4y-»). 

9. (x-«+32/»)(a;-*-32/»). 10. (5V'+4T^)i5V'-AT^). 

11. (A;'*"Hp')(A;'*""^-p'). 12. (56+sin2a;)(56-sin2a:). 

13. (26+ COS* x) (26 -cos8 x). 14. (46+tan x) (46 -tan x). 

16. (.02x-»+.52/2)(.02x-»-.52/2). 16. (Aa;+.2A2/)(Ax-.2A2/). 

17. (.8a2t;-3+.09i/)(.8a*z;-3_.o%). 

18. (x^-*+15r-^y){x^y'^-15r-^). 

19. (a;-"+3o2i/-i)(a:-"-3o2i/-i). 

20. (^'"'+|s"«)(6--2_|,-sy 

21. [{c^+d^)+(e'+PMc'+d') -(e'+/^)]. 



§ 4. THE DIFFERENCE OF TWO CUBES 

91. Law. In the examples in division when you divided 
the difference of two cubes by the difference of the roots, 
what was the result? 

Write the law. 



Digitized by 



Google 



92 



LAWS OF NUMBEES 



123 



92. Examples for Sight Work. Copy the following 
examples in the work-book, followed by an equality sign, 
and write the results by inspection. 



x-y 

3437»-8s« 
4972+i4s7+4s2* 

27<»~125 
^- 3«-5 • 

64^ 72r 
4^"9^ 



9. 



11. 
13. 

16. 

17. 



64d8n-27y-» 
4dn*-32/-i * 

g 
27 ^ 



2 



m— rTn^p*""* 



1331g»-1728e» 
12122+ 13262+ 1446»' 

x-y 



^^- ^.^ 



4. 
6. 
8. 

10. 

12. 
14. 

16. 

18. 
20. 



64 -L» 



4-L* 

8-64L» 
4+8L+16L2' 

125ig»-2167» 
25i2«+30i2F+36F«' 

fe»r»-8t;--« 
^2r«+2An;-«+4t>-«* 

8a»6--»-512y» 
2a6-i-82/ • 

125<-»-216m;" 

64a;»-125y-» 
16a;2+20a;y-»+25y-«' 

(3r+2s)8-(4p)» 
(3r+2s)-4!; ' 

{x+yy-(a+hy 
,(x+t/)-(a+6)- 



Digitized by 



Google 



124 



TECHNICAL ALGEBRA 



93 



§ 6. THE SUM OF TWO CUBES 

93. Law. Refer to your work in division and write 
the law. 

94. Examples for Sight Work. Copy these examples in 
the work-book, followed by an equality sign, and write 
the results by inspection. 



1. 



6. 



x+y' 

216A»+343a;» 
• 36A«-42/ix+49a;«* 

27x^+125y* 



9. 



11. 



13. 



16. 



17. 



19. 



3x+5y ' 

2162"+ 1 
62^+1 • 

216d-»+27e» 
36(i-«-18d-»e+9e«' 

64d»a;+8a;»y» 
4dx^+2xy ' 

d»+17282» 
d+12z ' 

8a;+64y 

2x*+42/** 

(5a;+2y)»+2» 
(5x+22/)+2* 

(x+yy+{v+zy 
(x+y)+(v+z) • 



2. 



6. 



8r»+278» 
2r+3« • 

512a«+7296» 
8a»+96» • 

64a-»+8fe« 
4a-i+26«* 

C+d 



10. 



12. 



14. 



16. 



18. 



20. 



1728/ft+64r-« 
144/g*-48i2«a;-«+16a;-** 



g^-gh+h^' 

512e-»+27g« 

8e-i+32« * 

r+{t+v)' 

x+y 
<^+</y' 

(a+b)+{r+s) 
\^a+h+\^r+s 



Digitized by 



Google 



96 LAWS OF NUMBERS 125 

§ 6. THE SQUARE OF ANY POLYNOMIAL 

96. Law. Perform the operation indicated by 
(2n+36+4a:3_ 52/2)2, 

In the result do you have the sum of the squares of each 
of the terms? 

Do you also have plus twice the product of each term 
into each which successively follow it? 

Square any other polynomial and see if the result cor- 
responds. 

Write the law. 

96. Examples for Sight Work. By application of the 
law expand* the following: 

1. {a+h'-2c+d)K 2. (e+Sf-Ag-h^. 

3. 0*-^-5m-fi+4)«. 4. (3p-5g-2f«-9)«. 

6. (V7+2y-4z«-l)«. 6. (3ax^+2y-5by+l)K 

X -8-I6-2) • 
9. (10-llx-12a;»-13a;»~14a;*)». 10. (4m+2r-3s-2t;~8)«. 
11. (y-"+32/'*-*+92/'*-62/-'»-0». 12. (7a;*-6a;»-2a:«-5a;-9)«. 
13. (\/d-V4f-3x»+7x~8Vx)*. 14. iax+hx+cx+ay+dy)\ 

16. (-3y*-62/-7y»-2y»-3)«. 16. {ax^+br^-2cx-d*-'9)K 

17. (V3x-V^+3a-V46)« 18. (a;'»-a:»'»-2a;*'»-a;'*+i)«. 

♦ Expand means to raise to the indicated power. 



Digitized by 



Google 



126 TECHNICAL ALGEBRA 97 

§ 7. THE EXACT DIVISOR OF A POLYNOMIAL 

97. The Factor Law. What value of x will make 
3(x— 7) equal to zero? 

If any factor of a product is zero what does the product 
equal? 

What value of x will make a:— 4 equal to zero? 

If a:— 4 is a factor of any expression will the expression 
equal zero when x— 4 equals zero? 

In (a:— 3)(aj — 6) name two values of x which will make 
the expression equal to zero. 

What value substituted for x will reduce to zero, every 
expression in which a:— 2 is a factor? 

An expression having a:— 9 as a factor will reduce to zero 
when what is substituted for a:? 

An expression having a: — 12 as a factor will reduce to 
zero when what is substituted for. a:? 

Therefore an expression having a:— a as a factor will 
reduce to zero when what is substituted for a:? 

Therefore if any rational integral expression containing 
a:, does not reduce to zero when a is substituted for x, is it 
divisible by a:— a? 

Law. Any rational, integral expression in x, which reduces 
to zero when x equals a, is exactly divisible by x -a. 

98. Illustration. (1) To determine an exact divisor or 
factor of 6x^ — 6a:^+l we substitute 1 for x in the ex- 
pression. 

Since it reduces to zero we know that it is exactly divisible 
by a: — 1. 

(2) To determine an exact divisor of x^+Gaj^+llar+G 
we substitute for x, some factor of 6. 

But since the signs are all positive it is evident that no 
positive factor of 6 will reduce the expression to zero. We 
therefore try —3 instead of +3 and obtain the following: 

-27+54-33+6 = 0. 



Digitized by 



Google 



99 



LAWS OF NUMBERS 



127 



The expression is therefore exactly divisible by a:— (—3), 
which by the law of subtrahends becomes x+S, 

(3) To determine the exact divisor of x^— Tx^+lGx — 12 
we substitute for z some factor of 12, say 3. When this is 
substituted for x we have 27-63+48-12 = 0. 

Therefore x— 3 is an exact divisor of the expression. 

When the known term has several factors several trials 
must sometimes be made before the correct factor is deter- 
mined. 

99. Examples. Determine an exact divisor or factor 
of each of the following expressions: 



1. x*+Qz+5. 
3. a:»-43x+42. 

6. 36a;«-61x+25. 

7. x*+x^-20. 
9. 22/»-%+39. 

11. y»-13y-12. 
13. x»-21x+20. 

16. x»-10a;2+29a;-20. 

17. 3A»-6A«-3A+6. 
19. «<-252«+60«-36. 



2. a;»-3x*+4. 

4. 4c«-7x+3. 

6. 16a:«+49x-60. 

8. a:»-19a:+30. 
10. 6a;8+7a;+13 
12. 3y*-7y^-20. 
14. a;»-14a;«+35a;-22. 
16. r»+9r«+26r+24. 
18. s*+s»-16s«-4s+48. 
20. 4x*-19x»-37x«+44r-12. 



Digitized by 



Google 



CHAPTER VI 

FACTORING 

Section 1, A Common Factor. Section 2, Grouping. Section 
3, The Difference of Two Squares. Section 4, The 
Difference of Two Cubes. Section 5, The Sum of Two 
Cubes. Section 6, The Trinomial. Section 7, The 
Polynomial. Section 8, Special Expressions. 

100. Definitions. The factors of a number or expression 
are the quantities whose product equals the number or 
expression. 

Factor means maker and factors are numbers which 
make other numbers when multiplied together. 

Factoring is the process of resolving an expression into 
its factors. To resolve an expression into its factors is to 
determine its factors. 

101. Cases of Factoring. This subject will be presented 
under the following cases: 

Case 1. A common factor. 

Case 2. Grouping. 

Case 3. The difference of two squares. 

Case 4. The difference of two cubes. 

Case 5. The sum of two cubes. 

Case 6. The trinomial. 

Case 7. The polynomial. 

Case 8. Special expressions. 

128 



Digitized by 



Google 



108 FACTORING 129 

§1. A COMMON FACTOR 

102. Illustration. In the expression 

25x*+15x3«30x2-40x+100 

what is the largest factor common to all the terms? 

Write this factor before a parenthesis within which is 
the result obtained by dividing the expression by this 
common factor. 

What is the largest factor which is evenly divisible in 

Express the factors in the same form as in the first 
illustration. 

103. Examples. Factor the following expressions: 

1. 3a«c-27a<c»+9a»c»-15a^^ 

[ 2. 5ax^—Sx*+cx\ In the result what is the coefficient of x'? 
Why? 

3.phy*-fh^^+ph% 

4. Sz—hz+cx—tx. In the result what is the coefficient of x? 
Why? 

6. ^^^^^^^+Wu 6. ^+Wt. 

7. Ci+Cin-2Ci. 8. ^x^-VIixy^—rx^yK 

9. U^-ai^+U^--t^+tK 10. 8x«-ax«-7x»-(c+d)x«. 

11. ay^-by^-Ay^+dy^-yK 12. 8a:«-16x«+24x-8. 

13. acx*-hx^+dx^-x\ 14. -7 ^+^. 

t Of 7t* 



Digitized by 



Google 



130 TECHNICAL ALGEBRA 104 

7s 2s» 6rs» ^ ^ az 

17. C^-^+a-CK 18. ^-8.^.. 

7t;» 16r 

125t;Ht;» 
19. 5cH«' — - — — -. 20. 3rsma-6isma-12»sma. 

8ar* 

21. il tan ^+ila tan S+SA^ tan d. 

22. 34oa;*-156a;*-rx«-x«. 

23. 8a62cA~46«A+1662^»m-1246». 

24. C0w«6a;-72m*62+126«m«. 

26. 8tana+16tan«a-12tan»a. 

26. a cos* ^ -3 cos 6+r cos^ 6. 

27. 5V X —y —awx —y+h\/x — y. 

28. 26V4a+«+3dV4a+« -7/V4a+i. 

29. x^Vcos ^-1 -i/^Vcos ^-1 -Vcos d-1. 

30. x«(x~t/)*--2xy(a;-2/)«+2/*(aJ-y)*. 

31. 12x«-172&r»-156x+x*. 

3a6x» _ 9a«&» ' 27a6»y 
4t; 16 "^ 8x * 

-_ Sfi 25fiX . ^^_ 

34r2^_51r5^_68r^* 
' a6c 86c a6c ' 

9 5c 

104. Removal of Common Factors. In the examples 
in the subsequent cases and in all work involving factoring, 
common factors if present should be removed before the attempt 
is made to factor under other cases. 



Digitized by 



Google 



106 PACTOKtNG 131 



§2. GROUPING 

106. Definition and Illustration. In conventional lan- 
guage, grouping is the rearrangement of the terms of an 
expression so that terms having common factors follow 
each other, the terms which have a common factor being 
inclosed (grouped) in parentheses. This makes errors 
possible both in copying the expression and in changing 
the signs of terms which are to be grouped in a negative 
parenthesis. By the use of grouping linos as shown in 
the illustration which follows, copying and inclosure and 
therefore the mistakes mentioned, are avoided. 

Factor ax — ay+cy — cx, 

ax—ay+cy—cx = 
a{x-y)-c{x-y) = 
{a-c){x-y). 

106. Explanation. In the preceding illustration the 
first term and the second term have the common factor a; 
therefore we group them. 

The third term and the fourth term have a common 
factor ~c; therefore we group them. 

a is contained in the first term x times and in the second 
term —y times. Therefore the factors of the first and the 
second term are a and x—y. 

The factor common to the third and to the fourth term 
is — c; contained in the third term x times and in the fourth 
term —y times. Therefore the factors of the third and the 
fourth term are —c and x—y. 

Adding the coefficients of x—y we obtain {a—c){x — y). 

Observe that — c instead of c was used as a factor of the 
second group because the factoring of the first group shows 
us that x— 2/ is a required factor of the second group. — c 



Digitized by 



Google 



132 TECHNICAL ALGEBRA 107 

gives this by laws of division, while +c would give — x+y. 
This means that in factoring by grouping, the signs of all 
common factors after the first are determined by the first paren- 
thesis and in no other way. 

107. Another Solution. The expression in paragraph 
105 may also be grouped and factored as follows: 

ax—ay+cy--cx = 
x{a'-c)—y{a—c) = 
(a-c){x-y). 

The first and last are grouped because they have x as 
a common factor. The second and third are grouped be- 
cause they have — y as a common factor. 

X is contained in the first group, a—c times; — y is con- 
tained in the second group, a—c times. 

The factors are therefore a—c and x—y. 

108. Examples. In the following indicate the grouping 
by light grouping lines, and factor: 

1. ac—as—rs+rc, 2. vx+yz—vy—xz. 

3. v^x^-yz-v^+xH. 4. KL+KiLi-KLi-KiL, 

5. gH^+wf-ft^-ghJO, 6. 2e^+^h^-'^eh-'2eh, 

7. 4a5«+5a62-5a26-465«. «. ^0g''r^-2ay+\0ag'y-^rK 

9. 1 Ic^r Vx -45a2c'p«r< - 15a«c2c'a;+33p*r<t? V. 

10. * ax —csy+rx —ry+csx —ay, 

11. 6ac-x2-3cx+2ax.. 12. 8x«-rc-2rx+4cx. 

13. 2/'+2/*+2/+l. 14. cz'^-cz-cyz+cy+z-l. 

15. ac-t-a-3c2-3c-4c-4. 16. pg-p-6g+6-<5+<. 

17. 2a(i«-dV'+/+2d/-4ad-2a. 18. a{x+y)-h{x+y). 

19. {s+c)k+{8+c)v. . 20. 7r-6r+6s-7s. 

* Group by threes as follows : ax—csy-\-rx—ry-{-c8x—ay. 



Digitized by 



Google 



Ill FACTORING 133 

21. hx+l-h^x. 22. 12+182/ -72y«-48y«. 

23. 2l2«+2+2+422«. 24. fx -bx -kx -kx^ -bx^+fxK 

25. t{v-d)-(d-v). 26. 3(x-2/)«-9(2/-x). 

27. x^+2xy+y^+cx-x-y+cy. 28. 6»+6«-36-3. 

29. 1-^+i-^i. 30. 5-5t-t^-{-tK 

§ 3. THE DIFFERENCE OF TWO SQUARES 

109. Illustration. Perform the operation indicated by 

{x+y){x-y). 

By what law of numbers could you determine the prod- 
uct without the necessity of multiplication? 

Therefore the difference of two squares will always 
resolve into what two factors? 

Therefore 

^2— c^ = (a-|-c)(a— c). 

110. Explanation, a^—c^ is the difference of two squares. 
It is therefore resolvable into two factors; one, the sum 

of the roots, the other, the difference of the roots. 

The first root is o, the second is c. 

The sum is a+c; their difference a—c; therefore the 
factors are a+c and a—c, 

111. Examples. By inspection write the factors of 
the following: 

1. a»-6*. ^2. c»-d*. 3. r*-s» 

4. C2-M 5. D^-EK 6. A*-/«. 

7. V^-TK 8. V^-RK 9. W^-X^. 

10. 64x8-81y-». 11. 2/*-2». 12. g^-bH^ 

13. 4x«-92/». 14. 25««-49r2s«. 15. x-y. 

16. m^-n*. 17. s«-r«». 18. h^-k-K 



Digitized by 



Google 



134 TECHNICAL ALGEBRA 112 

19. z-^-y*. 20. 32/«-162»». 

21. (x-y)«-(r+s)«. 22. (3t^+40»-(2c+6)«. 

23. (x»-2/)«-(a+6-c)«. 24. 24(r-e«)«-24(A-3A;)«. 

26. (3d+2e»)*-(7r*-5/)«. 26. (x+2/)«-s«. 

27. 9x^'-3QaH*-2ixz+lQzK 28. r«-4s«+«*-2re. 
29. l-x^+2xy-yK 30. a*+2a«6«+6«-a«6». 

31. * D^+F^ -E^ -m+2DF-2EH. 

32. *a2+2a6+62-(c«+2ce+e2). 

33. *SF5+4-12i«J-52+9i«J«-16F«. 

34. *2t;M;-t(;«+a;«+2/'-t^*+2xt/. 

§ 4. THE DIFFERENCE OF TWO CUBES 

112. Illustration. Divide t?—]^ by x—y. By what 
law of numbers could you have written the quotient by 
inspection? 

Therefore the difference of two cubes may always be 
resolved into what two factors? 

Therefore 8C3-64r3 = (2C-4r)(4C2+8Cr+16T2). 

113. Explanation. 803-647^ is the difference of two 
cubes. 

It is therefore resolvable into two factors: one, the 
difference of the roots; the other, the sum of the squares 
of the roots plus their product. 

The first root is 2C; the second is 47. 

Their difference is 2C— 47; the sum of their squares 
plus their product is 4C2+8Cr+16T2. 
' Therefore the factors are 2C-4r and ^C^+^CT+l&T^. 

* Group the terms by threes in two parentheses so as to show the 
difference of two squares. 



Digitized by 



Google 



116 FACTORING 135 

114. Examples. By inspection write the factors of the 
following: 

1. c*-dK 2. 8y»-27s». 

3. x^-SzK 4. 64^»-r». 

5. L»-125D«. 6. y3_2162-». 

7. Sp-k-^ 8. 343a»-6- »*. 

9. l-xK 10. 8y»-l. 

11. l-1728r-»s». 12. 8a;»y"-272/*. 

13. sin» a - cos» a. 14. 125 tan» 6 -343 sin» 6. 

15. x'-l. 16. 27g-*-h\ 

17. l-64A;«-«. 18. A^'-B" 

19. (p+g)*-(s-0*. 20. l-(l+2/)». 

21. 64 cos» ^ -8 sin» e, 22. (3a+z)^ - 1. 

23. (56«-6y)»-r«. 24. (x-y)» -(««-»«)». 

26. C-"-(L2+8M«)«. 26. (7a6c+l)»-64d-». 

27. (29ey-30)«-125^-«'. 28. x^-iSy^-lU^y^^ 
29. (k^+Ugy-Sr-^^. 30. 1 -(2x-32/+52)«. 

31. l-(6a;«-a2-y)-«. 32. (116A-12B»+1)'-1. 

33. 64 tan» 6 -343 cot» d. 34. (sin* d+ cos« ^)» -1. 
35. 8(sin« <t>+ cos* <^)» -512 tan» <t>. 



§ 5. THE SUM OF TWO CUBES 

116. Illustration. Divide a^+l^ by a+b. By what 
law of numbers could you have written the quotient by 
inspection? 

Therefore the sum of two cubes may always be resolved 
into what two factors? 

Therefore 27J53+125Z)3 = (3J5+5Z))(9J52-15J5Z)+25Z)2). 

116. Explanation. 275^ + 1251)3 is the sum of two cubes. 
It is therefore resolved into two factors: one, the sum 



Digitized by 



Google 



136 



TECHNICAL ALGEBRA 



117 



of the roots; the other, the sum of the squares of the roots, 
minus their product. 

The first root is SB; the second is 5D. 

Their sum is 3J5+5D; the sum of their squares minus 
their product is 9J52-15J5Z)+25Z)2. 

Therefore the factors of 27B^+125I>^ are ZB+5D and 
9B2-15J5Z)+25Z)2. 



117. Examples. By 

the following: 

1. x^+y\ 

3. R*+S\ 

5. L»+F». 

7. Sx'+l. 

9. MA*+B*. 
11. 125C-»+8i)-». 
13. aY+^"d"". 
15. x^+2lQz-». 
17. l+2/». 

19. 1728^»+729F-«. 
21. (5 sin a— cos a) *+l. 
23. (7a26»c+6d)»+(a;+y)«. 
26. (V»-y)»+l. 



inspection write the factors of 

2. l + (l+2/)». 
4. l+Mh^-^^\ 
6. (x+2/)»+(^+A)«. 
8. (e-/)»+((7+/i)». 
10. (6r+l)»+l. 
12. l+(2ax+6-6«)». 
14. (A;2-6^«)»+8A-»3^+«'. 
16. (sm2^+cos*^)»+l. 
18. 343+8(sin2 ^)». 
20. {A^+B^)^+{E*^F*)K 
22. l+(V2^+36)^ 
24. a-«6-9+(a-6+c2)3. 
26. a;3+3a;22/+3a;i/2+2/3+l. 



§ 6. THE TRINOMIAL 

118. Illustration. Perform the operation denoted by 
(a+3)(a-4). 

The product has how many terms and therefore is 
called what? 

Each factor has how many terms and therefore is called 
what? 



Digitized by 



Google 



119 FACTORING 137 

The first term of the product is the product of what 
terms of the factors? (Answer first terms or last terms 
as may be correct.) 

The last term of the product is the product of what 
terms of the factors? 

The second term of the product is the product of the 
inner terms plus the product of the outer terms as follows: 

(a+3Xa-4). 

The product of the inner terms is 3a 
The product of the outer terms is— 4a 

The sum of the products is — a which is what 

term of the trinomial? 

Therefore (a+3)(a-4) =what? 

119. How a Trinomial is Factored. In the preceding 
illustration a trinomial resulted from the multiplication of 
two binomials. 

The trinomials under this case of factoring are all re- 
solvable into two binomial factors which may be determined 
by the inverse process of that just given. 

For example, L^ — 15L+56 is factored as follows: 

This is a trinomial because it has three terms. 

It is therefore resolvable into two binomial factors. 

We therefore make two parentheses each large enough 
to inclose a binomial. 

This gives L2-15L+56 = ( )( ). 

The first terms of these binomials are two numbers 
whose product is L^, therefore L and L. 

This gives L2-15L+56 = (L )(L ). 

The last terms of these binomials are two numbers whose 
product is 56 and the algebraic sum of whose products 
when multiplied by the first terms of the binomials, is — 15L. 

The last terms are therefore 7 and 8. 



Digitized by 



Google 



138 TECHNICAL ALGEBRA 120 

This gives L2-15L+56 = (L 8)(L 7). 

The product of the inner terms (L 8)(L 7) is 8L. 

The product of the outer terms (L 8)(L 7) is 7L. 

But their sum must be — 15L. ^^* 

Therefore both must be negative and the two minuses 
will give +56 in the product. 

Therefore the factors are L— 8 and L— 7. 

This gives L2-15L+56 = (L-8)(L-7). 

The order and method of operation here given will 
factor any trinomial under this case. 

120. Pairing of Terms. In a trinomial like 6+7Q-5Q2 
care must be taken that the terms are properly paired and 
the signs properly placed. 

The factors of this trinomial are (3+5Q)(2— Q). 

But in trying to determine them we might have written 
(3-5Q)(2+Q), or (3+Q)(2-5Q), or different factors for 
6 or different signs, none of which, however, would give 
6+7Q-5Q2. 

121. Special Instructions. In factoring a trinomial 
observe first whether all terms have a common factor; 
if they have, remove it and then factor as a trinomial. 

Observe second whether the trinomial is a square. If 
it is, it may be factored on sight by the special laws for the 
square of the sum or the square of the difference of two num- 
bers, more quickly than by the general method here given. 

For instance ij^^—^rsx+a^ is the square of 2rs—x and 
the factors {2rs^x){2rs—x) may be immediately written. 

In a trinomial like L^— 15L+56, after the factors of 
L2 are written, the problem is merely to determine two 
numbers whose product is 56 and whose sum is —15. 

This is always the case when the coefficient of the first 
or the last term of the trinomial is unity. 

122. Examples. Enter the following in the work- 
'"ook with an equality sign and two parentheses, each large 



Digitized by 



Google 



123 



FACTORING 



139 



enough to inclose a binomial. If there is a common factor 
place it before the first parenthesis. 



1. z*+Sz+2. 

3. 2S^+i2S+lSO. 

5. A2+54A+729. 

7. 9y'-2Wxy+mx. 

9. 6r2+3r-45. 
11. 4a;+4\/^+l. 
13. 2p+l7f+35. 
15. 12aA;«+69aA;+45a. 
17. 9«2'-2-36r-V+36i 
19. 7a2-14a5+76*. 
21. 54A2-15A-56. 
23. 3662-836d+35d«. 
25. 21a^b*+20abd-9MK 
27. 24a*x^-39aH'x-99i^ 
29. 15.T«+224x-15. 
31. z*+it+V)z+tV. 



^v 



2. t?«-llt^+30. 

4. c*+9c«f«+14<«. 

6. c*-22c«d+105(i«. 

8. 3W^+nW+6. 
10. 12L2-68L+40. 
12. 2y«+3a;2/-2a;«. 
14. a:^+31x'-32. 
16. 156«-776+10. 
18. 9a;«+43a;-10. 
20. 60a«62+300a6c+375c«. 
22. xV -7x2/ -120. 
24. 10R^+19RV -l^VK 
26. 35x^+34x^2 -1442«. 
28. «.+|a+^. 
30. 44(7/i-20A«+15(7«. 

32. (x+2/)«+2(x+2/)-15. 



§ 7. THE POLYNOMIAL 

123. Explanation. Since a poljrmonial by definition is 
an algebraic expression of three or more terms and there- 
fore includes the trinomial, it is necessary to state that the 
poljTiomials which are considered under this case are of the 
same type as those in paragraph 99. They are factored 
by the factor law as given in paragraph 97. When one factor 
has been determined by this law the second factor is deter- 



Digitized by 



Google 



140 TECHNICAL ALGEBRA 124 

mined by division. If this second factor is not prime 
it is often factorable as a trinomial or by some other case 
of factoring. Should it prove troublesome it may be read- 
ily factored by the factor law. 

124. Examples. For this list of examples use those 
in paragraph 99. Copy each in the work-book and deter- 
mine the factors. 



§ 8. SPECIAL EXPRESSIONS 

126. Sum or Difference of Same Power. The cases 
presented in the previous sections include those that are 
essential in the subsequent study of mathematics. Some- 
times, however, one is asked to factor the sum or difference 
of the same odd powers or the difference of the same even 
powers when the odd powers are not cubes nor the even 
powers squares. In such instance it is only necessary to 
remember the following in which a and b denote any munbers 
and n denotes an integer: 

1. a**— 6** has the factor a—b whether n is odd or even. 

2. a^—b^ has the factor a+b when n is even, 

3. a^+b^ has the factor a+b when n is odd, 

4. a^+b^ has not the factor a+6 or a— 6 when n is even. 

In other words, 

(1) The difference of the same even powers is divisible 

by the difference of the roots and by the sum of the 
roots. 

(2) The difference of the same odd powers is divisible 

by the difference of the roots and not by the sum. 

(3) The sum of the same odd powers is divisible by the 

sum of the roots and not by the difference. 

(4) The sum of the same even powers is divisible by 

neither the sum of the roots nor the difference of 
the roots. 



Digitized by 



Google 



126 



FACTORING 



141 



Under the paragraph number and heading copy the fpl- 
lowing and complete the entries, specifying whether the fac- 
tor is a+b or a— b or both or neither, and give reason: 



No. 


Example. 


Factor. 


Reason. 


No. 


Example. 


Factor. 


Reason. 


1 


a^-b^ 






6 


x^^y* 






2 


a^i-¥ 






7 


x'^-y'^ 






3 


a»-6» 






8 


x'^+y' 






4 


a»-h6« 






9 


R^-S^ 






5 


x*-y* 






10 


/e«+/S« 







126. Special Methods. Some expressions are made 
easily factorable by the following methods: 

(1) An expression may sometimes be factored by first 
adding and subtracting the same quantity as in the case of 

If the middle term were 2x^y^ the expression would be 
factorable. We therefore add x^y^ and indicate the sub- 
traction of sc^y^ as follows: 

(a^+2a:2^+t/*)-xV. 

The expression may now be factored as the difference 
of two squares. 

In the case of 4:a'^ — 21a^hf^+9b^ we add and denote the 
subtraction of Qa^fr^ as follows: 

(4a* - 12a262+9&4) -9a^1y^ 

when the expression is seen to be the difference of two squares. 
Factor the following: 



1. 4^:4-13x2+9, 

2. Sly^-3^a^y^+y^. 

3. 121R^-UR^S^+S\ 

4. x^+7^y^+y^. 



5. c(^+^. 

6. (^h^-2l(?h^+SQ. 

7. 36eH49. 

8. xio+642/2. 



Digitized by 



Google 



142 TECHNICAL ALGEBRA 126 

(2) The factoring of expressions like sfi—y^ is simplified 
by observing that x^—y^ = {3i:!^)^ — {y^)^. 

Write the factors. 

(3) A polynomial which is a square may be factored 
by the inverse of the law for the square of any polynomial. 

For example 

4x2+16x2/--402/z+162/2-20a;z+2522 

is seen to be a square because three of its six terms are 
squares and each of the other three terms is plus twice the 
product obtained by multiplying each root into each of 
those which follow. 
Thus 

4r2+162/2+2522+l6a:2/-2Oa:2-4O2/0 = (2a;+42/-52)2. 

This inverse law * makes it possible to extract by inspec- 
tion the square root of any algebraic expression which is 
a square (usually called a perfect square by way of emphasis). 

Determine the factors of the following: 

1. 3ex^+z^+25y^+l2xz+myz+mxy. 

2. (P+e^+^P+h^+2ed+Mf+2dh+^ef+2eh+^fh. 

3. 9fc2+64m2+36n4-48fcm-36fcn2+96mn2. 

4. 121p8+81gi«+49r6+16s4+198pV-56r3s2-126(/V 

- 154p4r3+88pV+72g5^. 

5. a^a^+10axz+2al^xy+25z^+l0b^yz+b^y^. 

6. Mcdyz+Q^c^z^ - 2^d7?y+^7^ - A&cx^z+ IGcPi/^. 

7. 100x2^i50a:2/3_|.35a65-4_96^^5-22^_^54^6__i20a36"2a;. 

8. 25L2+169F2+150LS-4+130LF+225>S-8+390F>S-4. 

9. 196a:2-252a:2/+8l2/2. 

* See paragraph 95. 



Digitized by 



Google 



126 FACTORING 143 

10. 324^-756r-3^2+441r-^ 

11. 576x^-15S&a?y^+102^y^. 

12. 9fc2+25j2+49m^+81g2+30fc;-42fcw2-54Jfcg-7Q/m2 

-90ig+126m2g. 

(4) A polynomial like 16y^-^0yz+25z^+32xy -4:0x2 
+153? which at first sight might be thought to be a square, 
may be factored as a trinomial by the foUowing^ arrange- 
ment: 

(42/-52)2+8x(4?/-52)+15a:2^ (Write the factors.) 

(5) An expression like 32a^^-b^^ if written (2a3)5-(62)6 
may be factored as the difference of the same odd i>owers 
of two numbers. 

Write the factors. 



Digitized by 



Google 



CHAPTER VTI 

FRACTIONS 

Section 1, Reduction. Section 2, Addition and Subtraction. 
Section 3, Multiplication and Division. Section 4, 
The Complex Fraction. 

§ 1. REDUCTION 

127. Definition. A fraction is an indicated division of 
one number or expression by another number or expression, 
the first number being placed above the second with a hori- 
zontal (fraction) line between. 

A fraction may also be denoted by one number following 
another with an oblique line between. 

The terms of a fraction are the numerator and the 
denominator. 

The mathematical basis of most of the work of this 
chapter is the 

Fraction Axiom: If both terms of a fraction are 
multiplied or divided by the same quantity the value of the 
fraction is unchanged. 

Reduction to lowest terms is the division of both nu- 
merator and denominator by the same factor until they 
can no longer be divided because prime to each other. 

Numbers are prime to each other when not evenly 
divisible by the same factor. 

128. Illustration of Reduction. The fraction ^ » ^ _.., 

is reduced to its lowest terms by factoring both numera- 

144 



Digitized by 



Google 



129 



FRACTIONS 



145 



tor and denominator and by canceling the same factors 
as follows: 

(a;+5)(a;-6) 



(x+5)(2x'-S) 

129. Examples in Reduction. Reduce the following 
fractions to their lowest terms. If cancellation marks 
are shown slant them when possible, in the same direc- 
tion. The factors may be placed above and below the 
numerator and denominator as shown in the illustration, 
or written following the given fraction. 

Small, carefully made parentheses are desirable. 



1. 



3. 



9. 



11. 



13. 



16. 



17. 



h+k 

4x«-19a;-5 
2x*-7a;-15' 

2x^+3x-U 
l(te»+33x-r 

2av+bt+2hv+(U 
2cv-2dt+ct-4dv' 

6ar~5A;s+15A;r-2fla 
21H+s-3r-7st ' 

5a6+753«-135r-« 
90a6-180s* ' 

Sx*-8y* 



6x»+12xy+6y*' 

15x»-6a;«-12x 
6a«-246x 



2. 



6. 



10. 



12. 



14. 



16. 



18. 



mx^yY 

x^+9x+U 
' 3x«+21a;+30' 

2x»-27x+13 



3x«-37x-26* 

6x«+7x~20 
12x2+32x+5* 

(c-2d)16? 
400 ' 

17(^L»+17dLi» 
85L+85Li • 

96«+4o«-196r-«-12a6 
4a-28r-*-66 

15x»-llx«-10x+4 
10x«-4x-8 ' 

2a;3-4y2-4xy+2x«y 
ax*-'2ay+bx^+2by' 



Digitized by 



Google 



146 TECHNICAL ALGEBRA 

2a:»+7a;»-7a:-12 



130 



19< 
20 
21 
22 



24. 



2x2+5a;-12 * 

3ax+2r»-7x+6-6a 
3x+eax-Q-12a ' 

20dr+25r^-30rx+id^ - 12dx+9x* 
5r*-8rx+3x*+2dr-2dx 

Mx+y)^+Sx+ S y--m 
12a;+122/+60 * 

2a«V^3-h tan ^-5V ^3+ t an e+h\^3+ tan g 
7V^3+ tan ^ 

2fl(i2-rf2;-4flrf--f+2a+2d/ 
5/-10a 



§ 2. ADDITION AND SUBTRACTION 

130. Addition of Integer and Fraction. The algebraic 

sum of an integer and a fraction is called a mixed expression. 

The process of reduction to simple fractional form is the 

same as the process of reducing a mixed number to an 

improper fraction. Each is merely the addition of the 

integer and the fraction. 

5 

Thus 3f is only a convenient expression for 3+^ and 

o 

5 
the addition of the 3 to the x- gives the so-called improper 

29 
fraction -^. The process may be explained in two ways: 

8 



(1) 



Therefore 



1=8- 

24/5^29 
8'*"8 8' 



Digitized by 



Google 



181 ■ FRACTIONS 147 

(2) Every integer may be considered a fraction whose 
denominator is 1. 

Therefore 3f=3+|=|+|. 

3 

To reduce j to 8ths the denominator must be multiplied 

by 8. 

Therefore the numerator must also be multiplied by 8, 
the principle involved being the 

Fraction Axiom. If both numerator and denominator 
are multiplied by the same quantity the value of the frac- 
tion is unchanged. 

The preceding explanation should make clear the reason 
for the following rule of both arithmetic and algebra: 

Rule. To reduce a mixed number or expression to 
fractional form, that is, to add an integer to a fraction, 
multiply the integral part by the denominator, add the 
product to the numerator, and write the result over the 
denominator. 



Thus 3x4 



x+5 x+5 



. r o 7x-42/ 4a^lP-Uab+6-7x+4y 
aft— 3 ao—3 

131. Addition of Fractions. In order to add fractions 
they must be reduced to the same denominator. That 
the result may be as small as pospible, the fractions should 
be reduced to the least common denominator which may be 
defined as the smallest number in which each denominator 
is evenly contained. The simplest way of determining 
the least common denominator is to resolve each denomi- 
nator into its factors and write each factor as many times 
in the least common denominator as it is used in the given 
denominator. 



Digitized by 



Google 



148 



Thus 



TECHNICAL ALGEBRA 
3a+5 a-1 



id2 



(a+x)(a+x) {a+x){a-ax+z^) 

{3a+5)(a-ax+a^)-(a+z)(a-l) 
{a+x)(a+z)(a—ax+a^) 

As shown, a+x is used twice as a factor in the first 
denominator; therefore it must be used twice as a factor in 
the L.C.D. 

Fractions, and fractions and integers, are therefore 
added by determining the L.C.D., multiplying both terms 
of each fraction by it, and collecting like terms in the 
numerator of the resulting fraction. 

132. Examples in Addition and Subtraction. Solve 
the following examples: 



'-•+¥-¥• 



3. 2|/-3+ 



2y+5 



y 



5. 3- 



7. 



x+2y 
X''2y 



Bx-y 
x-S 



-3a;+2. 



x+2 



x'-x-e a:»-2a;-8 3a; 



11. 



a«-9 



a+2 



a«+3a o«+6a+9 



+a+L 



4 4 4 * 

IB .J 3 _5_ 4_ 

^^' a«+l o«-l |l-o«^l+a 



2. 3-+a-X- 

4. ^J+2-6. 
a— 

2-t;* 
8. x«-6a;+6+ 
5 



3a;«-2 

10. -- +'J^±^. 

bi-si-sr+hr^ t+r 

|/^4|/+7^16i/«-49' 

* xy 22/ a:2 2* 



16. — -;; 



x*+3x 2a;«-2a; 



+2. 



Digitized by 



Google 



134 FRACTIONS 149 

X 2a+26 a ^ 2^_1_ 

' a+h a«-a6+6« a»+6» (a+i;)« a«+r« 2* 

^r. a 5X-46 ^ 7a;+26 ^ x 2a;+l . 3 

19. ^^-—^2y^-—, 20. iIZ^-3i^+8. 

oi o . . ^+y . 1 no 1 -49a;» . 1 

21. 2-^+2,+2i^-2^,+— . 22. i^28x+49:c*+l+7i- 

3a+6 _1_ ^^ 2^ 3 _21 _5_ 

^' a^-h^^ia^h)^^"^' '"^ 2^d-2^(d-2)«- 

«^ c 1-c. 2c 1 

25. 1 . 

\+c c c* — 1 1—c 



§ 3. MULTIPLICATION AND DIVISION 

133. How Fractions are Multiplied. As in arithmetic, 
fractions are multiplied by multiplying their numerators 
together and multiplying their denominators together. 
Before this is done, however, all numerators and denomi- 
nators should be factored and all factors canceled as far 
as possible. 

When one fraction is to be divided by another, the divisor 
should be inverted before factoring , 

134. Examples in Multiplication and Division of Frac- 
tions. When possible factor both numerator and denomi- 
nator of the following fractions; apply the law of divisors 
if necessary, and simplify by cancelation: 

2r«+3r 4r«-6r 
4r» 12r+18' 

18t?g -Zav -6a\ ^ !?» -2flt;+fl\ ^ 3a»+2a« 



Zav-Za^ 92;«-4a« (2i;+a)(t;-a)* 

. ^2-^S--20^/S*~/S-2 . S+l 



««-25 ^/S2+25-8 S^+bS\ 



Digitized by 



Google 



150 TECHNICAL ALGEBRA 134 

• x*-y* ^\ lh^x^-l¥y^ ldx-20y )' 

L«~5L-14 . / 2L»-32L+126 L»-11L+18 \ 
(L-2)» " \c«L2-4c2L+4c*^ L*-4 /* 

^ 3x» ^ / 2x»+2a:« 86x^ \ 
3+3a;** \ 2a; 46+46a;V* 

fa+dy 



3+3a;* 

-d« 9a+9c 



-c» ' 3a2+3ac+3c2' 



a;g+a;?/+2/\ ^ 5x»+5y\ ^ 4a« 
o. ~~; ; "X I r~X^ 



X*— xt/+y* x'— 2/' rx+ry 
a;+2 ' x^^z^ \ x—y/' 



a« -7fl+10 . 2a -4 8a»+64 



a2-10o+25 a -5 o«-2a+4' 



y.2_5s ' r«-9r+20 r2-10r4-2r 

6(c-2d)» c^+5c^d^+4ri< ^ c^-4 c»d +4r2cg'' 
c(c2+4d')^ 3(i(c-4d2) • c34-8(i3 

a»+2a6+&'-c\ ^ a-6+c 
13. , 1 . X " 



a+b+c a^-2ah-c^' 

-- x^-y ^ (x-yy ^/ x^-'y^ x'^- xy+xjA 

x*-2xy+y^ \x+y/ ' \x^-{-y^ x^+xy+ijy' 

7a+21 o«-4 . 5a2 



a+3 14a«+56a+56 3a2-6a-24 

x+x^-e 2(x»+2x+4)(x^-x?/+2/^) 
2x+2x*-12 x»+2/3 

a«+5fl+6 a^-25 
5a«+25a 2a2-8* 



Digitized by 



Google 



136 FRACTIONS 151 

^y^-i8y+90 2y'-S Qy^-'30y-{-24:' 

xi-yi-zi-2yz ^ 2x^ -2y^+4xz+2z* 
* x^+z^-2xz-y^ ' 4x2+82/2-422-42/*' 



-(!-)(f-')(^.)- 



§ 4. THE COMPLEX FRACTION 

136. Definition. A complex fraction is a fraction 
having fractions in either numerator or denominator, or 
in both. 

To simplify a complex fraction means to make it integral 
or a simple fraction. 

A simple fraction is a fraction whose numerator is 
integral and whose denominator is integral; that is, a 
fraction both of whose terms are integral. 

In simplifying complex fractions application must be 
made of the following law. 

Law of Divisors: 

(1) Invert every divisor. 

(2) Use it as a multiplier. 

136. Illustration. Given the complex fraction. 

a ar 
1-^. 

This is a complex fraction because both terms contain 
fractions. 

To simplify it is to make it integral or one simple fraction, 
i.e., a fraction whose numerator is integral and whose 
denominator is integral. 



Digitized by 



Google 



152 TECHNICAL ALGEBRA 136 

Operations: 

(1) Simplify the numerator. 

(2) Simplify the denominator. 

(3) Apply the law of divisors. 

(1) Simplify the numerator. 

To simplify the numerator is to make it integral or one 
simple fraction. 

The numerator is now a mixed expression because 
it has both integral and fractional terms. 

A mixed expression is simplified by adding its terms. 

This is done by reducing them to a common denomi- 
nator. 

This is effected by multiplying the integral terms by 
the least common denominator, adding the nimaerators, 
and writing the result over the least common denominator. 

The least common denominator is a^. 

Multiplying 1 by a^ we have a^. 

Adding to this the numerators we have a^— 2a6+&^. 

Writing this over the denominator we have ^ • 

The numerator is now simple because it is one simple 
fraction. 

(2) Simplify the denominiEitor. 

To simplify the denominator is to make it integral or 
one simple fraction. 

The denominator is now a mixed expression. 

Multiplying the integral part 1 by the denominator a^ 
and adding the numerator — 6^, we have a^—V^, 

a^—}p 
Writing it over the denominator we have — g — • 

The denominator is now simple because it is one simple 
fraction. 

(3) Apply the law of divisors. 

Invert every fractional divisor and use it as a multiplier 
of that which it divided. 



Digitized by 



Google 



137 



The divisor is - o 



FRACTIONS 



153 



Inverting it and factoring we 



have 



(a-6)(a-6) 



a~& 
(a+6)(d-6). 
137. Examples. Simplify the following: 



1. 



7. 



9. 






c* c 



24 



2 ^h 

36-9 

6 



11. 1+- 



l+r+ 



2r« 
1-r 



L-1 



+1 



1- 



L • 



3. 



5. d+ 



1-L 
d 



--ax 



d+ 



1 



4e 
^+3 



R-4+ 



8. 



10. 



R- 



/g-f 1 . /g' -3/^+2 
6 * i_^+5 • 
72-1 



1- 



JB*-1 






1 



a;-?/ 



12. 



1 . 1 



x+- 



1 



y+ 



1 !/ 



13. -4^X?'-^' 



6 a 



a'+b' 



(p«+f«-g»)« 

nX ■ 



14. 



4<» 



{p+t)'-y' 



(p-t+v)* 



Digitized by 



Google 



154 TECHNICAL ALGEBRA 1S7 

, 2zy S-2 

15. ^I(i^. S-2-^' 






17. ' 



^ h*-l ' ^„ 3d-2 3d+2 

* T "• i ' 



20 I C^\^^CZA^(AC-1\ 

\A'C'-A*C^I\ ,24 /\AC+1/ 



Digitized by 



Google 



CHAPTER VIII 

THE QUADRATIC EQUATION 

Section 1, Introduction. Section 2, Solution by Factoring. 
Section 3, Solution by Completing the Square. Sec- 
tion 4, Equations in Quadratic Form. 

§1. INTRODUCTION 

138. Definition. A quadratic equation is an equation 
having the second power of the unknown quantity but no 
higher power when cleared of fractions and reduced to its 
simplest form. 

3x2+8a; = 10 is a quadratic because it contains the 
second power of the unknown. 

5x^—3i?+2x = S although having the second power of 
the unknown, is not a quadratic because it has a higher 
power also. 

— r-^+x— 1 = 16 in its fractional form has only the 
x-\-o 

first power of the unknown and might seem not to be a 

quadratic. When cleared of fractions, however, and 

collected, it has a second power of the unknown and is 

therefore a quadratic. 

139. Classification. There are two classes of quadratics: 

(1) Incomplete, or those having the second power 

only. 

(2) Complete, or those having both first and second 

powers of the unknown quantity. 

155 



Digitized by 



Google 



156 TECHNICAL ALGEBRA 140 

The names applied to these classes show that the second 
power of the unknown constitutes a quadratic and that a 
quadratic is regarded complete only when having the 
first as well as the second power. When the first is want- 
ing the quadratic is incomplete. 

Quadratics of the first class are sometimes called pure; 
those of the second class, adfected or affected. 

140. Essentials to Solution. Two things are essential 
to the solution of any equation: 

(1) Its recognition as simple, simple fractional, or 

quadratic. 

(2) Knowledge of the operations by means of which 

the value of unknown quantities can be found. 
Observe that it is the highest power of the unknown 
quantity which determines whether an equation shall be 
called simple, quadratic, or cubic, etc., the powers of the 
known quantities having nothing to do with it. One's 
first thought, therefore, when an equation is to be solved, 
should be what kind of an equation is it, and this is deter- 
mined from the exponents of the unknown quantity. 

141. Methods of Solution. (1) Incomplete Quadratics. 
If a quadratic has only the second power of the unknown, 
the value of that unknown is determined by the extraction 
of the square root after solution for the second power. 
In other words, an incomplete quadratic is solved by finding 
the value of the square of the second power of the unknown 
quantity, followed by extraction of the square root. 

Thus 5x2-18 = 107 

X=*i:5. 

(2) Complete Quadratics. When an equation contains 
both the first and the second power, solution may be effected 

* Every square root must be written with a double sign. 



Digitized by 



Google 



142 THE QUADRATIC EQUATION 157 

either by transposing all terms to the first member and 
factoring; or by transposing all known terms to the second 
member and completing the square. The latter method 
is employed when factoring is impossible or when the 
factors are hard to determine and should not be used 
under any other conditions. 

There are therefore two methods of solving a complete 
quadr&tic: 

(1) Solution by factoring. 

(2) Solution by completing the square. 

§ 2. SOLUTION BY FACTORING 

142. Illustration. To solve 5x2+6a:=32 by factoring 
we transpose all terms to the first member, which gives 

5x2+6a:-32=0 

The first member is now a trinomial; therefore if fac- 
torable it may be resolved into two binomial factors. 
Therefore we have (5x+16)(x~2)=0. 
Dividing both members of the equation by the first 
factor 6a:+16 we have 

a:-2=0, 
.-. x=2. 

Dividing both members by the second factor x— 2 we 
have: 

5x+16 = 

16 

x=2 or — =-. 
5 

Check: 6x2+6a:=32 

Substituting 2 for x, 5 • 22+6 • 2 = 32 

20+12=32. 



Digitized by 



Google 



158 TECHNICAL ALGEBRA U3 

1 n 

Substituting — — for x, 
o 



i-WH-f)-- 



5 

32 = 32. 

Following is a complete model for the solution of a quad- 
ratic by factoring:* 

(a) 5x2+6x=32 



(1) 53^+ex-d2= 


—Trans 32. 


(2) (5x+16)(a;-2)= 0— 


Factor 1st member (1). 


(3) x= 2— 


-— Div (2) by 5a;+16 and trans. 


(4) 5x+16= 


— Div (2) by a; -2. 


(5) x=-^- 


In (4) trans 16 and Div by 5, 


(6) .-. 1=2 or ■ 


-y— (3) and (5). 



143. Short Method of Determination of Roots. If 

the values of x in (6) are compared with the factors in (2) 
it will be seen that x in (6) equals the second terms of the 
binomials in (2) with the signs changed, divided by the 
coefficient of x in each binomial. 

For example, in (6) x = 2. 

But —2 is the second term of the second binomial in (2). 

Therefore x equals the second term -^2 with the sign 
changed, divided by 1 which is the coefficient of x in that 
binomial. 

Therefore x = 2. 

1 f\ 
X also equals — =-, 16 being the second term of the first 

o 

binomial in (2) and 6 the coefficient of x in that binomial. 



Digitized by 



Google 



144 THE QUADRATIC EQUATION 159 

— 1 A 

With the sign of 16 changed this gives x=—=— which 

o 
1 /» 

by the law of signs in division gives — =-. 

5 

Observe that when the numerator is negative it is written 
as positive, and the minus sign is given to the fraction. 

To solve a quadratic therefore by factoring, transpose 
all terms to the first member. 

If that member resolves into two binomial factors, 
X (the unknown) will equal the second terms of these 
binomials with the signs changed, divided by the coefiicients 
of X, 

It is therefore unnecessary to show the solution in the 
long form of paragraph 142. Instead, use the form shown 
in the following illustrations: 

1. 3x2+121 =44a: 
3x2-44x4-121=0 

(3x-ll)(x-ll)=0 

x = ll or—-. 
3 

2. 7r2-7cr = 84c2 
7r2-7cr--84c2 = 

r2-cr-12c2 = 
(r-4c)(r+3c)=0 

r = 4c or —3c. 

3. 2v^-av-ab=-2bv 
2v^'-av+2bv'-ab = 

2v(v+b)-a(v+b)=0 

{2v-a){v+b)=0 

a 
«;=— or —6. 
2 

144. Instructions for Solution by Factoring. 
1. If the equation is fractional, clear it of fractions by 
multiplying by the least common denominator. 



Digitized by 



Google 



160 



TECHNICAL ALGEBRA 



146 



2. Transpose all terms to the first member, arranged 
in the order of the descending powers of the unknown quan- 
tity, followed by the known term or terms. 

3. If all terms have a common factor divide the equa- 
tion by it. 

4. Factor the first member. 

6. Write the values of the unknown. 

145. Examples. Solve the following by factoring. 
Number all equations but do not specify operations. 



1. a;«-12a:+30=3. 
3. iS«- 169 =24/5 -144. 

6. 5d«-6d-16=4d«-2d-4. 

7. Gt^-6t+9=5t^+l. 

9. |-|-42f=-20i. 



11. 



a.-^-5=16i. 
4 



13 2_5^^15 
5 2i 4i«' 

15. w^+2bw-2b =w (w unknown). 

16. s*— cs+es— cc=0 (s unknown). 

17. L2+r+(r+l)L=0 (L unknown). 

18. k^ -2ek =k -2e (k unknown.) 

19. P+-=a-\-- (P unknown). 

a r 

20. — |-T7= — hr (M unknown). 

T M r 

21. D«+eD+6D+6e=0 (D unknown). 

2cF 2c^ 

22. F^-cF 1 =0 (/^unknown). 

a a 

ab 

23. a+h=H+-zz (^unknown). 

a 



2. L«+16L-17=0. 

4. 2F«-2F-9 = F«-1. 

6. 4Q2+19Q-25=3Q«-4(?+25. 

8. 25h^+^+?0h-Sl=9h*+ih. 

10. |D«-|D-7f = -7. 

12. y«-|t/+14i=16. 
, . 5 3^+1 1 , 11 

z z^ 4: 42* 



Digitized by 



Google 



146 THE QUADRATIC EQUATION 161 

24. Zb;+-J- =^ (Ax unknown). 26. 9'+^-^ =0. 
26. /•+! +|-=0. 27. z«-(n+l)z= -n. 

27 a*6* 

30. 52/(2/-3)=j-122/. 31. -^ — a«=6«-x. 



§ 3. SOLUTION BY COMPLETING THE SQUARE 

146. How the Sqtiare is Completed. What must be 
added to a^+2ab as a third term in order to make it a 
trinomial square? 

In a^+2ab what is the coefficient of a? 

What is half the coefficient? 

What is the square of half the coefficient? 

Is this the term which must be added to make a^+2ab 
a trinomial square? 

What quantity must be added to a^—2ab to make it a 
square? 

Is this quantity the square of half the coefficient of a? 

In the equation a;2_i2x=— 20 what is the coefficient 
ofx? 

What is half the coefficient of x? 

What is the square of half the coefficient of x? 

Indicate the addition of this quantity to the first member 
as a third term. 

Is the first member now a square? Why? 

Was it a square in the original equation? 

The square (of the first member) was completed by 
adding what? 

Completing the square therefore means to do what? 

If a quantity is added to the first member of an equation 
what must be added to the second member in order to 
preserve equality? 



Digitized by 



Google 



-Add Ax. 



162 TECHNICAL ALGEBRA 147 

Indicate the addition to the second member and collect 
the terms in that member. 

Extract the square root of the resulting equation. 

Solve. 

In the solution of a quadratic, do not fail to place the 
double sign ± before the square root of the second member. 
This will give two values of the unknown quantity; one 
from the positive root, the other from the negative root. 
Always show in full the operations by which the two roots 
are obtained. 

Thus: (a) x2-18x = 144 

(1) x2-18x+81 = 144+81 = 225 Add Ax. 

(2) a: - 9 = =t 15 ^Root Ax. 

(3) a: = 9+15 = 24 

(4) x=9-15=-6j 

147. Instructions for Solving by Completing the Square. 

1. Transpose: 

(a) All unknowns to the first member, arranged in 
the order of the descending powers of the 
unknown. 

(6) All knowns to the second member. 

2. Collect the coefficients: 

(a) Of the terms containing the second power of 

the unknown number. 
(6) Of the terms containing the first power of the 

unknown number, 
(c) Of the known terms when possible. 

3. Divide the equation by the coefficient of the second 
power of the unknown with its sign. 

4. Indicate the addition to both members, of the square 
of half the coefficient of the term containing the first power 
of the unknown. 

5. Combine* the terms in the second member. 

* Very important in solution of conventional equations; usually 
not done in solution of formulas. 



Digitized by 



Google 



148 THE QUADRATIC EQUATION 163 

6. Extract the square root of both members. 

7. Solve. 

8. Reduce the result to the simplest form. 

148. Examples. Solve the following by completing the 
square. When an equation in conventional symbols has 
more than one letter, it is understood that the last letters 
of the alphabet denote unknown quantities. 



1. 5x«-69 = 10a;+7. 


5 x^ X x^ 1 * 
12 12' 6 6 4* 


3. -^ = 10-3x. 


. 20 4(x+5) 


K "il ^ 1 9i 


3x +24 12 
2x«+ 2 "*2x" 


*-5*-4x._2'2*- 


'■'-1-^4 


8. 5x^-l0ax-5^10a. 


••f-'-f* 


10. llfx-3Jx«+41i=0. 


"■ f -f -■ 


12 -2^+64 -2^' 
12. ig+t>4- ^g^. 


13. x^-a+ax==z. 


14. ax^-c=bx. 


16. 2ax^b-'cx\ 


16. 36a«+4ax«=36aa;. 


3a Qx(a-x) 
4" 3a-2x' 


18. a*-l=2a»x-o«x». 



cr 

19. (r+s)L2-cL=— -— (L unknown). 

20. (e-h)R*-hR=h (/2 unknown). 

„ . f« ht a\ab ^^ . . 

21. 1 — =— :H — W unknown). 

m* c m^ c 

22. a;*-2ox = (6-c+a)(&-c-a). 

* Solve also by factoring. 



Digitized by 



Google 



164 TECHNICAL ALGEBRA 148 

23. (F-l)«-(3F+8)«-(2F+5)«=0. 

24. \/20+H-^«=2^-10. 

Square both members to remove the radical. 

26. -^=5-2\/F. 

Clear this equation of fractions, transpose, arrange in the 
order of the descending powers of D, and divide by the 
coefficient of D. Complete the square by adding to both 
members the square of half the coefficient of \/d, 

26. aa:*+6x~c=0. 

^ A;+l a+1 ,, , ^ 

27. —p= =—7=- (A; unknown). 
y/k Va 

Ao r..3a»x 6a»+a5-26« 6«x 

28. a6x«H = —- . 

c c^ c 

^^ F-a F+a 5aF-3o-2 ,_ , , 

29-* f+^+rra'^^ZpT- (f unknown). 

^^ /S«-2n5+2a5-n* 5+2n 1 ,« , 

^- ^^^:^i +^H^^+^«=S=-a (««^«--)- 

«• 10 1 14 

33. 



6(r-2) 2 3(r-l) 

8+C 8—C 

36. aa;*+cx«=da;«H — — . 
a+c 

36. 56» = 156 - 1 1 (Determine result to nearest thousandth.) 

* Before multiplying to clear of fractions change the signs in the 
denominator of the second member. This will change the sign before 
the fraction by the law of signs in division. 



Digitized by 



Google 



148 THE QUADRATIC EQUATION 165 



87./*+2a/+6«=26/+2af (/unknown). 
38. Lz^^-^-Rx. 

39. hite » -r . 

g h 

40. In example 39 compute the value of z when 
W^j, flf=32.2, h^^, and ifc=0.02. 

«. •+l_£+i.o. 

43. \/6+i+V6^= ^^ 



5V6+X 
44. V=-T-(ri«+rir2+ri*) (n unknown). 

46. i2»=a«+6»+2a6cosa. 
Solve for cos a. 

Note. — ^If further practice in the solution of quadratics is 
desired at this point, see Chapter XVII. 



Digitized by 



Google 



CHAPTER IX 

THE FRACTIONAL SIMPLE EQUATION 

Section 1, Denominators Numerical. Section 2, Some 
Denominators Literal. 

§ 1. DENOMINATORS NUMERICAL 

149. Definition. A fractional equation is an equation 
which contains one or more fractions. 

A fractional simple equation is a fractional equation 
which has only the first power of the unknown quantity 
when cleared of fractions and collected. 

150. Solution. A fractional simple equation may be 
solved by clearing it of fractions and by collecting and 
dividing by the coefficient of the unknown. 

It may be cleared of fractions by multiplication by 
the least common denominator provided it contains only 
simple fractions. 

If any of its fractions are complex they should be 
simplified before the least common denominator is 
determined. 

161. Instructions for the Solution of Fractional Equations. 

1. Reduce all fractions to their lowest terms. 

2. Simplify * all complex fractions. 

3. Factor all literal denominators before attempting 
to determine the least common denominator. 

* It is sometimes better to multiply a complex fractional equa- 
tion by the L.C.N.D. of the lower row of denominators. 

166 



Digitized by 



Google 



163 THE FRACTIONAL SIMPLE EQUATION 167 

4. When a minus sign precedes a fraction, change the 
signs of all terms resulting from the multiplication of the 
fraction after canceling its denominator. 

5. Never multiply any equation until all like terms 
are collected. 

6. When an eiquation has both literal and numerical 
denominators, 

(1) Factor literal denominators if factorable. 

(2) Multiply every term of the equation by the 

least common denominator of the numerical 
denominators, L.C.N.D. 

(3) Collect all integral terms. 

(4) When possible, divide the equation by the 

largest number which will evenly divide both 
members. 

(5) Multiply by the least conmaon denominator of 

the literal denominators. 

162. Change of Signs. In the subsequent work attention 
is called to the following: 

By the law of signs in division, 

(1) All signs in either numerator or denominator 

may be changed provided the sign before 
the fraction is changed. 

(2) All signs in both numerator and denominator 

may be changed without changing the sign 
of the fraction. 

153. Examples. Solve the following equations: 

z+2 14 3+5a; 5a; 5a; 9 S-x 

*• 2 " 9 4 * 2 4 ~4 2 • 

3. ^I^+2=a:-^. 4. 55^-?^=3.-14. 



4 ' 2 * '2 3 

r — 
6 



^ « 5a; -4 ^ l-2x ^ 2x+7 9a; -8 x-11 
6. 2a;— r--=7— — — . 6. — z 7— = — ^r-- 



Digitized by 



Google 



168 TECHNICAL ALGEBRA 153 

_ 7x+3 5a; -6 8 -5a; ^ 10a;+3 6a; -7 ,^, ,, 

7. -6— ^=-12-- 8. _^— ^=10(x-l). 

^ 8a;-15'llx-l 7a;+2 ^^ 5a; -3 9-x 5a; , 19, ,^ 
9. -3 ^ =-^. 10. — ^=-+-(x-4). 

5x+3 3 -4x X ^31 9 -5a; 
8 3 "*"2 ~ 2 6 • 

7a;+9 3x+l _ 9a;-12 249-9a; 
8 7-4 14 • 

13 in? x_20_x-12 7 2x-?5±Z-i+i 

^^' 2 +3"3 2+2- ^^-^"^ 11 "".2+^- 

^, 6a;-4 ^ 18-4a; . ^^ 7a;+9 / 2x-l\ ^ 

n. '4^-(l-^) =7x. 18. ?^-?^-.4.-14i. 



19. 



4 \ 9 ./ '4 

a; -3 2x -5 41 3a; -8 5a;+6 
4 6 60"" 5 15 • 



20. 7x+13f-|=|-8f+^. 

2(x-8) 3(9 -x) 5(x-ll) ^ 3(x-17) 
^^- 3 ~ 4 ~ 6 "^~ 8 • 

22. ?5±^-??±Z+io4=0. 

7 o o 

23. ^(3x-4)+^(5x+3)=43-5x. 

24. |(27-2x)=|~(7x-64). 



«= = /7 2\ X 3x-(4 
« 4x 7x 

x-2_^-4^ 
.05 .0625 



-(4-5x) 



Digitized by 



Google 



158 THE FRACTIONAL SIMPLE EQUATION 169 

lOo+ll 12a -13 7 -6a 

6 3 ^" 4 • 

4 5 4 3 ^ 

^ .e ..135a: -.225 .36 .09x-.18 
30. .15.+-—^—=- —. 

2(a;-18) 2(x+10) 

33./-^.10|. 

'^^ * I + 3 8 ~°- 

5L-11 L-1 llL-1 
85. ' ~'' ' 



5 12 ' 

A;-l 3fe-4 JL^ 6fc+7 

o« 3 "^ 5 16 . 2 
36. 



8 4 ' 32 • 

„.,+_L+,(|».j,j._V?_^. 

m+3 3m -5 

^^ 2 m-2 12 , 1 

^- "1 — 4-=-r"+3- 

3n-l 2n+l 2n-5 7n~l 

^ ^ 4"^3 ^ 3'''8 
39. 22 z =20- 



«•¥-¥-+¥• "'Vn-SkllM. 



Digitized by 



Google 



170 



TECHNICAL ALGEBRA 



153 



§2. SOME DENOMINATORS LITERAL 



4a;4-3 _ 2x-5 2x~l 
' 10 "Sx-l' 5 ' 

14 "^6x4-2 " 7 ' 
^^ 9a;+20 4a; -12 x 

36 5a: -4 ^4 
,^ 17+4/ 7/+26 , f+13 



^^ Qx+7 .7a: -13 2x+4 
43. — h" 



45. 

47. 

10-/ 



9 ' 6a:+3 3 
3x+2 _ 2x-l X 
6 ""3a:-7™2* 

7x+16 x-\'S 



21 



4a:-ll~3* 



49. 



51. 



53. 



64. 



55. 



56. 



57. 



/+21 ' 7 
2a: -4 2a: -1 



21 
6x+l ^ 

15 7x-16" 5 • 
6a:+7 _ 2x-2 ^ 2x+l 

15 Ix-Q" 6 • 
10a:+17 12x+2 5a: -4 



^^ 4a:+3 . 7x-29 8a:+19 
60. — :; — \-- 



9 •5a:-12 



18 



6x+7 7a:-13 _ 2a:+4 
9 "^ 6a:+3 3 * 



18 
llx-13 

14 
llc-13 

14 28 

4(a:+3) _ 8a:+37 

9 " 18 
x^+13 3x+5 



9 * 
13x+7 



5a: -25 



13a: -16 
22a: -75 _ 

28 "2(3x+7)' 
22c -75 _ 13C+7 
"2(3c+7)' 
_ 7x-29 
5X-12" 
2 
5* 
4a: -3 



15 
a: -3 



59. 



5 ■ 2a: -15 10 
2x+H 2fa:-l x-^ 
5 ""50a: -10 - 2J ' 



-liV. 



9x+5 8a:-7 _ 36x+15 lOj 
^' 14 "^6^+2 56 "^14* 



' See instruction 6, paragraph 151. 



Digitized by 



Google 



153 THE FRACTIONAL SIMPLE EQUATION 171 

2x+8^ _ 13a; -2 x Jx _ x-\-l^ 
9 17x-32"^3 "12 36 • 

%-bx 7-2x^ _ l+3x lOx-ll 1 

• 15 Uix-l)" 21 30 "^105' 
18^-22 i + 16x 101 -64x 
39 -6x ^ ^ 24 "^ 24 

2d+8^ 13d -2 d _7d rf+16 

• 9 17d-32'*"3 "12 36 ' 

65 6-5x 7-2a:« l+3x _ 2x-2i 1 
' 15 14(a;-l)" 21 6 "^105* 

66. -^^i:Ii+2x+y^=4A.i?i::55. 

13-12x^ ^ 24 ^ 3 

67. (a;+3)-?^=7x 



.|3..fcM)' 



0.25i~1.5 _2 1 , 2 _x+2 

* 0.15C;-5) 1.5' 2"^a:+2 2x ' 

70. (x+l)«=x[6-(l-x)]-2. 71. ^ ^ ^ 



x-2 a;4-2 a:«-4' 

„« x+1 x-1 2 „^ 1 1 x+1 ^ 

72. — ■ = . 73. — ' — =0. 

x-1 x+1 x^-1 x-2 x+2 X2-4 

3a; 2x _ 2x«-5 7 _ 6a;+l _3( l+2a;») 

2a;+3 2x-3"4a;2-9' a;-l"a;+l a;2-l ' 

^^ 3 x+1 x^ „.T 4 , 7 37 

76. 7 —r^^z ;. 77. —77:+- 



a;-l a;-l *1— x«' * a;+2 a;+3 x2-|-5a;+6* 

„^ Sx+1 x-2 „^ x-7 2a;-15. 1 

78. ■ — = . 79. =0. 

3(x-2) a;-l a;+7 2a;-6^2a;+14 

80. -^Z^^^Z^+l, 81. ,+^J^-2)(x+4) 

4(x-l) 6(a;-l)^9 x-1 x+1 

2(2x+3) 6 5x+l 



2. 1- 



9(7 -x) 7-x 4(7 -x)* 



^ x«-(x+l) x«+x-l _ ^ 

X-l X— 1 



* See paragraph 152. 



Digitized by 



Google 



172 TECHNICAL ALGEBRA 153 

Sx+1 x-1 
86. A;+^ ' 



86. 
87. 



* 2x-3 z-2 3x+2" 
3x-l 4a: -2 1 



2:i;-l Sx-2 6* 

1 1 x-1 



2(x-3) 3(x-2) (x— 2)(x-3>* 



• x+3 3x+9 ' ' x+7 2x-6 2(x+7)* 

QA ^±i4.ii=3£±? 91 3a:-l 4x-2_l 

Sx+S"^^ 2x+3' 2a:r-l 3x-2 6* 

3+x_2+x_l+x^ _J 6_ J_ 

3-x 2-a; l-x 2x-3 3a;+2"^l-a;' 



i 



a;-8 2a;-16 24 3a;-24 

X + 4: 
r.= 1 ^'^ 1-X 

5 x-1 3 

g-3f 5-4g 3?-i(3-2g) 

«, 2 7 1 , _, 2 

97. ^ =3-9+7* . 



4r 9 lf)r-81 6r-18 4r-9 



Oft 5_2_ 6 _ 5 10_ JL 

**• 3 6 , .4 9^2 ^4i* 



t 



Digitized by 



Google 



CHAPTER X 

SIMULTANEOUS SIMPLE EQUATIONS 

Section 1, Two Unknowns. Section 2, Three or More 
. Unknowns. 

164. Definition. .Simultaneous equations are equations 
having the same unknown quantities with the same respect- 
ive values. 

Simultaneous simple equations, therefore, are simple 
equations containing the same two or more unknown 
quantities so related that the values of the unknown quan- 
tities in one of the equations are the same respectively as 
in the other equations. 

155. Solution. Generally speaking, solution is possible 
only when the number of given equations is the sam£ as the 
number of unknown quantities. 

For example, any given equation like 5x— Sy = 7 in which 
there are two unknowns x and y, cannot be solved. To 
make solution possible a second equation must be given in 
which X and y have the same respective values as in the first 
equation. 

If the first equation contained three unknowns solution 
would be possible only in case two more equations were 
given, each having the same respective values of the 
unknowns. 

In other words, if there are two unknowns, two equa- 
tions are necessary to solution; if there are three unknowns, 
three equations are required, etc. 

173 



Digitized by 



Google 



174 TECHNICAL ALGEBRA 156 



§ 1. TWO UNKNOWN QUANTITIES 

156. Methods of Solution. In order to solve simultane- 
ous equations having two unknown quantities, one of the 
unknowns must be made to disappear. The process by 
which this is accomplished is called elimination. 

There are three methods of elimination: 

(1) Elimination by addition or subtraction. 

(2) Elimination by substitution. 

(3) Elimination by comparison. 

167. First Method. Elimination by addition or sub- 
traction. 

Under chapter and paragraph headings enter the fol- 
owing example in the work-book: 

2x+3y = 17 
4x+5y=-31 

Number the equations (a) and (6) respectively. 

Multiply (a) by 2, giving (1). 

Are the coefficients of x in (1) and (6) equal or unequal? 

They were made so by what operation? 

Are the signs of the x terms like or unlike? 

In order to eliminate x must (1) and (6) be added, or 
subtracted? 

Perform the operation which will eliminate x. Solve 
for y. 

In either (a) or (6) substitute the value of y and solve 
for X, 

Why is this method called elimination by addition or 
subtraction? 

All of the examples which follow in paragraph 160 may be solved 
by eliminating one of the unknowns by addition or subtraction. 



Digitized by 



Google 



159 SIMULTANEOUS SIMPLE EQUATIONS 175 

Usually both of the given equations must be multiplied in 
order to make the coefficients of one of the unknowns equal, with- 
out which elimination is impossible. 

Sometimes less work is required by dividing one of the equations 
by some number, or by adding or subtracting the given equations, 
before multiplying. 

Each example must be studied to determine the simplest and 
quickest operation which will give equal coefficients of the same 
unknown in two different equations. 

When there are three or more unknowns solution is effected 
by successive eliminations of the same unknown, care being taken 
that each elimination involves a different pair of equations. 

158. Second Method. Elimination by substitution. 

Enter this heading. 

Elimination by addition or subtraction is not always the 
simplest means of solution. In some instances, especially in 
work with formulas, it is better to solve the simpler equation 
for one of its unknowns in terms of the other quantities and 
to substitute the resulting value in the other given equation. 

Solve (a) from paragraph 157 for x in terms of the other 
quantities. 

In (6) substitute the value of x and solve for y. 

In either (a) or (6) substitute the value of y and solve 
for X, 

Why is this method called elimination by substitution? 

169. Third Method. Elimination by comparison. Enter 
this heading. 

It is sometimes convenient to eliminate by solving 
each given equation for the same unknown in terms of the 
other quantities and by comparing the results by applying 
the equality axiom. 

Solve (a) for x or y. Solve (6) for the same quantity. 
Apply the equality axiom and solve the resulting equation. 
In (a) or (6) substitute the resulting numerical value 
and solve. 



Digitized by 



Google 



176 



TECHNICAL ALGEBRA 



160 



160* Examples. Solve the first five by each of the three 
methods; solve the others by any method. Time will be 
saved in most examples by indicating the multiplication 
which gives equal coeflScients, and not performing it. 



1. 5x+y='21. 
2x+3y=2i. 


2. 3x+7y=27. 
5i+2y=16. 


8. l(te+9j/=-37; 
9x-llj/=24. 


4, 3z+4y = 10. 
4x+y=9. 


6. 7x-3y=67. 
4x-10y=88. 


6. 3x+y=3. 
5x+2y=4, 


7. I5x+8y = -3. 
Qx -4j/ = -4. 


8. 42-% =8. 
6x-j/=28. 


9. Sx-7y=8l. 
10a;-24y=25. 


10. 5x-5y=25. 
7x+4j/=2. 


11. 36x+63y=-Q9. 
30x+5% = -60. 


12. 7x+2y=31. 
3x-4y=23. 


13. 8a-c=34. 
o+8c=53. 


14. 9x+%=9. 
2x-3y=24, 


IB. 2e-5d=60. 
7e-12d=342. 


16. 6x-10j/=26. 
5x-7j/ = 15. 


17. 3h=7L. 
12L-5A-1. 


18. 2x-3y = -17. 
6x+82/=66. 


19. 8x-&y=2x+3Q. 
llx+32/=80. 


20. 12x-4y=84. 
17x-19j/=79. 


21. 9x-y=4x+Uy. 
lZx-7y = U0. 


22. 15x-13j/=20-5j/. 
19x-8y=36. 


23. 5x+7y=U. 
3Jx-%=5. 


24. 17x+7y = l(Xi. 
llx-2y = 19. 


26. 19ix-13y=65. 
12x-14y=-2. 


26. llA;-12y=-207. 
-5A;-7y = -155. 


27. 6m+3y=78. 
12m -5y =2. 


28. 17x-21j/=60. 
24x-72/=130. 


29. 5x-5y=70. 
-7x+8y=-95. 


30. 8x-9j/ = -31. 
12x+2j/=62. 


31. 9a: -3% = 111. 
8x+13j/=118J. 


32. 0+56=34, 
7a -66 =33. 



Digitized by 



Google 



160 



SIMULTANEOUS SIMPLE EQUATIONS 177 



33. 3c -86 =24. 
8c-106=86. 

35. 14€-62/=24. 
13e+ll2/-188. 

37. 39i~62/-36. 
18i-9^=0. 

39. 13§a;-152/=9. 
-24a;+2l2/=-84. 

41. 4§a;-3.5n=7.5. 
ll|x-8n=32. 

43. 23a: -5r =9. 
17a;+4r=99. 

46. 18.2«-5t;=-14. 
29.6«-6t;=22. 

47. -122+4x=-32. 
-17z+8x = -15. 

49. 3fA+4y=98. 

12A-lli=-79. . 

61. 1000.55 -1002) =49.026. 
525+4002) =6.60. 

63. |+|=3x-7y-37 

x+3 S-y 
5 " 4 • 

55. 2r-.?±5=7+?i^^ 
4.^«^^=24i-?^. 

10»+8+j|-^+«. 



a 



»2a6. 



^+^-«+*- 



34. 100d-18y=2. 
33d -52^ = 11. 

36. 120A-8y=0. 
15^+61/ = 105. 

38. 7x+92/ = 122. 
101a;-102/=82. 

40. 6x-52^ = 13. 
lla;-8y=39. 

42. 7.2a; -84.4p = 137.8. 
5a;+26p = 138. 

44. 3.375s+7a:=24.7. 
7.75s -2.5a; =27. 

46. 225.5m; -81 j2/= -111. 
45.1m; -lOy =90.6. 

48. 11.875L-9.56=-95. 
3.375L+5.26 = 131. 

60. 100a:-802=-15. 
125:c+160 =39.25. 

62. 2000a;+i2002/=46.4. 
3000a: -9002/ =-16.8. 

64. ?5+?l = l. 
X y 



56. 



20(2+^) =7. 

1 _ 11 

L+P 
L-P 



=a. 



7+a: 2a:— V 
68. ^-^^+d=3y. 



5a: -18+ 



4 
4a:-3 



5y-7 



2©-r „^, 2r-59 
60. r -:;::; — =20+- 



23 -r 

t;-- ^=30- 
r-18 



2 
73 -3t; 



Digitized by 



Google 



178 TECHNICAL ALGEBRA 161 

61. ay -hx=0. 62. x+y=a, 

h—y=a—x. hy=x, 

63. X cos 70° -y cos 40° =0.* 64. R cos 20° -T cos 45° =2120. 
X sin 70° -y sin 40° =2000. R sin 20°+^ sia45° =5520. 

66. AR\ sin 25° -AB sin 67° =0. 
AR cos 25° -AB cos 67° = 19.4. 



§ 2. THREE OR MORE UNKNOWN QUANTITIES 

161. How Solved. Simultaneous equations having any 
number of unknown quantities, are solved by successive 
eliminations of the same unknown by the methods already 
described. 

For example: 

2x+42/+52=19. ^ 
-3ir+5i/+7z=8. ^ 
'8a;-3i/+5z = 23. ^ 

X may be eliminated in (a) and (6), then in (a) and (c), 
or (6) and (c). By this means two equations having the 
same two unknowns will be obtained. 

The equations constituting an example are called a set or 



* In equations of this kind, time is saved by indicating^ instead of 
performing the multiplication which gives equal coefficients. 

Thus to eliminate x the first equation is multiplied by sin 70° and 
the second by cos 70°. 

Therefore x cos 70° sin 70° -y cos 40° sin 70° =0. 

X sin 70° cos 70° -y sin 40° cos 70° = 2000 cos 70°. 

Subtracting, the x terms are eliminated because the coefficients 
are equal and the signs ahke. It is therefore necessary to perform 
the computations in the y terms only. 

t AR and AB are unknown. 



Digitized by 



Google 



162 



SIMULTANEOUS SIMPLE EQUATIONS 



179 



162. Examples. Solve the following: 



= -4. 



66. 12a;+52/-42=29. 
13a;-22/+52=58. 
17a; -2/ -2 = 15. 

68. a: -32/ -22 = 1. 
2a;-32/+52=-19. 
dx+2y-z^l2. 

70. 3a:-5=2y. 
32/ -7 =42. 

42=2+5a;. 

^^ a -6 c+b 1 

g+fe c+fe 

4 2 
c— a_c— 6 
~5~""~6~' 

74. 1+^=5. 
o; 2/ 

i-i-6. 

2/ 2 

2 a; 

76. 3a; -22 =2. 
42+32/ =41. 
5w-72/ = ll. 
2u+32=39. 

77. i+f = 1. 

22/ 4w 

22/ 32* 

4tA 32* 



67. ay+bx=c. 
oz+cx=b, 
cy+bz =a. 

69.A+3^6=443 
4a; 2/ 2 24 

1 =12a«. 

6a; 2/ 2; 

2 _^ 1_^85 

X 32/2 "27* 

72. a;— 03+0^2 =a2/. 
x-M=^by-¥z. 
c^z=cy—x. 



73.^!^±^=.2-2. 



2a-Aa;= — r — • 
(o+A)i'+j/=oA(z+2+a;). 



76. 


2 1 3 




c 




c 3 a' 


78. 


a 6 2/ 




bay 




y a b 



Digitized by 



Google 



CHAPTER XI 
EXPONENTS 

Section 1, A Zero [Exponent. Section 2, A Negative 
Exponent. Section 3, A Fractional Exponent. 

163. Classification.' In paragraph 24 an exponent was 
defined as a number which, when positive and integral, 
shows how many times the number affected by it, is used 
as a factor. 

The appHcation of the latter part of this definition to a 
negative or fractional number when written in the position 
of an exponent, is meaningless. For example, by the expres- 
sion T* we understand that x is used 5 times as a factor. 
All attempts fail, however, to interpret expressions like 
x^, x"^, or x*, in terms of factor, for the reason that a num- 
ber used zero or —5 or i times as a factor is unthinkable. 

The four expressions x^, x^, x~^, x*, represent the 

Four Kinds of Exponents: 

Positive integral) 
Zero, 
Negative, 
Fractional. 

The first is already understood; the significance of the 

last three will now be determined. 

180 



Digitized by 



Google 



164 EXPONENTS ISl 



§1. A ZERO EXPONENT 

164. Meamng of A®. If a denotes any quantity whatever, 
then aP denotes any quantity with an exponent zero. 

The meaning of aP may be illustrated by the operations 
of multiplication and division. 

First Illustration. By the law of exponents in mul- 
tiplication, 

but oTxi =ar. 

Therefore by the equality axiom, 

arXaP=arxh^ 
Therefore d9=l. 

Second Illustration. Under suitable heading write 
this illustration in full in the work-book using the same 
symbols as in the first illustration but dividing instead of 
multiplying. 

In working out the illustration number all equations 
and specify all operations. 

In both illustrations a represented what? 

Therefore any quantity with a zero exponent equals 
what? 

Underline or otherwise emphasize your answer to this 
question. 

Therefore aP equals wljat? 

999^ equals what? 

.OOOl® equals what? 

25x^y^ equals what? 

(34^)2 equals what? 



Digitized by 



Google 



182 TECHNICAL ALGEBRA 166 

165. Zero Exponent Theorem. Any quantity with a zero 
exponent equals unity. 

Following is another proof of this theorem: 

Given any quantity whose exponent is zero, 
i.e., Given a^, 
Prove aP = l. 



(1) 


a 


Law of exponents in division. 


(2) 


but -=1 -™^ 
a 


Law of division. 


(3) 


.-. 00=1-^ 


""- Equality axiom. 



§2. A NEGATIVE EXPONENT 

166. Meaning of A"*". If a denotes any quantity and r 
denotes any quantity then a"'^ denotes any quantity with any 
negative exponent. 

The significance of the negative exponent will be dis- 
covered by doing the work which follows. 

Enter the work in standard form according to page 8, 
paragraph 10. 

Multiply a"*" by a*". 

But aP equals what? 

Equate and solve for a""*". 

But a represents what? 

And r represents what? 

Therefore any quantity with any negative exponent is 
equal to its reciprocal with the same exponent with what sign? 

Letter, underline, or otherwise emphasize the answer. 

167. Negative Exponent Theorem. Under this heading 
write for the theorem your answer to the last question. 

Under it write hypothesis and conclusion in the same 
form as in paragraph 165. 



Digitized by 



Google 



168 EXPONENTS 183 

Write the demonstration in the same form also, as 
follows: 

a^X— equals what? 
but (fi equals what? 

Finish the demonstration. 
168. Use of Negative Exponent Theorem. The fact that 

a' 

means that an algebraic expression having negative expo- 
nents may be written with positive exponents by the appli- 
cation of the negative exponent theorem. 

' Thus (1) aH-^ = ^. 



(2) 






(3) 6-i+3a-2 = l+|. 



(4) 



%x^y-^ ^ 6x2 



In like manner 



by" 



(5) i^ = 7a%-^ry^. 



We therefore have the principle that any multiplier 
may be written as a divisor and any divisor may be written 
as a multiplier, provided the sign of its exponent is changed. 

In other words any factor may be transferred from 
numerator to denominator or from denominator to numerator, 
provided the sign of its exponent is changed. 

Thus .829_^829_ 

^•^"^ 10000 10* ^^^^" • 



Digitized by 



Google 



184 



TECHNICAL ALGEBRA 



169 



169. Examples. 

and zero exponents: 

1. a-K 
3. tH"K 

6. 7d-»/8. 

7. 10-^«i>-». 

9. ISz-^Vr-^ 
11. 3a-*c\ 
13. .00078960. 
16. *10-*X75000000. 



Express the following without negative 



17, 



19. 



21. 



25. 
27. 
29. 
31. 



2ar-»+5y-^ 

72a6"2c4 ' 



x-y 

Acy^z " 2 
Ic-Hj-^z^' 



* Observe th' 
denotes a shift of 
of units in the c: 



2. 


x*y-K 


4. 


6ax-K 


6. 


34L«i>-». 


8. 


1 

X-*' 


10. 


5bo. 


12. 


^oaH)-*. 


14. 


98299889799'. 


16. 


7a— d*/'. 


18. 


14r-V 


20. 


b-'+c' 
3x-« • 


22. 


4x^-' 


5x'yi+ab-i' 


24. 


7r»y-» 


9r'y-'+4r-^'' 


26. 


d'f-' 


3e-»d-'-2a-»' 



2r-2.' 




171 EXPONENTS 186 

33. -; — .^,, . . 34. 



36. 



1 -150/1-1 * 3am+6s-»» 

18a6-\;-id« 8f-»c-7+3<»c-< 

9o^»c-* ' 15«2c-i 



§ 3. A FRACTIONAL EXPONENT 

1^ 

170. Meaning of A**. Solve the following problems: 

1. In the expression (a*)^ the exponent 3 denotes that 
a* is used how many times as a factor, by the definition of 
a positive integral exponent? 

And by the law of exponents in multiplication, 

a^Xa^Xa^ equals what? 

Therefore a* is one of the three equal factors of a. 

Therefore a* is what root of a? 

Therefore the cube root of any number may be denoted 
by what exponent? 

2. Show that the square root may be denoted by expo- 
nent ^. 

3. Show that the fourth root may be indicated by ex- 
ponent J. 

4. How may the nth root be denoted by the use of a 
fractional exponent? 

171. Meaning of bpth Numerator and Denominator 
of a Fractional Exponent. 

1. In the expression (a*)^ the exponent 2 denotes that 
a* is to be used how many times as a factor? 

And a^Xa^ equals what? 

But a number is squared by using it how many times 
as a factor? 



Digitized by 



Google 



186 TECHNICAL ALGEBRA 172 

Therefore in a fractional exponent does the numerator 
indicate a power, or a root? 

What does the denominator of a fractional exponent 
indicate? 

2. In X* what does 3 denote? What does 4 denote? 

3. In y* what does 1 in the fractional exponent denote? 
What does 2 denote? 

4. In 2""* what does the miniLS denote? 
Write the expression without a negative exponent. 
What does 2 denote? 

What does 5 denote? 

5. In 12 a®6*2/"^ what does the zero exponent denote? 
What does the fractional exponent denote? What does 

the negative exponent denote? 

Write the expression as interpreted by the three laws. 

6. What is the value of 64*? 

By what law is its value determined? 

7. Express x* with a radical. What law did you apply? 

8. Express Sr^v^ with a radical. 

Did you apply the same law as in problem 7? Why? 

9. Express Qx~^y^z~^ without negative and without 
fractional exponents. 

How do you know that your result is correct? 

8*a~^6~*c^ Simplify this fraction in two differ- 
8 "^a -^6 ""^c ~^* ent ways : 

(1) By applying the law of exponents in division. 

(2) By applying the laws of negative and fractional 
exponents. 



Digitized by 



Google 



173 EXPONENTS 187 

172. Fractional Exponent Theorem. 

Given a". 

Prove r denotes the power of a and n denotes the root of a. 

W (a ) --a -^a | Law Exps in Mul. 

i- n/- 

(2) .-. an = Va (1) and Def nth Root. 

/o\ XT -i r \\r [Def Pos Integ Exp and 

(3) Now a''=(a'*y \ j ^ • ivyr i 

^ ' [ Law Exps m Mul. 

(4) /. In a ",r denotes 

the power and,v.v,^,v I Def s Power and Root. 

n denotes the 

root. J 

173. Examples. Express the following with radicals 
and without negative exponents. 

1. ax*. 2. {ax)K 

3. 26*d*. 4. 5ris*. 

6. 8(06)1. 6. ^j^^, 






7. -^. . 8. 



x-*y- 



9. -7=. 10. 6o"*X2a-*. 

11. 3a- «x*. 12. 7h-*xK 

13. -^^. 14 "*'"* 



16. (6c«a-»)^ 16. (6c«a-»)"^ 

17. 32*. 18. 32-». 

19. 56-«c"*X86-i. 20. 21x-^y*Xix-^-K 



Digitized by 



Google 



188 TECHNICAL ALGEBRA 174 

174. Examples. Express the following with fractional 
exponents in place of the radicals: 

1. Vei^. 2. v^. 

3. Vsix-*. 4. X^Sly^z-K 

6. </xhi-\ 6. v^o^. 

7. Vx^. 8. ^(a;+2/)*. 
9. (r->5-»i*)*. 10. y/ia-hy. 

13. \^XV^». 14. v^Tofcf+v^i. 

17. V^64a;82/-". 18. V^32r-iop-« 

19. ^7a*6-*. 20. V5a«6-»+2a-^ 

21. V^18a;-*+32/-». 22. V^6c*d-*. 

23. V^(2a«-36-2)*. 24. V^(9x-*-52/)l 

26. V^(15t;«-2«-»)*. 26. v^x-V+^a*. 

27. 4V|a;»y-|x-*2/-*. 28. V^(x«-2/«)*+(a-6)V. 
29. 3Vl46a;»-5aa;->. 30. v^(r» -«-»)» -2(a«+6»). 
31. >J^(x«+2y»)*. 32. V7(4a6«-5a«6-«). 

175. Root or Power of a Fraction. In raising a fraction 
to any power and in extracting any root of a fraction be sure to 
raise both numerator and denominator to the power and to 
extract the root of both numerator and denominator. 

rpu /3\2 9 , , 9 3 

Thus I -r I =77: and not -r or 



/3\2^^ 
U; 16 



4 "* 16" 

-r and not -tt- or -r-. 
4 2 4 



Digitized by 



Google 



176 EXPONENTS 189 

\36 6- 

F „ /T 3 • - 1 

,-? = 3A/rv or -7= or 3- 



15 \15 Vl5 Vl5" 

A factor which is not a power of the indicated root is 
left under the radical. 



„. /i47 f~49 7 /3 . 

Thus Vl8=V2X¥=3V2- 



PRACTICAL METHOD 

From the practical point of view the operation last shown 
is ridiculous. If in any computation it were necessary to 

/i47 
determine \j^r^ no one would think of reducing it as indi- 
cated but would directly determine it in one of six different 
ways: 

(1) Compute it on the slide rule. 

(2) Find the quotient and extract the square root 
arithmetically. 

(3) Find the quotient and extract the square root by 
logarithms. 

(4) Find the quotient and read the square root from a 
table. 

(5) Read from a table the square roots of both 147 
and 18 and then find the quotient. 

49 1 

(6) Reduce the fraction to its lowest terms -r-, take - 

o o 

of 49 by inspection, and extract the square root by one 
of the several methods. 



Digitized by 



Google 



190 TECHNICAL ALGEBRA 176 

Since a square root is one of the two equal factors and a 
cube root is one of the three equal factors of a number, 
any ROOT of a monomial quantity is obtained by extracting 
the required root of the numerical factors, and dividing the 
exponents of all the literal factors by the index of the root 



^ 






in which the exponents of the letters were divided by the 
root index 2. 

Similarly, any POWER of a monomial is obtained by rais- 
ing the numerical factors to the required power, and multiply- 
ing all the exponents of the literal factors by the exponent of 
the power. 

/ 3a;Vz-5 ^3_27^6yi2^-i5 

^^""^ \ 2v^ ) ~' 8^6 • 

176. Miscellaneous Examples. Express each of the 
following in one or more different forms: 

L a-^K 2. x-^K 

3. sH\ 4. x^y-K 

^ r - a-^c^ 

6. . 6. . 

x-2 262 

7. V36a26-*. 8. \^32g^^h-^. 

9. V98a-»68. 10. {5a%-^)K 

11. {2x-Y)^. 12. (3a-362c)l. 

13. 4b-id^\/2x. U. (7-*a36J)«. 

16. V^25o6-ic-*. 16. V2a^y^8r-*v^\ 

17. ^306 -id 18. V^729a-«6-«. 

19. V 384x^2/ -10. 20. yj 



32a- 



15 -5c 



1+X-* 



Digitized by 



Google 



CHAPTER XII 
THE BINOMIAL THEOREM 

177. A New Symbol. In the study of this chapter we 
are to derive and apply a formula whose expression is simpli- 
fied by the use of a symbol not before used. In this formula 
we shall wish to indicate 1 times 2, 1 times 2 times 3, 1 
times 2 times 3 times 4, and so on. Whenever this is 
necessary in mathematics, that is, when we wish to indicate 
the yroduci of the integers in succession from unity ^ instead 
of writing 1X2X3X4 or 1-2-3-4 and so on, we write 4! 
or |4, each of which means 1X2X3X4 or 1-2-3-4, and is 
called factorial 4. 

Likewise 6 1 or |6^ is called factorial 5 and means 

1X2X3X4X5. 

In modem books the first or exclamation point form is 
used almost exclusively to denote a factorial number 
instead of the older or angle form shown in the two pre- 
ceding illustrations. 

As the exclamation form is easier to make and is in 
common use it is preferable to the angle form. 

Our new symbol, therefore, like other operation symbols, 
is significant only when written in the proper position 
with respect to the quantities affected by it. - Following a 
word or sentence it denotes an exclamation; following a 
symbol for quantity it denotes that the symbol is factorial 
and therefore integral, 

191 



Digitized by 



Google 



192 TECHNICAL ALGEBRA 178 

178. An Example in Addition. In the derivation of 
the formula referred to in the preceding paragraph it will 
be necessary to add terms involving a factorial denomi- 
nator, like 

n(n— l)(n— 2) ^ «,« , n(n— 1) ^ oi^ 
-^^ ^ ^a""2ft3.j — 1^ — ^a""-2b3. 

Observe that with respect to a and b these are like 
terms because a and b have the same respective exponents. 
They may therefore be added by adding their coeflScients 

n(n—l)(n— 2) , n(n— 1) 

3! 2! * 

Show that the sum is 

n(n~l)(n~2) 3n(n-l) 
3! "^ S\ ' 

Show that this reduces to -^^ by the two 

following methods: 

(1) By adding the terms. 

(2) By removing the parenthesis and then adding. 

In like manner, addland simplify 

n(n-l)(n-2)(n-3) n(n~l)(n-2) 
4! "^ 3! 

179. Expansion. When an algebraic expression is oper- 
ated upon as denoted by an exponent it is said to be 
expanded and the operation is called expansion. 

1. (a+6)2 can be expanded by what law of numbers? 
Show the application of this law to the expression. 

Expand the following, showing the full multiplication: 

2. {a+b)\ 3. (a+6)*. 4. {a+b)\ 



Digitized by 



Google 



180 THE BINOMIAL THEOEEM 193 

Each expansion has how many more terms than the 
number of units in the exponents of the respective powers? 

Therefore the expansion of (a+by^ would have how 
many terms? 

The expansion of (a+b)^ would have how many terms 
provided n is a positive integer? 

The first term of the binomials is in all terms of the 
expansions, except what term? 

The last term of the binomials is in all terms of the 
expansions, except what term? 

Which term of the binomials has increasing exponents 
in the expansions? 

Which has decreasing exponents? 

What is the increase in each succeeding term? 

What is the decrease? 

In (a+b)^ what is the first term of the expansion? 

What is the first term of the expansions of 

(a+6)3, (a+6)^ {a+b)^ (a+by^l 

How do the exponents of these first terms compare with 
the respective exponents of the binomials? 

Therefore what is the first term of the expansion of 
(a+br? 

How do the coefficients of the second term of the expan- 
sions compare with the exponents of the respective powers? 

The exponent of a is how much less in the second term 
of the expansions than in the first term? 

What is the exponent of b in the second term of all the 
expansions? 

Therefore what is the second term of the expansion of 
{a+br? 

180. Determination of the Coefficients of the Terms of 
a Binomial Expansion by Inspection. What is the coefiicient 
of the third term of the expansion of (a+b)^? 



Digitized by 



Google 



194 TECHNICAL ALGEBRA 181 

Show by expansion of (a +6)^ whether this coeflScient 
equals the following: 

Coef preceding termXEzp first letter in the term 
Number of the term 

Apply the same test to the coefficient of the third term 
in the expansion of (a +6)^; also to the coeflScient of the 
third term of (a+6)^. 

Apply the same test to the coefficients of all the other 
terms except the first in all of the expansions. 

Law for the Coefficient. Write this law by answer- 
ing the following question: 

The coefficient of any term except the first in a binomial 
expansion, equals what? 

181. Summary of Facts Regarding the Expansion of a 
Binomial. 

In the work-book fill in the following outline: 
In every binomial expansion. 

(1) Number of terms = 

(2) Coefiicient first term = 

(3) Exponent first letter in first term = 

(4) Exponent second letter in first term = 

(5) In each succeeding term, decrease in exponent 
first letter = 

Increase in exponent second letter = 

(6) Coefiicient any term except first = 

182. The Binomial Theorem. This celebrated theorem 
is a law for the expansion of a binomial. If written it 
would state substantially what has been summarized in 
paragraph 181, provided your summary is correct. The 
theorem is true for all exponents whether integral or fractional, 
positive or negative. 

It will be best understood as summarized and therefore 
need not be written in the work-book. 



Digitized by 



Google 



IM THE BINOMIAL THEOREM 195 

183. The Binomial Formula. This formula is best 
expressed by the expansion of (a+t)", as indicated in 
paragraph 181. 

Thus (a+b)»=a"+na"-^b+^^^j^a"-V 

_^ n(n-lKn-2) ^„_3^^3_^ . . .* to (n+l) terms. 

Write the first six terms of this expansion. 

184. Proof that the Formula Applies to any Ntimber 
of Terms. By the expansion of (a+b)^ by multiplication 
you have discovered a law for the expansion of any binomial 
to the fifth power. This law was applied in the preceding 
paragraph in obtaining six terms in the expansion of (a +6)**. 
The work of this paragraph, if correctly done, will be a 
proof that the law applies to all powers beyond the fifth. 

Expand (a+b)^ to six terms and multiply both members 
by a +6, writing like terms with respect to a and 6 under 
each other. 

Simplify by adding the like terms in the second member. 
(If necessary refer to paragraph 178 in the work-book.) 

You will now observe that 

(1) Paragraph 183 gave the expansion of (a+b)** when 
n = 5. 

(2) This paragraph gives the expansion of (a +6)**"^^. 

(3) The expansion of (a+6)'*"^\ follows exactly the 
same law of development as (a +6)". 

The law therefore applies to the expansion of 

(a+6r+^ = (a+6)^+^ = (a+6)6. 

But if n = 6, the law applies to the expansion of a power 
1 greater and therefore to the seventh and eighth and ninth 
and higher powers. 

* Dots so written in an expression signify " and so on." 



Digitized by 



Google 



196 TECHNICAL ALGEBRA 185 

Therefore the statement of paragraph 182, has been 
shown to be correct and it should be now clearly understood 
why the binomial formula applies to all powers. 

186. Change in the Formula when the Binomial is a 
Difference Instead of a Simi. If instead of (a+6)'* the 
binomial is of the form (a— 6)", will the terms of the expan- 
sion containing even powers of b be positive or negative? 
Why? 

Will the terms of the expansion containing odd powers 
of b be positive or negative? Why? 

Write the first four terms of (a— 6)". 

186. How to Apply the Formula. When the coefficients 
and exponents of the given binomial are all unity the expan- 
sion may be written directly without the use of the formula. 

Thus (x+y)^=o^+Sa^y+Sxy^+y^. 

and (x — 2/)^ = a:^ — Sx^y+dxr/^ — y^. 

by the binomial theorem. 

If however the binomial is of some other form, as 

(2x2+4t/3)3^ 

the expansion may be obtained as shown, or as follows: 

(1) Expand (a+b)^ to four terms. 

(2) Substitute the given values, a = 2x^, 6=4i/3, n = 3, 
and simpHfy the resulting equation. 

Thus 

/ I l.^» « . « ir . n(n— 1) ^n.o , n(n— l)(n— 2) ^ ,,, 
{a+b)''=a''+na''^^b'\ — ^^ ^ a'^-^fe^H — ^^ ^ ^a^-^j^. 

.-. (2x2+41/3)3 = (2^2)3+3(2x2)3-141/3+ . . . to 4 terms. 



Digitized by 



Google 



187 



THE BINOMIAL THEOREM 



197 



187. Examples. Solve the following examples either 
by applying the binomial theorem or by substitution in the 
binomial formula under the conditions stated in the pre- 
ceding paragraph. 

An expansion which will be too long for the width of 
the page should be written lengthwise. 

1. Copy and completely solve and simplify the last illustrative 
example of paragraph 186. 



2. ir+s): 


3. (2r+8)». 


4. (2r«+5s)«. 


6. (2r»+5s«)». 


6. (x»-y)«. 


7. (»•-«;»)< 


8. {bx+dy*)'. 


9. (5x-'+j/«)«. 


10. (t-d)'. 


11. (g+h): 


12. ixi-y^)'. 


13. (3n-«-6A;«)'. 


»■ H)' 


»■ (f-)"- 


.«. (t-^.y. 


"• (^"-a-T-. 


18. (2+2/)* to 3 terms. 


19. (x-3)' to 4 terms. 


20. (3v-2w*)-i to 4 terms. 


21. (e«+2c»)-* to 3 terms 


-(-.-)•■ 


23. lx-{ — j to 4 terms. 


24. (2x-'+3j/«)». 


- (-!?.)•• 


26. (7o»-26-»)», 


27. (9x»-7j/-«)». 


28. (12»'-lla;«)-». 


29. (4r»+5«»)> to 3 teiTOB, 


30. (8c»+id-')». 


31. (6x-»+3z')«. 


32. (ll6-»+13d«)'. 


«■ (-':?• 



34. (3«.--) . 



Digitized by 



Google 



198 TECHNICAL ALGEBRA 188 

188. How to Determine any Term in a Binomial Expan- 
sion. By reference to paragraph 183 write the fourth term 
of (a+6)". 

For n substitute 6 and simplify by performing the 
indicated subtractions and denoting the multiplications 
in the numerator of the coefficient. 

Thus ^^a363. 

1. Analyze the coefficient of this fourth term as follows: 
(a) How does the number of factors in the numerator 

compare with the number of the term? 

(6) How does the first factor compare with the exponent 
of the power? 

(c) The last factor in the numerator is how many more 
than the exponent of a? 

(d) How does the number of units in the factorial num- 
ber compare with the number of units in the exponent of 6? 

(e) How docs the decrease in the exponent of a compare 
with the number of units in the exponent of 6? 

2. Write the third term of the same expansion and state 
and illustrate whether these five facts are true in the third 
term. 

3. Write the fifth term of the same expansion of (a+b)^ 
and state whether the five facts recorded in 1 are true 
for the fifth term. 

4. Write the fourth term of (r+s)^ and determine 
whether the analysis of 1 applies in this case. 

5. By reference to the analysis of 1 write a law for any 
term in a binomial expansion. 

189. Examples. Solve the following by the preceding 
law: 

1. Write the 6th term of (2x -5y^yK 

2. Write the 10th term of {5y^-Szy*. 



Digitized by 



Google 



THE BINOMIAL THEOREM 199 



3. Write the 7th term of {bx^-2d*y\ 

4. Write the 3rd term of (5a:»-82/)». 



5. Write the 8th tenn 



of (.+'-)% 



6. Write the middle tenn of (2«»-t;»)". 

7. Write the nth term of (a+h)\ 

8. Write the (n+l)th term of (x+y)\ 

9. Write the 8th term of (1+z^)-^^ and reduce it to its simplest 

form. 

10. Write the 5th term of (1 —a:*)-* and reduce it to its simplest 

form. 

11. Expand (c -ox) - ^ to four terms. 

12. Expand (1 —nx)^ to three terms. 

13. Compute the sum of the coefficients of the 4th and 7th terms 

of (3a;+l)". 

14. Compute the sum of the coefficients of the 8th and 12th terms 



-H")'" 



Digitized by 



Google 



CHAPTER XIII 

POWERS AND ROOTS 

Section 1, Powers. Section 2, Square Root. Section 3, 
Cube Root. Section 4, Other Roots. 

§ 1. POWERS 

190. Power of a Monomial. In Chapter XI the rule was 
given that a monomial may be raised to any power by multi- 
pljring each exponent of its factors by the exponent of the power. 
The reason for the rule will now be developed. 

In (40^63)2 the exponent 3 denotes that ^a^l^ is to be 
taken how many times as a factor? 

Therefore 4 must be used how many times as a factor? 

And a^ must be used how many times? 

And 6^ how many times? 

Therefore the cube of 4ia^lP equals what power of 4 times 
what power of a times what power of 6? 

Could these powers of 4, a, and 6, have been written 
directly by multiplying the exponents of these three factors 
by 3, the exponent of the power? 

Therefore (4a263)3 equals what? 

In order, therefore, to raise a monomial to any power 
what operation should be performed 

(1) on the numerical coefficient? 

(2) on the exponents of its literal factors? 

191. The Binomial Formula. As shown in the pre- 
ceding chapter a binomial may be raised to any power 
by substitution in the binomial formula. By the use of 

200 



Digitized by 



Google 



192 



POWERS AND ROOTS 



201 



marks of parenthesis this formula may also be used for 
detennining any power of a trinomial and an expression 
having four terms. 

Thus (a+b+c)^ = (^+c)^, 



and (a+b+c+d)^ = {a+b+c+d^; 

the vinculum being used in both illustrations to transform 
the given expressions into binomials. 

In this connection it should not be forgotten that the 
square of any polynomial is most readily obtained by the 
law given in Chapter V. 

192. Examples. By inspection expand the following. 
In each example state what law was applied. 



1. i2x+3y-^y. 


2. (2a-462+2c-5d')«. 


3. (ix^-2a%)*. 


4. (2a;-y+3c2-r«)*. 


5. {5v-^+Qw^)-\ 


6. (ax'-cy«)'- 


7. {2ab^+x^)^ to 4 terms. 


8. (a;«-4a2y)-J to 4 terms, 


9. {-2)K 


10. (-2)-«. 


11. (-2)3. 


12. (-2)-'. 


13. (-3a^x*zi)\ 


14. (-3oi^<3»)». 


16. (-7a36'»d«-2)2. 


16. (-6c-2diA"-3)3. 


17. (5a26-4c)~. 


18. (4a;i^-i)-3». 


19. (^)^ 

xy 


20. ^-«r 
26* 


21.W'. 
2 


22 ^' 
22. 2 . 


23. (^)"*. 
4 




26. (y-2): 


»■ H); 


«■ (!+-)•■ 


28. {y+-j to 4 terms. 



Digitized by 



Google 



202 TECHNICAL ALGEBRA IM 

29. (c+d-r-sy. 30. (x^-y^-v+w^)*, 

31. iv+x-izy. 32. {a-3x^-iy-2c»y. 

33. (^3r+26-2)-». 34. (46-i-2c-J)-2. 

193. Any Root of a Monomial. A monomial may be 
raised to any power by what operation on 

(1) its numerical coefficient, 

(2) the exponents of its literal factors? 

The extraction of a root is the inverse of the determination 
of a power. Therefore any root of a monomial may be 
extracted by what operation on 

(1) its numerical coefficient, 

(2) the exponents of its literal factors? 

Thus v^8xV equals what? 

and v^lGa^fe-^c equals what? 

Reduce each of the two results to a form having no 
fractional and no negative exponents. 

194. Sign of the Root. Perform the following indicated 
operations: 

(2x22/)2, (-2x22/)2, (-2x22/)4. 
(2x22/)3, (-2x22/)3, (^2x^y)\ 

The results show that: 

(1) Even powers have what sign? 

(2) Odd powers have what sign? 

(3) Even roots of positive quantities have what sign? 

(4) Even roots of negative quantities are impossible. 
Why? 

(5) Odd roots have what sign? 



Digitized by 



Google 



196 POWERS AND ROOTS 203 

196. Examples. Extract the indicated roots of the 
following and reduce the results to forms with no fractional 
and no negative exponents: 

1. V25a*6«c-». 2. (25a^«c-»)*. 

3. \^8rH^K 4. (8r»<«z;«)*. 

5. \^81x-^-K 6. (81x-^-»)*. 

7. \^Slx-^-\ 8. (81x-^-»)"*. 

9. V^(2r-3i2)io. 10. (2r-3/»)*. 

11. <^-216a»6-». 12. (-216a»6-»)*. 

^^ 5/ 243a:ioy-» .. /243xiV-*\* 

^^- \"32i^- ^*- V^2^^/- 

17. V-4a«6^ 18. (-4a«60*. 

§2. SQUARE ROOT 

196. Square Root of a Polynomial. The following 
instructions should enable you to extract the square root 
of any polynomial. 

(1) Arrange the terms in descending or ascending order 
with respect to the same letter. 

(2) Enter at the right the square root of the first term 
of the polynomial. 

(3) Subtract its square from the first term and bring 
down the next term of the polynomial. 

(4) At the left set down a trial divisor determined 
from the following law: 

Trial divisor = 2 Xroot. 

(5) Enter in the root the number of times the trial 
divisor is contained in the first term. ^ 

(6) Add to the trial divisor to form the complete divisor, 
last term of root. 



Digitized by 



Google 



204 TECHNICAL ALGEBRA 197 

(7) Multiply the complete divisor by the last term of 
the root and subtract, bringing down more terms as may 
be necessary. 

(8) Repeat (4), (5), (6), and (7) until the root is 
obtained. 

Thus 9a^ - 24x3 « 14a;2+40x+25( 3x2-4x~5 

9^* 

6x2 -4a;) _ 24x3 - 14x2 
24x3+16x2 
6x2-8x-5) -30x2+40x4-25 
-30x2+40x+25 

197. Examples in Square Root. Extract the square 
root of the following: 

1. 166-24a+9a«+16+46«-12a6. 

2. 25x«-120fox+1446«. 

[3. v*-vy+jy\ 

L ^ 

'4. (x+2/)«-6(x+y)+9. 

^^5. 12ac+9c*+6«+4a«+4a6+66c. 

6. 9x-24x2/+16y. 

7. 4x*+~-+|x+4x3+|x«. 

r- 1 

8. x+— to 3 terms. 
[9. 5-2x to4 terms. 

10. r*-6r»+^-2r+9?r2. 

11. m^-SOtv' -2^t^v+m*v^+2^*. 

12. a«+462-8cd+2ad+16c2-8ac+d«+4a6+4W-166c. 



Digitized by 



Google 



199 POWERS AND ROOTS 205 

§3. CUBE ROOT 

198. Cube Root of a Polynomial. The following laws 
should enable you to extract the cube root of any polynomial: 

(1) Arrange the tenns in descending or ascending order 
with respect to the same letter. 

(2) Enter at the right the cube root of the first term 
of the polynomial. 

(3) Subtract its cube from the firslb term and bring 
down the next two terms of the polynomial. 

(4) At the left set down a trial divisor determined 
from the following law: 

Trial DivisoR=3Xroot2. 

(5) Enter in the root the number of times the first term 
of the trial divisor is contained in the first term of (3). 

(6) Add to the trial divisor to form the complete divisor: 

(a) 3 X last term root X all terms root except last, 
(6) last2. 

(7) Multiply the complete divisor by the last term 
of the root, subtract, and bring down as many terms of 
the polynomial as may be necessary. 

(8) Repeat (4), (5), (6) and (7) until the root is obtained. 

Thus 27a3-54a26+36a&2-863(3a--26^ 

27a3 
27a2-18fl6+4b2 ) -54a26+36a&2-8b3 
-54a2b+36ab2-8fe3 

199. Examples in Cube Root. Extract the cube root 
of the following: 

1. x^+Sx^+3xy*+yK 

2. 33x2-63x»+66a;*+l+8a;«-9a;-36x«. 

3. -18a2+13a»+3a6+a«+8+12a+9a<. 

4. 2/«-127/-1122/»-482/-f542/*-f 1082/2+8. 
6. r«+3r5+2r»-3r7+3r«-6r*-3r+3r«+l. 



Digitized by 



Google 



206 TECHNICAL ALGEBRA 200 

6. a»+3a+~+-. 

a» o 

7. 368+106«-126*+126^-106»-36+l+66«. 

8. 42s^-9s-36s»+9s»-l+8s«-21s«. 

9. 2/'-l to 3 terms. 
10. 64r«-2to4tenns. 



§4. OTHER ROOTS 

200. What Roots Can be Extracted. If a root higher 
than the cube root is required it can be determined by the 
processes of this chapter only in case its index is factorable 
into 2s and 3s. 

Resolve the following root indexes into 2s and 3s as 
far as possible: 

4, 5, 6, 7, 8, 9, 10, 11, 12, 13, 14, 15, 16. 

What root denoted by these indexes cannot be extracted 
by applications of square root and cube root? 
How can the 4th root be obtained? 

201. Method of Signifying Successive Roots. If x 
denotes any quantity the process of finding the 12th root 
may be indicated as follows: 



Indicate the process of finding the following roots: 

1. v^ 2. v^ 3. \^x. 

4. v^ 5. v^ 6. v^ 

In computation all roots are determined by logarithms and 
checked on the slide-^ule. 



Digitized by 



Google 



CHAPTER XIV 

RADICALS 

Section 1, Reduction. Section 2, Addition and Subtrac- 
tion. Section 3, Multiplication and Division. Sec- 
tion 4, Powers and Roots. Section 5, Rationalization. 
Section 6, Radical Equations. 

202. Operations with Radicals. In computation and 
in the further study of mathematics little use is made of 
radicals as conventionally presented. 

In this chapter the following fundamental operations 
are discussed and whenever the conventional and practical 
methods of operation differ, both are shown. 

(1) Reduction of radicals. 

(2) Addition and subtraction of radicals. 

(3) Multiplication and division of radicals. 

(4) Powers and roots of radicals. 

(5) Rationalization of radicals. 

(6) Radical equations. 

The study of this chapter will give excellent practice 
in the simplest arithmetical processes, the use of expo- 
nents, the interpretation of mathematical expression, and 
an increased ability to transform the equation, all of which 
is worth while even to the most practical. 

203. Kinds of Radicals. A radical is an indicated root of 
any quantity. 

Thus V25, v^, 1728*, v1[%: 

207 



Digitized by 



Google 



208 TECHNICAL ALGEBRA 204 

There are two kinds of radicals: 

(1) Rational, or indicated roots which can be exactly 
determined. 

(2) Irrational or surd, or indicated roots which cannot 
be exactly determined. 

Thus V49, v^ are rational, 

and VT?, V^156 are surd. 

In two different ways denote 

(1) four rational radicals, 

(2) four surds. 

State why the first four are rational and why the second 
four are surds. 

§ 1. REDUCTION 

204. Processes of Reduction. A radical is said to be 
reduced when changed to a different form having exactly the 
same value. 

Thus V98 = V2X49 = 7 V^. 

The processes of reduction are as follows: 

(1) Reduction to simplest form. 

(2) Reduction to full radical form. 

(3) Reduction to same degree. 

205. Reduction to Simplest Form. A radical is in its 
simplest form, only when the quantity under the radical is 

(1) integial and 

(2) as small as possible. 

It is as small as possible only when 

(a) it is not a power of the indicated root and 

(Jb) has no factor which is a power of the indicated root. 



Digitized by 



Google 



205 RADICALS 209 

Thus 'vS has the first requirement of simplest form 
because it is integral. It does not have the second require- 
ment because it is not as small as possible. It is not as 
small as possible because it is a power of the indicated root. 

Thus v^8=2. 

^^432 has the first requirement but not the second 
because it has a factor which is a cube. 

Thus v'l32 = v^2X216 = 6v^. 

6v^ is in simplest form, being both integral and as 
small as possible. 

-W- is not in its simplest form because a fraction. It 

can be reduced by multiplying both numerator and denomi- 
nator by the smallest number which will make the denomi- 
nator a power of the indicated root. 

-^15 is in its simplest form because the quantity under 
the radical is integral and as small as possible. 



When the radical has a coefficienti the root of any factor 
under the radical must be multiplied into the coefficient 

Thus (o+6)V3(a2+2a6+62) = (a+b)^Vs, 

and 8V32=8Vi6X2=8X4V2=32V2. 



Digitized by 



Google 



210 TECHNICAL ALGEBRA 206 

206. Examples in Simplest Form. Solve the following 
by reduction to simplest form as indicated In the preceding 
models: 

.1. v^. 2. v^ 3. V72. 

4. <^i, 6. Vm, 6. \^m, 

[7. v^. 8. V96. 9. V20OO. 

10. V392. 11. 3V^3456^ 12.T2V^12432. 

13. {a+z)Vs{a+x)K 14. Vsx^-lGx. 

16. V3z^+Qxy+Sy\ 16. a"'\/i^- 

, 2(7^ ^^ 4/l6a« 



19. 4a;'\/5aa;-*. 20. V{r^-s^)(r-s). 

21. \^i728^. 22. xf^. 

\x~y 

\. ^I^\ 24. '/-^^^. 



23 



207. Simplest Form from the Practical Standpoint. 

From the practical side the simplest form of a radical, as 
of any other expression, is the form which is the simplest 
for computation. 

Thus V^192 whose conventional simplest form is 
'>J^64X3 = 8v^, is in a simpler form before reduction than 
afterwards for the reason that it is just as easy to determine 

v^l92 as to determine v^ 

and in the latter form the root must be multiplied by 8. 

Look over the examples worked in paragraph 206 and 
if any would be simpler in computation in the form to 
which they were reduced, specify which and^ state reason. 



Digitizegl by 



Google 



209 RADICALS 211 

208. Reduction to I*ull Radical Form. This process is 
accomplished by introducing the coefficient of the radical, 
under the radical sign. The operation required to do this 
is determined by the root index. 

If the index is 2 the coefficient must be squared; if the 
index is 3 the coefficient must be cubed before being placed 
under the radical sign. We therefore have the following 

Rule. To introduce the coefficient of a radical, under the 
radical sign, raise it to a power of the same degree as the 
radical. 

Thus 5 V2 = V25X2 = V50, 

and 3 vY= v^27X7 = v^O", 

and (3 +x) V3+2x = V(S+x)^(S+2x) . 

From the practical standpoint this process is the height of 
absurdity and would never be employed, 

209. Examples. In the following introduce the coeffi- 
cients under the radical; express the results in simplest 
form. 

1. 5V8. 2. 3\/i2. 

3. (a+b)V^. 4. |v^. 






2a -26 



3 

8 ' *" a;»' 



9. |(2J»-2/^)*. 10. -XS-x)-K 



11. (a+6)(a«+6«)-«. ^2. |^ 



8 



1416* 



Digitized by 



Google 



212 TECHNICAL ALGEBRA m 

13. ZabxV{3abx)-K 14. ^V^. 

o 

15. 9^. 16. 5y-.^. 

17. 3-iV27ai 18. 26*^346-*. 

210. Reduction to the Same Degree. The degree or 
order of a radical is denoted by the root index. With no 
index the radical is of the second degree; with three as index 
the radical is of the third degree, and so on. 

Radicals are therefore reduced to the same degree by 
transforming them to radicals of the same root index. This 
may be easily done by the following process: 

(1) Express the radical quantities with fractional 
exponents. 

(2) Reduce the fractional exponents to the least com- 
mon denominator. 

(3) Express the results with radical signs. 

Thus V3, v^", and v^ 

are reduced as follows: 

V3=3* = 3A = v^, 

211. Examples in Reduction to Same Degree. By the 

process of the preceding paragraph reduce the following to 
equivalent radicals of the same degree. Express all final 
results in simplest form. 

1. V20, <^m, ^M. 2. ^, <^S, <^. 

3. v^, v^, VK. 4. v^, Vxb, </y. 



Digitized by 



Google 



li: RADICALS 213 

6. Vsi, v^26, V^3y^ 6. v^2xS v^o+6. 

7. Vsi, v^, v^. 8. ^|, \^{x-'S)'K 

9. V^, V^2x-«. 10. \^x^+y*, y/2x^+ixy+2y*. 

§ 2. ADDITION AND SUBTRACTION OF RADICALS 

212. Similar Radicals. If radicals of the same degree 
have exactly the same quantity under the radical sign, they 
are similar. 

Thus Sy/2ax, cV2ax are similar because both are of 
the second degree and have the same quantity 2ax under 
the radical sign. 

Similar radicals are added by collecting their coefficients, 
the same as in the addition of like terms. 

Add the following radicals: 

1. 4V5+2V5-5V5. 

2. sVxy-aV^+bVxy-sVTy. 

3. bVa-c+xVa-c+sVa+c-Va+c. 

Radicals which are not similar may sometimes be made 
similar by reduction to simplest form. 

Thus V27+\/T8+V98 = 

V9X3+V9X2+V49X2 = 

3\/3+3V^+7V2= 

3V3+IOV2. 

In computation the original indicated roots would be 
directly determined and the sum found wiih no attempt at 
reduction to similar form. 



Digitized by 



Google 



214 TECHNICAL ALGEBRA 213 

213. Examples. Reduce the following to simplest form 
and add: 

1. V20+V45+ J|. 

2. V24-Vi50+3V54. 

3. V98+V2OOO-2V72. 

4. 5V48-6V75-.-\/27. 

6.^ 7V2(x+2/)2 - V 128+ Wm, 

6. 2V^-oV^+6V^^. 

7. rV^4(x-2/)3+t;V^108(r3+3r22/+3n/2+2/'). 

8. V5x2 - 10xy+5y^ - V20x2+20cc+l. 

9. V^2a« - 6a26+ 6a62 _ 2b' + V^6466«. 

11. V^+ V^TisS - \/2x^\ 

12. 4V^^-5V^8^+|^-. 

§ 3. MULTIPLICATION AND DIVISION OF RADICALS 

214. Multiplication. Radicals of the same degree are 
multiplied by multiplying together 

(1) their coefficients and 

(2) the quantities under the radical. 

Thus {a-b)V2xX{a+b)\/5xy = 

(a2 - 62) Vl0x2// = x(a2 - 62) Vlb7. 

When the radicals are of different degree the multipHca- 
tion may be indicated; or the radicals may be reduced to 
equivalent radicals of the same degree and then multiplied. 



Digitized by 



Google 



216 RADICALS 215 

Thus 3V5^X4v^ = 12 Vs^- \^, 
or 3(5a)i = 3(5a)S = 3\/(5^ 

4(6a)» = 4(6a)» = 4v^(6^ 

It should be unnecessary to state that the second method 
should not be used in the work which follows and that it wculd 
not be tolerated in computation, 

215. An Important Principle. The product obtained 
when a radical is multiplied by itself is best shown by the 
use of fractional exponents. 

Thus •\/a+2xxVa+2x = (a+2x)*X(a+2x)» = a+2x. 

If therefore, the square root of a quantity be multiplied 
by itself the result is the quantity without the radical. 

Thus V2x-32/XV2x-32/ = 2x-3?/, 

and 2V29X3V29 = 6X29 = 174. 

216. Examples. Perform the following indicated multi* 
pUcations and express the results in the simplest form: 

1. VT2XVJE. 2. V^xVa+6. 

3. 8V3X2V5. 4. 2v^X7V^5l 

6. v^XV^. 6. 4V^X3V^7! 

7. VTsx^y/^. 8. aV^Xft^V^. 

o 

9. VxyX\^. 10. \^Sa^X\^isiu\ 

11. (V^+l)(vV-5). 12. (V2b-y){V2b+4) 

13. (4V^-3)(5Vx+l). 14. (4V^+r)(4V^-r). 

16. (VxT^+2)(V^-2). 16. (\/x2+3 -5){Vx^-\-'S -2). 

17. (V^+^-\/^)2. 18. (2V^-4Vh-2)2. 



Digitized by 



Google 



216 



TECHNICAL ALGEBRA 



217 



217. Division. Since division is the inverse of multi- 
plication it should be possible to solve the examples in the 
following paragraph without special instructions. 

One important principle may be noted: 

a+x (a+xy f\ \. ^/ — I — ' 



Therefore if any quantity be divided by its square root 
the quotient is the square root of the quantity. 

Show this by two illustrations, one literal and the other 
numerical. Number all equations and give the mathemati- 
cal authorities for all operations. 

218. Examples. Perform the following indicated divi- 
sions and express the results in simplest form: 



Observe that — ^ = 



Vb' 



1. V24^V6. 
. V12T 



2. V729^V8T. 
V5OO 



7. 



10. 



\/02.5' 



6. 



V24-A/i6+\/36 



8. 



V20* 





V4 


Wx- 


■8 
-2 


</^ . 





12. ,— ^ , 



14, 



V28* 



3. 



6. 



V58 
V29* 

<r^x 
x-\-y 



^x+y 



11. 



13. 



15. 



38 



I9V3-57* 

4V75 
5V56' 

7V24Sx*y-^ 
28V363x-2y** 



Digitized by 



Google 



221 RADICALS 217 



§4. POWERS AND ROOTS OF RADICALS 

219. Any Power of a Monomial. Write the answers to 
the following questions: 

In order to square a monomial what operation is per- 
formed 

(1) on the numerical coefficient; 

(2) on the exponents of the literal factors? 
Illustrate by working out the square of bah''^(? with 

all equations numbered and all mathematical authorities 
for the operations. 

In the same form work out the square of VGab. 

Show the full work with authorities for (\/bx^y)^. 

The square of the square root of a quantity therefore 
equals what? . 

The cube of the cube root of a quantity equals what? 

A monomial may be raised to any power by what opera- 
tion on its numerical coefficient? 

By what operati on o n the exponents of its literal factors? 

Illustrate by (VGox)®. 

220. Any Power of a Binomial. By what law can a 
binomial be raised to any power? 

Illustrate by expanding {y/x+2VyY. 

Can a trinomial be expanded by the same law? 

Illustrate by (Va- Vb+Vc)^. 

What is the law for the square of any polynomial? 

Illustrate by (V2^-2V6+4V3c- Vd)2. 

221. Examples in Powers. The following examples 
may be worked in the form given or with the roots denoted 
by fractional exponents, as preferred. 

1. (aV2a6)«. 2. (-3aV^«)». 3. {-Za</wy. 

4. {?x<^^y. . 6. (3-V6)«. 6. (3V2-4)«. 

7. (x+2Vy)». 8. (2o+3V^)». 9. (3V^-4V6)«. 



10. (V^-5V7)». 



Digitized by 



Google 



218 TECHNICAL ALGEBRA 222 

222. Roots of Monomials. Any root of a monomial 
radical may be readily determined by the use of fractional 
exponents in place of the radical signs. When this change 
is made, in order to determine the root what operation 
must be performed. 

(1) on the numerical coefficient; 

(2) on the exponents of the literal factors? 

Illustrate by \^21a^x^ 

also by Va*»6*»+^. 

§6. RATIONALIZATION 

223. What Rationalization is. To rationalize any expres- 
sion, is to free it from radical signs. 

Thus Vx — 1=0 can be rationalized by squaring. 

Why? 

Show the complete solution for x. 



In 



\42/ 



the denominator can be rationalized by multiplication by a 
quantity which will make it a power of the indicated root. 

Show the process of rationalization of the denominator, 
being sure to multiply the numerator by the same quantity 
so that the value of the fraction shall not be changed. 

224. Denominator a Binomial. When a radical denomi- 
nator has two terms it is rationalized by multiplication by itself 
with the sign changed of one of its terms, the numerator being 
multiplied by the same rationalizing factor. 

Thus 

V3 - V5 ^ (V3 -VI) (\/2 - Vg) ^ 
\/2 + V6 (\/2 + Vg) (V2 - Vg) 

(V3-V5)(V2-\/6) _ (\/3-V5)(\/2~\/6) 
2-6 -4 



Digitized by 



Google 



227 



RADICALS 



219 



Show this illustration in full and reduce to the simplest 
form. 

State why the factor V2— Vg rationalized the denomi- 
nator. 

225. Examples. Rationalize the denominators of the 
following: 



6. 



Va+Vb 



-y 



-Vx+y 



^x—y+\^x+y 



2. 



6. 



10. 



3Vx" 

_y36+2 
V3b^4x^' 

2-V3 

V5+W2' 

2-Va ^ 
3+Va^x 
Vx^-x y +y^+2 
Vx^+xy+y^*-2 



§6. RADICAL EQUATIONS 

226. Principle of Operation. A radical equation is one 
in which the unknown quantity is under a radical sign. The 
general method of solution is to transpose the radical to 
one member (entirely by itself) and then to raise the 
equation to a power of the same degree as the root index. 

Illustrate by solving \^2x+l -3 = 0. 

also by solving Vx+S =4. 



227. Examples. Solve the following equations: 

1. 2V^ = 14. 2. \V9+x^=3. 

o 

3. V6»-a;=7. 4. aVsb+x* =a». 



Digitized by 



Google 



220 TECHNICAL ALGEBRA 227 

6. 8-2VJ=6. 6. 7-4V3x=0. 

7. 3vV+2 = vV+5. 8. 2V3x-14=8. 

9. Va;»-7a;=a;-V5". 10. V5x«-10a;+20 = V5x». 

When a radical equation is not cleared of radical signs 
in one operation the operation is repeated after the remain- 
ing radical is transposed to on e member entirely by itself. 

Illustrate by solving Vx— 32-j- Vi = 16. 

Solve the following: 

11. V3y+10-V3y+25=-3. 

12. ^4a;+12=6. 13. V2x-7-8= -V2x+9. 

14. .L_pi-~=7. 16. 3VT-10=-V25+8A. 

\ 5r+7 

16. v^2y-8=2. 17. 3-Va+n = -Vo-3. 

18. V2/+11-3 = VV^. 19. .^ =\/2a+9~V2a. 

V2a+9 

20. \/6"-V6^=-^. 



Digitized by 



Google 



CHAPTER XV 

LOGARITHMS 

Section 1, Logarithm op a Number Greater than Unity. 
Section 2, Logarithm op a Number Less than Unity. 
Section 3, Naperian or Hyperbolic Logarithms. Sec- 
tion 4, lyOGARITHM OP A PRODUCT. SECTION 5, LOGARITHM 

OP A Quotient. Section 6, Logarithm op a Power. 
Section 7, Logarithm op a Root. Section 8, Solu- 
tion OP AN Exponential Equation. Section 9, Model 
Solutions. Section 10, Logarithmic Computation. 

228. Two Ways of Multiplying. 100 may be multi- 
plied by 1000 as follows: 

(1) 100X1000=100000, 

(2) 102 X 103 = 10^ = 100000. 

Observe that in (2) the product is obtained by addition 
of the exponents of the powers of 10 which equal 100 and 
1000. It is therefore possible to multiply together numbers 
which are integral powers of 10, by addition of the exponents 
of these powers. 

In like manner 472 may be multiplied by 67.5 by addi- 
tion of the exponents of the powers of 10 which equal 472 
and 67.5. But here are two difficulties: 

What powers of 10 equal 472 and 67.5, and what does 
10 equal when raised to the sum of these two powers? 

The answer* to this question and the removal of the 
difficulties follow: 

100=102 
472 = 10^ + 
1000 = 103 
67.5 = 10^ + 
10=101 

221 



Digitized by 



Google 



222 TECHNICAL ALGEBRA 229 

472 is greater than the 2d power of 10 and less than the 
3rd; 67.5 is greater than the 1st power of 10 and less than 
the 2d. 

Therefore 472 = 10^+" ^^^^™^ 

r*fj r 1 r\l +a decimal 

A table of logarithms is an arrangement of numbers in 
sequence with the decimal parts of the powers of 10 which 
equal the numbers. 

Taking the decimals from the table we have: 

472=102-673942 

67.5 = 10^-»2Q^Q^ 
Therefore 472X67.5 = 10^503246 

By the table 104503246 = 31860. 

Therefore ^472 X 67.5 = 3 1860. 

In practice the labor of this method of multiplication 
is reduced by setting down only the exponents of 10 as 
follows: 

472X67.5 = 31860 2.673942 
1.829304 
4.503246 

In like manner by the use of a table of logarithms, one 
number may be divided by another and any power or root 
of a number determined. 

229. What a Logarithm is. Logarithms are used as a 
means of shortening and simplifying the mathematical 
processes of multiplication, division, powers, and roots. 

In every system of logarithms all numbers are regarded 
as powers of another number which is called the hase of 
the system. 

Therefore the definition of a logarithm: 

The logarithm of a number is the exponent of the power 
to which the base of the system must be raised to equal the 
ntmiber. 



Digitized by 



Google 



230 LOGARITHMS 223 

In the system of logarithms in common use, called the 
common or Briggs' System, the base of the system is 10. 
Hence in this system all numbers are regarded as powers o/ 10. 

Consider any number, as 306. In the Briggs' system 
the logarithm of 306 is the exponent of the power to which 
the base 10 must be raised to equal 306. 

Now 102 = 100 and 10^ = 1000. But 306 is greater 
than 100 and less than 1000. Therefore in order to obtain 
306 from 10, 10 must be raised to a power between the second 
and the third. 

Therefore 306 = 10^ +^ ^^"^"^^^ 

As shown 2+ a decimal is the exponent of the power 
to which the base* 10 must be raised to equal 306. 

But by definition the exponent of the power to which 
the base must be raised to equal a given number is the 
logarithm of that number. 

Therefore log 306 = 2+a decimal. 

This decimal is given in the table of the Logarithms of 
Numbers. 



§ 1. THE LOGAMTHM OF A NUMBER GREATER THAN 

UNITY 

230. A Number Having Three Figures. Direction I. 
In the table of the Ix)garithms of Numbers find 306 in the 
column headed N, 

Place the index finger of the left hand directly under 
306 and move the hand to the right in a horizontal line 
until it is under the number in the column headed 0. This 
number with the two figures prefixed (called leading figures) 
which are immediately above the blank space to the left 
of it in the same column, is the decimal part of the logarithm 
306. 

Therefore lo^ ^85721 ^ 3q^ . j^g ^^ ^ 2.485721. 



Digitized by 



Google 



224 TECHNICAL ALGEBRA 231 

The log of 306 therefore consists of two parts: 

(1) Integral, called the characteristic, 

(2) Decimal, called the mantissa. 

The decimal part only, is given in the tables. 

231. Accurate Use of the Tables. The above direction 
for using the index finger of the left hand is given in order 
to secure speed and accuracy in the use of the tables. It 
makes possible the unobstructed use of the right hand for 
writing the figures from the tables and the index finger can 
be kept in its position on the page until the required number 
has been written from the table and the written number compared 
with the printed number. 

Another excellent method is to move a straight-edge 
or a blank sheet of paper up or down the page until the 
required number can be read just above the upper edge. 

232. Reason for the Characteristic. The integral part 
of the logarithm of 306 is 2, but the number of figures in 
306 is 3. 

In this instance the integral part of the logarithm is 
one less than the number of integral figures in 306, the 
natural number. 

This is also true regarding any other number, for example 
4798. 

This number is greater than 10^ and less than 10*. 
Therefore as in the case of 306 the integral part of the 
logarithm is one less than the number of integral figures 
in the natural number. 

233. Characteristic of the Logarithm of a Number 
Greater than Unity. Rule 1. The characteristic (integral 
part) of the logarithm of any number greater than 1 is one 
less in unit value than the number of integral figures in 
that number. 

By this rule the characteristic of the logarithm of any 
number greater than unity may be determined without 



Digitized by 



Google 



286 LOGARITHMS 225 

the necessity of locating it with respect to integral powers 
of 10. 

234. Position of Decimal Point. Since the division of 
a number by 10 is made by moving the decimal point 
one place to the left and division by 100 is made by moving 
the decimal point two places to the left and so on, the position 
of the dedrhal point affects the characteristic only. 

For example, log 306 = 2.485721 

log 30.6 = 1.485721 
log 3.06 = .485721 
log .306 =*I. 485721 
log .0306= 2.485721 

Each number, being one-tenth the preceding, represents 
one less integral power of 10. Therefore, every shift of 
decimal point to the left means one less integral power 
of 10 and every shift to the right, one more integral power 
of 10. 

In both cases the mantissa is the same. In reading 
mantissas, therefore, disregard decimal points in the natural 
numbers. 

236. A Negative Characteristic. The mantissas of the 
logarithms of all numbers are positive but the character- 
istics may be positive or negative. They are positive for 
numbers greater than unity; they are negative for decimals. 

It is therefore impossible to denote the logarithm of a 
decimal by a minus sign written in the usual position since 
that would indicate the entire logarithm as negative. 
Accordingly negative characteristics are indicated by a minus 
sign ABOVE them. 

Thus log .401 = 1.603144. 

*The minus sign above the characteristic is used to denote that 
the characteriatic is negative but that the mantissa is positive. 



Digitized by 



Google 



226 



TECHNICAL ALGEBRA 



237 



236. Examples. About one inch and a half from the 
top of the page rule the following table with the columns 
long enough for twelve entries and sufficiently wide for the 
niunbers in this paragraph and in paragraphs 238 and 243. 

Table VII 
LOGARITHMS 



3 Figures. 


4 Figures. 


>4 Figures.* 


Number. 


Logarithm. 


Number. 


Levari thm. 


Number. 


Logarithm. 



























In the first column enter the following numbers: 375, 
189, 208, 784, 999, 118, 510, 619, 200, 907, 666, 103. 
In the second column enter: 

(1) The characteristics (determined by rule I), and 

(2) The mantissas (determined by direction I). 

237. A Number Having Four Figures. Direction II. 
To read the logarithm of a number consisting of four figures, 
as for example, 5848, find the first three figures, 584, in 
the N column. 

Place the index finger of the left hand immediately 
under 584 and move the hand in a horizontal line to the 
right until it is under the number in the column headed 8. 
This number with the leading figures prefixed from the 
zero column is the mantissa of the log 5848. 

The mantissa of the logarithm of any number containing 
four figures is always to be found in the same horizontal 
Hne as the first three figures, in the column headed by the 
fourth figure of the number. 

■ * This symbol means " greater than." 



Digitized by 



Google 



2S9 LOGARITHMS 227 

238. Examples. In Table VII enter the following 
numbers and their logarithms: 2084, 3009, 6000, 1289, 
9104, 1059, 7855, 8899, 5123, 7248, 6167, 1070. 

239. A Number Containing any Number of Figures. 

Consider the number 84678. This number is greater than 
84670 and less than 84680. 

log 84680=4.927781 
and log 84670 = 4.927730 

10^ 51 

That is, a difference of 10 in these natural numbers 
corresponds to a difference of 51 in their logarithms. 

In other words a difference of 10 between 84670 and 
84680 is expressed by a difference of 51 in their logarithms. 

Since the difference of 10 in the natural numbers equals 
a difference of 51 in their logarithms, a difference of 1 in 

natural numbers equals a difference of ttt of 51, or 5.1 in 

logarithms. 

Therefore a difference of 8 in natural numbers equals 
a difference of 8 times 5.1, or 40.8 in logarithms. 

Therefore the logarithm of 84678 is 40.8 greater than 
the logarithm of 84670. 

Therefore log 84678 = 4.927771. 

The same result would have been obtained by multi- 
plying 51 (the difference between log 84670 and log 
84680) by 8 the last figure of the given number 84678, 
with a decimal point before that figure. 

A continuance of this investigation with numbers of 
more than four figures will give the following rule: 

Rule. The logarithm of a number having any number 
of figures equals the logarithm of the first four figures as 
given in the tables, plus the product obtained by multiply- 



Digitized by 



Google 



228 TECHNICAL ALGEBRA 240 

ing the difference between the log of a number 1 greater 
than the first four figures of the given number, by all the 
remaining figures of the given number with a decimal 
point before them. 

Therefore, Direction III: To find the logarithm of a 
number consisting of more than four figures, for example, 
642147, find the logarithm of the first four figures, as in 
Direction II. 

Then multiply the difference between the mantissa 
of the next higher number of four figures, 6422, and the 
mantissa of 6421 (the first four figures), by the remaining 
figures of the given number with a decimal point before 
them. 

Add this product to the logarithm of the first four figm-es, 
placing it for performing the operation of addition so that 
the right integral figure of it is under the right figure of the 
mantissa of the logarithm to which it is to be added. 

Thus 



log 6422= 


=807670 


log 6421 = 


=807603 


difference 


67 




.47 




469 




268 




31.49 


.807603 



log 642147 = 5.807634 

240. Tabular Difference. In some tables the average 
difference between successive mantissas, called tabular 
difference, is given in a column headed D. In using such 
tables it is customary not to ascertain the exact difference 
by subtraction, but to multiply the tabular difference in the 
same horizontal Une as the mantissa of the given number 
(if there is no tabular difference in the same horizontal line, 
use the tabular difference immediately above), by all the 



Digitized by 



Google 



242 LOGARITHMS 229 

figures of the natural number except the first four, with a 
decimal point before them. 

241, Proportional Parts. In modem tables of log- 
arithms the last column is used for the proportional parts 
which must be added to the mantissa when the number 
has more than four figures. 

The entries in the column are as follows: 

The numbers at the top of each group are differences, 
those at the left are the figures of the natural number, 
and those in the several groups are the proportional parts 
required for a fifth figure of a natural number, provided the 
mantissa table gives readings for the first four significant 
figures. 

If the reading is for a sixth figure, one figure must be 
pointed off in the number read; if the seventh figure, two 
places must be pointed off, and so on. This is due to the 
fact that the value of a figure in the sixth place is only 
one-tenth of its value in the fifth place, and its value in 
the seventh is only one-hundredth of its value in the fifth 
place. 

The column of proportional parts saves time and labor 
and does away with the necessity of multiplying the dif- 
ferences as required in the preceding paragraph. 

In the Author's six-place tables, proportional parts 
are given at the bottom of the page, arranged with differ- 
ences at the top and figures of the natural number at the 
left. 

242. How to Use the fable of Proportional Parts. In 

this paragraph the logarithm of 7145863 will be determined 
by 

(1) Direction III, 

(2) The table of proportional parts. 

(1) The logarithm of the first four figures 7145, is 
6.85400i. 



Digitized by 



Google 



230 TECHNICAL ALGEBRA 242 

Multiplying the dijBference 61, from the D column by 

the remaining figures with a decimal point before them 
we have: 

.863 6.854002 

61 53 

— 863 •'• log 7145863 = 6.854055 
5178 
52.643 

(2) The readings from the table give: 
6.854002 
49 
37 

18 

6.854055 

In the reading, 

49 was the reading for 8, the fifth figure of the given 
number, and was therefore set down directly under the 
mantissa; 

37 was the reading for 6, the sixth figure of the given 
number, and was therefore shifted one place to the right; 

18 was the reading for 3, the seventh figure of the given 
number, and was therefore shifted two places to the right 
when set down for addition. 

Observe that in the result only 6 places were retained 
because the readings were taken from a 6-place table. 

Always use exact difference. An inspection of any 
page of the table of the logarithms of niunbers will show 
that the entries in the D column are not exact but are the 
average differences between successive mantissas for one 
or more lines. 

Therefore, whenever a reading is taken which requires 
the use of the table of proportional parts, determine the dif- 
ference between the last figure of the mantissa which is read 
and the last figure of the mantissa in the next column, and 
take the other figures of the difference from the D column. 



Digitized by 



Google 



244 LOGARITHMS 231 

When reading in the 9 column use the last figure of the 
following zero column to determine the exact difference. 

It is frequently the case that the table of proportional 
parts gives no difference corresponding to the actual dif- 
ference between the successive mantissas. No rule can 
be given as to what difference to use in such cases, as some- 
times the next larger difference will give a more nearly 
correct reading, and sometimes the next smaller, but this 
could not be determined without a table which gives the 
next figure of the mantissas. To secure uniformity of 
results it is suggested that the next smaller difference be 
invariably used under such circumstances. 

In other words, when the exact difference is not in the 
proportional parts table, read proportional part under the 
next smaller difference. 

Therefore when using the table of proportional parts 

(1) Note the actiuil difference between the last figure of 
the mantissa read and the last figure of the mantissa in the 
next column, and take the other figures from the D column. 

(2) If this difference is not given in the proportional 
parts table, take P.P. readings under the next smaller 
difference. 

243. Examples. In Table VII' enter the following num- 
bers and their logarithms, taking all readings from the table. 

Show all computations on a page following Table VII. 

70829, 238495.06, 1007.61, 500.0089, 8799920.678, 
4900.3240, 30005, 800062, 4300091, 699.714, 5.30792, 
19.9049. 

244. Miscellaneous Examples. On the same page with 
Table VII rule Table VIII for the entry of the examples 
below and those in paragraph 249. Make entries as in 
Table VII and show all computation. 

11007, 30000, 54431, 472, 7980, 3466145, 427214.96, 
1C97, 20.0084, 7290.632, 3.008, 47.979. 



Digitized by 



Google 



232 TECHNICxVL ALGEBRA 246 



§ 2. THE LOGARITHM OF A NUMBER LESS THAN 

UNITY 

246. Sign of the Characteristic. Suppose that we wish 
Co determine the characteristic of the logarithm of.l; in 
other words suppose we wish to know what power of 10 
will equal .1. 



Now 


100 = 1 


and 


101 = 10 


but 


1 = 10 times .1 


and 


10 = 100 times .1 



When raised to the zero power, therefore, 10 becomes 
10 times .1 and when raised to the first power it becomes 
100 times .1. 

As even a zero power gives a result ten times too large 
it is evident that 10 can equal .1 only when raised to a 
power less than 0, and therefore to some negative power. 

This power may be determined as follows: 

.1=^=10-. 

01= — = — = 10-2 
100 102 ^^ f 

•^^ " 1000 " 103"^^"^' 

Therefore .1 can be obtained by raising 10 to a power 
indicated by the exponent —1, that is, .1 = 10"^. 

But by definition, the exponent of the power to which 
the base of the system must be raised to equal a given 
number, is the logarithm of the number. 



Therefore 


(a) log .1=-1 


and 


(6) log .01 = -2 


and 


(c) log .001 = -3 



Digitized by 



Google 



247 LOGARITHMS 233 

In (a) it is seen that .1 is obtained from 10 by raising 
10 to a power indicated by the exponent —1. 

In (6) .01 is obtained by raising 10 to a power indicated 
by the exponent —2. 

In (c) .001 is obtained by raising 10 to a power indicated 
by the exponent —3. 

By continuing this analysis with any decimal it would 
be foimd that the characteristic of the logarithm of a deci- 
mal is always negative. 

246. Unit Value of Characteristic. The unit value of 
the characteristic may be determined from a consideration 
of some decimal, for example .306. 

It is evident that .306 is greater than .1 and less than 1. 

But log .1= -1 and log 1 = 0. 

Therefore .^06 can be obtained from 10 by raising 10 
to a power indicated by an exponent greater than —1, 
and less than 0, i.e., —1+ a positive decimal; in other 
words the logarithm of .306 equals — 1+a decimal. 

Therefore log .306 = 1.485721. 

The minus sign is placed immediately above the character- 
istic to show that the characteristic only, is negative, while the 
mantissa is positive. 

It must never be placed in front of but always above 
the characteristic. 

In the preceding work it has been shown that the char- 
acteristic of the log .306 is —1. But the first figure in .306 
is one place from the decimal point. In this instance 
therefore, as in (a), (fe), and (c),the characteristic of the 
logarithm of the decimal is negative and in unit value 
equals the number of places of the first significant figure, 
from the decimal point. 

247. Another Illustration. The decimal .042 is less than 
.1 which equals 10"^ and is greater than .01 which equals 
10-2. 

The power therefore to which 10 must be raised to give 



Digitized by 



Google 



234 TECHNICAL ALGEBRA 248 

.042 is less than the power indicated by the exponent — 1 
and is greater than the power indicated by the exponent — 2. 

But by definition, the exponent of the power to which 10 
must be raised to equal a given number is thr logarithm 
of the number. 

Therefore the logarithm of .042 is less than —1 and 
greater than —2. 

Therefore .042 = 10"^+'*^*^^°^^ 

(See also paragraph 234.) 

248. Characteristic of Logarithm of a Number Less than 

Unity. It is evident from the preceding illustrations that 
the characteristic of the logarithm of a decimal is negative 
and in unit value equals the number of places of the first 
significant figure of the decimal from the decimal point. 

The first significant figure of .042 is two places from the 
decimal point; the characteristic is —2. Therefore 

Rule 11. The characteristic of the logarithm of a number 
less than 1, in sign is negative and in unit value is equal to 
the number of places of the first significant figure of the given 
number from the decimal point. 

249. Examples. Under the heading Decimals enter 
the following with their logarithms in Table VIII and on a 
subsequent page show all computations: 

.428, .0343, .00072, .0700486, .00009915671, .000008409, 

.17, .0056007, .95064, .70147, .314087, .0127078. 

Show also the readings for the logarithms of the follow- 
ing numbers: 

1. 1.009678. 2. 581.639. 

3. .00049637. 4. .072385. 

5. 3891.739. G. .8276309. • 

7. 18.46708. 0. 4.72381. 

9. .008372008. 10. 542.39X10"*. 



Digitized by 



Google 



261 LOGARITHMS 235 

250. Summary. Following is a summary of the essen- 
tial facts presented in the preceding pages of this 
chapter. 

1. Logarithms are exponents. 

2. A logarithm consists of two parts: 

(a) Integral, called the characteristic^ ascertained by 
inspection; 

(6) Decimal, called the mantissa, given in the tables. 

3. The characteristic of the logarithm of a number 
greater than 1, in sign is always positive and in unit value 
is one less than the number of integral figures in the 
number. 

4. The characteristic of the logarithm of a number 
less than 1, in sign is always negative and in unit value is 
equal to the number of places of the first significant figure 
of the number from the decimal point. 

5. The mantissa of a number is not affected by the 
position of the decimal point in that number. 

6. The mantissa of the logarithm of a number having 
less than four figures is in the column in the same hori- 
zontal line with the given number. 

7. The mantissa of the logarithm of a number having 
four figures is in the horizontal line with the first three 
figures of the number, in the column headed by the* fourth 
figure of the given number. 

8. The logarithm of a number having more than four 
figures is obtained by adding to the logarithm of the first 
four figures, either 

(1) the product of the tabular difference by the remain- 
ing figures preceded by a decimal point; or 

(2) the readings from the table of proportional parts. 

251. Antilogarithms. The natural number corresponding 
to any given logarithm is called an antilogarithm. It can 
be obtained by the inverse of the processes employed in 
determining the logarithm of a natural number. 



Digitized by 



Google 



236 TECHNICAL ALGEBRA 251 

In the work-book enter the following: 
Antilog 2.589703 = 

Disregarding the characteristic 2, find the leading 
figures 58 in the table of logarithms of numbers in the 
column. 

Place the index finger of the left hand under these figures 
and move it down the column until it is under 9, the 
third figure of the given mantissa .589703. 

Move the finger to the right along the line or the line 
above, until it is under the mantissa next smaller than 
.589703. 

Write this mantissa in the work-book under the given 
mantissa. 

After it write an equality sign followed by the natural 
number to which it corresponds, the first three figures 
of which are in the N column, in the same horizontal line, 
and the fourth figure is at the top of the colmnn in which 
the mantissa was read. 

(1) Subtract the second mantissa from the given man- 
tissa. 

Determine the figure of the natural number to which 
the remainder is equal, by the P.P. table as follows: 

Subtract the mantissa read, from the mantissa in the 
next column. 

Under this difference in the table of proportional parts, 
find the niunber equal to or smaller than the given remain- 
der. Enter this niunber under the remainder obtained 
in(l). 

The figure at the left end of the line in which the number 
was read is the fifth figure of the natural number. 

Write the five figures now found, as the antilog* 
2.589703 as follows: log-i* 2.589703 = . 

* Antilog is symbolized log~^ the dash 1 signifying anti. Observe 
that the S3anboI is not minus 1, but dash 1. 



Digitized by 



Google 



263 LOGARITHMS 237 

262. Position of Decimal Point. The number obtained 
in paragraph 251 may be pointed off by reference to the 
characteristic 2. 

Does its positive sign indicate that the natural number of 
whose logarithm it is the characteristic, is integral or decimal? 

What is indicated by its unit value? 

Point off the required number of integral figures and under 
the work now entered write reason for the operation as follows: 

Sign of the characteristic 2 is (state whether + or — ). 

Therefore the natural number is (state whether greater 
or less than 1). 

The unit value of the characteristic is (state how many). 

Therefore the natural number has (state how many 
integral figures). 

263. Examples. The following arrangement is suggested 
for the work of this paragraph: 

I.80G259 

248 =*6401 
11 

7 =1 
40 

41=6 
.640116 



What to do when the leading figures change in the line 
and when a difference gives no reading is shown below: 

3.700364 

58 =5016 
6 =0 



.00501607 



60 
60=7, 



Determine and fill in the omitted entries in the table on 
page 238, showing all readings. 

* This symbol is a combination of a dash (from log-* meaning 
antilog) and an equality sign. It means and should be read ** whose 
antilog equals.'' 



Digitized by 



Google 



238 



TECHNICAL ALGEBRA 



254 



Table IX 
ANTILOGARITHMS 



No. 


Logarithm. 


Number. 


No. 


Logarithm. 


Number. 


1 
2 

3 
4 
5 
6 


2.706184 
2.706184 
1.580462 
3.418051 
3.280096 
4.121873 




7 

8 

9 

10 

11 

12 


2.900173 
1.607585 
3.526319 
i. 470008 
1.342004 
.087267 





§ 3. NATURAL OR HYPERBOLIC LOGARITHMS 

254. Systems of Logarithms. There are three kinds or 
systems of logarithms: 

1. Napierian logarithms, pubUshed 1614. 

2. Common or Briggs' logarithms, pubUshed 1617. 

3. Natural, hyperholic, or Speidelian logarithms, pubUshed 
1619. 

In elementary, practical computation by logarithms, com- 
mon or Briggs' logarithms are used almost exclusively and are 
the logarithms always meant when one speaks of logarithms. 

The last two systems have the foUowing bases: 

Briggs' or common, base 10. 

Nattiral or hyperbolic, base e = 2.718284. 

The logarithms of numbers in these systems are denoted 
as follows: 

Common, log 28.341. 

Natural, log. 28,341 or hyp * log 23.341. 

In the computations of higher mathematics where it 
is understood that the logarithms employed in formulas 
are to base e, a common logarithm is denoted by subscript. 

Thus logi3 a. 

255. Natural or HyperboUc Logarithms. Many en- 
gineering formulas have been derived by higher mathe- 
matics and therefore involve logarithms to base e. 

* '* hyp " means hyperbolic. 



Digitized by 



Google 



266 



LOGARITHMS 



239 



When such logarithms are required they may be deter- 
mined in two ways: 

(1) From a table of hyperbolic logarithms. 

(2) By multiplying the common logarithms by 2.3026, 
generally used as 2.3. 

From (2) it will be noted that the logarithm of any 
number to base e equals 2.3 times the logarithm of the number 
to base 10. 

Thus loge a = 2.3 logio a. 

Solve the equation for logio a. 

Therefore to reduce Briggs' logarithms to Natural, mul- 
tiply by what? 

To reduce Natural to Briggs', divide by what? 

266. Examples. Any number whose use as a multiplier 
converts one quantity into another quantity is called a 
coefficient, conversion factor, constant, or modulus. 

The computation of the omitted entries in the following 
table will give practice in the use of the conversion factor 
2.3 in transforming common or Briggs' logarithms into 
Natural or hyperboUc logarithms. 

Rule the table in the work-book and compute and fill 
in the omitted entries. 

Table X 
USE OF CONVERSION FACTOR 



Ex. 


Number. 


^°«io 


I^ogg 


1 


296.4 






2 


31.065 






3 


5.834 






4 


1796.3 






5 


85478 






6 


60.007 






7 


138.009 






8 


4.6342 






9 


71324.6 






10 


5009.61 







Digitized by 



Google 



240 



TECHNICAL ALGEBRA 



267 



257- How the Modulus is Obtained. Following is a 
proof that the Natural ^o-ir'thm of a number equals 
2.3 times its common logarithm. 

Given n any positive nimiber, 
a=logen, 

and l>=logion. 

Prove a = 2.30266, 

or log«n = 2.3026 logj^n. 

(1) a = log«n 
and 6 = logjQn 

(2) /. €f=n 
and 10* =n 

<=10* 

6=10« 



(3) 
(4) 
(5) 



log,n« =r 



Hyp 

Def log 

=ity Ax 
-Root Ax 
Def log 



(6) /. 

(7) But 

(^) •'• "^logj^2.71828 

(9) But log 2.71828 = .434294 

(10) /. 



a = i 

logio^ 

6 = 2.71828 

h 
a = ; 



a=- 



.434294 
(11) /. log* n = 2.3026 logj^n 



Mul Ax and Div Ax 

Notation 
Subs Ax 

Table 

= 2.30266 Subs Ax 

(1) and Subs Ax 



The number in the denominator of (10) is known in 
mathematics as the modulus of the common system of 
logarithms becaitse by its use Natural logarithms are reduced 
to common logarithms. 

268. Hyperbolic Logarithm of a Decimal. A simple 

way of determining the logarithm of a decimal to base e 



Digitized by 



Google 



2G9 LOGARITHMS 241 

by the use of the conversion factor 2.3026 or 2.3 is to 
multiply the mantissa and characteristic separately as shown 
below. Observe that the characteristic is written following 
the mantissa. 

Required the hjrp log .09685. 

.986100-2 
2.3 2.3 



2958300 
1972200 



2.2680300-4.6 
4.6 



3.668030 
hyp log .09685=3.6668030 

259, Ways of Denoting Ciphers. When several ciphers 
precede the first significant figure of a decimal they may be 
denoted by: 

(1) One cipher and a subscript whose unit value equals 

the nimiber of ciphers. 

(2) Indicated multiplication of the significant figures 

by 10 with a negative exponent having the same 
number of units as the number of decimal places. 

(3) Indicated multiplication of the significant figures 

as a decimal, by 10 with a negative exponent hav- 
ing the same number of imits as the number of 
ciphers immediately following the decimal point. 

Thus .000498 and .0000007345 may be written 

(1) .03498 and .067345, or 

(2) 498X10-6 and 7345X10'!^ or 
* (3) .498X10-3 and .7345X10-«. 

If a number ends in several ciphers it may be denoted 
by an indicated multiplication by 10 with an exponent 
whose unit value equals the number of ciphers. 

Thus 1832000000 is conveniently denoted by 1832 XIO^. 



Digitized by 



Google 



242 



TECHNICAL ALGEBRA 



260 



260. Examples. Compute the Natural logarithms of 
the following numbers, arranging the work as in paragraph 
256. 



1. .0864. 

4. .O3731. 

7. .O48O95I. 
10. .0,9812003. 
13. .0,720041. 
16. .00826054. 



2. .48064. 

6. .005614. 

8. .0062149. 
11. .817216. 
14. .631142. 
17. .049671. 



3. .32196. 

6. .073485. 

9. .0(50009. 
12. .0981467. 
16. .05480072. 
18. .0029346. 



§ 4. LOGARITHM OF A PRODUCT 

261. How Logarithms are Used. Logarithms are expo- 
nents. Their use is therefore governed by the laws of 
exponents in the following operations: 

Multiplication, 

Division, 

Involution, 

Evolution, 

Solution of an Exponential. 

. # 

262. Multiplication by Logarithms. Since in the 
Briggs' system all numbers are regarded as powers of 10, 
the multiplication of two or more factors is only the multi- 
plication of two or more 10s whose exponents are the 
logarithms of the respective factors. 

The exponents (logarithms) are therefore added. 

Thus 43.4 X 2.91 = 10^ ^37490 x 10463893 

101-637490 
10 -463893 
102.101383 

Therefore 43.4X2.91 = 102ioi383 



Digitized by 



Google 



262 LOGARITHMS 243 

What this product is, may be determined by reading 
the antilogarithm of 2.101383 from the table. 

Thus 2.101383 

059 sl262 
324 

310 =9 
140 
138=4 



The characteristic 2 denotes three integral figures. 

Therefore 43.4X2.91 = 126.294, which is exactly the 
same as .would be obtained by arithmetical multiplication. 

In practice, logarithms are never shown as powers of 10 
bttt are set down directly. 

Thus 43.4X2.91 = 126.294 

1.637490 
■463893 
2.101383 

059 =1262 

324 

310 =9 



140 
138=4 



One or More Factors Decimal. 

If one or more of the factors of a product are decimals 
and their logarithms therefore have negative characteristics, 
there are three ways of doing the work: 

(1) Set down the logarithms directly with their char- 

acteristics as determined. 

(2) Both add and denote the subtraction of 10, before 

adding the logarithms. 

(3) Set down the logarithms of the decimal, increased 

by 10. 



Digitized by 



Google 



244 TECHNICAL ALGEBRA 263 



us 


2454.1 X. 00568 


(1) 


18 




3.389875 




3.754348 




1.144241 


(2) . 


18 




3.389875 




7.754348-10 




11.144241-10 


(3) 


18 




3.389875 




7.754348 




*1 1.144241 




3951 =15 




291 




281 s9 




100 




94s3 




13.939* 



Whenever logarithms having negative characteristics ore 
to he added or subtracted it is suggested that they he set duun 
as shown in (3). 

263. Examples in Multiplication. Solve the following 
as indicated in the preceding paragraph: 

1. 10.09X687. 2. 381.56X16.9217. 

3. 7.298X1.654. 4. .5341X13.908. 

6. 181.96X31.148. 6. .00715X1.0083. 

7. 5.037X236.84. 8. 29.53 X. 42159. 

9. .0331287X12.64. 10. .0428961 X. 084507. 

11. .81965X .037964X 15.823X .073854. 

12. .067864X1384.19X.78113X144.58X.0834. 

* Two integral figures are pointed off in the result because the 
actual characteristic is 1, characteristic 11 being 10 too large. 



Digitized by 



Google 



265 LOGARITHMS 24J 



§6. LOGARITHM OF A QUOTIENT 

264. Division by Logarithms^ or the Logarithm of a 
Fraction. In the division of algebraic quantities the 
exponents of the quantities in the divisor are subtracted 
from the exponents of the same quantities in the dividend. 

Therefore, in division by logarithms the logarithm of the 
divisor is subtracted from the logarithm of the dividend. 

A fraction is an indicated division of the numerator 
by the denominator. 

Therefore the value of a fraction may he determined by 
subtracting the logarithm of the denominator from the logarithm 
of the numerator and by reading the antilogarithm of the 
remainder. 

When the logarithm of the dividend is smaller than the 
logarithm of the divisor it should be increased by one or 
more 10s, as may be necessary. 

Thus 

(1) 3.728^145.73=^^558 (2) ^J=^^ffii£ 

10.571476 460 

2.163549 89 7.754348 75 

8.407927 3.389893 18 

01_=2558 4.364455 ~ 
26 363 =2314 

17^=1 92 

90 75 =4 

84=5 170 

169=9 

265. Examples in Division. Solve the following examples 
by logarithms: 

1. 12.803^1.728. 2. 39.1857 -1-7.264. 

3. 3.7201-^14.96. 4. .082305 -^ 4.7812. 



Digitized by 



Google 



246 



TECHNICAL ALGEBRA 



266 



9. 
11. 
13. 
15. 
17. 



19. ;^ 



54.<sro7 
50.7002* 

.O3I2O8 
.047862* 

.970008 
1.20705' 

42.984 
.90358* 

.30876 
.00043581* 

.083451 
.0041486* 

11.6309 
.72184 ' 

45.8671 



21. 



23. 



27. 



29. 



31. 



33. 



318.0075* 

.014592 
.0056847* 
21.8429 
178.0096' 
1396.75 
23815.62' 

.063581 
.0043217* 
398.41 X 10 -» 

.54189 X10»" 
15.7847 X. 35685 
.048129X47.842' 

5173.96X10-'^ 
5300.48 X. 066814* 

.084056 X. 0054734 



6. 



10. 



12. 



14. 



16. 



18. 



20. 



22. 



21. 



26. 



28. 



30. 



32. 



34. 



1.9684 
8.17201* 

.00835 
.74802' 

.98305 
14.0091' 

1.72914 
16.3429' 

182.639 
73.6425' 

514.292 
3084.43* 

.3183 
.074129' 

.00096425 
.0472638 ' 

.0690723 
.O46I895' 

.44571 
.0059412" 
408.039 
3423.086' 

.03465125 
14962.9X10-*' 

.076485X10-' 
17.632 X. 340084* 
.017358X5.63419 
496.431 X. 0054866* 
.914184X30.639X84.69 
.071945X.18726X44.069* 



35. 17.28X 



.021968X1.70345 



Digitized by 



Google 



267 LOGARITHMS 247 



§ 6. LOGARITHM OF A POWER 

266. Involution by Logarithms. Involution is the 
process of finding any power of a number. 

The process and the reason for it should be evident 
from the following: 

(3.968)3 = (10-598572)3 = 62.476 

.598572 

3 

1.795716 

672 =6247 
44 

41 =6 
30 
28=4 

In practice the power of 10 is not shown. 

Thus ( . 09867)4 =.0494785 

2.994185 

4 

5.976740 

17=9478 

23 

23=5 

The characteristic of the product is 5 because 4X2=8: and 
8+3 which was carried, gives 5. 

267. Multiply Characteristic and Mantissa, Separately. 

When the exponent of the power of a decimal has more than 
one figure the computation is best made by separate multi- 
plication of mantissa and characteristic. 



Digitized by 



Google 



248 TECHNICAL ALGEBRA 268 

Thus .006348 g^ = . 037300 

.802637-3 
.65.65 



. 4013185 
4815822 



.52171405-1.95 
1.95 



5.571714 

09^ =3730 

50 
^=4 

In the two preceding illustrations the logarithm of the 
given number was multiplied by the exponent of the power 
* because to raise a number to a power is only ta raise 10 
with some exponent to the given power. In Chapter XIII is it 
shown that any power of a monomial is obtained by multi- 
plying its exponent by the exponent of the power. (See 
paragraph 190.) Therefore the following 

Rule for Logarithm of a Power. To obtain any 
power of a number, multiply its logarithm by the exponent of the 
power and read the antilogarithm. 

268. Examples in Powers. Determine the following 
powers by logarithms as shown in the illustrations of the 
two preceding paragraphs. The first four examples, as 
will be seen, are different powers of x. 



1. 3.1415926«. 


2. 3.1415926». 


3. 3.1415926*. 


4. 3.1415926*. 


6. .71834^ 


6. .05738^^ 


'■ a)* 


'■ (^)' 


9. 713.6042S. 


10. .0,74209'. 



Digitized by 



Google 



269 



LOGARITHMS 



249 



11. 



13. 



/ i.i84 y 

V,5631/ • 

)■• 



/7854 
V231 



^^ /.86211X42.19\» 
^^- V 2.296 ) • 

/ 1.2986X.124 Y 
V .70201 / ' 

19. 1942.683*. 

21. 37.6342X8.0097*. 

23. 4317.92^X72834 J. 

26. .63591 «^2.7314». 

27. (.49518»X6.70158*)». 

.5 4631 »X34.6 05» 
696.432X. 0009248* 

.82176 A » 



31 



/ 7.143^X.S 
\ .00782 



824* 7 • 
33. 8.0053942«-6.7084«. 



12. (^^'-)\ 
\. 012685/ 

14. (29.608X.60312)». 



16 



/ .08433XlO» y 
\ 3.406 / • 

^.' /27045X10-A* 
^^ \ 2.298 ; • 

20. 15761. 8*. 

22. .08379«X. 92046*. 

24. (2.7803»X. 84507*) «. 

26. (1.7046»X3.9.5167)*. 

17.964»X. 00952X1728 
34.762« 

/ 7rX9.345» y 
• \ 4.00732 / ' 

21.436^.04573' 
208.92* -^5.7814^• 

19.783X.24685»-1.9632» 
5609.41* 



30. 



32. 



34. 



§ 7. LOGARITHM OF A ROOT 

269. Evolution by Logarithms. In Chapter XIII it is 
shown that a root is only a power whose exponent is 
fractional. 

Write a rule for the determination of any root of a 
number by logarithms. 

How TO Extract the Root of a Decimal when 
Root Index is Positive and Evenly Divisible. When the 
root of a decimal is required and the root index is posi- 
tive and evenly divisible in the negative characteristic, the 



Digitized by 



Google 



250 TECHNICAL ALGEBRA 269 

division may be performed with the negative characteristic 
written in its usual place preceding the mantissa. 

Thus v^.0072861 3) 5.862495 489 

1.28749^ 6 

V .0350187 2 )4.700591 531 

2.350296 60 

What to do when the Root Index is Negative, or 
when not Evenly Divisible. When the root index is nega- 
tive, or not evenly divisible in a negative characteristic, 
there are several ways of determining the quotient as 
shown in subsequent paragraphs. 

The two best are the following: 

(1) When the root index is positive and not evenly divi- 
sible, both add to the logarithm of the given number, and 
indicate the subtraction from it, of the smallest integral 
multiple of the root index which will eliminate the negative 
characteristic. Divide the result by the root index. 

(2) When the root index is negative, even though it is 
evenly divisible in the characteristic, find the excess of 
the negative characteristic over the positive mantissa. 
Divide the result by the root index. 

(1) Positive Root Index, not evenly Divisible. 



V.0007154 = .042896 

4.854549 
4.6 



2.3 )1.454549 -4.6 
.632413 -2 
356=4289 
57 
61=6 

In the preceding illustration, the smallest integral multiple 
of 2.3 which will eliminate 4 is 2X2.3 =4.6. 



Digitized by 



Google 



270 



LOGARITHMS 



251 



4.6 was therefore added and its subtraction denoted, before 
the division was performed. 

The characteristic of the quotient, as shown, is 2. 

(2) Negative Root Index. 



"V.077584 = 236.247 

8.879898 
-3 ) -7.120102 * 
2.373367 

280 =2362 
87 
74 =4 



130 
129=7 

270. Examples in Roots. Compute the following roots 
by logarithms as shown in the illustrations of paragraph 269: 



1. V.002948. 
3. *V714629. 



6. ^•V.004157. 



2. V.052915. 
4. V545.I7. 
6. *V.061208. 



7. V3.1415926. 

9. "V.27806. 

11. V.004248. 

13. 'Vl294.63. 

16. V.O46O92I8. 

17. "V.012983. 
IS. "V29.856. 
21. V.017648. 
23. "*V;0()63057. 



8. V.7821. 

10. V2.38007. 

12. V.867502. 

14. V.096317. 

16. V12.OOO87. 
18 
20 

22. Vl604.965 

24. V.091872. 



V.71405. 
V.0678201. 



* —7.120102 is the excess of minus over plu9 in 3.879398 »nd is 
obtained by subtracting .879898 from 8. 



Digitized by 



Google 



252 



TECHNICAL ALGEBRA 



871 



26. 'V29.6294. 
28. Vl78.4219. 



26. *V341.738. 

27. "V.013968. 
29. "V.0085621. 
81. V.83176. 

33. V2.85009x3V.044216. 
36. V.0017298 X V21.0634 x Vl42.833. 



30. ^V4498.706. 

32. *V37.642L 

34. 175-2V.061287. 



§ 8. SOLUTION OF AN EXPONENTIAL EQUATION 

271. Definition and Illustration. An exponential equation 

is one in which the unknown quantity is the exponent. 

Thus 24.5*=?12.298 

Equations of this kind are solved as follows: 
(1) Base Greater than Unity. 



log 24.5* 
.'. X log 24.5 

X- 
X- 


= log 12.298 
= log 12.298 
log 12.298 
log 24.5 
1.089835 
1.389166 


552 
283 


.037028 


.142702 




318 


31 




119 


187 




199 


187 




.037360 


.142754 , 




.142754 






1.894606 






593=7845 






13 






lls2 

x=. 78452 







Digitized by 



Google 



271 LOGARITHMS 253 

(2) Base Decimal. Since negative numbers have no 
logarithms, the following illustrates what to do when both 
of the known quantities are decimal and when therefore the 
algebraic sum of characteristic and m/intissa is negative: 

.0543''=. 2347 

1.370513 "^ 



= 



o= 



2.734800 

- .629487 
-1.265200 

.629487 
1.265200 



(3) One Term of Fraction, Negative. When only 
one number is decimal, the operations are as follows: 

1.342» = . 08763 
2.942653 



6= 
6= 
6= 



.127753 

-1.057347 
.127753 

1.057347 
.127753 



As shown b is negative. The value of the fraction is 
therefore computed by logarithms and the antilog is written 
with a minus sign before it. 

In practice, work like that in the preceding illustrations 
should be set down as follows: 



(1) 24.5'= 12.298 




1.089835 


552 


1.389166 


283 


and so on as shown in (1). 





Digitized by 



Google 



254 TECHNICAL ALGEBRA 272 

(2) .0543° = . 2347 

1.370 513 

2.734800 

.629487 

1.26520U 

and so on as shown in (1). 

272. When the Exponent is Negative. In case the 
unknown exponent is negative , the operations are as follows: 

1.342 -«»= .08763 

_ 2^942653 

.127753 

1.057347 

.127753 

Observe that the last equation is obtained from the 
second equation by multiplying by —1. 

273. Examples. Solve the following equations: 

1. 19.87' =345.2. 2. .2467*' = 12.675. 

3. 3.7983^=449.31. 4. .00434' = .097504. 

6. .01432^^ = .18335. 6. .645001'' =27.6404. 

7. .5264-^ = 1.9234. 8. .01872 "'^ = .80441. 

9. .0734-' = 1.11054. 10. .708-^ =22.007. 

11. .00651-2' =3.7841. 12. 56.908' = .15925. 

13. .4084' =7.0382. 14. .07329'' = .23194. 

274. Summary of Laws with Formulas. There are jive 
laws for logarithmic computationj which may be formulated 
as follows: 

I. Logarithm of a Product, ab. 

log (ab) =log a+log b. 



Digitized by 



Google 



276 LOGARITHMS 255 

a 
II. Logarithm of a Fraction, -- . 

b 

a 
log r T^og a— log b. 
b 

III. Logarithm of]a Power, a^. 

loga^=bloga. 

IV. Logarithm of a Root, Va. 

ft/- log a 
log Va=— — . 
b 

V. Logarithm of an Exponential, a' =b. 

X log a =log b. 

If either b or a is decimal, 

logb 

log a' 

If both 6 and o are decimal, or greater than 1, 

logb 



x=: 



log a 



§ 9. MODEL SOLUTIONS 

275. Multiplication. (1) Factors Greater than 
Unity. 

346.84X93.72 = 32505.8 



2.540079 

50 
1.971832 
4.511961 

883 =3250 

78 

67 =5 

110 

107=8 



Digitized by 



Google 



256 



TECHNICAL ALGEBRA 



276 



(2) Factors Decimal. 

.034684 X .9372 = .032506 
50 
8.540079 
9.971832 



and so on as in (1). 

276. Division. 

346.84 



2.511961 



93.72 



=3.7008 



2.540129 
1.971832 



.034684 

.9372 
8.540129 
9.971832 



= .037008 



.568297 


2.568297 


02=3700 


02=3700 


95 


95 


94s8 


94=8 



277. Powers. In the determination of a power by 
logarithms, there are two cases: 

1. Power of a ntimber greater than unity. 

2. Power of a decimal. 

Model solutions for each of these cases follow: 

1. Power, Characteristic Positive or Zero. 
3.46842.3 = 17^4702 

.540129 
2.3 
1620387 
1080258 



1.2422967 

3 =1747 

47* 
50=2 

* Observe that figures in excess of the 6th are retained and used 
in the differences. 



Digitized by 



Google 



277 LOGARITHMS 257 

2. Power, Characteristic Negative. 
Following are illustrations of six different methods of 
determining the power of a decimal: 

(1) By logarithms. 

(2) By separate multiplication of mantissa and char- 
acteristic, and algebraic addition of the two products. 

(3) By separate multiplication as in the second method, 
but instead of algebraic addition of the products, sub- 
traction of the product of the characteristic from the product 
of the mantissa. 

(4) Same as the third method, except that the mantissa 
product is determined by logarithms instead of by arith- 
metical multiplication. 

(5) By multiplication after algebraic addition of char- 
acteristic and mantissa. 

(6) By multiplication of the logarithm increased by 10. 

First Method. Entirely by Logarithms. 

.900483-^1 = 0*. 

log a* = 6 log a = fc X 1.954473 

log a* =-*. 0455276. 

logo* =-(.0455276). 
log log a*= -(log .045527+log 3.51) 

67 

8.658202 

.545307 





9.203576 








TL- 


= 1598 




logo' 


= -.1598= 


= 1.840200 








169 s 


56921 






31 








323 


=5 


.900488S1 


= .69215 







♦Obtained from T.954473 by finding the excess of the negative 
characteristic over the positive mantissa. 



Digitized by 



Google 



258 TECHNICAL ALGEBRA 277 

As shown, — .1698 the logarithm of a*, is a negative decimal. 
Therefore it is not in tabular form, is not a mantissa, and 
its logarithm cannot be read in any table because all mantissas 
are positive. It was therefore reduced to tabular form by sub- 
traction from a number, one greater than its integral part, zero. 
It was therefore subtracted from 1, which gave .840200 
as follows: 

1. 
.1598 
.840200 

But this result is 1 too large; the subtraction of 1 is 
therefore denoted by the negative characteristic I, giving 
1.840200 as the tabular logarithm of a*. 

Every negative logarithm may be reduced to tabular 
form by the same process. Therefore the 

Rule for Reducing a Negative Logarithm to 
Tabular Form: 

Subtract the negative logarithm from a number one greater 
than its integral part and write for the negative characteristic 
of the result, the number from which subtraction was made. 

Thus - 3.285634 = 1.714366. 

In the preceding solution, the mathematics of the entire 
process has been shown. 

In practice the following arrangement is preferable: 

.90048351 = .69215 
1.954473X3.51 
-.045527X3.51 
-(.045527X3.51) = -.1598 = 1840200 

169=692.1 
67 31 

8.658202 32=5 

.545307 
9.203576 

77 s 1598 



Digitized by 



Google 



m LOGARITHMS 259 

Second Method. Separate multiplication of mantissa 
and characteristic, and the algebraic addition of the two 
products. 

.900483 51 = .69215 



.954473 -1 
3.51 3.51 
954473 
4772365 
2863419 

3.350200 -3.51 

3.350200 



• .159800 

1.840200 

169=6921 
31 
32s5 



Third Method. Same as the second except that instead 
of determining excess of negative over positive after multi- 
plication, the product of the characteristic is subtracted directiy 
from the product of the mantissa. 

.900483" =.69215 



.954473 -1 
3.51 3.51 
954473 
4772365 

2863419 

3.350200 -3.51 

3. 51 

1.840200 

169=6921 
31 
32=5 



Digitized by 



Google 



260 TECHNICAL ALGEBRA 277 

FouBTH Method. Same as the third except that the 
product of the mantissa is determined by logarithms. 

.900483 " = .69215 



3.51 X. 954473 -3.51 

9.979730 

32 

14 

.545307 

.525070 

45=3350 
25 

26=2 
3.350200-3.51 
3.51 
1.840200 

169=6921 
31 
32s5 

Fifth Method. Multiplication after algebraic addi- 
tion of characteristic and mantissa. 

.900483 51 = .69215 

1.954473 

-.045527 

3.51 

45527 

227635 

136581 

-.15979977 

' 1.840200 

169s6921 
31 ' 

32s5 



Digitized by 



Google 



278 LOGAEITHMS 261 

Sixth Method. Multiplication of the logarithm in- 
creased by 10. 

.900483" = .69215 



9.954473 
. 3.51 
9954473 
49772365 
29863419 

* 34.94020023 
35.1 



1.840200 

169=6921 
31 
32=5 

278. Roots. In the determination of a root by loga- 
rithms there are two cases: 

1. Root of a number greater than unity 

2. Root of a decimaL 

Following, are model solutions for each of these cases: 
1 Root, Charactbbistic Positive or Zero. 

''•V3;4684= 1.71727 



By Division. 


By Logarithms. 


2.3).540129(.234839 

46 770 =1717 




.540129 
9.732474 


80 


69 




16 


69 


51 =2 




73 


111 


180 ^ ' 




9.732497 


92 


177=7 J 




.361728 


192 






1.370769 


184 
89 




.234839 698 =2348 
770 =1717 71 


69 
200 




69 
51=2 


55=3 
160 






180 


166=9 






177=7 





♦Log .90048 was increased by 10 and then multiplied by 3.51. 
Therefore the starred number above, is 35.1 too large. 



Digitized by 



Google 



262 TECHNIC.VL ALGEBRA 278* 

2. Root, Characteristic Negative. 
How to operate when the root of a decimal is wanted 
depends on the root index which may be 

(0) Evenly divisible in the negative characteristic, 

(6) Not evenly divisible in the negative characteristic. 

(a) Root Index Evenly Containable. A root index 
evenly containable in the negative characteristic should be 
directly divided into the logarithm without transformation 
of any kind. 

Thus v^.007286 and V.08723 and v/.00006504 

3 )5.862498 ' 2 )2.940666 5) 5.813181 

1.287499 1.470333 1.162637 

(6) Root Index Not Evenly Containable. Following 
are illustrations of seven different methods of determining 
the root of a decimal when the root index is not .evenly divisible 
in the negative characteristic. 

(1) By algebraic addition of negative characteristic and 
positive mantissa, with division by logarithms. 

(2) Same as first except that the division is performed 
arithmetically. 

(3) By separate arithmetic division of mantissa and 
characteristic, with algebraic addition of the two quotients. 

(4) Same as the third except that the quotient of the 
characteristic is subtracted directly from the quotient of 
the mantissa. 

(5) By separate logarithmic division of mantissa and 
negative characteristic, with direct subtraction of the 
characteristic quotient from the mantissa quotient. 

(6) By adding to the logarithm of the given number, 
and by denoting the subtraction from it, of the smallest 
integral multiple of the root index which mil eliminate the 
negative characteristic^ followed by arithmetical division, 
and reduction as in the fifth method. 



Digitized by 



Google 



278 LOGARITHMS 263 

(7) Same as the sixth except that the division of the 
positive number is performed by logarithms. 

FiKST MsTHOD. Algebraic addition of the negative 
characteristic and the positive mantissa, with division by log- 
arithms. 

, »/- log a 1.954473 

1 .»/- - 045527 
log Vo = 



log Vo = ■ 



b 
045527 



b 
log log v^a = -(log .045527 -log 3.51) 

8.658269 202 

.545307 67 



8.112962 

40=1297 

22 

33sl 

log v^= - .012971 = 1.987029 

6996 =9705 
33 

31=7 

'•"V.90048 = . 970574 20 

— 18=4 

Tn the preceding, the mathematics of the entire process 
has been shown in order that it may be clearly miderstood. 



Digitized by 



Google 



264 TECHNICAL ALGEBRA 278 

In practice, the following arrangement is preferable: 
"V .90048= .970574 



1.954473 -.045527 



3.51 

.045527 
3.51 

8.658269 

.545307 

8.112962 



3.51 






= -.012971 = 


= 1.987029 






6996 1 


=9705 




33 






31 : 
20 


=7 


= 19.07 


18t 


s4 



22 
33=1 

Second Method. Same as the first except that the division 
is performed arithmetically. 

Show in full the application of this method to the example 
employed in the first method. 

Third Method. Separate arithmetical division of man- 
tissa and characteristic. 

3si\/.90048 = >?^a 

1 »/- logo 1.954473 

^-1 .954473 
3.51"^ 3.51 

^ .954473 1_ 

3.51 3.51 

Show in full the application of this method to the example, 
and reduce the result by algebraic addition of the two 
quotients. 



Digitized by 



Google 



278 LOGARITHMS 265 

Fourth Method. Same as the third except that the 
result is reduced by direct subtraction of the characteristic 
quotient from the mantissa quotient. 

Show in full the application of this method to the 
example. 

Fifth Method. Separate logarithmic division of mantissa 
and characteristic, with direct subtraction of the quotient of 
the characteristic from the quotient of the mantissa. 

Show in full the application of this method to the 
example. 

Sixth Method. Addition to the logarithm of the given 
number and by denoted subtraction from it, of the smallest 
integral multiple of the root index which will eliminate the 
negative characteristic, with arithmetical division by the 
root index and direct subtraction of the quotient of the char- 
acteristic from the quotient of the mantissa. 

3-5V.90048 = ^ 

, 6A- log a 1.954473 

1.954473 
3.51 
3.51 )3.464473-3.51 
.987029-1 

In the preceding/ 3.51 the root index was not evenly divisible 
in the negative characteristic T. 

Therefore, the negative characteristic was eliminated by add- 
ing to it, 3.51. 

This gave 3.464473. 

But this was 3.51 too large. 

Therefore, the subtraction of 3.51 was denoted as shown, and 
the quotient obtained by arithmetic division. 

Show in full the application of this method to the example. 



Digitized by 



Google 



260 TECHNICAL ALGEBRA 279 

Seventh Method. Same as the sixth except that the 
division of the positive number, is performed by logarithms. 

Show in full the application of this method to the example. 



§10. LOGARITHMIC COMPUTATION 

279. Miscellaneous Examples. This paragraph contains 
complex and difficult work, but by patience and study of 
the model solutions, even the student to whom mathematics 
is particularly difficult can solve any example in the list and 
will be able to make any computation from any formula 
of whatever kind. 

Many of the results will be found absurd from a practical 
standpoint but are intentionally so, in order that it may 
be definitely known that the position of the decimal point in 
an antilogarithm is determined by the characteristic and 
not by imagination of what the result ought to be. 

However much you may use tables, never be satisfied 
merely to copy a number. Always compare the copied 
number with the number in the table and be sure it is 
correct. 

Do not dash ahead and take one or more readings the 
moment an example is seen. Take no readings at all until 
the example has been studied to determine exactly what 
it is, and not only by what operations it may be solved 
but in what order the operations are best performed and 
what is the best method. 

In each of the examples from 1 to 70 inclusive, the 
first factor is a natural number and the second factor is a 
logarithm. This means that the first seventy examples 
in this paragraph are examples in powers. Why? Because 
any power of a number is determined by multiplying its 
logarithm by the exponent of the power, and reading the 
antilogarithm of the product. 



Digitized by 



Google 



279 LOGARITHMS 267 

Thus in example 1 on page 268, .72X2.786340, 
.72 is the exponent of the power, 

2.786340 is the logarithm of the number whose .72 power is 
wanted. 

Now log-^* 2.786340 =.061142. The operations, therefore, 
are exactly the same as though the example were given in the 
form 

.061142'^^ 

By the first method of paragraph 277 we have the following 
solution: 



2.786340X.72 = 








-1.213660X.72 = 
-a.213660X.72) = - 
.083861 
215 
. 215 
9.857332 


.873836 = 


: 1.126164 

31 = 1337 
33 = 1 


1.941430 








12 =8738 








18 








15 =3 








30=6 








.13371 


=.061142 


.72 





It is suggested that the results for the first seventy be 
expressed as shown, that is, as an equation specifying to 
what power of what number the final result is equal. 

Confusion may be avoided by not attempting all six 
methods of solution until at least twenty examples have 
been worked by the first method and in every instance 
checked by the second or third. Then at least ten should 

*Log~^ denotes antilog. See page 236. 



Digitized by 



Google 



268 



TECHNICAL ALGEBRA 



279 



be worked by all methods, after which, use the method 
which gives the least trouble and takes the least time. 



1. .72X2.786340. 

3. 2.19X3.717784. 

5. 1.72X1.423380. 

7. 3.17X2.463418. 

9. 3.9X4.617388. 
11. 4.08X5.980071. 
13. 2.86X2.670531. 
15. .83?X 3.405039. 
17. 2.16X5.208764. 
19. 3.076X1.181923. 
21. 4.581X5.390067. 
23. .735X1329671. 
25. 3.17X1.007314. 
27. 2.08X3.412953. 
29. .389X1324615. 
31. 1.78X2.814337. 
33. 1.98X1.422891. 
35. 4.9X2.909097. 
37. 3.42X2.739451. 
39. 2.76X5.238088 
41. 9.07X2.734012. 
43. 3.26X7.981264. 

45. 1.56X2.448892. 



47. 7^X2.352864. 
Id 

49. i^Xi.000465. 

51. 3.742X7.007869. 



2. 1.98XL301030. 

4. 3.98X5.396318. 

6. 5.32X6.223846. 
. 8. 7.89X3.327864. 
10. 3.7X1.385209. 
12. 7.31X8.290004. 
14. 9.87XL548119. 
16. 3.009X2.084361. 
18. 4.007X3.304812. 
20. 2.98X6.423057. 
22. 3.3X2.408568. 
24. 6.219X3.700638. 
26. 1.73X2.029975. 
28. 3.07X1.730451. 
30. 3.29X5.418927. 
32. 7.91X6.317235. 
34. 2.71X4.074698. 
36. 3.99X7.089674. 
38. 2.74X5.630425. 
40. .889X5.329416. 
42. 2.145X5.754112. 
44. .69X1.726345. 

46. ^X3.698113. 
4 

48. ltXS.147365. 

50. 13.76X2.000091. 
52. 1.69X2.396214. 



Digitized by 



Google 



279 



LOGAEITHMS 



269 



63. 4.097X^.720899. 
55. 2.48X4.739542. 
67. .3198 X. 009384. 
59. 2.999X0.300179. 
61. 5.008X2.940837. 
63. .691X1.700008. 

66. 2.471X7.900079. 

67. .1892X2.147826. 
69. .2567X0.842087. 

71. '^V.oeiu. 

73. ^•^V;00522i. 
75. ^•^V.000001674. 
77. ^'^ V.029068. 
79. ^V .00041437. 
81. ^•°V.000095515. 
83. ^•^V.04683. 
85. -^V .002541. 
87. ^■^Vxi00013572. 
89. ^•°^V.152028. 
91. ^•^^V.000024551. 
93. •^^V.21363r 
95. ^•"V.00010169. 
97. •^°V.00258794. 
99. •^^V.211102. 
101. ^•^V.065212. 
103. ^^V .000264784. 

106. ^•V.081113. 

107. ^'^V.054885. 



54. 1.782X2.047856. 
56. .786X1.007694. 
68. 2.^15X5.607284. 
60. 1.583X2.078149. 
62. 1.9004X4.931700. 
64. .238X1.000001. 
66. 2.128X5.300481. 
68. 1.723X5.009308. 
70. 1.2098X0.341723. 

72. ^•^^v:2. 

74. ^•®V.00002491. 

76. ^•^V.25508. 

78. ^-^V .0021747. 

80. ^V .24272. 

82. '•^V.000000019498. 

84. ^^V.35328. 

86. ^•^V.012144. 

88. ^•°°V.0020175, 

90. '^^V.00000264885. 

92. ^•V.025619. 

94. ^•^^V.00501925. 

96. ^•^V.0107146. 

98. ^"V .537590. 
100. ^•^V.000262378. 
102. ^^V.0000020764. 
104. ^•^V.001187. 
106. ^'®^V.000000122934. 
108. ^•^V.0000426997. 



Digitized by 



Google 



270 



TECHNICAL ALGEBRA 



279 



109. 


'•'"V.oooiTaoi. 


110. 
112. 
114. 
116. 

US. 

120. 
122. 
124. 
126. 
128. 
130. 
132. 
134. 
136. 

138. 
140. 
142. 
144. 
146. 
148. 
160. 

162. 
164. 
166. 
168. 
160. 


■""V.00002135. 


111. 


'•"^.0542016. 


""'V.00056171. 


113. 


''•*°V.0000009578. 


■**V.53253. 


116. 


'•*"V.028112. 

3 


■'*V.0049901. 


117. 


wV.022535. 

.408 


'■V.001404. 

13.76 


119. 


1W8V.000108107. 


s«'V.0100021. 


121. 


'*"^V.0872632. 


^"^V.000854476. 


123. 


V.501197. 


V.100021. 


126. 


^•'"V.00000079448. 


""'V.000019975. 


127. 


•'"'"V.014055. 


' '^''V'.OOP0102166. 


129. 


•'^V6.95165. 


i.^««V2.19646. 


131. 


'•"V.ouios. 


•''"V398.726. 


133. 


■'"'V.3002. 


•*'Vl.007. 


136. 


'•"^56.7859. 


'"•'^V.087264. 

1 


137. 


••"^Vl. 34009. 


'V'.87744. 


139. 


•"™V.06349. 


'■'"V.0093008. 


141. 


"''V5.30477. 


•""V.9721. 


143. 


■""V.07263X1.3467«. 


•*"V.02345. 


146. 


'•"V.000725. 


'■''V.6345. 


147. 


'■"V.38961. 


•''V.8397. 


149. 
161. 


'•"Vl.2963. 

.301 

1.936^/ .0968. 


'•'*Vl.307. 
"V.425. 


163. 


1728 X "V4.965. 


•*'*'V.098006. 


166. 


•■'"Vl.0007. 


'^''V56.7859. 


167. 


•'*'V398.726. 


■*^V.0017238. 


169. 


'•"^Vl. 34009. 


'•''V.3417X. 04128. 



Digitized by 



Google 



279 



LOGARITHMS 



271 



161. 
163. 
166. 
167. 

169. 

171. 
173. 

174. 
175. 

176. 
177. 

178. 

179. 

180. 
181. 
182. 
183. 
184. 

185. 
186. 



^•*V.002873X 1.968. 

•'''V.063492^ 

•^^V.00039253-28. 

'®'''^V3.7346^»X.072. 

2.760582 
.52632-^* ' 

8" V92. 163 X. 009459. 
(.008726X19.8005) 10-'- 

3.008009 
.00768* X.973i* 
2 .723081 



162. 



1' 



0004215 



082428 



164. ^•^V.0093008-®®^ 
166. 2:71 \/l.30843X .0009672. 

168.^ 

170. 

1. 923'' X. 8452 

1 
172. (32.0807 X.08267)-396. 



9.400009 
-4.761 X. 9983" 
12.728406 



®^V;3482l' 



.573 

1-208 V.00076008 + .8546 X 1.985. 

i.600085 



•^V.092807X.4062i' 
5.726008 



/ -. 07341 X log .401 

\ 84.5 



1.9672'*ixV9.126 

.00765., ..56,+i^g4oi 

396.72 X'^V3.127X.493X1.7842. 

27.98 X "^^ V.0972 X .385^ X .0446^ 

^Vl348X "•^V.0726X.309-2. 

.7204^X "•^^Vl.0732X.08293X6.19-*X10-8. 



3.018724 X V7.218+.1728'. 
1.073961 -329.8^-^^ 

^ V;0726 
^•^ V.018292X .1928*X 10*. 



Digitized by 



Google 



272 TECHNICAL ALGEBEA 279 

187. 17.921^ " -34.96^ ""^xVs.lSOe. 

3 

188. 2V799l2xi.980072"<>»+.006834-2, 



1.78 



189. I7.634XV 1900.0082 X 10"*. 

1 

190. 7 76 V.084176J - 728.1009X 10 -5. 

.007568 



191. 



193. 
194. 



.0075^ X 1000.967^"^^ X'V90.872»* 

.000842007 

..K)9615 \/72.389^ 
2.006346 +^-°V.07384» 



1.19253X10-3 

.08273 X. 09638 ^X. 01728-' 
1705.61^^^ 



3Vl6.0892+*^V6.2401 
19u. 



197. 



10-'^xV.01829 

^7 8X10^+6.2172 
1.7348-3X159.2008-^-ixV.0841 



196. .00827 8X105+6.21731-729. 



V.08293^-^^ 



3 2 

198. 50.096"8 4-.5729"'^+V3.9638X10-2. 



7 



jgg 3.80193 X\ . 9731 5X3. 9204 « 

V3.8172^+. 17292-7 



>««• -Sx-«^2^<^^"- 



201 
202 



4 
4^ 



/.083622X^"V3.4'l69 
72.684 -.08126^-^^ * 
2.007345 



V.028912X.7236* 
203. log 83.21 93 X. 6873 '7 x^"V-.8o2P log .728. 



Digitized by 



Google 



279 LOGARITHMS 273. 



J3.1416«+.1892 8 



3^ 

8 

1.436 -*X^'V.T9307 



204. \ y\_ 2.8 .- ==+log 7.9281 ^ 



206. .895-*Xl.586-3x7.64^*X --3^^.008226. 

206. •*®^V54.43 log 64.15*. 

207. .0l728-2-**8X.0361-2 39. 

208. ^^V(.2481t log 39.41)+362.58 log 572. 
00496»X 14.723* 



209. 
210. 



.0896* X. 3516-** 

4563^^^^ 
•^Vj3r9Xl8.5-i' 



mV7431 X V.6245XV34.963 
185.96 hyp lo£ .3461 

212. .0329"*^^V.682""^^X'*V6.129X.000317-*. 

213. I-X.98 log .31642 -.0892-*. 

o 

214. 66X*^V.0638X48.1*. 

215. .00539 X 30.08 --^2^ 

216. 1.04^ loge 16.178X10-^ loge.0923-3-2^ 

^_ .4398-^^^X.045843-2«+42.97 
128.1 hyp log .3973 

218. " •^V.0008429X '"^^^VrndEQ X hyp log .4086 - «. 

219. .0173"3"X25.09"'^^^X'^^V.00928^-'^V.09^. 

220. .7583"*X.0046r^X7.69 hyp log •^V;217\ 

221. V.6018'iiX.0046l"^X7.69hyplog 'WaU. 

222. '^^V3184.7-2X.897PX.09426-8-V;0142+Vj. 



Digitized by 



Google 



274 
823. 

224. 
226. 

226. 
227. 



TECHNICAL ALGEBRA 






1.41 r 



0263 -«X 



.08175 
.075 



L962X7^'.01298 
396.34-^X4.0189 ? +.3118-*X7.914--^. 



-3.18 



v-i 



.0268^X V-.74123 log .41722. 

1.009 / f 



V .00845 3 log 3.1 V -.4993xhyp log .0892-^^. 
7.3219 



3.17 log .07893 

2.93 



1-. 



^ 6.4208 2.09 16 
7196 .0823-2 018* 
3.961 -•3»2_^.0458-»29 iQg Qg9i2 



- 1.38 



V.012963+ .01723 " -^^^ X .4779 " ^' 



279 



MODEL SOLUTIONS 



174. 



3.008009 



- 2.991991 



2.991991 



.00768* X .973* " .00768* X .973* " .00768* X .973* 

f^ log .00768 , log .973 
3 



= -34.454155 



io,2.mm-(^^^+'^f-^) 



.475816 
131 

13 

20.475960 

18.938719 

1.537241 

189 = 3445 
52 

50=4 
20 

13 = 1 
70 

63=5 
70 



3.885361 

j4 

2)1.885361 -4 
.942681 -2 



1.988113 
3. 



3 )2.9881 13-3 
.996038-1 



8.942681 

9.996038 

18.938719 



Digitized by 



Google 



279 
177. 



1.600085 



LOGARITHMS 

.399915 



-'V.092807X.4062* "^ V.092807 X .4062* 

logc\ 



275 

-16.48253 



flog6+- 



loga-l 



.72 



8.967548 
33 

8.967581 

9.869580 
18.837161 

2.837161 1.162839 



.72 

.065206 
299 
112 
336 

.065520 
9.857332 

.208188 

73 = 1615 
15 = 
150 
134=5 

160 
161=6 



.72 



1.608740 

J 

3)2.608740-3 

9.869580 



= -1.615056=2.384944. 



9.601951 
11 

55 

9.601968 

8.384944 

1.217024 

957 = 1648 
67 
53 = 2 

140 

132 = 5 
80 
79=3 



r_ . 07341 X log .401 ^, j-aXi-h) . I ab 
179. .00765=X^ -84.56^+l^r=N-?+(^=Nc-^ 



Digitized by 



Google 



276 TECHNICAL ALGEBRA 279 

„n« 2.007345 L992655 oq raqoh 

202. » , = - o / = -23.563211 

V.0289PX.7236 V .02891 ^X. 7236 - 

WlQQ9fi^^ 2 log .02891 +log .7236 

.299289 ^^^ ^-^^^^^^ 3 

131 2.461048 

109 2 

. 109 4.922096 



_.299432 1.859499 

2.927198 4.781595 

1.372234 J 

175=2356 3)2.781595-6 

59 2.927198 

55 = 3 

40 

37 = 2 
30 

18 = 1 
. 20 

206. •^^^V54.43 log 64.15^ = 1465.9 

4)1.807197 

.451799 

3 



1.355397 
•^^''''V54.43X 1.355397 
1.735838 .271377 

.132064 46 

96 209 

288 10.271384 
224 9.770859 



1.868029 .500525 

.59 n = 3166 

14 

3.166100 

5838 =1465. 
262 
266 =9 



Digitized by 



Google 



279 



LOGARITHMS 




'.2481* log 39.41+362.58 log 572 




.394627-1 






3 






8)1.183881-1 


1.595606 


.147985 




.202761 


.375 




163 


9.772985 


163 


.202926 




.202926 


9.975911 






891=9460 






20 




2.757396 


18=4 




.440437 


20 




47 
140 


999.777 




94 


.9460 




.440499 


1000.723 




2.559308 
96 


3.000000 




2.999903 


304 




870=9997 


87 




33 


130 




30=7 


3.000314 




30=7 


129 






.477121 






43 






14 


.023258 


58 




_2=1055 


477166 






2.110590 






2.366576 






423 = 2325 






153 






149=8 








= 1.055 




^^V 1000.723 = 





277 



Digitized by 



Google 



278 



TECHNICAL ALGEBRA 



279 



226. ' **V.00845-»Xlog ^' V -.499^ xhyp log .0892-^^. 



-3 ) 3.926857 
1.-308952 

1.009).691048(.684884 
6054 

8564 
8072 

4928 

4036 



8920 
8072 

8480 
8072 



.950365-2 

.17 .17 
6652555 
950365 -.34 

.16156205 
.34 

1.821562 

-.178438 

2^ 

535314 
356876 
-.4104074 



4080 
4036 



3.18 



V -.499»X( -.4104074) ='*^V.499»X .4104074. 

'•^V .00 845 ^X( -.40W408) = 
-C'^V.00845-iX.40&4408) 



1.613207 
74 
4 



1.613215 



1.698101 

3 

1.094303 
1.613215 
2.707518 

3.18) -1.292482( -.4064408 
1272 

2048 
1908^ 
1402 
1 272 
1300 
1272_ 

2800 



-1.96735 



1.608954 
43 

^^ 9 

1.60S99S 
.6S48j^ 

.293SS2 
04= 
7S 

66=3 
120 
110=5 



1967 



Digitized by 



Google 



CHAPTER XVI 

THE SLIDE-RULE 

Section 1, Introduction. Section 2, Sines. Section 3, 
Tangents. Section 4, Multiplication. Section 5, Divi- 
sion. Section 6, Proportion. Section 7, Logarithms. 
Section 8, Powers and Roots. Section 9, Gage-Points. 
Section 10, The Log Log Rule. 

§ 1. INTRODUCTION 

280. Historical. Within less than fifty years after 
Napier's invention of logarithms in 1614, a slide-rule of 
two fixed strips held together by brass plates at the ends 
with a sliding strip between, the first duplex, had been 
manufactured and sold in England. 

Six years after Napier's invention John Gunter made 
the first logarithmic scale, known ever since as Gunter's 
scale. In addition to trigonometric scales this had a scale 
of numbers from 1 to 10, the divisions of the scale being 
proportional to the logarithms of these numbers exactly 
as in the modern slide-rule. Instead, however, of having 
some scales on a movable piece so that computations 
could be performed by sliding one scale along another, 
all the scales were on the same piece and computations 
were made by means of a pair of compasses. 

In the summer of 1630* while visiting his teacher WiUiam 
Oughtred, an English clergyman and mathematician, a pupil 
spoke to him regarding Gunter's scale. Oughtred replied 

* See Cajori's " History of the Slide-rule." 

279 



Digitized by 



Google 



280 TECHNICAL ALGEBRA 281 

that Gunter's scale was a " poor invention and cumber- 
some in performance; '* and thereupon showed some mechani- 
cal devices of his own, including a circular and a straight 
slide-rule; the former so constructed that one disk rotated 
within another with two pointers from the center, and the 
latter having one scale on a piece which slid along another 
scale. The pupil, Foster, in his account of this incident 
says that Oughtred had invented these devices about 
thirty years before, but as this would antedate the invention 
of logarithms by fourteen years the statement is undoubtedly 
incorrect. The probabilities are that William Oughtred 
invented the slide-rule shortly after Gunter's scale first 
appeared in 1620. 

281. What the Slide-Rule is. A slide-rule is a mechani- 
cal device for performing logarithmic computations, by 
sliding (if straight), or rotating (if circular), the logarithm 
of one number along the logarithm of another so that their 
sum or difference may be determined. 

It consists of a fixed part having logarithmic scales, 
called the stock, a movable piece called the slide having 
logarithmic scales, and a movable m^tal piece called the 
runner having a hair-line for vertical readings of the scales. 

282. Arrangement of Scales. The simplest arrange- 
ment of scales, of which all others are modifications, is 
that of the Mannheim rule which was designed about 1850 
by Lieutenant Mannheim of the French army. The face 
of the Mannheim rule has four logarithmic scales, two on 
the stock and two on the slide. The two upper scales on 
stock and slide, usually designated as the A and B scales, 
are exactly alike and are used twice in the length of 
the rule. 

The lower scales on stock and slide, usually designated 
as the C and D scales, are exactly alike, with the correspond- 
ing divisions double the length of the divisions of the two 
upper scales. The readings on the two upper scales are 



Digitized by 



Google 



THE SLIDE-RULE 



281 




=3 



03 

d 

O 



1 

I 






Digitized by 



Google 



282 



TECHNICAL ALGEBRA 



therefore the squares of the aligned readings on the two 
lower scales when the indexes are in alignment. 

On the reverse side of the slide are sine and tangent 
scales and a scale of logarithms, which can be read by the 
index marks on the end notches or transparent plate at the 
back of the rule, or by inserting the slide turned over, and 
using the runner. 



fmLt. \ ' i wwyyTOppwywyi 




l^iMMn'|nH'i'i';'jjj'i^i'f|||n'i|imfi;iw^ 



iif! iiiilniilnnliMilim 



lltnlltilTllllljHIilllillMI ItlllilCt u\\\\ 



^^ 



53 



Fig. 93. — Section of Linear scale on beveled edge of Mannheim rule. 



On the beveled edge of the Mannheim rule is an English 
scale, and on the opposite edge a metric scale which, in some 
rules, is continued along the channel of the rule. 



1 • 




5 € 7 , « , ? 

'-.I'h'N-'imnMHimiilimiiiiih 



Fig. 93A. — Section of Metric scale on lower edge of Mannheim rule. 

Some of the modifications of this arrangement of scales 
are as follows: 

(1) An inverted scale in place of the direct B scale. 

(2) The sine scale instead of being on the back of the 
slide is placed in the middle of the face of the slide, between 
scales B and C. In place of the sine scale on the back of 
the slide » is an inverted B or C scale. 



Digitized by 



Google 



288 



THE SLIDE-RULE 



283 



(3) Instead of the metric scale on the edge of the rule 
a scale of the cubes of readings on the D scale, the hair-line 
being carried down on a tongue on the runner, as in the 
POLYPHASE and the multiplex. 

(4) The stock widened to cany the cube scale below 
the D scale, A and C scales on the upper part of the stock, 
and on the slide two C scales with inverted C or CI between, 
as in the triplex. 




nhHilmmnihlii 



LnlHi^^ 




Fig. 94. — Section of an Inverted B scale. 



(5) A two-faced rule with A, B, sine, C and D scales on 
one face; and A, BI, tangent, C7, and D scales on the reverse, 
with an L scale (scale of logarithms) on the lower edge or 
lower stock, as in the duplex. 

(6) Stock widened to carry in addition to the scales of 
(5) on the reverse, a continuous log log scale mounted in 
three sections on the upper stock, giving direct readings 
for exponentials, logarithms to base e or any other base, 
and any power or root, as in the log log duplex. 

This rule has a C scale in place of the BI scale, and the 
widened lower stock has the L scale on its face below the 
D scale. 

283. Graduation of the A and B Scales. Examine the 
upper or A scale on your rule. Observe the graduation 



Digitized by 



Google 



284 



TECHNICAL ALGEBRA 



283 



fc^ 


;,:,,.-:. ^ . ^.^:;r 


- 




'- 


c:; 5 


^■ 


, -0 


~ 


;■::,.:: 3 


- 


„.:rt:i-^ 


1 


t^ " '■■*'"■ 


r 




-^ 


,:,::s a-::i:;5-s 








v-rr-— -rr ^, ™. — 


T 


..,■;-- rs — ■ ^ 


i- 







i 


r 


^' ; d 


^. 


■-:^. c^ :? 


„ 


< 


r 


h 


■r 


- . - : ■ < 


r* 


■"■■ ■ - ■;. ; - a 


^ 




■^ 


■^-§— -!2 ci.;, : ; m 




— —^■.. r:4:r 


r 










- ■'.,■■■' 


ri'.. -i- i' 


■" 




♦ 




r^ ■ '^■". 


^> J^."* -^ K' 


i. ; 


' 


r. 












"^ 




- 


- 


r 


^ ' '■ 


ii^ 






a> 










.: 


I. a 


- 




» 


^■. 


*-•< 






« 






. 


?: '" '''* . " 






r> 




K 




« 








» 


^ , '' 


" 


, -:fe....o. — -■ 


r 


3! J • :r: r 


- 


^ ; --^ ^ 


t 


-a( a « 


\.._ 





o 
a 

.a 

13 



5 
1 

.a 

""I 
II 






•I 



T 







a 



:r — 1 ^::_ 


Hxr • :^ 


— ^a,__:-t^- "~ 


^..^J^-i 


^i M- 


i 




I'V-'- 


Z^:-- ' i£i 




g- [^" 


s l-j 



^ 



Digitized by VjOOQ IC 



283 



THE SLIDE-KULE 



285 




r^ 






""^^ — J^- « O 



a= 



J 






a 

Q 



^ K « O Q 







:=— 3^ 



< 



M H *-« fi 



§ 



I 
I 



§ 



T 

00 






r 5] 



Digitized by 



Google 



286 



TECHNICAL ALGEBRA 



288 




c3 
Xi 



"3 



C 



o 









Digitized by 



Google 



THE SLIDE-RULE 



287 



marked 1 at the beginning, center, and the end. These 
marks are called indexes and are known as the left, center, 
and right A indexes. 

Fig. 101.— il-scale from 1 to 2. 

The distance between the graduations nmnbered 1 and 2 
is divided into how many large graduations? 

Each of these large graduations is divided into how 
many parts? 

If the left index is taken as 1, the first large graduation 
will denote what? 

Each small graduation therefore denotes how many 
hundredths? 

Set down the reading for each graduation from 1 to 2 
inclusive. 

If the left index is taken as 10, each of the smallest 
graduations therefore denotes how many tenths? 

Set down the reading for each graduation from 10 to 11 
inclusive. 

If the left index is taken as 100, the first graduation 
will be read what instead of 11? (If unable to answer, 
notice what it must be in order that the graduation numbered 
2 or 20 may be read 200.) 

Write the reading for each graduation from 100 to 200 
inclusive. 



'i' 



Fig. 102. — ^ A-scale. from 2 to 5. 



Examine the A scale from 2 to 5. 



Digitized by 



Google 



288 TEUiUNlUALi AAJJhllStiA 284 

Is the number of the large graduations the same as from 
lto2? 

Are the small graduations the same? 

Write the reading for each graduation from 2 to 3 
inclusive; from 3 to 4; from 4 to 5; from 20 to 30 inclusive; 
from 400 to 500 inclusive. 



Miii|iiimiii|iiimiii|iiiiiii 



10 
Fig. 103. — ^A-scale from 5 to middle index. 

Examine the A scale from 5 to the end. 

Is the number of the large graduations the same as in 
other parts of the scale? 

Are the small graduations the same? 

Write the reading for each graduation from 7 to 8 
inclusive; from 70 to 80 inclusive; from 800 to 900 inclusive. 



6 7 C 9 1 3 

Fig. 104. — B scale showing left and middle indexes. 

Examine the B scale. This is the second scale on the 
rule, being the upper scale on the slide. 

Is it the same as the A scale? How is it read? 

284. Graduation of the C and D Scales. Examine the 
D scale. 



Fig. 105. — Section of D scale. 



Digitized by 



Google 



286 THE SLIDE-RULE 289 

This is the fourth scale on the rule, being the scale on 
the lower stock. 

By reference to the U. S. linear scale on the edge of 
the rule state the relation of the distance from 1 to 2 on the 
D scale, to the distance from 1 to 2 on the A scale. 

What is the relation of the number of graduations on 
the D scale from 1 to 2, to the number of graduations on 
the A scale from 1 to 2? 

Write the reading for each graduation on the D scale 
between the following limits inclusive: 



(1) 


lto2, 


10 to 20, 


100 to 200. 


(2) 


3 to 4, 


30 to 40, 


200 to 300. 


(3) 


5 to 6, 


50 to 60, 


500 to 600. 


(4) 


7 to 8, 


70 to 80, 


600 to 700. 



Examine the C scale. This is the third scale on the 
rule, being the lower scale on the slide. 



Fig. 106. — Section of C scale. 
Is it exactly the same as the D scale? How is it read? 

§ 2. SINES 

286. Graduation of the S Scale. The S or sine scale 
on most rules is the lower scale on the reverse of the slide 
when the rule is held in position for reading the face scales. 
When the rule is turned over without reversing ends the S 
scale therefore is the upper scale, the lower scale being a 
tangent scale, with a scale of logarithms between the two. 

Draw the slide to the left and examine the left end of the 
S scale. Looking back from the first numbered graduation 
which is 40', observe that the scale begins at slightly less 
than 35', or at 34' 18". 



Digitized by 



Google 



290 TECHNICAL ALGEBRA 286 

Write the readings for each graduation from 35' to 2°, 
disregarding the irregularly placed graduations which 
may immediately follow the 1° 10' and precede the 2** 
graduations; these are gage-points as explained in para- 
graph 290. 

r; rv/ ». *«•;■ :.!• • » . afe vl -i 

pii|ll!||iU!i.l!i..^ ^ jiliHpp.,^™- 

Fig. 107. — Back of slide, showing the first 10® of the sine scale. 



What is the graduation imit from 35' to 10®? From 
10° to 20**? From 20° to 40°? From 40° to 70°? From 
70° to 80°? From 80° to 90°? 

Observ-e that the S scale is a scale of angles and not of sines. 

286. How Sines are Read with Unreversed Slide. On 

all rules sines are read on the B or the A scale. When the 
back of the rule has a notch at the end, sines may be read 
by moving the slide so as to bring the given angle on the S 
scale directly under the index mark on the edge of the 
notch in the right end of the rule. 

The rule is then turned over, and the numerical value of 
the sine of the angle is read on B, directly under the right 
A index, 

287. Rule for Pointing off a Sine Reading. (1) Place 
.0 before all sines which are read on the first B scale. 

(2) Place decimal point only, before all sines which are 
read on the second B scale. 

Thus sin 24° 15' is read as follows: 

(1) Set 24° 15' under the right notch index. 

(2) Turn the rule over. 

(3) Move runner to right A index, and read .411 on B. 



Digitized by 



Google 



288 



THE SLIDE-RULE 



291 




! : 1 ; 1 i ' i i, ' i I! t MiinMI i I [ i ' t ii ilMm!ini!!Hl!i!l!i!i!ir 



:inillnlilliiliilllii(JHiil!lliiiiM]il|jili|]!fi!ii!j|j||{|^ -'^ jj 



Fig. 108. — Back of Mannheim rule, showing 24° 15' on S, under S 
index on edge of notch. 

A decimal point is placed before the reading because 
it was read on the right or second B scale. 
Verify this reading on your rule. 




ii ill fmi 



IjlTTTfMjMp— 
B 9 _1 



Fig. 109.— Reverse of Fig. 108, showing .411= sin 24° 15' under right 

A index. 

If your rule has an index mark on the edge of the notch 
at the left end of the rule, set 24° 15' on S under the left 
index mark and take the reading on B under the left A index. 

Is the reading the same as before? 

288. Examples in Sines. Enter the following examples 
in the work-book with the readings for each. 

1. sin 39° 10'. 2. sin 9° 12'. 3. sin 40° 25'. 

4. sin 5° 20'. 5. sin 5° 55'. 6. sin 3° 15'. 



Digitized by 



Google 



292 



TECHNICAL ALGEBRA 



289 



7. sin 28° 30'. 
10. sin 60° 35'. 
13. sin 21° 24'. 
16. sin 70° 31'. 
19. sin 28° 40'. 



8. sin 4° 25'. 
11. sin 75°. 
14. sin 8° 25'. 
17. sin 45° 30'. 
20. sin 64° 35'. 



9. sin 19° 35'. 
12. sin 4° 28'. 
16. sin 32°. 
18. sin 55° 25'. 
21. sin 80° 45'. 




i|i|^nrtiiui|un|iHijnii|ii^ii 



Fig. 110.— Reading of sin 24** 15' on LL duplex. 
24® 15' is under runner on 5; .411 is under runner on A and B. 

289. Sines with Reversed Slide. Remove the slide and 
insert it with the reverse side up and with the indexes of 
all scales in alignment. 

Move the runner so that the hair-line exactly covers 
24° 15' on the S scale, and read the sine on A under the 
hair-line. 

Point ofif exactly the same as when sines are read on B, 
Is the reading the same as in paragraph 288? 

Read all the sines of the preceding paragraph with the 
slide reversed. 

Are the readings exactly the same as before? Why? 

Which seems the simpler way to read sines, with the 
slide unreversed or reversed? 

290. Gage-Points for Sines and Tangents. If the S 
scale is examined on some makes of rule theie will be found: 

(1) A gage-point denoting seconds and therefore marked 
with the seconds symbol, immediately following the 1° 10' 
graduation. 

(2) A gage-point denoting minutes and therefore marked 



Digitized by 



Google 



891 



THE SLIDE-RULE 



293 



with the minutes symbol, immediately preceding the 2° 
graduation. 



p|}rfpiifif!fi!|i}«ipiip!|r : '■\m 


1 


! •lllillillliiili: ^ ^'!;ll 


liiite^^^ 1 



Fig. 111. — Section of LL duplex showing seconds and minute gage- 
points on S scale. 

These gage-points on the S scale are used in reading 
both sines and tangents of angles less than 34' 18". 

291. Rules for Gage-Point Readings: (1) Angles less 

THAN 1'. 

Move the slide until the seconds gage-point is in align- 
ment with the given number of seconds on the A scale. 

With the runner read sine or tangent as required, on 
A directly over the S index. 

(2) Angles from 1' to 34' 17" inclusive. Move the 
slide imtil the minutes gage-point is in alignment with the 
given number of minutes on the A scale. 

With the runner read sine or tangeiit as required on A 
directly over the S index. 

The following tabulation shows how many ciphers 
precede readings for sines and tangents within the inclusive 
limits: 

Table XI 

POINTING OF SINES AND TANGENTS 



Angles from 


Ciphers 
Preceding 


Angles from 


Ciphers 
Preceding 


sin 


tan 


sin 


tan 


l"to 2" 
3" to 20" 


5 
4 


5 
4 


21" to 3' 26" 
3' 27" to 34' 17" 


3 
2 


3 
2 



Digitized by 



Google 



294 TECHNICAL ALGEBRA 292 

292. Examples. Read the following functions on the 
slide-rule by the use of the gage-points: 



1. 


sin 42". 


2. tan 28". 


3. smSO'. 


4. 


t^,n 6'. 


5. sin 16'. 


6. tan 41". 


7. 


sini^V. 


8. tan 4' 15". 


9. sin 34' 27". 


10. 


sin 45". ^ 


11. tan 18' 30". 
§ 3. TANGENTS 


12. tan 11' 12". 



293. How Tangents are Read. How tangents are read 
depends on whether the angle is 

Less than 34.3', 

Between 34.3' and 5° 42' 37" inclusive, 
Between 5° 42' 38" and 45° inclusive, 
Greater than 45° and less than 90°, 
Greater than 90 and less than 180. 

(1) Angles Less Than 34' 18". Use seconds or min- 
utes gage-points on the S scale, as explained in the pre- 
ceding paragraph. 

(2) Angles Between 34.3' and 5° 42' 37" Inclusive. 
Use S or sine scale, the readings being the same as those 
for sines within the same limits, as may be verified by 
reference to a table of natural functions. 

(3) Angles Between 5° 42' 38" and 45° Inclusive. 
Use T or Tangent scale as follows: 

Mannheim, Slide Unreversed. Turn rule over and set 
given angle on T under left notch index. On face of rule 
read tangent on C over D index. 

Mannheim, Slide Reversed; also Duplex. Align indexes. 
Set runner to given angle on T; read tangent on D under 
runner. 



Digitized by 



Google 



293 THE SLIDE-RULE 295 

Pointing Off. Place decimal point before all tangent 
readings on D for angles from 5° 42' 38" to 45°. 
Observe that tan 45° = L 



^n^^TTTTTT^^ 



Mijto^^ 



Fig. 112. — Section of T scale on back of slide in Mannheim rule. 

(4) Angles Greater than 45° and Less Than 90°. 
The tangent scale necessarily reads only to 45° because 
tan 45° = 1, and therefore the reading is the right D index. 
By the definition of cotangent 

tan 6 = 



cot^ tan (90-^)* 

Therefore if tangents above 45° and less than 90° are 
required, use the reciprocal of the tangent of 90° minus 
the given angle. This means that when the tangent of an 
acute angle greater than 45° is required, the slide-rule 
reading must be the reciprocal of the tangent of the com- 
plement of the angle. 

Mannheim. Invert * slide. Align indexes. Set runner 
to 90°— given angle on T; read tangent on D under the 
runner. 

The tangent thus read is the tangent of the given angle because 
read with one scale inverted, which always gives a reciprocal. 

Duplex. Set runner to 90°— given angle on T; read 
tangent on CI under the runner. The reading is: 

1 

= tan a. 



tan (90 -a) 



* Invert means remove, turn over, and insert the slide " the other 
end to." 



Digitized by 



Google 



296 



TECHNICAL ALGEBRA 



294 



Observe that tan 90'' = oo*. 

Pointiiig Oflf. Apply rule for a quotient. 

(5) Angles Greater than 90° and Less than 180®. 
The tangent of an obtuse angle equals minus the tangent of its 
supplement. Therefore, if 6 is between 90 and 180°, 

tan ^=-tan (180° -6^). 

(180°-^) Less than 46°. Read exactly as in (3) or (2) 

or (1). 

(180° — 0) Greater than 46°. Read exactly as in (4). 

Be sure to place minus before the readings. 

See illustration in the next paragraph. 

Observe that tan 180° =0. 

294. Illustrations. (1) Required the tangent of 79°. 



tan 79° = 



1 



1 



cot 79° tan (90° -79°) tan 11 



J = 5.145. 



Fig. 113 shows this reading on a Mannheim rule with the 
slide reversed but not inverted. 




Fig. 113. — Tan 79° on Mannheim rule with slide reversed. 

The runner is set at 11° on T. The reading 5.145 is on 
D under the right T index. 

The setting is diagrammed as follows: 



T 


11° 


It 


D 


1 


5.145 



* oo denotes infinity. 



1 1 denotes index. 



Digitized by 



Google 



294 



THE SLIDE-RULE 



297 



Fig. 114 shows the reading on a duplex rule. Observe 
that the T scale is direct, the C scale inverted and therefore 
marked CI at the left end; thus called CI scale and not C. 



^^ ttiiiiiiiiiiiii 




Fig. 114.— Tan 79** on duplex. 



The runner is set at 11° on T. The reading 5.145 is on 
CI under the runner. 

This setting is diagrammed as follows: 



T 


*I 


11° 


CI 


1 


5.145 



(2) Required the tangent of 134°. 
tan 134°= -tan (180°-134°) = -tan 46°. 



tan 46 



o 



1 



1 



cot 46° tan (90° -46°) tan 44 



:= -1.0355. 




Fig. 115. — Back of Mannheim rule showing 44° on T under right 

notch index. 



Digitized by 



Google 



298 



TECHNICAL ALGEBRA 



2M 



The setting in Figs. 115 and 116 is diagrammed as 
follows: 



T 


*44n° 




C 




1 


D 




-1.0355 




Fig. 116.— Face of rule in Fig. 115 showing -1.0355= tan 134°, oa 
D under sUde index. 



Diagram of Same Reading on Duplex. 



T 


I 


44° 


CI 


1 


-1.0355 



Observe that a reciprocal reading requires an inverted 
scale. On a Mannheim rule which has no inverted scale, 
this is obtained by inverting the slide and therefore the T 
scale. On a duplex rule, which is a modification of the 
plain Mannheim, the CI scale gives the reciprocal reading. 

Readings from direct scales are called direct, and from 
inverted scales are called inverse. 

Fig. 117 shows a reading for 
tan 134°=-: 



tan 44°* 
* n means notch index. 



Digitized by 



Google 



296 



THE SLIDE-RULE 



299 



on a plain Mannheim rule with the slide reversed but not 
inverted. 



r 



' ^-^^-^■^'^^-•^ 




Fig. 117. — Tan 134° on Mannheim rule with slide reversed. 



The figure at the right shows 44° on T in alignment with 
the right D index. 

The figure at the left shows the reading —1.0355 on D 
under the left T index. 

Diagrammatic setting. 



T 


I 


440 


D 


-1.0355 


1 

— 



295. Miscellaneous Examples in Ftmction Readings. 

Write the readings for the following functions, diagram 
each setting, and check by the table of functions. 



1. tan 30° 45' 
4. tan 18° 30'. 
7. sin 14' 16". 

10. tan 4° 29'. 

13. tan 59°. 

16. cos 63° 25'. 

19. tan 22' 14". 

22. tan 156° 20'. 

25. tan 128° 40'. 

28. tan 107° 45'. 



2. sin 30° 45'. 

6. tan 3° 12'. 

8. tan 21'. 

11. sin 4° 29'. 

14. sin 75° 30'. 

17. cos 31° 42". 

20. sin 125°. 

23. cos 130° 15'. 

26. tan 131° 30'. 

29. tan 98° 50' 30". 



3. tan 30' 45". 

6. tan 45". 

9. tan 21° 18'. 

12. sin 4' 29". 

15. tan 72° 25'. 

18. cot 27° 30'. 

21. tan 165° 40'. 

24. cos 172° 18'. 

27. tan 112° 25'. 

30. tan 155° 15' 45". 



Digitized by 



Google 



300 



TECHNICAL ALGEBRA 



29e 



§ 4. MULTIPLICATION 

296. What Scales are Used in Multiplication. Numbers 
are multiplied by the A and B scales, or by the C and D 
scales. The advantage of using the C and D scales is that 
they have a greater number of graduations than the A and B 
scales, which makes it possible to read a result more closely 
than on the A and B scales. 

The disadvantage of using the C and D scales is that a 
factor is frequently " off the rule,'* which makes it neces- 
sary to change indexes. This disadvantage however is only 
apparent and the C and D scales are recommended for all 
multiplication except when one of the factors is a sine or a 
cosine. 

297. The Principle of Slide-Rule Multiplication. The 
right and left ends of the logarithmic scales on the slide- 
rule, marked 1, are called the indexes. The first graduation 
is marked 1 because the graduations are proportional to the 
logarithms of the numbers by which the graduations are 
denoted, and the logarithm of 1 is zero. 

The reason why the graduations continually decrease in 
length as the scale advances, and the principles of slide«-rule 
computation, will be evident from the following series of 
numbers and their logarithms: 



No. 


1 


2 


3 


4 5 


6 


7 


8 


9 


10 


log. 





.301 


.477 


.602 .639 


778 


.845 


.903 


.954 


1.0 



Observe the decreasing interval between the logarithms. 
Imagine the C and D scales divided into 1000 equal parts: 

Then 1 is placed at the zero graduation, 

2 at the 301st, 

3 at the 477th, 

4 atthe602d, 



Digitized by 



Google 



1 



299 THE SLIDE-RULE 301 



5 


at the 699th, 


6 


at the 778th, 


7 


at the 845th, 


8 


at the 903d, 


9 


at the 954th, 


10 


at the 1000th. 



Therefore the product of 2X3 may be delennined by 
placing 301 equal divisions of the D scale end to end with 
477 equal divisions of the C scale, making a total of 778 
which is the graduation marked 6 on the D scale. 

298. Integral Figures. In computations on the slide- 
rule, results may sometimes be pointed off by inspection. 
In other cases, use is made of the integral figures of the 
numbers involved. 

For this purpose the integral figures of all numbers are 
determined as follows: 

(1) Number Greater Than 1. The number of integral 
figures in a number greater than unity equals the number of 
figures which precede the decimal point. 

(2) Number less than 1. The number of integral figures 
in a decimal equals minus the number of ciphers between the 
decimal point and the first significant figure of the decimal. 

Thus 113.92 has 3 integral figures, 

.7896 has integral figures, 

.00845 has —2 integral figures, 

.O46I59 has —4 integral figures. 

299. Rules for Pointing oflf a Product. All products 
obtained on the slide-rule may be pointed off by the fol- 
lowing rules: 

(1) The number of integral figures in a product obtained 
on C and D (or B and left A) with the slide to the left^ and on 
B and right A with the slide to the right, equals the sum of 
the integral figures in the factors. 



Digitized by 



Google 



302 TECHNICAL ALGEBRA 300 

(2) The number of integral figures in a product obtained 
on C and D (or B and left A) with the slide to the right, or 
on B and right A with the sUde to the leftj equals the sum 
of the integral figures in the factors, minus 1. 

Thus (a) L2X. 0005 = .0006. 

SUde to the right; l-3-l = -3. 

—3 means .000 precede the number read. 

(6) 42.9 X. 854 = 36.6. 

SUde to the left; 2+0 = 2. 

2 means two figures precede the decimal 

point. 

300. Illustrations of Multiplication. The following exam- 
ples illustrate the use of the slide-rule in the determination 
of a product: 

1. To multiply L25 by 17.3 on the sUde-rule. 

Move the slide to the right until the index of the C scale 
is exactly over (in alignment with) the graduation marked 
L25 on the D scale. (Use the runner to align C index with 
L25.) 

Move the runner until the hair-line is in alignment with 
the graduation marked 17.3 on C. 

Under 17.3 on C read the product 216 on D, 

Set the rule as specified and observe that log 17.3 is end 
to end with log 1.25 and that graduation 216 is therefore 
the product because it is the antilogarithm of the sum of the 
logarithms. 

The equation for pointing off, as determined by para- 
graph 299, is as follows: 

Slide to the right l-|-2-l = 2. 

Therefore the product has 2 integral figures. 

Fig. 118 shows the setting. The C index is over 1.25 
on D; the runner is at 173 on C. The product 21.625 is 
on D under the runner. 



Digitized by 



Google 



300 THE SLIDE-RULE 

The diagram is as follows: 



c 


1 


17.3 


D 


1.25 


216 


* 




> 



1+2-1=2 



303 




Fig. 118.— 1.25X17.3 on LL duplex. 

Determine the same product by a different setting, 
write directions in the work-book for both settings, and 
give the equation for pointing off. 

2. To multiply 72.5 by .832, since .832 is off the rule 
when the C index is set to 72.5 D, the slide is moved to the 




MADE IN f^PPMAN 




Fig. 119.— Section of Mannheim rule showing 72.5 on DX.832 on C. 
The product 60.3 is on D under the runner. 

left instead of to the right, until the right index of C is in 
alignment with 72.5 on D. 

Set the rule as specified and move the runner to bring 
the hair-line into alignment with .832 on C. 

* Arrow shows direction of slide. 



Digitized by 



Google 



304 



TECHNICAL ALGEBRA 



801 



Under .832 read the product 60.3 on D. 
Fig. 119 shows the setting. Be sure to reproduce it on 
your rule. 
Diagram: 



c 


.832 


1 


D 


60.3 


72.5 



2+0=2 



The slide is to the left. Therefore the number of integral 
figures in the product equals the sum of the integral figures 
in the factors. See paragraph 299 (1). 

Determine the same product by a different setting, 
show diagram, arrow, and equation for pointing off. 

3. To multiply 32.8 by 6.52 move the slide to the left 
to bring the C index into alignment with 32.8 on.D. 

Move the runner to bring the hair-line over 6.52 on C 
and read the product 213.8 on Z>, under the hair-line. 

Set the rule as specified. 

Determine the product by another setting, and write 
directions for, or diagram both settings. 



c 


1 


6.52 


D 


32.8 


213.8 



2-Fl=3 

301. Examples in Multiplication. Solve the following 
examples on the slide-rule, showing all settings and equations 
for pointing off. Check by logarithmic computation. 

For pointing off see paragraph 299. 

1. 18X34.2 2. .18X3.42. 3. .018X.342. 

4. .018X.00342. 5. 18.8X3.8 6. 1.45X85. 

7. 76.4X29. 8. 845X7.5. 9. 39.2X53. 



Digitized by 



Google 



S02 



THE SLIDE-RULE 



305 



10. .00059X.805. 
13. 72X3.47. 
16. .54 X. 0908. 
19. .63X42.9. 
22. 13.8X7.6. 
26. 5.43X85. 
28. 29.2X18.5. 



11. 54.8X614. 
14. 386X15.8. 
17. 1.44X8.14. 
20. .472X.129. 
23. 31. 9 X. 0046. 
26. .4309X3.7. 
29. 74.4X.75. 



12. .129X48.6. 
15. 25.8X.0063. 
18. 708X2.96. 
21. .067X12.4. 
24. .00657X15. 
27. .0^512X392. 
30. 391 X. 935. 



302. How to Mtiltiply Sines and Tangents. Sines. 

To multiply sines use A and S Scales with the slide reversed. 
(Duplex requires no reverse.) 

Thus to multiply 29.6 by sin 18 J ° take settings as shown 
below: 



29.6 



18i° 



Determine the product, give the equation for pointing 
oflf, and explain why the setting gives the product. 
Fig. 120 shows the setting as diagrammed. 




Fig. 120. — Mannheim rule showing 29.6 sin 18 J ° with reversed slide. 
Tangents. To multiply tangents use D and T scales with 
the slide reversed. 

Thus to multiply 19.5 by tan 21° set rule as follows: 



Ij means left index. 



T 


Ii 


21^ 


D 


19.5 


' 



Digitized by 



Google 



am 



TEGECNICAL ALGEBRA 



Detefiniiie tbe product, gLve equation for poifoting off, 
and explaitt why the setting gives the result. 
Figs. 121 aDd 122 shows the setting. 




Fig. 121. — Section of Mannheim rule with shde reversed showing 
h T over 19.5 D. 







Fig. 122. — Mffit end of mle in. Fig: 121 showing nimier at 21" on 7, 
ami 7.485 oni L^ undie3r the runner. 



303. fiitegrar Vtgfares in a Sine or Tangent. Tlie fol- 
lowing summary of the number of integral figures in a sine 
or tangent reading, will f iacilitate the solution of the examples 
in the ne5it paragraph: 

(1) The number of integral figures in a sine or a tangent 
which is read from a guge^oint, may be determined from the 
table in paragraph 291. 

(2) A sine or a tangent reading on the first A scale is pre- 
ceded by .0 and therefore has — I integral figures. 



Digitized by 



Google 



SOS THE SLIDE-RULE 307 

(3) A sine reading on the second A scale is preceded 
directly by the decimal poiiit and therefore has zero integral 
figures. 

(4) A tangent reading on ^e D scale is preceded direcfly 
by a declmfd point throughout the scale, except the reading for 
tan 45° which is 1. 

A tangent reading on the D scale therefore has zero integral 
figures, 

304. Examples. Solve the following examples on the 
slide-rule, showing the diagrammatic settings and the equa- 
tions for pointing oflf. Check by logarithmic or arithmetic 
computation. 

1. 34.19 sin 42°. 2. 19.21 sin 29° 20'. 

3. 9.85 sin 51.6°. 4. .65 tan 14°. 

5, 1,89 tan 34-5°. 6. 12.8 tan 42°. 

7. 11.5 tan 45°. B. 7.26 sin 39.1°. 

9. 427 tan 28.2°. 10. 57.2 sin 56° 18'. 

11. 345 sin IS* tan 40°. 18. 18.7 fiin34i° tan 29i°. 

13. 91.7 sin 50° tan 13.1°. 14. .639 sin 75° t.n 43°. 

16. 450.2 sin 15". 16. 824.5 tan 34.5°. 

17. 17.28 tan 20'. 16. .€71 sin 5° tan 50'. 
200 
16!' 



200 
19. 77^ sin 47.6°. ». 20.05X1.805 tan 118°. 



305. Products of More than Two Factors. A product 
having several factors may be read by the use of the scales 
already specified. 

On a rule having C, C/, and D scales on the same face, 
the product may sometimes be obtained with fewer settings 
than on rules not having this arrangement. 

Following are general diagrammatic settings in which F 
with subscripts denotes the factors in succession, R denotes 



Digitized by 



Google 



308 



TECHNICAL ALGEBRA 



306 



the rimner, and 1 denotes the index of C, either left or right 
being used as may be necessary to give reading: 

(1) 



c 


1 


Rio Ft 


1 to/2 


RioFi 


D 


Fi 






Product 



(2) 



c 




F, 


CI 


Fi 




D 


F2 


Product 



Illustration. To determine 12.5X24X4.56, take set- 
tings as follows: 



(1) 












c 


1 


ii;to24 


ItoR 


R to 4.56 


D 


12.5 




1368 



2+2-1+1=4 



(2) 



c 




4.56 


CI 


12.5 




D 


24 


1368 



2+2-1+1=4 

< 

In the preceding, the product is read by the use of the 
CI scale from one setting because the third factor is so located 
on the slide that it does not project beyond the D scale. 
Sometimes another setting must be made. This is always 
the case when the third factor is located on the C scale so 



Digitized by 



Google 



3i)6 



THE SLIDE-RULE 



309 



that it projects beyond the D scale or is off the rule when 
the setting is made for the first two factors. 

Thus to determine 12.5X24X1.56, take settings as 
follows: 



(1) 












c 


1 


Rto2A: 


ItoR 


Riol.m 


CI 


12.5 






468 



2+2-1+1-1=3 



(2) 



c 




Rio I 


1 toi2 


R to 1,56 


CI 


12.5 








D 


24 






468 



2+2-1+1-1=3 

< < 

Take the preceding readings on your rule in one or both 
of the two ways illustrated in each instance. 

Take the readings also on the A and B scales, showing 
the diagrammatic setting and the equation for pointing off. 

306. One Factor a Sine or Cosine. If one of the factors 
of a continued product is a sine or a cosine, the diagrammatic 
setting is as follows: 



A 


Fi 






Product 


B 






ItoR 


RtoF2 


S 


1 


Rtod* 







* 6 denotes the angle. 



Digitized by 



Google 



310 



TECHNICAL ALGEBRA 



307 



Thus to determine 3.68X18 sin 45° take the following 
settings: 



A 


3.68 






46.8 


B 






lioR 


18 


S 


1 


R to 45° 







1+0+2-1 = 2 



307. One Factor a Tangent. If one of the factors of a 
continued product is a tangent the diagrammatic setting 
is as follows: 

(1) 



c 






ItoR 


RtoF2 


T 


1 


RtoO 






D 


fi 






Product 



(2) 



T 




d 


CI 


Fi 




D 


F2 


Product 



Thus to determine 3.68X18 tan 40° take the following 
settings: 

(1) 



c 






ItoR 


18 


T 


1 


72 to 40 






D 


3.68 






55.6 



1+0+2-1=2 



Digitized by 



Google 



THE SLIDE-RULE 



311 



(2) 



T 




40 


CI 


3.68 




D 


18 


55.6 



1+2-1+0 = 2 



308. Caution about Setting the Runner. When several 
settings are necessary, use special care so that accumulated 
errors may not affect the final reading. 

Keep the hair-line free from dust and take no reading 
from the runner except when it is set so that the graduation 
mark for the reading is exactly coincident with the hair-line. 
When set properly no trace of the graduation can be seen. 

309. Examples. Compute the following on the slide- 
rule, showing the diagrammatic settings and equations for 
pointing off. 

Take two different settings for each example so that one 
may be used as a check for the other. Check the slide-rule 
computations by logarithmic solution from the tables. 



1. 3.78X72.5 tan 24°. 
3. 29.8X6.12 tan 18.4°. 

6. 44.6X53.7 tan 29.3^ 

7. 842X9.65 tan 31.2°. 

9. 55.6X.139 tan 22° 35'. 
11. 17.28X16 sin 35°. 
13. 472X12.8 sin 4° 12'. 

16. 2630X.192sin2°34'. 

17. 546X3.27 cos 18° 12'. 
19. 8.27X58 cos 60° 20'. 
21. 28.6X3.47X29.75. 
23. 53.8X4.8X6.12. 

25. 485X. 349X6315. 



2. 17.4X51.8 tan 361°. 

4. 34.7X.169tan9° 15'. 

6. .789 Xtan 15.5°. 

8. 51.8X1.64 tan 9° 20'. 
10. 689X.185tan41° 10.5'. 
12. 29.8X39 sin 56° 30'. 
14. 584X3.125 sm 3° 8.5'. 
16. 90.8X26.01 sin 1° 45'. 
18. 3.86X.142cos40°31'. 
20. 54.9X2.446 cos 72° 9'. 
22. 72.6X5.408X12.9. 
24. 8.126X73.5X17. 
26. 6.07 X 50.9 X. 00845. 



Digitized by 



Google 



312 



TECHNICAL ALGEBRA 



310 



§ 6. DIVISION 

310. How a Quotient is Determined on the Slide-Rule. 

Since division is the inverse of multiplication, a quotient is 
obtained by setting the rule so that the logarithm of the divisor 
may be subtracted from the logarithm of the dividend. Division 
may therefore be performed on the following scales: 

A and B, and A and BI, 
C and Z), and CI and D. 

. BI means the B scale inverted, an inverted scale being graduated 
from right to left instead of from left to right. If your rule has no 
BI or CI scale,* it will have both if the slide is inserted with the 
ends reversed. This is not reconunended, for the graduations 
stamped on the slide are then upside down, the BI scale lies next 
to the Z), and the CI next to the A. But, if the user of a plain 
Mannheim rule must have an inverted scale, this is the only way 
to get it without buying a more expensive rule. 

Below are general diagrammatic settings for both C and 
2>, and CI and D scales: 

(1) 



(2) 



Illustration. To divide 76 by 4 take settings as 
follows: 

(1) . • 



c 


Divisor 


1 


D 


Dividend 


Quotient 




CI 


1 


Divisor 


D 


Dividend 


Qoutient 



C 


4 


1 


D 


76 


19 



Digitized by 



Google 



310 



THE SLIDE-RULE 



313 



CI 


1 


4 


D 


76 


19 



<2) 



Enter all these diagrams in the work-book. Show also 
corresponding diagrams for the A and B, and A and BI 
scales, and take the readings on the rule for all settings. 

Explain why these settings give the quotient. 
Figs. 123, 124 and 125 show the setting on the multi- 
plex, duplex, and log log duplex. 



^- 7-^^^::r.--. 9 •■ 1 • n 



16 17 




■7-^ 



J 



Tig. 123. — Multiplex rule, showing division of 76 by 4 on A and BI 

scales. 




|g{|^h!d!illihl| mjih 




IB 



mIp 



76 



Fig. 124. — Duplex rule showing — =19 by CI and D 

4 



tP 



'■nm|iM! 




Fig. 125. — LL duplex showing 76 divided by 4 on C and D. 



Digitized by 



Google 



314 



TECHNIOAL ALGEBRA 



dU 



311. Rule for Pointing Off a Quotient The rules for 
pointing off a quotient obtained cm the slide-rule are the 
inverse of the rules for pointing off a product: 

C and D, or A and B scales. 

(1) The number of integral figures in a quotient obtained 
when the slide is moved to the left, equals the number of 
integral figures in the dividend minus the number of integral 
figures in the divisor. 

(2) The number of integral figures in a quotient when 
the slide is moved to the right, equals one more than the 
difference between the number of integral figures in dividend 
and divisor. 

CI and D« or A and BI Scales. 

(1) The number of integral figures in a quotient obtained 
when the slide is moved to the left, equals one more than the 
difference between the number of integral figures in the 
dividend and divisor. 

(2) The number of integral figures in a quotient obtained 
when the dide is moved to the right, equals the difference 
between the number of integral figures in dividend and 
divisor. 

These rules may be tabulated as follows: 

Table XII 
POINTING OFF RULES FOR DIVISION 



Scales. 


Direction of Slide. 


Integral Figures in 
Quotient. 


C and Z> • 
^ andB 


Left 


Difference 


Right 


DifFerence+1 


C/andZ) 
AI and B 


Left 


Difference +1 


Right 


Difference 



Digitized by 



Google 



31S THE SUDE-RULE 315 

312. Do not Memorize but Think. Be sure to observe 
that when the divisor on C is set over the dividend on Z), 
the quotient is always on D under the C index. 

In other words, when the divisor is on (he slide the quotient 
is on the stock. 

Avoid thinking this must be remembered. Of all instru- 
ments the slide-rule is the one which necessitates that 
nothing be remembered, for everything is before you on the 
rule. The one fact you must know for division is that in 
division logarithms are subtracted, because logarithms are 
exponents. For the rest, study the rule. The necessity 
now is not to remember or to consult models or diagrams, 
but to think. The models and diagrams are for reference 
when you are tmable to think. 

Do not be satisfied to solve the examples in this list only. 
Have the ambition to masrier slide-rule division and other 
operations. The work in shop, laboratory and classroom 
will furnish the material. If not, try all sorts of numbers. 
Work of this kind steadies the mind in idle moments, 
makes one more intelligent, and will probably increase one's 
earning capacity. 

313. Examples in Division. Solve the following exam- 
ples on the slide-rule, showing all diagrammatic settings 
and equations for pointing off for one set of scales. 

Check by logarithmic or arithmetic computation. 

1. 18.94-3.1. 2. 144.5-^6.5. 3. 596-S-121 

649 58.2 89.1 

78.1* 3.21' 95 * 

7.64 58.2 1728 

2 28 • 2.61* 14.4* 

./v 200 ., 4.39 ,^ .396 

^^•16-f ^'•:212- ' ^^'WS' 

5L6 m .m^ 

1.27 5.67 .349 



Digitized by 



Google 



316 TECHNICAL ALGEBRA Sia 

.^ ,608 ^„ 7.84 -^ 1626 

16. . 17. . 18. . 

L47 16.3 342 

.A 12.96 ^ .2982 ^, 9.46 

19. . 20. . 21. . 

51.85 1.78 .575 

200 im3 ^, .0484 

ICf 18.5 • 



25. 



^«. 


.6815' 


27. 


.483 


sin 23" 30'' 


30. 


743 


sin 60.5°* 


oo 


743 



420.5 .3917 

8.075* .0562* 

.275 31.4 sin 32** 

• CSC 41.2°* tan 30° * 

7.8 cos 18° . 1 

sin 18° ' tan 18°' *^' sin 60.5°' 

34. 41.5 tan 71°. 36. 2.28 tan 80.2°. 36. 5.25 taa 57° 40'. 

«« 78.9 «. 29.6 ^^ .6871 

37. . 38. . 39. . 

3.18 .0875 12.9 

^^ 5.62 ^, 4.76 ^„ .0328 

40. . 41. . 42. . 

.731 .0029 17.4 

^^ .0096 ^^ 76.8 ^^ 2.46 

43. . 44. . 46. . 

.000751 125.2 .533 

^^ 92.1 ^^ .805 ^^ 349 

46. -— . 47. — — . 48. 



.756' • .508* ; .1828' 

,^ 5965 ^^ 27.15 ^/ 48.5 

49. - — r—To' 60. : — -—I, 61. 



tan 18.4°* ' tan 58^' ' cot 17° 15' 

70.4 ^^ .704 ^^ .516 

52. ; — TTx^v,. 63. 7— :^;^rTV7. 64. 



tan 34° 17'* tan 20° 15'* tan 2° 18'" 



Digitized by 



Google 



iiB 



THE StIDE-RULE 



3ir 



§6. PROPORTION 

314. How to Set the Rule. One of the simplest use» 
of the slide-rule is for the solution of a simple proportion 
in which two factors are divided by a third. The setting 
is as follows: 

(1) 



(2) 



When a proportion is given for solution by the slide- 
rule there is no necessity of algebraic solution if one will 
notice whether a mean or extreme is unknown. The 
moment this is done the two factors and the divisor may be 
immediately determined and the reading taken. . 

. Thus if 12.5 : 8.2 :: 24.3 : R, 

12^^24^ 
8.2" i2 ' 



c 


Divisor 


Ft 


D 


Fi 


Result 










CI 


F, 


Divisor 






D 


Fi 


Result 





or 



the factors are 8.2 and 24.3 and the divisor is 12.5, since in 
every numerical proportion the product of the means equals 
the product of the extremes. 

315. What to do when a Number is " Off the Rule." 
If an attempt is made to determine R by the preceding 
diagrammatic setting, the second factor 7^2 = 24.3 will be 
found to be " off the rule," since when 12.5 on C is set over 
8.2 on D, 24.3 on C is beyond the index of D. Whenever 
this is the case the final reading can be taken only by a. 
setting from the index which is off the rule. 



Digitized by 



Google 



31S 



TECHNICAL ALGEBRA 



SM 



This setting is always made as follows: 
(a) Move the runner to the C index which is on the rule 
denoted cm the diagram by /2 to 1. 

This is necessary in order that the hair-line shall be set at the 
number which is the product or quotient or other function of the 
quantities involved in the previous setting. Do not read this 
number, for it is not wanted. 

(6) Keeping the runner stationary, bring the projecting 
C index to the hair-line, denoted on the diagram by 1 to /2. 

(c) Move the runner until the hair-line is over the 
required reading. 

Thus (1) 



c 


12.5 


Riol 


1 to/2 


24.3 


D 


8.2 






15.94 






1-2+1+2 = 


2 




(2) 






CI 


24.3 


i2tol 


1 tofi 


12.5 


D 


8.2 






15.94 



2+1-1=2 



316. Explanation of the Preceding Process. The quick 
and intelligent use of a slide-rule is possible only when a 
mastery of the principles involved makes one independent 
of prescribed models. 

Therefore study diagrams (1) and (2) and the instructions 
for the settings imtil the following is clearly understood: 

In (1), 8.2 is divided by 12.5. Therefore the setting 
is made so that from the logarithm of 8.2, is subtracted the 
logarithm of 12.5, the difference being imder the left index 
of C. 



Digitized by 



Google 



317 THE SLIDE-RULE 319 

The result is to be multiplied by 24.3. 

If this were *^ on the rule " it would only be necessary 
to move the runner to 24.3 on C because this would place 
log 24.3 end to end with log 8.2 -log 12.5. 

Since 24.3 is not ^* on the rule/' 

(a) the runner is moved to log 8.2— log 12.5 under the 
left C index, 

(6) the slide is moved its whole length to bring the right 
index under the hair-line, 

(c) the logarithm of the reciprocal of 24.3 is subtracted 
from the difference previously obtained, 

(d) the antilogarithm of the final difference, which is R 
in the proportion, is read on the D scale imder the left C 
index. 

In (2), 24.3 is multiplied by 8.2 on the CI scale. There- 
fore, the setting is made so that from the logarithm of 8.2 
on Z), the logarithm of the reciprocal of 24.3 on CI is sub- 
tracted, the difference being under the left C index. 

The result is to be divided by 12.5. 

If 12.5 were on the rule it would only be necessary to 
move the runner to 12.5 on Cly which would place the 
logarithm of the reciprocal of 12.5 end to end with the dif- 
ference of logs previously obtained. Since 12.5 on C is 
off the rule, the final reading is taken only after a change of 
indexes, as explained in (a), (6), and (c). 

317. The Ratio Method. The usual method of solving 
a proportion on the slide-rule is based on the principle that 
whenever two numbers are aligned on C and D, or A and B, 
all pairs of aligned numbers throughout the two scales are in 
exactly the same ratio. 

Thus, when the indexes are aligned, 

12 3 
the ratios are — ,— , -, etc., each being equal to 1, the ratio 

1 ^ o 

of the aligned indexes. 



Digitized by 



Google 



320 TECHNICAL ALGEBRA 817 

When 2 on C is aligned with 4 on Z), 

1 4 3 15 , , . , 1 ^ 
the ratios are -, -, -, — , etc., each being equal to -, the 

2 8 6- 30 2 

ratio of 2 to 4. 



Therefore the unknown term R in 

12.5^24.3 
8.2 R • 

is determined by '* setting up *' the ratio 



12.5 

8.2 



on A and B 



or on C and D, The runner is then moved to 24.3 on the 
wsame scale as 12.5, and the number aUgned with 24.3 
under the runner is the unknown term. 

Why? Because the ratio of 12.5 to 8.2 is exactly the 
same as the ratio of 24.3 to the number in alignment with 
it, or 15.94. 




Fig. 126. — Sections of Mannheim rule illustrating ratio setting for 
solution of a proportion. 

The right figure shows the ratio 12.5 : 8.2. The left 
figure shows the slide indexes interchanged, and the equal 
ratio 24.3 : 15.94 under the runner. 

It will be obvious that the numerators cf the two ratios 
must be on one scale and the denominators on another. 

This is undoubtedly the simplest way of solving a pro- 
portion and is recommended. 



Digitized by 



Google 



318 THE SLIDE-RULE 321 

318. Examples. Solve the following proportions, with 
diagrammatic settings and equations for pointing off: 



1. 


19 : 34::55 : x. 


2. 


75 : 29:: 11.8 : x. 


3. 


4.29 18.2 
6.1 X ' 


4. 


X 2.1 
50.8 41.2* 


6. 


1.27 18.5 
X 78.6* 


6. 


342 X 
193 8.4* 


7. 


sin 47° 124.5 
X 2.56 • 


8. 


9.1 50.9 
tan 29° x ' 


9. 


tan 32° x 
18.24 "".61' 


10. 


398 tan 60° 
X .536 ' 


11. 


cos 48° 3' X 
34.6 7.32 


12. 


8.69 7.06 
tan 34° 16' " x ' 


13. 


X tan 41.2° 
93.4"" 4.29 * 


14. 


3.47 X 
sin 18° 20' ".831' 


16. 


.723 .82 
tan 29° 41' ~lx ' 


16. 


13.45 914.9 
4.21 " X * 


17. 


5.4 tan 75°=^. 

X 


18. 


X 429 
tan 27° 24' 61.7* 


19. 


306 X 
cos 43° 8' "184' 


20. 


.0734 sin 54° 
.00426" X ' 


21. 


34 : 85::a; : 296. 


22. 


45.2 : 130.1:: 16 : 


23. 


534 29 

296~>S' 


24. 


T 78.2 
341 61.3 • 


26. 


182 608 
71.3" T' 


26. 


556 19.2 
V 4.75* 


27. 


70 760 
y "740* 


28. 


125 760 
112 mm 


29. 


.428 X 
1.09 "3.76' 


30. 


♦22 X 

7 "r 



X. 



♦Align 22 on A with 7 onB. The index 1 nearly aligns with what 
gage-point? Why? 



Digitized by 



Google 



322 TECHNICAL ALGEBRA 819 

„/ 78.9 461 ^^ 308.5 12.84 

* 594.5 «; 21.9 W ' 

„« .0893 X ^^ .00429 52.45 

.624 192.5' * .00265 x ' 

319. Combined Multiplication and Division. In com- 
putation it is sometimes necessary to determine a result 
when several factors are divided by several other factors. 
An analysis of the diagranmiatic settings for the examples in 
the preceding paragraph will show how the result is obtained 
under such conditions. 

Thus, in the examples in proportion, the runner was set 
to one of the factors on D and the slide was moved to bring 
the divisor on C to the runner. The runner was then 
moved to the second factor on C and the result was read on D, 

Or, by the ratio method, the known ratio was set up. 

The application of this process to the determination of 
the value of 

34X41X65X19 



52X18X75X31.5* 
is as follows: 

(1) Set the riumer to a factor in the numerator, for 
convenience the first factor on D. Bring a factor of the 
denominator to the runner, for convenience the first factor 
on C. This gives 52 C over 34 D, 

(2) Bring runner to 41 C. 

(3) Bring 18 C to the runner. 

(4) Bring runner to 65 C. 

(5) Bring 75 C to the runner. 

(6) Bring runner to 19 C 

(7) Bring 31.5 C to the runner. 

(8) Read result on D under C index. 

This means briefly: 

Bring the runner to all multipliers. 
Bring all divisors to the runner. 



Digitized by 



Google 



320 



THE SUDE-RULE 



323 



Whether multipliers and divisors are used alternately 
as above, or whether all multipliers first and then divisors, 
makes no difference in the result. 

Following is the diagrammatic setting: 



c 


52 


i2to41 


18toi2 


iJ to65 


75 to R 


i^to 19 


31.5 to R 


^ 


34 












.778 



2-2+2-2-hH-2-l -2+1+2-1 -2=0 
< > < 

Make these settings on the slide-rule with the equation 
for pointing off. 

Check by logarithmic computation from the table. 

When the number of factors in the numerator is not the 
same as the number of factors in the denominator, proceed 
as follows: 

(1) Bring the rimner to the C index for each missing 
numerator factor. 

(2) Bring the C index to the runner for each missing 
denominator factor, in both cases using the C index that 
is not " off the rule." 

In applying (1) and (2) neither subtract nor add unity 
to the number of integral figures, because a multiplier or 
divisor of 1 does not change the result. 

320. Examples. Solve the following examples on the 
slide-rule and check by logarithmic computation from the 
table. 



1. 



5. 



29.2X77X12.3 
3.45X53X8.6* 

8.2X91.9X1 65 
42X3.4X87 ' 

493X3.72X12.8X. 64 
5.7X29.4X. 39X16.3* 



50.5X31X18.2 
48X91.2X3.75' 

.78X14.5X22.6X83 
15.2 X 14.9 X. 082 X. 634' 

61.8X9.2X18.45 sin 35° 
29.2 X. 806X5.36X25 cos 35** 



Digitized by 



Google 



324 



TECHNICAL ALGEBRA 



321 



7. 



11. 



13. 



16. 



17. 



19. 



492X18.3 tan 28° 20^ 
n.5X2.16sin29' ' 
217 X. 824 



515 X. 00634X9. 12X11.6* 
91.5X36X184.5X 75 

33000 
5.96 X 18.2 X. 349 
.784X2.68X1.78' 

.674X14.4 X8.12 
3.39 X. 092X1721* 

14.6X29.4X4.37 
1296X15.9X6.72* 
.581X84.2X13.9 
64.1 X.128X. 049* 



8. 



10. 



12. 



14. 



16. H^: 



.0718X9.6X22.8 C08 35° 
.0078X613 
84.5X21.8 tan 63° 28^ 
4.19X.302X61.7 sec 29.4° 
1875X21.4X63.2 
746X 18.9k. 831* 
.C09X 1345X78. 1 
1.245X12.7X346.5* 
.567 XI 8.9 X. 00436 



18. 



20. 



7.47X9.02X. 00573 
3.48 X .792X40.9 
18.24X34.71X15.9' 
72.8X6.55X1728 
40.3X.64X3.09* 



§ 7. LOGARITHMS 

321. The L Scale of Logarithms. On the slide, usually 
between the S and T scales, or on the face or edge of the 
stock is an L scale or scale of logarithms. 




' TilltliillliliillfilllllillililiiilllKlllilllillill 



illllllllllllllllltllllllllillilllllllllllllllllllillllll^ 




Fig. 127. — Left end of L scale on edge of duplex rule. 

When the zero of the L scale is in alignment with the index 
of th3 D or C scales, the mantissa of the logarithm of any 
numher on those scales may be read on the L scale: and 



Digitized by 



Google 



321 



THE SLIDE-RULE 



325 



inverselyj the antilogarithm of any number on the L scale may 
be read on the D or C scales. 

On some rules the D and L scales are both on the stock 
with the L zero to the left of the D index. When this is 
the case the D index is aligned with the L zero by a mark 
on the tongue of the runnei . 

Examine the L scale on the rule. (If the rule is a Mann- 
heim, remove the shde.) 

The entire length from zero to 10 has how many 
graduations? 

If the number of graduations were doubled, the scale 
would then be divided into how many equal parts? 

Therefore the 301st division would represent the loga- 
rithm of what number? (See paragraph 297.) 



[ 



1 1 2 3 * :> . 'i . . ' 



Fig. 128. 

Insert the slide in its usual position so that the face 
scales are A, B, C, and D. Move the runner to 2 on D and 
bring the left slide index to the runner. 

Turn the rule over and read the logarithm of 2 on the 
L scale under the L index on the right notch or on the 
celluloid plate. What is the reading? * 




Fig. 129. — L notch (lower) in right end of Mannheim rule. 
* Do not fail to notice in what direction the L scale is graduated. 



Digitized by 



Google 



326 



TECHNICAL ALGEBRA 



If thie rule is a duplex, what is the reading on L when 
the L and D indexes are aUgned and the runner is set at 2 
onD? 

How therefore is the logarithm of a number determined 
by the slide-rule? 

How is the antilogarithm determined? 

What part of paragraph 297 exactly illustrates the read- 
ing of logarithms on a sUde-rule? 

322. Examples. By sUde-rule readings, fill in the omitted 
entries in the following table, determining characteristics by 
the same law as when a table of logarithms is used: 



Table XIII 
LOGARITHMS 



No. 


N 


log. 


No. 


iV 


log. 


1 


38 




13 




2.745 


2 


125 




14 




1.862 


3 


82.3 




15 




1.086 


4 


.546 




16 




.534 


5 


112 




17 




3.620 


6 


16 




18 


17 




7 


18 




19 


19 




8 


23 




20 


25 




9 


26 




21 


27 




10 


28 




22 


29 




11 


31 




23 


32 




12 


35 




24 


4.75 





Digitized by 



Google 



323 



THE SLIDE-BULE 



327 



§ 8. POWERS AND ROOTS 

323. The Square of a Number. Move the runner to 
each of the following numbers on D and enter in the table 
the aUgned A readings: 



D 


A 


1 D 


A 


D 


A 


2 




1 3 




11 




4 




5 




13 




6 




7 




15 




8 




9 




; 16 





The readings on A are what power of the readings on 
D? Explain why. 




Fig. 130.— 2« on duplex. 

324. How a Square is Pointed Off. The number of 
integral figures in a square when read oh the D and A scales 
is as follows, when n denotes the number of integral figures 
in the number to be squared : 

(1) On the left A scale, 2n-l. 

(2) On the right A scale, 2n. 

Test this rule by reading the squares of the following 
numbers, with the A and D scales: 13, 20, 60, 8, .1, .01, .31. 
Is the law correct? 



Digitized by 



Google 



328 TECHNICAL ALGEBRA 326 

Read 18^ with the C and D scales and show the diagram- 
matic setting with a statement of the principle involved. 
If the principle is not clear, refer to paragraph 266. 

325. The Square Root of a Number. As might be 
inferred, the square root of a number on A may be read on 
D. The process is as follows: 

(1) When the number of integral figures is odd^ read 
the given number on the left A scale. 

(2) When the number of integral figures is even or zero, 
read the given number on the right A scale. 

326. How a Square Root is Pointed Off. The number 
of integral figures in the square root of a number, read 

(1) On the left A scale = ^. 

Yl 

(2) On the right A scale = ^ . 

n denotes the number of integral figures in the number whose 
square root is to be read. 

327. Examples. When possible solve the following 
both by the A and D scales and the C and D scales. Show 
not less than one diagrammatic setting for left A, right Ay 
and C and D readings, with equations for pointing off: 

2. 282. 3. 522. 

6. 28*. 6. V525^ 

8. Vs^ 9. 7.5*. 

11. Vtan 18.5°. 12. Vcos 60** 14'. 

14. 3082. 16. 842. 

17. .00242. 18. .6252. 

20. V.0046. 21. \/.0125. 

23. Vtan 40° 20'. 24. (sin 52.3°)*. 



1. 


192. 


4. 


Vl9. 


7. 


31.8*. 


10. 


Vsin 35°. 


13. 


4.82. 


16. 


.0172. 


19. 


V.79. 


22. 


.00085*. 



Digitized by 



Google 



THE SLIDE-EULE 



329 



328. How to Read a Cube. With a table of logarithms 
the cube of a number is obtained by multiplying the logarithm 
of the number by 3, and reading the antilogarithm. On 
the slide'Tule the antilogarithm of three times the logarithm 
is determined by the Z), A, and B scales as follows: 



immmmmimiiiima 



fi|i|rji|l(iti.|i|!K 

" . : 3 7t 

;ljllllllll(l|ll|llill!l|lll!l)lf 



Fig. 131.— 19.68=2.7'. 

/ 

(1) Set the runner to the number on D. Twice the loga- 
rithm is then under the runner on A, 

(2) If the given number is on the left half of D, move the 
left index of the slide to the runner; if on the right half of 
D, move the right index of the slide to the runner. 

(3) Move the runner to the given number on that scale 
of B whose index was set to the runner in (2). 

The reading on A under the runner is the antilogarithm 
of 3 times the logarithm of the given number and is there- 
fore its cube. 

Take the following setting on the rule and explain why 
it gives the cube of 7: 



A 




343 


B 


1 


7 


D 


7 





Digitized by 



Google 



330 



TECHNICAL ALGEBRA 



329 



Place the left B index over 27D and set runner as 
shown in Fig. 131. 

Why is the A reading under the runner, the cube of 2.7? 





7 d B- 1 



Fig. 132. — First third of a cube scale. 

329. The Cube Scale. Some rules have a cube or K 
scale on the stock, either on the vertical edge or on the face. 



mi 



mimmmiim 



II 



-tUi: 



jnUltJiilllJJji/ 




Fig. 133. — 2.7» on Triplex rule. Observe 2.7 under runner on C; 
the cube, 19.68 is on K (lower scale) under runner. 

The K scale consists of three equal logarithmic scales placed 
end to end, with the two end indexes in alignment with the 
D indexes. 



Digitized by 



Google 



830 



THE SLIDE-RULE 



331 



The cube of any number on Z), therefore, is read on K 
by the use of the runner on which the hair-line is carried 
down to the K scale by an index mark on the side; and 
inversely, the cube root of any number on K is read on D, 

For convenience the three K scales may be denoted 
from left to right by iCi, K2y and iCa. 

330. How to Point Off the Cube of a Numb^. If 

n denotes the number of integral figures in the number 
whose cube is required, the number of integral figures in the 
cube is as follows: 

(1) When read on the K scale; 

On Ki, 3n-2; 
OniC2, 3n-l; 
On Ksy 3n. 

(2) When read on the A scale. (See Table XIV.) 

Table XIV 
POINTING OFF A CUBE 



Cube read on. 


Slide Pi ejecting 
to the 


Integral Figures 
in Cube. 


Left A 


Right 


3n*-2 


Left 


3n-l 


Right A 


Right 


3n-l 


Left 


3n 



The table applies only when readings on left A are the 
cubes of numbers on the left half of D, and when readings 
on right A are cubes of numbers on the right half of D, 

(3) When the L scale is used. When a number is 
cubed by multiplying its logarithm by 3, the mantissa being 

*n denotes the number of integral figures in the given number. 



Digitized by 



Google 



332 



TECHNICAL ALGEBRA 



331 



read on the L scale, if c denotes the number of units in the 
characteristic of the resulting logarithm, the number of 
integral figures in the cube is as follows: 

(a) When the characteristic is positive, c+1. 
(6) When the characteristic is negative, — c. 

331. Examples. By the use of the A^ B, and D scales, 
determine and fill in the omitted entries in the following 
table. 

If your rule has a K scale, use it for checking the readings 
on the A scale; if not, check by the L scale. 

Table XV 
SQUARES AND CUBES 



















No. 


N 


JV2 


N* 


No. 


N 


N* 


isr« 


1 


13 






6 


3.4 






2 


9 






7 


53 






3 


14 






8 


.48 






4 


15 






9 


6.2 






5 


2.1 






10 


74 







Read the squares and the cubes of the foUowng num- 
bers and tabulate as above. 



11. 4. 


12. 9.1. 


13. 5.4. 


14. 7. 


16. .21. 


16. 3.7. 


17. 11. 


18. 1.3. 


19. 6.5. 


20. 1.5. 


21. .6. 


22. 2.4. 


23. 12. 


24. 8. 


26. 7.6. 


26. 6.1. 


27. 4.3. 


28. 5. 


29. 3.2. 


30. 2.9. 


31. 2.5. 


32. .4. 


33. 7.2. 


34. 5.3. 



332. The Cube Root of a Number. By the principles 
of logarithms the cube root of a number is obtained by 
dividing its logarithm by 3, and by reading the antiloga- 
rithm of the quotient. 



Digitized by 



Google 



333 THE SUDE-EULE 333 

The cube root of a number may therefore be computed 
on the slide-rule, in four different ways: 

(1) By ^, jB, and D scales. 

(2) By the L and D scales. 

(3) By the K and D scales. 

(4) By the LL and CI or C scales. 

Which of these ways should be used in determining a 
cube root depends entirely upon the kind of slide-rule used. 
If your rule has a K scale or a log log scale, the other ways 
of computing the root will be of interest only as a means 
to a more complete understanding of the principles of 
operation. 

333. Cube Root by the A, B, and D Scales. On all 

plain Mannheim rules the A, B, and D scales are used for 
the cube root. Before a reading can be taken, the number 
whose cube root is required must be divided o£f as in arith- 
metical cube root into periods of three figures each from the 
decimal point. 

Thus 3'917, .829'3, 1'.29, 14'.8, 42'674, .000763, .007'63, 
.008'56, .091'5. 

The part of the scale on which the readings are to be 
taken is determined as follows: 

When the first significant period has 

(1) One significant figure, use left A and left C index; 

(2) Two significant figures, use right A and left C index; 

(3) Three significant figures, use right A and right C 
index. 

The first significant period is the first period beginning 
at the left, which contains significant figures. 

Pointing Off. The only rule required for pointing off 
a cube root is the following: 

Each period in the number whose root is required gives 
ONE figure in the root. 

Thus vT728=12, >J^.000^= .0928, 



Digitized by 



Google 



334 



TECHNICAL ALGEBRA 



SU 



334. Cube Root by the A, B, and D Scales. The prin- 
ciples involved when the A, J5, and D scales are used, will 
be evident from the setting for the cube of a number, which 
always means the addition of once its logarithm to ttvice its 
logarithm. 

The cube of a number was therefore read on the A 
scale by placing once the logarithm on jB, end to end with 
twice the logarithm on Ay this latter being directly over 
once the log on D, 

Therefore when the cube was read on Aj the number on 
B under the runner and the number on D under the sUde 
index, were identical. 

We therefore have the following 

Rule for Cube Root on A, B, and D Scalm. 

(1) Consider the given number to be separated into 
periods of three figures each, in both directions from the 
decimal point. 

(2) Set the runner to the given number on A as specified 
in the table below. 

(3) Move the slide until the number on B under the runner 
and the number on D under the C index are exactly the same. 

This number is the cube root. 

CUBE ROOT WITH A, B, AND D 



Significant Figures in 
First Period 


Set Runner on 


Read Root under 


1 


Left il 


Left C index 


2 


Right A 


3 


Right C index 



Owing to the difference in the graduation of the D and 
B scales, the setting of the slide as required in (3) will give 
some difficulty and requires considerable practice. 



Digitized by 



Google 



836 THE SLIDE-RULE 335 

In case the edge of the runner hides the reading of a 
number under the slide index, the readings must be taken 
without the runner. 

335. Cube Root by the K and £) Scales. The cube root 
of a number is read as follows on the D and K scales: 

(1) Divide off the number, or consider it as divided off, 
into periods of three figures each in both directions from 
the decimal point. 

(2) Set the runner to the number on K and read the 
cube root on D under the hair-line, as follows: 

When the first significant period has: 

(a) One significant figure, set to Ki\ 
(h) Two significant figures, set to K2; 
(c) Three significant figures, set to K3. 

336. Examples in Cube Root. Extract the cube root 
of the following numbers by the simplest method which 
your rule permits. 

Those who have a log log rule are referred to section 10, 
page 342. 

1. 2. 2. 3. 3. 4. 4. 3.9. 

6. 78.5. 6. 100.5. 7. .00068. 8. .0068. 

9. .068 10. 3.25. 11. 18.9. 12. 2024. 

13. 15000. 14. 4.25. 16. 5.35. 16. 1.065. 

17. 165. 18. 1800. 19. 78.5. 20. 341.6. 

21. 3.456. 22. 5.09. 23. 61.4. 24. 49853. 

26. 23.8. 26. 2095. 27. 1785. 28. 480.7. 



Digitized by 



Google 



336 TECHNICAL ALGEBRA 887 



§ 9. GAGE-POINTS 

337. What Points are used. Computations involving t 
are facilitated by the following gage-points: 

IT =3. 1416. 
- = 1.273. 

^=.31.83. 




miUL 



kikikiiikitiiiijiiiii'iiiii-iiiifliii 



Fig. 135. — r gage-point on A scale. 

Almost any slide-rule has the w gage-point, but few have 
the other two, both of which are an advantage in cylinder 
computation. 

which will be recognized as 100 X—, is used instead 

TT IT 

of the reciprocal of w for the following reasons: 

When the w gage-point 3.1416 is marked on left A, — 

or .3183 cannot be shown on the rule because the first 
graduation on the rule, which is the left index, is then 
unity. 

If — or 3.183 were used, the gage-point would be so 

w 

close to 3.1416 that it would be confused with it. 

100 
Therefore — or 31.83 is used as the gage-point instead 

TT 

, 1 10 
of — or — . 

TT TT 



Digitized by 



Google 



338 



THE SLIDE-RULE 



337 



338. Reading from Gage-Point M. The gage-point 
31.83 is located on right B and is denoted by M. 

With a single setting it gives both the circumference 
and the lateral area of a cylinder from the formulas 



and 



C=tD 
A=tDL- 



PL ^ lOODL ^ lOODL 
1 100 31.83 • 



25 



30 M 



35 



40 



25 



30 M 35 

Fig. 136. — M gage-point. 



40 



If your rule has no gage-point ilf , cut and letter it 
carefully at the point specified, just under the B scale. 

Observe that in the use of the slide-rule a factor of IQ 
or an integral power of 10, affects only the position of the 
decimal point in the result, and therefore is not included 
in the settings. 

Observe also, that a setting for 



A = 



lOODL 
31.83 



is also a setting for C = wD 
because 



1 100 31.83* 



Digitized by 



Google 



S38 



TECHNICAL ALGEBRA 



339 



Take the following setting on the rule, diagram setting 
and readings, write the equation for pointing off, and explain 
by the principle of logarithms why it gives the circum- 
ference and the lateral area of a cylinder whose diameter 
18 12" and whose length is 15". 



A 


12 


Circumference 


Lateral Area 


B 


M 


100 


15 



339. Examples. In the following table, 

D = diameter of cylinder in inches, 

L = length in inches, 

C = circumference , 

i4 =area curved surface. 

Determine the circumference and lateral area by use of 
the M gage-point, and fill in the omitted entries. 

Table XVI 
CYLINDERS. M GAGE-POINT 



No. 


D 


L 


C 


A 


No. 


D 


L 


C 


A 


1 


3i" 


2i" 






6 


li" 


r 






2 


sr 


5r 






7 


2i" 


¥' 






3 


11.4" 


8.22" 






8 


7A" 


3i" 






4 


23.8" 


15.3" 






9 


9f" 


41" 






5 


40.7" 


20.2" 






10 


12.2" 


30.5" 







Diagram the settings and write the equations for point- 
ing o£f. 

340. Gage-Point C. This gage-point represents- = 1.273. 

IT 

When marked on the rule, it is located on the C scale 
at a distance from the left C index corresponding to 1.273 
on A, This is approximately at 1.1283 on the C scale. In 



Digitized by 



Google 




S40 THE SLIDE-RULE 339 

order that no readings involving this 
point may be off the rule, a second 
point Ci, is marked on the C scale at a 
distance from C equal to the distance 
from 1 to 10 on A, which is approximately 
at 3.563 on the C scale. * 

Take the following settings: 

Align slide and stock indexes. 

Move the runner to L273 on A. _ ^ 

Observe C scale reading under the ^^ ^ -^ ^ 

runner. _^ S 

If your rule is accurately set, this -3- q| § 

reading is where the gage-point C is 
marked both on the C scale and on IS, 

the D scale. " i m 

Move the slide so that this reading ^ - ^ Mi A 
is over left D index. ^ -^ i ^ 

Move runner to middle A index. «, " » I ^ 

Observe C scale reading under hair- 
line. 

If your rule is accurately set, this 
reading is where gage-point Ci is marked 
both on the C scale and on the D scale. 

As will be seen from the next para- 
graph, the proper location of C and Ci ?: ^- 2 
is on the B scale. ^ 

In regard to gage-points in general, 
every user of a slide-rule knows the diffi- 
culty of an accurate reading from the 
runner when the hair-line is between the 
gage-point graduation and the regular 
graduation. This difficulty would be 
obviated by placing gage-points im- ziz 

mediately above or below a scale instead 
of across it. 



:5^ 2 



n -- 



Digitized by 



Google 



340 



TECHNICAL ALGEBRA 



341 



Therefore if gage-points C and Ci are not graduated on 
your rule, and you wish them there, carefully cut and ink 
them just under the B scale. 

341. Reading from C Gage-Point. The problem below 
illustrates the use of the gage-point C 

A cylinder is 22.5 inches in diameter and 38 inches long. 
Formulate and compute the volume. 

y ^^^^ 4 1.273- 



A 


. 


Volume 


B 




38 


C 


gage-point C 




D 


22.5 





Observe that when gage-point C is set over the diameter 
22.5 on D by the use of the runner, 22.5^ is under the hair- 
line on A and L273 is under the hair-line on B, Therefore 
the distance on A from the left A index to the left B index, 
represents 

log 22.52 - log L273. 

If the runner is now moved to the multiplier 38 on By 
which is the length of the cyUnder, the volume may be read 
on A under the runner, because to the difference above has 
been added the logarithm of the length. 

Attention is called to the fact that in this instance, gage- 
point C was required on the B scale and nowhere else. If C 
were so placed, as it should be, the diagrammatic setting 
would be as in Table XVII. 



Digitized by 



Google 



842 



THE SLIDE-EULE 



341 



Table XVII 
VOLUME OF A CYLINDER. C GAGE-POINT 



A 




Vol 


A 




Vol. 


B 


C 


Length. 


B 


C 


38 


D 


Diam. 




D 


22.5 





342. Problems. Solve the following problems, in every 
instance showing diagrammatic setting and equation for 
pointing off. 

1. Determine the volume of the cylinder in paragraph 
341 and tabulate data and result. 

2. Determine the volume of the following cyUnders both 
in cubic inches, cubic feet, U. S. gallons, and liters. 



Table XVIII 
CYLINDERS. C GAGE-POINT 



No. 


L" 


D" 


Volume. 
















Cu. in. 


Cu. ft. 


Gal. 


Liters. 


1 


18i 


9i 










2 


12.8 


6i 










3 


8.5 


4f 










4 


13i 


5i 




- 






5 


26i 


15 










6 


30 


18.1 










7 


54.7 


31.4 










8 


38.6 


Hi 










9 


45 


21.3 










10 


76 


401 











Digitized by 



Google 



342 



TECHNICAL ALGEBRA 



343 



§10. THE LOG LOG RULE 

343. A Log Log Scale. Any rule having the usual 
scales is an effective instrument for the determination of 
the square or the square root, and the cube of a number. 
A cube root, as has been shown, may also be read, usually 
with some difficulty and an unsatisfactory approximation. 

Direct readings of any other powers or roots are impos- 
sible. Indirectly, a power or root not the square or cube 
may be determined by reading the logarithm of the number 
on the L scale, transferring the reading to the D or the A 
scale, and multiplying it on the rule by the exponent of the 
power or dividing it by the root index. 

The LL scale is a scale whose graduations are proportional 
to the logarithms of the logarithms of numbers. 

On the log log duplex rule, it is mounted in three sections, 
marked LL 1, LL 2, and LL 3, making a continuous scale 
from 



to e 



10 



e denoting the Naperian base 2.71828. 




Fig. 138. — Right index of back of LL duplex. 



Digitized by 



Google 



843 



THE SLIDE-RULE 



343 



The powers of e and the limiting numbers at the ends 
of the three sections of the LL scale, are as follows: 

LL 1; ei«o to e" or 1.01 to 1.105. 

LL 2; eio to e, or 1.105 to 2.71828. 
LL 3; e to e^", or 2.71828 to 22000. 

By reference to Fig. 139 or the LL rule, write the value 
of each graduation from 1.C8 to 1.09 and from 2000 to the 
end of the LL scale. 



1.08 



iiiiiiii nil 

2000 



III 



1.09 

ililulil 
lllllllllll 

5000 



1.10 



II I I I IMIMlllll 



I IIIIIIII 



lllllllll ^ 

10000 20000 

lllllll 



Fig. 139. — Right end of log log scale. 

Cube Root. The principles involved in the use of the 
LL and CI scales, and LL and C, for cube root, will be 
evident from the following illustration: 

Required the cube root of 2C0. This may be represented 
by the equation: 



log a: = 



log 260 



/. log log a: = log log 260 — log 3. 

What is wanted, therefore, is a rule so scaled that the 
log 3 may be subtracted from log log 260. A rule with a 
log log scale accomplishes this, and therefore makes the 
determination of a power an addition, and the determination 
of a root a subtraction. 



Digitized by 



Google 



344 



TECHNICAL ALGEBEA 



844 



If your rule has a log log scale, take the following settings 
-and explain why they give the cube root of 260: 



LL3 


260 


6.39 


CI 


1 


3 




LLS 


260 


6.39 


C 


3 


1 




Fig. 140. — Section cf LL rule showing setting for cube root of 260. 



344. On What Section of LL, to Read the Root. When 

the rule is set for the cube root of a number on any section 
of the LL scale, if the reading on the same section in align- 
ment with the C index or \vith 3 on C/, is greater than the 
number whose root is required, the root must be read on 
the preceding section; otherwise, read root on the same 
section as the number. 

Thus the cube root of 8 is read: 



LLS 


8 


LL2 


2 


C 


3 




1 




LLS 8 


LL2 


2 


CI 1 1 1 


3 



Digitized by 



Google 



346 



THE SLIDE-RULE 



345 



The root is read on LL 2 because the reading on LL 3 
in alignment with 1 C and 3 CI, is greater than 8, the number 
ivhose root is required. 

The cube root 8000 is read: 



LL3 


8000 


20 




LLZ 


8000 


20 


C 


3 


1 


CI 


1 


3 



In this instance, both the number and its root are read on 
the same section LL 3, because the reading in alignment with 
1 C and 3 CI is less than 8000, the number whose root is re- 
quired. 

Observe that the numbers on LL are absolute and must 
be used without shift of decimal point. 

The cube root or the square root of a number above or 
below the limits of the LL scale may be read if the number 
is separated into periods as in arithmetic root. 

Thus v/.02942 = v^MF42. 

Therefore read •v^2J.42. 

345. Examples. Refer to paragraph 336 and solve all 
the examples on LL and C, or LL and CI. Diagram, and 
give equations for pointing off. 

346. Any Power or Root. The equations in paragraph 
343 indicate how any power or root may be read with the 
log log scale. 

Thus the determination of x^ is formulated as follows: 

log x^ = 5 log X = log 5+log log X. 

Therefore log 5 on C is placed end to end with log log x 
on LL and the result is read on LL. 

A log log scale, therefore, makes it as easy to obtain any 
power or root as to perform multiplication and division, and 
by the same process. 



Digitized by 



Google 



346 TECHNICAL ALGEBRA 347 

347. Examples. Determine the following powers and 
roots by the use of the LL scale, and check by logarithmic 
computation. 



1. 4.7«. 


2. 2.9*. 


3. 4.7'. 


4. 3.6- «, 


6. 1.9«. 


6. 11.5'. 


7. V'l745. 


8. V734. 
11. -^9143. 


9. V2448. 


10. v' .07165. 


12. V649.5. 


13. V843.4, 


14. V.764. 


16. -^29.75. 


16. 32.75*. 


17. 464». 


18. 5064'- 



348. Logarithms to any Base. The common logarithm 
of a number may be /"ead on a log log rule by the use 
of the L and D scales, the same as on any other rule. 

Since LL 3 begins at e the Naperian base, logarithms to 
base e are read on C for any aligned number on LL, 

Logarithms to any other base are read by aligning the 
left C index with the required base on LL, If the reading 
on C is off the slide, change indexes. 

Thus, to read logio 40 set left C index to 10 on LLj 
and under 40 on LL read 1.60 on C This is the common 
log 40. 

To read to base 5, set left C index to 5, etc. 

348. How to Read a Naperian Logarithm. 

1. Ahgn indexes. This sets left C index under e on LL. 

2. Move runner to given number on LL, 

3. Read complete Naperian logarithm on C under the 
runner. 

How to Point Off. 

(1) Place .0 before all C readings from numbers on LL 1. 

(2) Place decimal point only, before all C readings^ 
from numbers on LL 2. 



Digitized by 



Google 



350 



THE SLIDE-RULE 



347 



(3) Point off one integral figure in all C readings from 
numbers on LL 3. 

Observe that the C reading is the entire loge, and therefore 
includes both characteristic and mantissa. 

349. Illustration. The loge650 is diagrammed as 
follows: 

(1) 



LLS 


e 


650 


C 


1 


6.47 



One integral figure is pointed off in the C reading 
because 650 is on LL 3. 
See preceding paragraph. 

(2) 



LL2 


e 


1.755 


C 


1 


.562 



Take this reading and explain the position of the decimal 
point. 

(3) 



LLl 




1.035 









Complete this illustration and explain. 

360. Napierian Logarithms of Numbers Less Than 
1.01. The LL scale begins at 1.01 which is therefore the 
lower limit of numbers whose logarithms can be read directly. 
The logarithms of numbers below this Umit may be read 
in two ways: 



Digitized by 



Google 



348 TECHNICAL ALGEBRA ssa 

L (1) Multiply the number by 10, or 100, or whatever 
power of 10 is necessary to bring the number within the 
range of the LL scale. 

(2) Read log« of the number obtained by the multiplica- 
tion. 

(3) Subtract log* 10 or log* 100 (log« multiplier) from 
the C reading. 

The difference is the Naperian logarithm of the given 
number. 

Thus log. .585 = loge 5.85 - log* 10, 

= 1.767-2.303. 

Therefore log* .585 = T.464. 

II. (1) Move runner to given number on C and read 
its reciprocal on C/. 

(2) Read log« reciprocal. 

(3) Subtract the reading on C obtained in (2), from a 
number one greater than its integral figure. 

The difference thus obtained is the loge of the given 
number. 



Thus 


.585 = -J-. 




.585 


Therefore 


log. .585 = log«l log. ggg 




= O-log.1.71. 




= 0-.536 


Therefore 


log. .585= 1.464. 



Digitized by 



Google 



361 



THE SLIDE-RULE 



34^ 



361. Examples in Naperian Logarithms. Read, and 
diagram the reading for each example in the following table 
and fill in the omitted entries. 

Read examples 11 to 16 inclusive, both ways. 

Table XIX 
NAPERIAN LOGARITHMS 



No. 


Number. 


Logarithme 


No. 


Number. 


Logarithm^ 


1 


1.0356 




11 


.5 




2 


1.062 




12 


.125 




3 


1.75 




13 


.456 




4 


2.17 




14 


.795 




5 


e 




15 


.028 




6 


4.58 




16 


.064 




7 


34.5 




17 




1.85 


8 


455 




18 




.185 


9 


2175 




19 




.0455 


10 


3 78 




20 




.3315 



Digitized by 



Google 



CHAPTER XVII 
TRANSFORMATION OF FORMULAS 

352. Definition. A formula is a statement in significant 
symbols, of the exact mathematical relation of two or more 
quantities. 

Transformation is the process by which formulas are 
simplified, solved for any or all of their quantities, or other- 
wise changed according to mathematical authority. 

353. Method of Transformation. A point has now 
been reached in the study of this book where it is necessary 
to classify an equation before attempting to solve it. This 
is due to the fact that while, in preceding chapters, equations 
have been grouped under their various names so as to lessen 
the confusion of learning the method of solution for each 
kind, in this chapter hardly any two of the same kind are 
grouped together and all are technically, instead of mathe- 
matically, named. 

Special methods of transformation adapted to various 
conditions can be learned only by considerable practice. 
There is a general method and order of operations, however, 
which will reduce the work to a minimum. 

Briefly, the method is as follows: 

1. Observe where the unknown quantity is, in the 
formula; whether in the first member or the second or both, 
in numerator or denominator of a fraction, or under a radical, 
and if so, whether as a factor or a term. 

2. Classify the equation as simple, quadratic, etc. 

3. Apply the method of solution which will give the 
required transformation most directly and simply. 

350 



Digitized by 



Google 



364 TRANSFORMATION OF FORMULAS 351 

Clearance of fractions is not always essential to solution, 
and in many instances only increases the work. 

The one essential to the transformation of any formula 
is the determination of the coefficient of the unknown 
quantity. If this can be determined without clearance, 
as often happens, clearance should not be resorted to. 

2cr 
For example, 0G = — may be solved for r by multiplica- 
tion by ^, the inverted coefficient of r. 

76 5cb 
625——=— may be solved for b by transposing, factor- 

7 5c 
ing, and dividing by ^+^r which is the coefficient of b. 

As soon as possible all equations of this kind should be 
solved mentally without the use of the pencil. 

When elimination of one of the unknowns in one or 
more simultaneous formulas is required, the method of 
elimination by substitution is usually the simplest. 

When solving for any quantity all others are regarded as 
known. 

Thus, when a^—ac = \/b + l is to be solved for a, 
b and c are regarded as known quantities. 

If solution is required for 6, 

c and a are considered known. 

It takes a long time to become skilful in transformation, 
but it is well worth while for it is the key to the interpre- 
tation of much of the technical material in books and 
periodicals which help one to become intelligent and there- 
fore of ^eater service. 

354. Simplest Form of Result. A result is in its simplest 
form, generally speaking, when so written that each quantity 
is used as few times as possible. In other words, simplest 
form is the form requiring fewest substitutions of known values 
and the least work in computation. 



Digitized by 



Google 



S52 TECHNICAL ALGEBRA 866 

Illustration. Suppose a given formula has been solved 
for Q with the result: 

^ ri^+2rir+r^ 

If the values of ri and r are known, say ri = 5 and r = 8, 
Q can be determined by six substitutions and seven 
operations. 

If however the equation is reduced to the form 



Q 



-m- 



Q is determined by four substitutions and four operations. 

Sometimes when a formula is reduced to a form requiring 
fewest substitutions, more diflScult or longer operations are 
involved after substitution is made. 

The equation just solved may be reduced to a still, 
simpler form as follows: 

\ r\T ) \T\r r\r} \r r\) 

Only two substitutions are now required to determine 
Q, yet these might be offset by the increased time necessary 
to perform the indicated operations. With a slide-rule, or 
tables of reciprocals and squares, Q would be determined 
with Uttle effort, whatever the values of r\ and r. 

366. Problems. Enter each formula under the heading 
given and solve as directed: 

1. Total Heat of Vaporization. 

^=108L94+.305«. 
Solve for i, 

2. Latent Heat of Vaporization. 
Solve for q and r. 



Digitized by 



Google 



355 TRANSFORMATION OF FORMULAS ' 353 

3.. Net Pressure on the Piston. 

CA-OD^DA. 

Solve for OD. 

Find the value of OD 

when DA =45.3. 

and CA =60. 

4. Work m Foot-Pounds. 

W=PV. 
Solve for P and V. 

5. Real and Apparent Cut-Off. 

, h+i 

i+t 

Solve for i and ki. 

6. The Pantograph. 

AB^CD 
L CE' 

Solve for each term of both fractions. 

7. Indicated Horse-Power. 

jjj^JLAN 
33000 • 
Solve for LN. 

8. Total Ratio of Expansion. 

V 

Solve for e, F, and v. 

9. Stress. 

Solve for P and A. ' ■ < 



Digitized by 



Google 



354 


TECHl 


^ICA 


L 


10. 


Strun. 










8' 


e 
L 




Solve for e and L. 






11. 


Coefficient of Elasticity. 








E- 


_S 
s' 



355 



Substitute for S and s their values from 9 and 10 and 

solve for E. 
In the resulting formula, compute the value of E 

when e =L and il = 1 sq. in. 
12. Factor of Safety. 







P- 






Solve for /and -4. 






13. 


Pressure on a Pipe. 










pd- 


=2fcS. 




Solve for d, t, p, and S. 




14. 


Cylinders. 










P-- 


=j7rd»p 




Solve for d. 






16 


4 
Solve for d and p. 






16. 


St 
Solve for r and t. 







5 

Compute the value of S when p = 163.8, r=7, i=r-. 

o 



Digitized by 



Google 



865 TRANSFORMATION OF FORMUX.AS 355 

17. Pressure of a Mixture of Steam of Different Pressures. 

VP=vp+ViPi. 
Solve for P, ©, and pi. 

18. Ratio of Cylinders. 



V 
^=2.72-. 

V 



Solve for v and V. 

19. -=V^. 

V 

Solve for v and E, 

20. The Mean Ordinate of Tangential Pressure on the Crank- 

Pin. 

2SV 



Solve for SV. 
21. Resistance. 



AM^-^. 



KL 



Solve for d and L. 

22. Measurement of Resistance. 

dr=diR-\-dir, 
Solve for i?, and cxpro&s the result in the simplest form. 

23. Electrical Equivalent of Heat. 

rr = .000^77 ECT, 
Solve for C and T. 

24. Magnetic Field within the Solenoid. 

Solve for N and L. 



Digitized by 



Google 



356 O'ECHNICAL ALGEBRA 356 

26. Sum of the Moments of Horizontal Stresses. 

Af--Ar«. 
c 

Solve for 8,C,A, r, and ArK 

26. Moment of Inertia. 

I^ArK 

In 25, substitute the value of Ar* and solve for c. 

27. Bending Moment. 

7 ^' 

Solve for — and/. 
c 

«.f. 

Solve for L. 

29. Deflection of Beams. 

aWL* 

Solve for a, W, E, and L. 

30. Comparison of Strength and Stiffness of Beams. 

bd* 

D 

Show that W«=Mi*. 

31. Speed Cones. 

Solve for d, Ui, and ni. 

32. Locknuts. 



a=l}d-l 



Solve for d. 



Digitized by 



Google 



866 TRANSFORMATION OF FORMULAS 357 

S3. Proportion of Keys. 

4 

Find the value of t in terms of d, and solve the resulting 
equation for d. 

34. The Cylinder. 

a = 1.21D+2e+1.22. 

but e=.0003PD+.375. 

In the first equation, substitute the value of e from the 
second, and solve for D. 



36. 


The Piston. 
Solve for D. 


c=.18V2D-. 


1875' 


86. 


The Connectmg Rod. 








g-e' 








. .32a; 
*= h- 





The equations are simultaneous. Solve for i, eliminating h. 
37. The Law of the Lever. 

w U 

Solve for F, TT, L, and Li. 
88. Quality of Steam. 



W+R+w* 
Solve for 5. 



Digitized by 



Google 



358 TECHNICAL ALGEBRA 355 

39. Factor of Evaporation. 

W{H^t+32) 
966.1 
Solve for W and L 

40. Thickness of a Fire-Box Plate. 

Solve for t. 

41. Strength of the Head of a Boiler Shell. 

2r=p 

Solve for p, r, and t. 

42. Efficiency of a Riveted Joint. 

h-d 



2/=- 



h 



Solve for d a.nd h. 
43. Double-Riveted Lap Joints. 



•«)'■ 



Solve for d and t. 



44. Rate of Combustion. 

F=J2,25\/ll-l. 
Solve for H, 

46. Safety Valves. 

Tf d+TTi a+W2C -pAa =0. 

Solve for d and A. 

46. Stress in Punch and Shear Frames. 

Solve for Ct and P. 



Digitized by 



Google 



3G5 TRANSFORMATION OF FORMULAS 359 



47. 


Diameter of a Pipe. 




,^5^(/L+Jd)y. 




Solve for h, d, and Q. 


48. 


Strength of a Stayed Surface. 




-'!• 




Solve for t and h. 


49. 


Pitch of a Boiler Stay. 




a = .835d^. 




Solve for T and d. 


60. 


Diameter of a Direct Boiler Stay. 




Ap^^d'T. 




Solve for d. 


61. 


Belting. 

ws 

^-900- 



Solve for S and W. 
62. Quantity of Air for Complete Combustion. 



("-!)■ 



TF = 11.6C+34.8 
Solve for H. 

63. Thickness of Pipe. 

. St 

Solve for L 

64. Pitch of a Screw in Terms of Outside Diameter. 

V16D+IO -2.909 
^■" 16.64 

Solve for D, 



Digitized by 



Google 



360 TECHNICAL ALGEBRA 365 

66. The Area of a Segment of a Circle. 

Solve for r, h, and C. 

66. The Inclined Plane. 

_ Wh 

but 7 = what function of the angle a? 

Therefore F= what? 

67. Resistance Measured by a Wheatstone Bridge. 

Ciri =Ctrt. 
Cix ^CiR. 
These equations are simultaneous. 

Solve for — in terms of x and R, 

68. Electromotive Force of a Battery. 

E 






R+r 

E 
Ri+r 



These equations are simultaneous; E and r are unknown. 
Solve for E, eliminating r. 

69. ^=/(«+r). 

Ei=lR. 

Find the value of r in terms of everything except /. 
60. Space Traversed in any Second. 

Solve for t and g. 



Digitized by 



Google 



366 TRANSFORMATION OF FORMULAS 361 

61. The Screw. 

Solve for TT, P, and r. 

62. The Safety Valve. 

Apb—Qa 

Solve for W, p, and A. 

63. The Compound Geared Lathe. 
gchi 



n = - ... 
afj 



Solve for t . 



64. Temperature-Coefficient of Resistance. 

Solve for n. 
66. Location of a Fault in a Cable. 

x+y 
Solve for x. 

66. Winding of Armatures. 



y-l{i+'')- 



Solve for C, 6, p, and a. 
67. Unit Acceleration of a Pulley. 



Solve for w. 



Digitized by 



Google 



362 TECHNICAL ALGEBRA 366 

68. Coefficient of Expansion of a Gas at Constant Pressure. 

Vi = Vo(l+Bt,); 
V.^Voil+BU). 

These two equations are simultaneous. 
B and Vo are unknown. 
Solve for B, eliminating Fo. 

69. Horizontal Intensity of the Earth's Magnetic Field. 

(1) mhJ-^, 

(2) but ^ = ^(i+?)' 

M 1 
(3) and -pz =^r* tan 8, 

Divide (1) by (3). 

In the resulting equation, substitute the value of K, 

Solve for K, 

70. Problem. By substitution in the preceding formula com- 
pute the value of H to three decimal places when 



r=30 W 


= 17.22 


5=3° 10' L 


=49 


«=3.65 a 


= .09 


71. Current in Series. 




, Ens 
ms+R' 
Solve for n„ r, and R, 




72. Current in Multiple-Series. 




Up 





Solve for n,, tip^ r, and R. 



Digitized by 



Google 



855 TRANSFORMATION OF FORMULAS 363 

73. Water Required by a Condenser. 

W{ti-k)^H-U+32. 
Solve for TT, Uy and H. 

74. Potential Difference. 

p n 

(1) 

(2) 

(3) 

Solve (1) and (3) as simultaneous equations, for P.D., 

eliminating /. 
In the resulting equation, substitute the value of r from 

(2), and simplify. 



/= 


' R ' 


/. 


E-P.D. 


r 


1- 


E 



76. 


Fly-Wheel Computation 








yo= 


2 ' 






E = 


y. * 




Solve for EV^K 






76. 


The Diameter of a 


Wire 








D = 


w 

.7854dL* 




Solve for W and L. 







77. Capacity of a Vessel Whose Form is the Frustum of a 
Cone. 



C=^(/2«+r«+/2r). 



Solve for R and r. 



Digitized by 



Google 



364 TECHNICAL ALGEBRA 866 

78. Velocity of Sound in terms of Temperature. 

F=333(l+.00370*. 
Solve for t. 

79. Measurement of Resistance of an Electrical Conductor. 

R+r+g+X tan a 
B'+r+g "tan o'* 



C' = 



B+r+g+X' 

E 

R'+r+g 



C 
Solve for —, in terms of tan a and tan o'. 

80. Measurement of Electromotive Force of a Battery. 

. _E E' 

• R+r+g R''+r'+g' 

E E' 



R+R'+r+g R-+R^^'+r'+g 


Prove 


^- R" 


81. Radius of Curvature of a Spherical Mirror. 




a^b R- 


Solve for R. 




82. 


"-6A+2- 


Solve for h. 





83. Thompson's Method of Comparison of Capacities of 
Condensers. 

Show that §=-'. 



Digitized by 



Google 



366 TRANSFORMATION OF FORMULAS 365 

84. Fall of Potential in a Series D3mamo. 

E^e+I{Ra+Rf). 
Solve for I and Ra. 

85. Air-Gap in a Dynamo. 

1,26^. 
a ^^ XA, 

In the first equation substitute the value of No from the 
second, and solve for A. 

86. Speed of a Continuous Current Motor. , 

2TTr:^ZNE-nZ*NK 
Solve for n, and ZN. 

87. Current in a Series Motor. 

c-f-o. 

Solve for C. 

88. Focal Distance of a Concave Spherical Mirror. 

— "T" — = — . 

ajb r 

Solve for /. 

89. Radius of Curvature of a Convex Spherical Mirror. 

a ^ jR 
b 'c+iR' 
Solve for R. 

90. Principal Focal Distance of a Convex Lens. 
Solve for r'. 



Digitized by 



Google 



366 TECHNICAL ALGEBRA 355 

91. Magnifying Power of a Lens. (Approximate formula.) 

Solve for /. 

92. The Focal Length of a Lens. 

V Pi 



Prove t / = 



4L • 



93. The Catenary. 

Transpose so that the radical is the only term in one of 
the members; then square to remove the radical, and 
solve for m. 

94. Stress Due to Longitudinal Tension. 

R f-fi ' 

Solve for/i. 



96. Wear of Wheels. 

& 
Solve for Ru 

96. Stress in a Rectangular Plate. 



Ri \Ri R2/ 



•' 2\L*-\-h*)t^ 



Solve for L. 



Digitized by 



Google 



366 TRANSFORMATION OF FORMULAS 367 

97. Effect of Temperature on Resistance. 



K = 



Solve for n. 
98. Cable Testing. 



riit^-tiY 



X+y^r". 

These equations are simultaneous. 

Solve for X and y in terms of r, r', and r". 

99. Thickness of Web of an I Beam. 

A^td+2s{b-t) + ^^~^lQl 
Solve for t and b. 

100. Girder of two Spans with Uniformly Distributed Load. 



^•-2r-4L(L+L.)J- 



Solve for L and Li. 

101. Radius of Gyration of a Ship. 

Solve for TT, D, and L. 

102. Diameter of Rivets in a Double-Riveted Joint. 

Uird^ = lSt{P-d). 
Solve for d and P. 

103. Ratio of Expansion. 

tt 
Solve forr^""^ and r. 



JXl. 



Digitized by 



Google 



368 TECHNICAL ALGEBRA 355 

104. Efficiency of Camot's Engine. 

^^ . Work done 

Efficiency =^~T n- 

Heat expended 

Work done =Heat expended -heat rejected. 

Heat expended ^cUloggr. 

Heat rejected ^cUhg^i, 

But fj =r. 

Derive the formula of efficiency. 

For highest efficiency what must be the value of 1%^ the 
temperature of the condenser? 

105. The Rankine-Gordon Formula for Colimins. 



P 



S--7+S,, 



Show that 



Mc 

M^Pf, 
I=Ar*, 

P S 



A ,,!,«• 



106. Internal Diameter of a Hollow Column. 
P_ S 



r'=^(i>'+d*). 



In the first equation, substitute for A and r» and solve for d. 



Digitized by 



Google 



S6S TRANSFORMATION OF FORMULAS 369 

107. Stresses Due to Impact. 

yQ 

e P' 

Solve these simultaneous equations for Q and y 

In the formulas obt^ed find the values of Q and y, 

when h=0; 

also when h =4e and 12e. 

108. Shrinkage of Hoops. 

S 

(D-d) 

Substitute for 8 from the second equation and solve for E. 

109. Elastic Resilience of Bars. 

K^lPe. 

PL 

P=SAf and €=-7-=. 

Alii 

Show that ^=^^- 

110. The Area of a Triangle. 

^w (,-16— ;• 

Solve for a, 6, and c. 

111. The Area of a Trapezoid. 

Solve for a, h, and h\ 



Digitized by 



Google 



} 



370 TECHNICAL ALGEBRA 366 

112. The Perimeter of an Ellipse. (Approximate Formula.) 



p /P'+d' (D-d)' 
^='\ 2 -"lis"- 



Solve for D and d. 

113. The Length of a Chord. 

Solve for r and h. 

114. The Helix. 



Solve for L, d, and n. 
116. The Surface of a Cone. 

^=|[L(D+d)+i(D«+d«)]. 

Solve for D, //, and d. 

116. The Volume of a Wedge. 

V=lwh{a+b+c). 



Solve for h and c. 

117. The Volume of a Prismoid. 

Solve for L and m. 

118. The Volume of the Frustum of a Regular Pyramid. 

V ^lh{A+a+VAa). 

^ o 

Solve for h and A. 



Digitized by 



Google 



355 TRANSFORMATION OF FORMULAS 

119. The Length of a Spiral. 

Solve for t and R, 

120. The Mean Velocity of Discharge of Water in Pipes. 



371 



4 



7m =2.315 
Solve for d and L. 
121. The Diameter of a Shaft. 



hd 



fL+125d' 



Solve for c and N. 



-Kil 



122. Relation of Temperature to Pressure. 



U2\ 

to \po/ 



0-1 




Compute the value of t when g = 1.408, to =520°, 

p=30, po=15. 



123. 


Work Done by an Engine. 






Work done =J(H-h), 








Work done =what in terms of all except H? 


124. 


Pressure with Clearance. 




--I 


\^*-y] 






[.+,T,J 






Solve for v and V. 







Digitized by 



Google 



. 372 TECHNICAL ALGEBRA 856 

125. Point of Cut-Off. 

vB „ vB 
Initial pressure =-^+Bpz —^, 

£jZ £i 

In the first equation substitute the value of z from the 
second, and prove 

(2\/^ l\ 
F /• 

126. Centers of the Valve Circles in Link Motions. 



'4F''+C'ir)*'°«4 



X-- 
Solve for k. 

127. Exposed Surface of Cylinders. 

Simplify this formula and solve for r and d. 

128. Inductance of Transmission Lines. 

L=^(9.211og..f+l). 

Solve for Lu, and logio S. 

129. Pressure of Saturated Steam. 

4079 71 
log p= 10.515354 - Z,' -0.00405096!r+0.ai39J9647'». 

Compute p when !r = 575°. 

130. The Specific Volume of Saturated Steam. 

p7"=475. 

Solve for log V, 
Evaluate V when p = 129. 



Digitized by 



Google 



356 TRANSFORMATION OF FORMULAS 373 

131. Mean Efiiective Pressure. 

_ l+hyplogjR 
V =P ^ . 

Compute the value of p when 
p'=125, 
and 12=3. 

132. Thickness of a Boiler Tube or Flue. 

^.18 

p=960000(H-r. 
La 

in which L, d, and ^ are all in inches. 

Compute the value of t when p = 170 lbs. per sq. in. 

L =3i feet. 

d =«4i inches. 

133. Breadth of a Rectangular Column. 

^= — ^ — • 

Solve for L, 

134. Effective Pull of a Belt. 

T 
hyplog^=/a, 

a=27m, 

P^Tx-Tt. 

T 
Compute ^ when / = .3 and n = .464, 

and find P in terms of Ti. 

135. The Area of a Parallelogram. 

The diagonals of a parallelogram are 81 and 106, and 

cross each other at an angle of 20° 18'. 
Formulate and compute the area of the parallelogram. 



Digitized by 



Google 



374 



TECHNICAL ALGEBRA 



366 



136. Height of a Weighted Pendulum Governor. 

BxXH =(B+Pr)r, 
but Bx=XmMBrN\ 
Solve for H in terms of PT, JB, and N*. 

137. The Equation of the Ellipse from the Trammel. 




Fig. 141. 

A trammel is a bar fitted with a pencil at D, and with pins 
at F and G which slide in grooved pieces at right 
angles to each other. 

DF =6, the semi-minor axis of ellipse, 

DG =a, the semi-major axis of ellipse. 

The pencil Z>, in one complete revolution describes an 
eUipse. 



Prove 



—+— = 1. 



Write proof in full, with authorities. 
138. Strength of a Riveted Joint. 

{12 ^nd)tSi 



P=~ 



Zird^Sz 



12 



Find d in terms of t, h, Si, and St, 



Digitized by 



Google 



366 TRANSFORMATION OF FORMULAS 375 

139. Deflection of a Magnetic Needle. 

e^^iB-A)[c-\iA+B)]. 
Solve for B and m. 

140. Potential of Needle in a Quadrant Electrometer. 



Solve for C. 
141. Charge of a Leyden Jar. 



^-i^A 



i+i+i 

Ci C2 Ci 



Solve for C2, and V, 

142. Force between Parallel Electrified Plates. 

Solve for 27rs*, the force per unit area. 

143. Excess of Potential in Parallel Plates. 

V=iirPh+^+iTP[d - {h+t)]. 

Simplify the second member. 

144. Energy per Unit Area. 

Using the equation in problem 142 derive a formula for 
Js7, the energy per unit area of the plates. 

145. Electric Capacity of Spheres. 
Simplify the second member. 



Digitized by 



Google 



376 TECHNICAL ALGEBRA 355 

146. Difiference of Electric Energy in Leyden Jars. 






Add the fractions, expressing the result in the simplest 
form. ^ 

147. Refraction of Lines of Force. 

Ri sin 01 =Ri sin 62. 
KiRi cos 61 KjRi cos 6^ 

Derive the simplest formula for tan ^1. 

148. Magnetic Force from a Small Magnet. 

][f . 

i2=— V4cos*^+sin«^. 
ft 

Substitute for sin 6 in terms of cos 6 and simplify. 

149. Equilibrium of Couples. 

HM' sm 6 = -^ . 

Solve for tan 6, 

150. Difference of Magnetic Potentials. 

M 
-Oi — ^(cos Bi -cos ^2), 

Z = cos 01, 

ft 

Zi = — COS 6t. 

ft 

Solve for -Oi in terms of Z, Zi, and r. 



Digitized by 



Google 



856 TRANSFORMATION OF FORMULAS 377 

161. Work done by an Electric Field. 



'^"oe^^^^-^Q)- 



Simplify the second member. 
162. Charges of two Concentric Spherical Shells. 

a 



Solve these simultaneous equations for Ei and Eu] 



Solve these simultaneous equations for Ei and J^j.^ 
154. Total Charge on Two Concentric Spheres. 



-^(-D- 



V- 

Solve for c' and c. 
155. Relation of Temperature to Heat. 



Qi-Q* 



Ti 



Determine the relation of Q to T and express the result 
with the variation symbol. 



Digitized by 



Google 



378 TECHNICAL ALGEBRA 

166. Measurement of Coefficient of Self-Induction. 



SJ6 




Fig. 142. 

Derive the formula for cos ft in terms of E^ Ety and Eu 
157. Capacity of a Condenser. 



C = 



( 



i^log.- 



Solve for logio — . 
168. Measurement of Power. 



-H'-i^y-'-] 





Solve for i^i. 


159. 


A Girder Stay. 




phi* SM^ 
8 " 6/ • 




Compute the value of d when 




V = 18000 and b=\d. 
f 4 


160. 


Effect of Variation of Resistance. 




r ^ 




Vr^+L*w* 




tantf=-. 



In the first equation, substitute the value of Lw from the 
second, and solve for tan 6, 



Digitized by 



Google 



365 TRANSFORMATION OF FORMULAS 

161. Coefficient of Self-induction. 
/= current, 

E = electromotive force, 
I/W?= reactance, 
/?= resistance, 
A B C issi right triangle. 



LW 



379 




The ratio of the electromotive force to the current equals 
the impedance. 

Write the formula for the current in terms of electro- 
motive force, resistance, and reactance. 

162. General Formula for Tangent Galvanometer. 

ff tan 8 

2wri^i 2'Kr<i^n2 

Solve for tan 3. 

163. Economic Coefficient in a Series Dynamo. 

R 



V = 



R+ra+Tm 



Solve for R. 



164. Economic Coefficient in a Shunt Dynamo. 

1 



^ = 



■^^^¥ 



Solve for r„ and ra. 



Digitized by 



Google 



380 TECHNICAL ALGEBRA 866 

166. Efficiency of a Motor. 

Wt—We-jWi+Wf) 
"-" Ws 

Solve for W,. 

166. Loss Through Heating of Conductors in a Shunt Dynamo. 

(E\t E* 
^-rJ ^+Ri 
Solve for E and R/. 

167. Theoretical Height of Chimney for Given Draft. 

7.65 7.65' 

Ta Tc 

Solve for Tc. 

168. Effective Area of a Chimney. 

Vh 
Solve for H and A. 

169. Twisting Moment of a Shaft. 



Ti^B+Vb^+TK 
Solve for B. 

170. Mutual Induction. 

4^SM 






L ' 



Determine the relation of L to S and use the variation 
symbol in the result. 



Digitized by 



Google 



366 TRANSFORMATION OF FORMULAS 381 

"1. |=Z. 

Substitute for B in formula 169, and solve the resulting 
equation for Z and T. 

172. Dimensions of a Journal. 



Solve for P and p. 
173. Bending Moment of a Wall Bracket. 



Solve for h. 


S^bh'. 
2* 


174. Solid 


Flange Coupling. 


(l) 






(2) 




2d=B-i 


(3) 







In the second equation, substitute the value of d from 

the first and solve the resulting equation for B, 
In the third, substitute for B and solve for d. 

176. Outside Diameter of a Hollow Shaft. 

di*-d2* 



d^~ 



A 



d. 



Prove di=-d 



31 1 



l-m* 



Digitized by 



Google 



3S2 TECHNICAL ALGEBRA 366 

176. Strength of Gear Teeth. 

, Sbt^ 

but t=-C, 

L = .7(7. 
In the first equation substitute these values and solve for C. 

177. Power Transmitted by a Rope. 






Solve for Z>. 
178. Diameter of a Pi&ton-Rod. 
IOttD^ 



*=.(|)'xf. 



4 
Solve for d, 

179. Per Cent of Moisture in Steam. 

Solve lot h, T, and Ti. 

180. Strength of a Stayed Surface. 
C(16^+l)* 



p=- 



s-6 



Solve for t. 



181. Intensity of Shearing. 

(r-p) cos ^=5 sin B. 

r sin ^=5 cos B. 

Solve for 5* in terms of r and p. 
Solve the q^ formula for r. 



Digitized by 



Google 



355 TRANSFORMATION OF FORMULAS 383 

182. Centrifugal Force of the Rim of a Fly-Wheel. 

Wihw^ 



but h = 

and w = 



g ' 
Rn sin a 



IT 

V 



R' 

Therefore F= what when A and w are eliminated? 
Solve for sin a. 

183. Direct Tension on Each Arm of a Fly-Wheel. 

frgR 2gr 

Solve for TFi and W2. 

184. Breaking Strength. 



A 
Solve for K. 






185. Resistance of a Riveted Joint. 

1 . A2L-dYiU 

Solve for d. 

186. Coefficient of Linear Expansion of a Solid. 

Solve for L and K. 

187. Stability of a Shaft. 

Solve for L, T, E, and /. 



Digitized by 



Google 



384 TECHNICAL ALGEBRA 865 

188. A Boiler Plate. 

Solve for D, 

189. Length of a Journal. 

a 
Solve for P. 

190. Strength of a Cotter. 

Solve for 7), d, and L 

191. Friction of a Pivot. 



|d.A=^+l^iVd. 



Solve for d. 



192. Friction of Worm and Wheel. 

l+/xtanfe 
tan^— /i ' 
Solve for tan ^i. 

193. Strength of a Crank. 

Solve for A and m. 

194. Radius of a Cast-Iron Spring-Ring of Unequal Thickness. 

11 24pr«8in«g 

Solve for t and p. 



Digitized by 



Google 



366 



TRANSFORMATION OF FORMULAS 



385 



195. Strength of Teeth. 

Solve for p and P. 

196. Equation of the Catenary. 




Fig. 144. 

^ 8 I ? 

Solve for s. 
197. Forces on the Crank Pin. 



tmax=P^ 



Solve for L, 



198. Sieman's Method of Comparison of Capacities of Con- 
densers. 



Solve for Ci. 


•-'^"Vc+cy 


199. Law of Deflection 


. 


Given 


/ PO 
H RO' 




PO=AR, 








f=2Mr: 


Prove y 


' 1 
M=^fftan5. 



Digitized by 



Google 



386 TECHNICAL ALGEBRA 355 

200. Distribution of Magnetic Force on the Earth's Surface. 
Approximate law: 

Magnetic force oc V L+3 sin« L. 

Express this law as an equation, denoting magnetic fore 3 
hyH. 

201. Induced Electromotive Force in a Closed Circuit. 



E = 
C 



t 
E 



Solve these two simultaneous equations for C, eliminating E, 

202. Electric Potential. 

Fp-Fg=£-(n-r). 
rri 

Simplify the second member. 

203. Force Exerted by a Charged Sphere. 

Find the value of / in terms of tt, r, and p. 

204. M. Love's Formula for Pressure. 

p =5358150 -^+41900^^-1323 1. 
Solve for t and d. 

205. Capacity in Electrostatic Units of two Concentric 

Cylinders. 

Solve for logio r'. Y^°f^ r I 



Digitized by 



Google 



356 TRANSFORMATION OF FORMULAS 387 

206. Difference of Potentials in a Special Condenser. 











Vn- ^ , 






K- ^ 






Solve for K in terms of r and r'. 




207. 


Capacity of two Condensers Joined in 


Series. 



Solve for — in terms of Ki and K%, 



208. Path of a Projectile. (Equation of Trajectory.) 

a:=dcos ao, 

(1) Solve for y when t is eliminated. 
But c=\/2i^. 

(2) Find y when c is eliminated. 

(3) Let J/ = 0, and solve for x, 

209. The Connecting-Rod. 

But n = .155D+.0623. 

Solve for L eliminating n. 



Digitized by 



Google 



388 TECHNICAL ALGEBRA 366 

210. Stress on a Plate. 

Solve for h. 



211. Tension in a Bolt. 



Q 



2 1 ( p+nird \ 
3^'^2\Td-tJLpJ 



R 
d' 



Solve for d and p. 

212. Velocity of a Projectile. 

VFTjTfr, ^ \/c*-2ctgBmao+gHK 
Solve for t, c, and g, 

213. The Attracted-Disk Electrometer. 

Difference of potential = work, 
But difference of potential = Fi — Vt, 
And work = force times distance, 
But force = attraction due to fixed plate 

plus repulsion due to movable plate. 
Attraction due to fixed plate =27rp. 
Repulsion due to movable plate =27rp. 
Distance =D. 
Write the formula for Vi — Fj. 

F=:2TpXSp, 

In the formula, substitute the value of p from the last 
equation and simplify. 

214 Formula for Economizers. 



x = 



30 , /SW+GC\ • 
-^+[-200-)' 



Solve for y, C, and a. 



Digitized by 



Google 



366 TRANSFORMATION OF FORMULAS 389 

215. The Heat Balance Equation. 

{W+w)T^W(L+ti)+wt. 
Solve for W, w, U, and L. 

216. Mean Pressure on the Piston. 

Wt^PaLid+hyplogR). 
Solve for p eliminating aLi, and W^ 

217. Adiabatic Expansion of Air. 

pyl.iOS ^p^ylAOB 

Write the formula for log P. 

218. Work done by Air in Adiabatic Expansion. 

pT^l.408 —py 1'408 

Solve for L when Pi is eliminated. 

219. Ratio of Absolute Temperatures of Air. 

PVPrVi 

T Ti' 
Eliminate V and Vi through the use of the first formula 

T 
in 217 and solve for — . 
ii 

220. Relation of Temperatures in Adiabatic Expansion. 

ftFs'^PiFA 



Show that P2F2=PiFi 



©'""• 



Digitized by 



Google 



390 TECHNICAL ALGEBRA 356 

221. Length of a Second's Pendulum. 



'^' 



in which r= period in seconds, 

i = length of pendulum in feet, 
g = acceleration due to gravity. 

The period of a pendulum is the time required for one beat, 
which is the time from the highest point on one side to the highest 
point on the other. The period for any given length depends 
on the latitude and the elevation above sea-level. 

In the latitude of New York the force of gravity at the sea- 
level is 32.14. 

By substitution in the formula, determine the period of a 
pendulum 39.1 inches long, in New York City. 

Use 6-place table and check by the slide-rule. 

222. Expected Mean Effective Pressure. 
M.E.P. 



F = — -. . 

p,j,+(,+c)l0ge^^|-B.P. 

(1) Solve for M.E.P. 

(2) Compute the M.E.P. 

when F = .8, 

r = .118, 
B.P.=5, 
Pi =160, 
c = .06. 

(3) Formulate the law in problem 13, paragraph 37. 

(4) In the formula obtained in (3), substitute the M.E.P. as 
computed in (2), and compute the expected indicated horse- 
power 

when L = 36 inches , 

r.p.m. =85, 

Z)* = 16 inches. 

* D denotes the diameter of the piston. 



Digitized by 



Google 



REFERENCE TABLES 



391 



TABLE I.— LENGTH 
United States and English Linear Measure 



Quantity. 


Unit. 


Symbol. 


12 inches 
3 ft. 

5iyd. 
16i ft. 

40 rd. 
8 fur. 1 
320 rd. 
5280 ft. J 


Ifoot 

1 yard 
r Irod 

Ipole 
I 1 perch 

1 furlong* 

1 mile 


ft. 

yd. 

rd. 
P. 

fur. 

mi. 



* Seldom used. 



Surveyor's Long Measure 



Quantity. 


Unit. 


Symbol. 


7.92 inches 
25 li. 
4rd. \ 
66 ft./ 
80 ch. 


Uink 
1 rod 

1 chain 

1 mile 


li. 

rd. 
ch. 
mi. 



Nautical Measure 



Quantity. 


Unit. 


6 feet 


1 fathom 


120 fathoms 


1 cable-length 


7J cable-lengths \ 
880 fathoms / 


1 statute mile 


1 . 153 statute miles ] 




1 minute of circum- \ 


1 geographic or nautical mile 


ference of earth J 




3 geographic miles 


1 league 


20 leagues | 
60 geographic miles [ 


1 degree of earth's circumfer- 


69.16 statute miles J 




Iknot 


1 nautical mile per hour. 



Digitized by 



Google 



392 



REFERENCE TABLES 



TABLE L^Continued. 
Metric Linear Measure 



1 meter » 39.37 inches. 




Quantity. 


Unit. 


Symbol. 


10 mm.* 


1 centimeter 


cm. 


10 cm. 


1 decimeter 


dm. 


10 dm. 


1 meter 


m. 


10 m. 


1 dekameter 


Dm. 


10 Dm. 


1 hectameter 


Hm. 


10 Hm. 


1 kilometer 


Km. 


10 Km. 


1 myriameter 


Mm. 



* mm. denotes millimeters. 

TABLE XL— AREA ^ 
United States and English Square Measure 



Quantity. 


Unit. 


144 square inches 

9 sq. ft. 

30J sq. yd. \ 

272J sq. ft. / 

160 sq. rd. \ 

43560 sq. ft. / 


1 square foot 
1 square yard 

1 square rod 

1 acre 

1 



Surveyor's 


Square Measure 


Quantity. 


Unit. 


625 sq. li. 




1 sq. rd. 


16 sq. rd. 




1 sq. ch. 


10 sq. ch. 




1 acre 


640 A. 




1 sq. mile 


36 sq. mi. 




1 township 
(U. S. pubUc lands) 



Metric Square Measure 


Quantity. 


Unit. 


100 sq. mm. 
100 sq. cm. 
100 sq. dc. 
100 sq. m. 
100 sq. Dk. 
100 sq. Hk. 
100 sq. Km. 


1 sq. centimeter 
1 sq. decimeter 
1 sq. meter 
1 sq. decameter 
1 sq. hektometer 
1 sq. kilometer 
1 sq. myriameter 



Digitized by 



Google 



REFERENCE TABLES 

'table III.— VOLUME 
Cubic Measure 



393 



Quantity. 


Unit. 


1728 cubic inches 
27 cu. ft. 
24i cu. ft.* 


1 CU. ft. 

1 CU. yd. 

1 perch* (masonry) 



* In some states. Not a legal standard. Should always be specified. 
Dry Measure 



Quantity. 


Unit. 


2 pints 

8qt. 

4pk. \ 
2150.42 cu. in. / 

2688 cu. in. 
2218.19 cu. in. 


1 quart 
1 peck 

^ g 1 bushel (struck) 

i 1 bushel (heaped) 
Eng. 1 bushel (struck) 



Liquid Measure 



Quantity. 


Unit. 


4 gills 
2pt. 

4qt. \ 
231 cu. in. / 
277 J cu. in. 
31i gal. 

2 bbl. \ 
63 gal. J 
42 gal. 


1 pint 
1 quart 

1 gallon (U. S.) 

1 Imperial gallon (Eng.)* 
1 barrelt 

1 hogshead 

1 bbl. refined oil 



♦ The exact imperial gallon has a capacity of 277.274 cubic inches, 
t The sise of a barrel or cask varies to such an extent that the capacity in gallons 
is sometimes stamped on the outside. 



Digitized by 



Google 



394 



REFERENCE TABLES 



TABLE IV.— WEIGHT 
Avoirdupois Weight 



Quantity. 


Unit. 


Symbol. 


16 drams 


1 ounce 


oz. 


16 oz. 


1 pound 


lb. 


100 lbs. 


1 hundredweight 


cwt. 


20 cwt. \ 
2000 lbs. / 


1 short ton (U. S.) 


T. 


112 lbs. 


1 quarter (Eng.) 


qr. 


20 qrs. \ 
2240 lbs. / 


1 long ton (Eng.) 


T. 


2204.6 lbs. 


1 metric ton 


T. 



Troy Weight 



Quantity. 


Unit. 


Symbol. 


24 grains 
20 pwt. 
12 oz. \ 
6760 grains J 


1 pennjrweight 
1 ounce 

1 pound 


pwt. 
oz. 

lb. 



Volume and Weight of Water 



Unit. 


Equivalent. 


Approximate. 


Exact. 


1 gal. (U. S.) 

1 cu.ft. 

1 liter ] 
1 kilogram 
1 kilo J 


/ .134cu.ft. 
I 8i lbs. 
/ 7i gal. 
I 62.4 lb. 

2.2 1b. 


.13368 cu.ft. 
8.3356* lb. 
7.480517 gal. 
62.35471* lb. 

2.204622tlb. 



* At 62° Fahrenheit, barometer 30". 
t By Act of Congress. Distilled (pure) water at maximum density, barom- 
eter 30". 



Digitized by 



Google 



REFERENCE TABLES 



395 



TABLE v.— DECIMAL EQUIVALENTS OF PARTS OF AN 

INCH 



Fraction. 


Decimal. 


Fraction. 


Decimal. 


Fraction. 


Decimal. 


A 


.01563 


a 


.32813 


*f 


.70313 


ii 


.03125 


a 


.34375 


a 


.71875 


A 


.04688 


a 


.35938 


a 


.73438 


1-16 


.0625 


S-8 


.375 


3-4 


.75 


A 


.07813 


a 


.39063 


a 


.76563 


A 


.09375 


a 


.40625 


a 


.78125 


A 


.10938 


u 


.42188 


a 


.79688 


1-8 


.125 


7-16 


.4375 


13-16 


.8125 


A 


.14063 


a 


.45313 


a 


.82813 


A 


.15625 


a 


.46875 


a 


.84375 


il 


.17188 


H 


.48438 


a 


.85938 


3-16 


.1875 


1-2 


.5 


7-8 


.875 


« 


.20313 


ji' 


.51563 


a 


.89063 


A 


.21875 


a 


.53125 


a 


.90625 


U 


.23438 


a 


.54688 


a 


.92188 


1-4 


.25 


9-16 


.5625 


16-16 


.9375 


H 


.26563 


a 


.57813 


a 


.95313 


A 


.28125 


a 


.59375 


tt 


'^. 96875 


« 


.29688 


a 


.60938 


a 


.98438 


6-16 


.3125 


6-8 

a 

11-16 


.625 

.64063 
.65625 
.67188 
.6875 


1 


1.00000 



Digitized by 



Google 



396 i^' ^c^-" EEFERENCE TABLES 



TABLE VL— U 


. S. AND METRIC 


EQUIVALENTS 




Equivalent. 


Unit. 


Approximate. 


Exact. 


1 acre 


. 40 hectares 


.4047 


1 bushel 


35 i liters 


35.24 


1 cm. 


.39 in. 


.3937 


Ice. 


.Oo cu. in. 


.0610 


1 cu. ft. 


.028 cum. 


.0283 


1 cu. in. 


16.4 cc. 


16.387 


1 cum. 


35.3 cu. ft. 


35.31 


1 cum. 


1.3 cu. yd. 


1.308 


1 cu. yd. . 


. 76 cum. 


.7645 


1ft. 


30i cm. 


30.48 


1 gal. (U. S.) 


3.8 Uters 


3.785 


1 grain 


065 g. 


.0648 


1 gram 
1 hectare 


151 gr. 


15. 4 i 


2 . 5 acres 


2.471 


1 inch 


2.5 cm. 


2.54 


Ikilo 


2.2 lbs. 


2.205 


1km. 


.62 mile 


.6214 


1 liter 


.91 qt. (dry) 
l.lqt. (liq.) 


' .9081 


1 liter 


1,057 


1 meter 


3.3 ft. 


3.281 


1 mile 


1.6 km. 


1.6093 


1 mm. 


.039 in. 


.03937 


1 oz. (avoir.) 


28i g. 


28.35 


1 oz. (troy) 


31 g. 


31.10 


1 peck 


8.8 liter 


8.809 


1 pint (liq.) 


.47 1. 


.4732 


1 pound 


.45 kg. 


.4536 


1 qt. (dry) 


1.1 1. 


1.101 


1 qt. (liq.) 


.95 1. 


.9464 


1 scm. 


. 16 sq. in. 


.1550 


1 sq. ft. 


.093 sm. 


.0929 


1 sq. in. 


6 . 5 scm. 


6.452 


1 §q. mile 


260 Ha. 


259. 


1 sm. 


1.2 sq.yd. 


1.196 


1 sm. 


11 sq. ft. 


10.76 


1 sq. rod 


25.3 sm. 


25.293 


1 sq. yd. 


.84 sm. 


.8361 


1 ton (U. S.) 


.91 m. ton 


.9072 


1 ton (Eng.) 


1 m. ton 


1.017 


1 ton (metric) 


1.1 t. (U. S.) 


1.102 


1 ton (metric) 


.98 t. (Eng.) 


.9842 


1yd. 


.91 m. 


.9144 



See also, page 400. 



Digitized by 



Google 



REFERENCE TABLES 



397 



rABLE VII.— INTERNATIONAL ATOMIC WEIGHTS, 1916 



A Argon 


... 39.88 


Mo Molybdenum. . . 


.. 96.0 


Ag Silver 


... 107.88 


N Nitrogen 


.. 14.01 


Al Aluminum 

As Arsenic 


... 27.1 
... 74.96 


Na Sodium 


.. 23.00 


Nd Neodymium 


.. 144.3 


Au Gold 


... 197.2 


Ne Neon 


.. 20.2 


B Boron 


... 11.0 


Ni Nickel 


.. 68.68 


Ba Barium 


... 137.37 


Nt Niton 


.. 222.4 


Bi Bismuth 


... 208.0 


O Oxygen 


.. 16.00 


Br Bromine 


... 79.92 


Os Osmium 


.. 190.9 


C Carbon 


... 12.005 


P Phosphorus 


.. 31.04 


Ca Calcium 


... 40.07 


Pb Lead... 


.. 207.20 


Cb Colimibimn. . . . 


... 93.5 


Pd Palladium 


.. 106.7 


Cd Cadmium 


... 112.40 


Pr Praseodymium . . 


.. 140.9 


Ce Cerium 


... 140.25 


Pt Platinum 


.. 195.2 


CI Chlorine 


... 35.46 


Ra Radium 


.. 226. 


Co Cobalt 


... 68.97 


Rb Rubidium 


.. 85.45 


Cr Chromium 


... 62.0 


Rh Rhodium 


.. 102.9 


Cs Caesium 


... 132.81 


Ru Ruthenium 


.. 101.7 


Cu Copper 


... 63.57 


S Sulphur 


.. 32.06 


Dy Dysprosium. .. 


... 162.5 


Sa Samarium 


.. 150.4 


Er Erbium 


... 167.7 ' 


Sb Antunony 


.. 120.2 


Eu Europium 


... 152.0 


Sc Scandium 


.. 44.1 


F Fluorine 


... 19.0 


Se Selenium 


.. 79.2 


Fe Iron 


... 65.84 


Si Silicon 


.. 28.3 


Ga Gallium 


... 69.9 


Sn Tin 


.. 118.7 


Gd Gadolinium 


... 157.3 


Sr Strontium 


.. 87.63 


Ge Germanium. . . . 


... 72.6 


Ta Tantalum 


.. 181.1 


Gl Glucinum 


... 9.1 


Tb Terbium 


.. 159.2 


H Hydrogen 


... 1.008 


Te Tellurium 


.. 127.5 


He Helium 


... 4.00 


Th Thorium 


.. 232.4 


He Mercurv 


... 200.6 


Ti Titanium 


. . 48.1 


Ho Holmium 


... 163.6 


Tl Thallium 


.. 204.0 


I Iodine 

In Indium 


... 126.92 
... 114.8 


Tm Thurium 


.. 168.5 
.. 238.2 


U Uranium 


Ir Iridium 


... 193.1 


V Vanadium 


.. 61.0 


K Potassium 


... 39.10 


W Tungsten 


.. 184.0 


Kr Krjrpton 


... 82.92 


Xe Xenon 


.. 130.2 


La Lanthanum. . . . 


... 139.0 


Yt Yttrium 


.. 88.7 


Li Lithium 


... 6.94 


Yb Ytterbium 


.. 173.5 


Lu Lutecium 


... 175.0 


Zn Zinc 


.. 65.37 


Mg Magnesium 


... 24.32 


Zr Zirconium 


.. 90.6 


Mn Manganese .... 


. .. 64.93 







Arranged Alphabetically According to Symbols 
By W. A. Ballou 



Digitized by 



Google 



398 



BEFERENCE TABLES 






(X4 



§ 



O 

H 

a 



o 

Q 
O 

2 









0. i 






." 






p . 



Pi 
o 



op 

P<0 



•c 

CO 









M 




«S5 


? 




2s 


B 




^8 


S 






O 




"1 
& 





lil 








sfi 




-.^ 


G 




Si 


■"ft 


Sg 


IS 




li 



S2 
I- 



Il 



2z 



01 



ir 

lis 



as 
It 

►^3 



•SI 



iS 



ss 



§o 



|3 



I! 
1^ 



lis 

§^ I 

a. ^ 



Be 

¥ 

8i 



I? 



S2 



6!9 



Zi 



Is 
«3 






S" 



a?"? 



Sod 



S I 



a- 

an 



Is? 

1" 






^ 
S 



SI 



a^ 
1" 



IS 

Oi 



1^ 



Digitized by 



Google 



REFERENCE TABLES ^ 



399 



TABLE IX.— SPECIFIC GRAVITIES AND WEIGHTS OF 
MATERIALS OF CONSTRUCTION 



No. 



Material. 



Specific 
Gravity. 



Average Weight in Lbs. 



Cu.in. 



Cu.ft. 



1 

2 

3 

4 

5 

6 

7 

8 

9 

10 

11 

12 

13 

14 

15 

16 

17 

18 

19 
20 
21 
22 
23 
24 
25 
26 
27 
28 
29 
30 
31 
32 
33 
34 
35 
36 



Aluminum, cast 

'* hammered 

* Ash, American white 

" red 

Asphalt 

* Beech 

* Birch 

Brass, cast 

'' rolled 

Brick, best pressed 

* * common, hard 

" soft 

Brickwork, pressed brick. . . 

* ' medium brick . . 

'* ordinary brick. . 

Bronze, copper 8, tin 1 ... . 

* Cedar, American 

Cement, hydraulic, ground . 

loose, Rosendale 

Portland, loose 

* Cherry 

Concrete 

Copper, cast 

* * wire 

♦Elm 

Glass, common window. . . . 

Granite 

Gravel 

* Hemlock 

* Hickory 

Iron, cast 

' ' wrought 

Lead, commercial 

Limestone and marble 

Mahogany, Spanish 

' * Honduras 

* Maple 



1 
2.7 



1 to 1.8 



7.8to8.4 
8.4 



8.5 



8.6to8.8 
8.8 to 9 



2.52 
2.56 to 2.9 



.85 

6.9to7.5 

7.79 

11.38 

2.46 to 2.84 

.85 

.56 



.075 
.099 



.29 
.303 



.306 



.314 
.321 



.26 

.278 



125 
170.6 

38 

40 

87.3 

43 

45.5 
504 
524 
150 
125 
100 
140 
125 
112 
529 

34.5 

56 



42 

130 

542 

555 

35 

157 

170 

117 

25 

53 

450 

480 

709. 

164. 

53 

35 

49 



* Seasoned. 



Digitized by 



Google 



400 



REFERENCE TABLES 
TABLE IX.—Cantinued 



No. 



Material. 



Specific 
Gravity. 



Average Weight in Lbs. 



Cu.in. 



Cu.ft. 



37 
38 
39 
40 
41 
42 
43 
44 
45 
46 
47 
48 
48 
50 
51 
52 
53 
54 



Mortar, hardened 

* Oak, live American . . . 

* * red American. . . . 
" white American.. . 

* Pine, white 

** yellow, Northern. 
'* *' Southern., 

* Poplar 

Sand, quartz 

** wet...* 

* ' well shaken 

Sandstone, building . . . 
Slate, American 

* Spruce 

Steel, structural 

* Wabiut, black 

Water 

Zinc, cast 

'' roUed 



1.4tol.9 



2.75 



2.41 
2.7to2.9 



7.85 



1 
6.9 
7.2 



.28 



103 

59 

40 

50 

25 

34 

45 

39 
90 to 106 
120 to 140 
99 to 117 
151 
175 

25 
490 

38 

t62.355 
376 
449 



* Seasoned. 



t At 62° Fahrenheit, barometer 30 inches. 



Pressure and Parts Conversion Factors 



Unit. 


Equivalent. 


Approximate. 


Kxact 


1 lb. per sq. in. 
1 kg. per scm. 
1 gram per gal. 


. 07 kilograms per scm. 
14.21b. persq. in. 
26 parts per 100,000 


.070308 
14.223105 
26.43737 



Digitized by 



Google 



REFERENCE TABLES 



401, 



TABLE X.— WIRE GAGE SIZES 
In Decimals of an Inch 









Washburn 














& Moen 






British 




Birming- 


American 


Manufac- 


Trenton 

Iron Co. 

Wire 

Gauge. 


American 


Imperial or 


Number 


ham or 


or Brown & 


turing Co. 


Screw Co. 


English 


of 


Stubs Iron 


Sharpe 


andJohnA. 


Screw 


Legal 


Gauge. 


Wire 


Wire 


Roebling's 


Wire 


Standard 




Gauge. 


Gauge. 


Sons Co. 


Gauge. 


Wire. 








Wire 






Gauge. 








Gauge. 








0000000 












.600 


000000 






.4600 






.464 


00000 







.4300 


"!456" 




.432 


0000 


!454 


'aqoooo 


.3938 


.400 




.400 


000 


.425 


.409642 


.3625 


.360 


.0315 


.372 


00 


.380 


.364796 


.3310 


.330 


.0447 


.348 





.340 


.324861 


.3065 


.305 


.0578 


.324 


1 


.300 


.289297 


.2830 


.285 


.0710 


.300 


2 


.284 


.257627 


.2625 


.266 


.0842 


.276 


3 


.259 


.229423 


.2437 


.245 


.0973 


.252 


4 


.238 


.204307 


.2253 


.225 


.1106 


.232 


5 


.220 


. 181940 


.2070 


.205 


.1236 


.212 


6 


.203 


. 162023 


.1920 


.190 


.1368 


.192 


7 


.180 


.144285 


.1770 


.175 


.1600 


.176 


8 


.165 


.128490 


.1620 


.160 


.1631 


.160 





.148 


.114423 


.1483 


.145 


.1763 


.144 


10 


.134 


. 101897 


.1360 


.130 


.1894 


.128 


11 


.120 


.090742 


.1205 


.1175 


.2026 


.116 


12 


.109 


.080808 


.1055 


.105 


.2158 


.104 


13 


.095 


.071962 


.0915 


.0925 


.2289 


.092 


14 


.083 


.064084 


.0800 


.0806 


.2421 


.080 


16 


.072 


.057068 


.0720 


.070 


.2662 


.072 


16 


.065 


.050821 


.0625 


.061 


.2684 


.064 


17 


.058 


.045257 


.0540 


.0626 


.2816 


.056 


18 


.049 


.040303 


.0476 


.045 


.2947 


.048 


19 


.042 


.035890 


.0410 


.040 


.3079 


.040 


20 


.035 


.031961 


.0348 


.035 


.3210 


.036 


21 


.032 


.028462 


.03175 


.031 


.3342 


.032 


22 


.028 


.025346 


.0286 


.028 


.3474 


.028 


23 


.025 


.022572 


.0268 


.025 


.3606 


.024 


24 


.022 


.020101 


.0230 


.0225 


.3737 


.022 


25 


.020 


.017900 


.0204 


.020 


.3868 


.020 


26 


.018 


.015941 


.0181 


.018 


.4000 


.018 


27 


.016 


.014195 


.0173 


.017 


.4132 


.0164 


28 


.014 


.012641 


.0162 


.016 


.4263 


.0148 


29 


.013 


.011257 


.0160 


.015 


.4395 


.0136 


30 


.012 


.010025 


.0140 


.014 


.4526 


.0124 


31 


.010 


.008928 


.0132 


.013 


.4658 


.0116 


32 


.009 


.007950 


.0128 


.012 


.4790 


.0108 


33 


.008 


.007080 


.0118 


.011 


.4921 


.0100 


34 


.007 


.006305 


.0104 


.010 


.6053 


.0092 


35 


.005 


.005615 


.0095 


.0095 


.5184 


.0084 


36 


.004 


.005000 


.0090 


.009 


.6316 


.0076 


37 




.004463 


.0085 


.0085 


.6448 


.0068 


38 




.003965 


.0080 


.008 


.6579 


.0060 


39 




.003631 


.0075 


.0075 


.5711 


.0062 


40 




.003144 


.0070 


.007 


.6842 


.0048 



From the Cambria Handbook. 



Digitized by 



Google 



402 REFERENCE TABLES 

TABLE XL— FOUR-PLACE LOGARITHMS OF NUMBERS 



N. 





1 


2 


3 


4 


5 


6 


7 


8 


9 


P.P. 


10 

11 
12 
13 


0000 


0043 


0086 


0128 


0170 


0212 


0253 0294 


0334 


0374 


22 1 21 


0414 
0792 
1139 


0453 
0828 
1173 


0492 
0864 
1206 


0531 
0899 
1239 


0569 
0934 
1271 


0607 
0969 
1303 


0645 
1004 
1335 


0682 
1038 
1367 


0719 
1072 
1399 


0755 
1106 
1430 


1 
2 
3 
4 
5 


2.2 
4.4 
6.G 
8.8 
11.0 


2.1 
4.2 
6.3 

8.4 
10.5 


14 
15 
16 


1461 
1761 
2041 


1492 
1790 
2068 


1623 

1818 
2096 


1653 
1847 
2122 


1584 
1875 
2148 


1614 
1903 
2175 


1644 
1931 
2201 


1673 
1959 
2227 


1703 
1987 
2253 


1732 
2014 
2279 


6 
7 
8 
9 


13.2 
15.4 
17.6 
19.8 


12.0 
14.7 
16.8 
18.9 


17 
18 
19 

20 

21 
22 
23 


2304 
2553 

2788 


2330 
2577 
2810 


2355 
2601 
2833 


2380 
2625 
2856 


2405 
2648 

2878 


2430 
2672 
2900 


2455 
2695 
2923 


2480 
2718 
2945 


2604 
2742 
2967 


2529 
2765 
2989 


1 
2 
3 
4 
5 
6 
7 
8 
9 


20 

2.0 
4.0 
6.0 
8.0 
10.0 
12.0 
14.0 
16.0 
18.0 

18 


19 

1.9 
3.8 
5.7 
7.0 

o.r* 

11.4 
13.3 
16.2 
17.1 

17 


3010 


3032 


3054 


3075 


3096 


3118 


3139 


3160 


3181 


3201 


3222 
3424 
3617 


3243 
3444 
3636 


3263 
3464 
3655 


3284 
3483 
3674 


3304 
3502 
3692 


3324 
3522 
3711 


3345 
3541 
3729 


3365 
3560 
3747 


3385 
3579 
3766 


3404 
3598 
3784 


24 
26 
26 


3802 
3979 
4150 


3820 
3997 
4166 


383S 
4014 
4183 


3856 
4031 
4200 


3874 
4048 
4216 


3892 
4065 
4232 


3909 
4082 
4249 


3927 
4099 
4265 


3945 
4116 
4281 


3962 
4133 
4298 


1 
2 
3 
4 
5 
6 


1.8 
3.6 
5.4 
7.2 
9.0 
10.8 


1.7 
3.4 
5.1 
0.8 
8.5 
10.2 


27 
28 
20 

30 

31 
32 
33 


4314 
4472 
4624 


4330 

4487 
4639 


4346 
4502 
4654 


4362 
4518 
4669 


4378 
4533 
4683 


4393 

4548 
4698 


4409 
4564 
4713 


4425 
4579 
4728 


4440 
4594 
4742 


4456 
4609 
4757 


7 
8 
9 

1 
2 
3 
4 
5 
6 
7 


12.0 
14.4 
10.2 

16 

1.6 
3.2 
4.8 
6.4 
8.0 
9.0 
11 2 


11.9 
13.0 
15.3 

15 

1.5 
3.0 
4.5 
0.0 
7.5 
9.0 
10.5 


4771 


4786 


4800 


4814 


4829 


4843 


4857 


4871 


4886 


4900 


4914 
5051 
5185 


4928 
6065 
6198 


4942 
5079 
6211 


4965 
5092 
6224 


4969 
5106 
6237 


4983 
6119 
6260 


4997 
6132 
6263 


6011 
6145 
6276 


5024 
5159 
5289 


5038 
5172 
5302 


34 
35 
36 


5315 
6441 
5663 


5328 
5453 
6576 


5340 
5466 
6587 


5353 
6478 
6699 


5366 
6490 
6611 


6378 
5502 
6623 


6391 
6615 
6636 


6403 
6527 
5647 


6416 
6539 
6658 


5428 
6551 
6670 


8 
9 

T 


12.8 
14.4 

14 

1.4 


12.0 
13.5 

13 

1.3 


37 
38 
39 

40 


6682 
5798 
6911 


5694 
5809 
5922 


6706 
6821 
5933 


6717 
5832 
6944 


6729 
6843 
6956 


6740 
6855 
6966 


6762 
6866 
5977 


6763 
6877 
6988 


6775 
6888 
5999 


6786 
6900 
6010 


2 
3 

4 
6 
6 
7 
8 
9 


2.8 
4.2 
5.6 
7.0 
8.4 
9.8 
11.2 
12.0 


2.6 
3.9 
5.2 
0.6 
7.8 
9.1 
10.4 
11.7 


6021 


6031 


6042 


6053 


6064 


6075 


6085 


6096 


6107 


6117 


N. 





1 


2 


3 


4 


6 


6^ 


7 


8 


9 



Note. — Proportional parts begin with N. 19. Determine P.P. from N. 10 to 
N. 19 by multiplying the actual difference between the mantissa read and the man- 
tissa immediately following, by all the figures of the natural number (except tho 
first three) with a decimal point before them. 



Digitized by 



Google 



REFERKNCE TABLES 
TABLE XI.— Continued 



403 



N. 





1 


2 


3 


4 


6 


6 


7 


8 


9 


P.P. 


40 

41 
42 
43 


6021 


6031 


6042 


6053 


6064 


6075 


6085 


6096 


6107 


6117 




6128 
6232 
6335 


6138 
6243 
6345 


6149 
6253 
6355 


6160 
6263 
6365 


6170 
6274 
6375 


6180 
6284 
6385 


6191 
6294 
6395 


6201 
6304 
6405 


6212 
6314 
6416 


6222 
6325 
6425 


44 
45 
46 


6435 
6532 
6628 


6444 
6542 
6637 


6454 
6551 
6646 


6464 
6561 
6656 


6474 
6571 
6665 


6484 
6580 
6675 


6493 
6590 
6684 


6503 
6599 
6693 


6513 
6609 
6702 


6522 
6618 
6712 




47 
48 
49 

50 

51 
52 
53 


6721 
6812 
6902 


6730 
6821 
6911 


6739 
6830 
6920 


6749 
6839 
6928 


6758 
6848 
6937 


6767 
6857 
6946 


6776 
6866 
6955 


6785 
6875 
6964 


6794 
6884 
6972 


6803 
6893 
6981 


1 
2 
3 
4 
5 
6 
7 
8 
9 


18 

1.2 
2.4 
3.6 
4.8 
6.0 
7.2 
8.4 
9.6 
10.8 


11 

1.1 
22 
3.3 
4.4 
5.5 
6.6 
7.7 
8.8 
9.9 


6990 


6998 


7007 


7016 


7024 


7033 


7042 


7050 


7059 


7067 


707C 
7160 
7243 


7084 
7168 
7251 


7093 
7177 
7259 


7101 
7185 
7267 


7110 
7193 
7275 


7118 
7202 
7284 


7126 
7210 
7292 


7135 
7218 
7300 


7143 
7226 
7308 


7152 
7235 
7316 


54 
55 
50 


7324 
7404 

7482 


7332 
7412 
7490 


7340 
7419 
7497 


7348 
7427 
7505 


7356 
7435 
7513 


7364 
7443 
7520 


7372 
7451 

7528 


7380 
7459 
7536 


7388 
7466 
7543 


7396 
7474 
7551 




57 

58 
59 

GO 

Gl 
62 
63 


7559 
7634 
7709 


7566 
7642 
7716 


7674 
7649 
7723 


7582 
7657 
7731 


7589 
7664 
7738 


7597 
7672 
7745 


7604 
7679 
7752 


7612 
7686 
7760 


7619 
7694 
7767 


7627 
7701 

7774 




7782 


7789 


7796 


7803 


7810 


7818 


7825 


7832 


7839 


7846 


1 
2 
3 
4 

r» 




0.9 
1.8 
2.7 
3.6 
4.5 


8 

0.8 
1.6 
2.4 
3.2 
4.0 


7853 
7924 
7993 


7860 
7931 
8000 


7868 
7938 
8007 


7875 
7945 
8014 


7882 
7952 
8021 


7889 
7959 
8028 


7896 
7966 
8035 


7903 
7973 
8041 


7910 
7980 
8048 


7917 
7987 
8055 


64 
65 
66 


8062 
8129 
8195 


8069 
8136 
8202 


8075 
8142 
8209 


8082 
8149 
8215 


8089 
8156 
8222 


8096 
8162 
8228 


8102 
8169 
8235 


8109 
8176 
8241 


8116 
8182 
8248 


8122 
8189 
8254 


6 
7 
8 



5.4 
6.3 
7.2 
8.1 


4.8 
5.6 
6.4 
7.2 


67 
68 
69 

70 


8261 
8325 

8388 


8267 
8331 
8395 


8274 
8338 
8401 


8280 
8344 
8407 


8287 
8351 
8414 


8293 
8357 
8420 


8299 
8363 
8426 


8306 
8370 
8432 


8312 
8376 
8439 


8319 
8382 
8445 




8451 


8457 


8463 


8470 


8476 


8482 


8488J 8494 


8500 


8506 


N. 





1 


2 


3 


4 


6 


6 


7 


8 


9 









Digitized by 



Google 



404 



REFERENCE TABLES 
TABLE XL— Continued 



N. 



3 



6 



8 



P.P. 



70 

71 
72 
73 

74 
75 
76 

77 
78 
79 

80 

81 
82 
83 

84 
85 
86 

87 

88 



90 

91 
92 
93 

94 
95 
96 

97 
93 
99 



8451 



8457 



8463 



8470 



8476 



8482 



8488 



8494 



8500 



8506 



N, 



8513 
8573 
8633 

8692 
8751 
8808 

8865 
8921 
8976 



8519 
8579 
8639 

8698 
8756 
8814 

8871 
8927 
8982 



8525 
8585 
8645 

8704 
8762 
8820 

8876 
8932 
8987 



9031 



9036 9042 



9085 
9138 
9191 

9243 
9294 
9345 

9395 
9445 
9494 



9090 
9143 
9196 

9248 
9299 
9350 

9400 
9450 
9499 



9542 



9547 



9590 
9638 
9685 

9731 
9777 
9823 

9868 
9912 
9956 



9595 
9643 
9689 

9736 
9782 
9827 

9872 
9917 
9961 



9096 
9149 
9201 

9253 
9304 
9355 

9405 
9455 
9504 



9552 



9600 
9647 
9694 

9741 
9786 
9832 

9877 
9921 
9965 



2 



8531 
8591 
8651 

8710 
8768 
8825 

8882 
8938 
8993 



8537 
859Y 
8657 

8716 

8774 
8831 

8887 
8943 
8998 



8543 
8603 
8663 

8722 
8779 
8837 

8893 
8949 
9004 



8549 
8609 
8669 

8727 

8785 
8842 

8899 
8954 
9009 



8555 
8615 
8675 

8733 

8791 
8848 

8904 
8960 
9015 



8561 
8621 
8681 

8739 
8797 
8854 

8910 
8965 
9020 



8567 
8627 
8686 

8745 
8802 
8859 

8915 
8971 
9025 



9047 



9053 



9058 



9063 



9069 



9074 



9079 



9101 
9154 
9206 

9258 
9309 
9360 

9410 
9460 
9509 



9106 
9159 
9212 

9263 
9315 
9365 

9415 
9465 
9513 



9112 
9165 
9217 

9269 
9320 
9370 

9420 
9469 
9518 



9117 
9170 
9222 

9274 
9325 
9375 

9425 
9474 
9523 



9122 
9175 
9227 

9279 
9330 
9380 

9430 
9479 
9528 



9128 
9180 
9232 

9284 
9335 
9385 

9435 
9484 
9533 



9133 
9186 
9238 

9289 
9340 
9390 

9440 
9489 
9538 



9557 



9562 



9566 



9571 



9576 



9581 



9586 



9605 
9652 
9699 

9745 
9791 
9836 

9881 
9926 
9969 



9609 
9657 
9703 

9750 
9795 
9841 



9930 
9974 



9614 
9661 
9708 

9754 
9800 
9845 

9890 
9934 
9978 



9619 
9666 
9713 

9759 
9805 
9850 

9894 
9939 
9983 



9624 
9671 
9717 

9763 
9809 
9854 



9943 
9987 



9628 
9675 
9722 

9768 
9814 
9859 

9903 
9948 
9991 



9633 
9680 
9727 

9773 
9818 
9863 

9908 
9952 
9996 





7 


1 


0.7 


2 


1.4 


3 


2.1 


4 


2.8 


5 


3.5 


6 


4.2 


7 


4.9 


8 


5.6 


9 


6.3 





5 


1 


0.5 


2 


1.0 


3 


1.5 


4 


2.0 


6 


2.5 


6 


3.0 


7 


3.5 


8 


4.0 


9 


4.5 



6 



8 



9 



6 

0.6 
1.2 
1.8 
2.4 
3.0 
3.6 
4.2 
4.8 
5.4 



4 

0.4 
0.8 
1.2 
1.6 
2.0 
2.4 
2.8 
3.2 
3.6 



Digitized by 



Google 



REFERENCE TABLES 



405 



TABLE XII.— FOUR-PLACE TRIGONOMETRIC FUNCTIONS 



Angle 


Sine 


Cosine 


Tangent 


Cotangent 


Angle 


o / 

00 


Nat. Log. 


Nat. Log. 


Nat. Log. 


Nat. Log. 


o / 

90 00 


.0000 


1.0000 0.0000 


.0000 




10 


.0029 7.4637 


1.0000 0000 


.0029 7.4637 


343.77 2.6363 


50 


20 


.0058 7648 


1.0000 0000 


.0058 7648 


171.89 2352 


40 


30 


.0087 9408 


1.0000 0000 


.0087 9409 


114.59 0591 


30 


40 


.0116 8.0658 


.9999 0000 


.0116 8.0658 


85.940 1.9342 


20 


50 


.0145 1627 


.9999 0000 


.0145 1627 


68.760 8373 


10 


1 00 


.0175 8.2419 


.9998 9.9999 


.0175 8.2419 


57.290 1.7581 


89 00 


10 


.0204 3088 


.9998 0999 


.0204 3089 


49.104 6911 


50 


20 


.0233 3668 


.9997 9999 


.0233 3669 


42.964 6331 


40 


30 


.0262 4179 


.9997 9999 


.0262 4181 


38.188 6819 


30 


40 


.0291 4637 


.9996 9998 


.0291 4638 


34.368 5362 


20 


60 


.0320 6050 


.9995 9998 


.0320 5053 


31.242 4947 


10 


2 00 


.0349 8.5428 


.9994 9.9997 


.0349 8.5431 


28.636 1.4569 


88 00 


10 


.0378 6776 


.9993 9997 


.0378 5779 


26.432 4221 


60 


20 


.0407 6097 


.9992 9996 


.0407 6101 


24.542 3899 


40 


30 


.0436 6397 


.9990 9996 


.0437 6401 


22.904 3599 


30 


40 


.0465 6677 


.9989 9995 


.0466 6682 


21.470 3318 


20 


60 


.0494 6940 


.9988 9995 


.0495 6945 


20.206 3055 


10 


3 00 


.0523 8.7188 


.9986 9.9994 


.0524 8.7194 


19.081 1.2806 


87 00 


10 


.0552 7423 


.9985 9993 


.0553 7429 


18.075 2571 


50 


20 


.0581 7645 


.9983 9993 


.0582 7652 


17.169 2348 


40 


30 


.0610 7857 


.9981 9992 


.0612 7865 


16.350 2135 


30 


40 


.0640 8059 


.9980 9991 


.0641 8067 


15.605 1933 


20 


60 


.0669 8251 


.9978 9990 


.0670 8261 


14.924 1739 


10 


4 00 


.0698 8.8436 


.9976 9.9989 


.0699 8.8446 


14.301 1.1554 


86 00 


10 


.0727 8613 


.9974 9989 


.0729 8624 


13.727 1376 


50 


20 


.0756 8783 


.9971 9988 


.0758 8795 


13.107 1205 


40 


30 


.0785 8946 


.9969 9987 


.0787 8960 


12.706 1040 


30 


40 


.0814 9104 


.9967 9986 


.0816 9118 


12.251 0882 


20 


60 


.0843 9256 


.9964 9985 


.0846 9272 


11.826 0728 


10 


5 00 


.0872 8.9403 


.9962 9.9983 


.0875 8.9420 


11.430 1.0580 


85 00 


10 


.0901 9545 


.9959 9982 


.0904 9563 


11.059 0437 


50 


20 


.0929 9682 


.9957 9981 


.0934 9701 


10.712 0299 


40 


30 


.0958 9816 


.9954 9980 


.0963 9836 


10.385 0164 


30 


40 


.0987 9945 


.9951 9979 


.0992 9966 


10.078 0034 


20 


60 


.1016 9.0070 


.9948 9977 


.1022 9.0093 


9.7882 0.9907 


10 


6 00 


.1045 9.0192 


.9945 9.9976 


.1051 9.0216 


9.5144 0.9784 


84 00 


10 


.1074 0311 


.9942 9975 


.1080 0336 


9.2553 9664 


50 


20 


.1103 0426 


.9939 9973 


.1110 0453 


9.0098 9547 


40 


30 


.1132 0539 


.9936 9972 


.1139 0567 


8.7769 9433 


30 


40 


.1161 0648 


.9932 9971 


.1169 0678 


8.5555 9322 


20 


60 


.1190 0755 


.9929 9969 


.1198 0786 


8.3450 9214 


10 


7 00 

o / 


.1219 9.0859 


.9925 9.9968 


.1228 9.0891 


8.1443 0.9109 


83 00 


Nat. Log. 


Nat. Log. 


Nat. Log. 


Nat. Log. 


o / 


Angle 


Cosine 


Sine 


Cotangent 


Tangent 


Angle 



Digitized by 



Google 



406 



REFERENCE TABLES 



TABLE XIL-— Continued 



Anoud 


Sine 


Cosine 


Tangent 


Cotangent 


Angle 


e t 
7 00 


Nat. Log. 


Nat. Log. 


Nat. Log. 


Nat. Log. 


O f 

83 00 


.1219 0.0859 


.9925 9.9968 


.1228 9.0891 


8.1443 0.9109 


10 


.1248 0961 


.9922 9966 


.1257 0995 


7.9530 9005 


50 


20 


. 1276 1060 


.9918 9964 


. 1287 1096 


7.7704 8904 


40 


30 


.1305 1157 


.9914 9963 


.1317 1194 


7.5958 8806 


30 


40 


. 1334 1252 


.9911 9961 


. 1346 1291 


7.4287 8709 


20 


50 


.1363 1345 


.9907 9959 


.1376 1385 


7.2687 8615 


10 


8 00 


.1392 9.1436 


.9903 9.9958 


.1405 9.1478 


7.1154 0.8522 


S2 00 


10 


. 1421 1525 


.9899 9956 


. 1435 1569 


6.9682 8431 


50 


20 


. 1449 1612 


.9894 9954 


.1465 1658 


6.8269 8342 


40 


30 


. 1478 1697 


.9890 9952 


. 1495 1745 


6.6912 8255 


30 


40 


. 1507 1781 


.9886 9950 


. 1524 1831 


6.5606 8169 


20 


60 


.1530 1863 


.9881 9948 


. 1554 1915 


6.4348 8085 


10 


9 00 


.1564 9.1943 


.9877 9.9946 


.1584 9.1997 


6.3138 0.8003 


81 00 


10 


. 1593 2022 


.9872 9944 


.1614 2078 


6.1970 7922 


50 


20 


.1622 2100 


.9868 9942 


. 1644 2158 


6.0844 7842 


40 


30 


. 1650 2176 


.9863 9940 


. 1673 2236 


5.9758 7764 


30 


40 


.1679 2251 


.9858 9938 


.1703 2313 


5.8708 7687 


20 


50 


. 1708 2324 


.9853 9936 


.1733 2389 


5.7694 7611 


10 


10 00 


.1736 9.2397 


.9848 9.9934 


.1763 0.2463 


5.6713 0.7537 


80 00 


10 


.1765 2468 


.9843 9931 


. 1793 2536 


5.5764 7464 


50 


20 


. 1794 2538 


.9838 9929 


. 1823 2600 


5.4845 7391 


40 


30 


.1822 2606 


.9833 9927 


.1853 2680 


5.3955 7320 


30 


40 


. 1851 2674 


.9827 9924 


. 1883 2750 


5.3093 7250 


20 


50 


.1880 2740 


.9822 9922 


. 1914 2819 


5.2257 7181 


10 


11 00 


.1908 9.2806 


.9816 9.9919 


.1944 9.2887 


5.1446 0.7113 


79 00 


10 


. 1937 2870 


.9811 9917 


. 1974 2953 


5.0658 7047 


50 


20 


.1965 2934 


.9806 9914 


.2004 3020 


4.9894 6980 


40 


30 


. 1994 2997 


.9799 9912 


.2035 3085 


4.9152 6915 


30 


40 


.2022 3058 


.9793 9909 


.2065 3149 


4.8430 6851 


20 


60 


.2051 3119 


.9787 9907 


.2095 3212 


4.7729 6788 


10 


18 00 


.2079 9.3179 


.9781 9.9904 


.2126 9.3275 


4.7046 0.6725 


78 00 


10 


.2108 3238 


.9775 9901 


.2156 3336 


4.6382 6664 


50 


20 


.2136 3296 


.9769 9899 


.2186 3397 


4.5736 6603 


40 


30 


.2164 3353 


.9763 9896 


.2217 3458 


4.5107 6542 


30 


40 


.2193 3410 


.9757 9893 


.2247 3517 


4.4494 6483 


20 


50 


.2221 3466 


.9750 9890 


.2278 3576 


4.3897 6424 


10 


13 00 


.2250 9.3521 


.9744 9.9887 


.2309 9.3634 


4.3315 0.6366 


77 00 


10 


.2278 3575 


.9737 9884 


.2339 3691 


4.2747 6309 


50 


20 


.2306 3629 


.9730 9881 


.2370 3748 


4.2193 6252 


40 


30 


.2334 3682 


.9724 9878 


. 2401 3804 


4.1653 6196 


30 


40 


.2363 3734 


.9717 9875 


.2432 3859 


4.1126 6141 


20 


50 


.2391 3786 


.9710 9872 


.2462 3914 


4.0611 6086 


10 


14 00 

o / 


.2419 9.3837 


.9703 9.9869 


.2493 9.3968 


4.0108 0.6032 


76 00 

o / 


Nat. Log. 


Nat. Log. 


Nat. Log. 


Nat. Log. 


Anolb 


Cosine 


Sine 


Cotangent 


Tangent 


Angle 



Digitized by 



Google 



REFERENCE TABLES 



407 



I 



TABLE Xll.^-Continued 



AifOLB 


SiNB 


Cosine 


Tangent 


Cotangent 


Angle 


o / 

14 00 


Nat. Log. 


Nat. Log. 


Nat. Log. 


Nat. Log. 


o / 

76 00 


.2419 9.3837 


.9703 9.9869 


.2493 9.3968 


4.0108 0.6032 


10 


.2447 3887 


.9696 9866 


.2524 4021 


3.9617 5979 


50 


20 


.2476 3937 


.9689 9863 


.2555 4074 


3.9136 5926 


40 


30 


.2504 3986 


.9681 9859 


.2586 4127 


3.8667 5873 


30 


40 


.2532 4035 


.9674 9856 


.2617 4178 


3.8208 5822 


20 


60 


.2560 4083 


.9667 9853 


.2648 4230 


3.7760 5770 


10 


15 00 


.2588 9.4130 


.9659 9.9849 


.2679 9.4281 


3.7321 0.5719 


75 00 


10 


.2616 4177 


.9652 9846 


.2711 4331 


3.6891 5669 


50 


20 


.2644 4223 


.9644 9843 


.2742 4381 


3.6470 5619 


40 


30 


.2672 4269 


.9636 9839 


.2773 4430 


3.6059 5570 


30 


40 


.2700 4314 


.9628 9836 


.2805 4479 


3.5656 5521 


20 


50 


.2728 4359 


.9621 9832 


.2836 4527 


3.5261 5473 


10 


16 00 


.2756 9.4403 


.9613 9.9828 


.2867 9.4575 


3.4874 0.5325 


74 00 


10 


.2784 4447 


.9605 9825 


.2899 4622 


3.4495 5378 


50 


20 


.2812 4491 


.9596 9821 


.2931 4669 


3.4124 5331 


40 


30 


.2840 4533 


.9588 9817 


.2962 4716 


3.3759 5284 


30 


40 


. 2868 4576 


.9580 9814 


.2994 4762 


3.3402 5238 


20 


50 


.2896 4618 


.9572 9810 


.3026 4808 


3.3052 5192 


10 


17 00 


.2924 9.4659 


.9563 9.9806 


.3057 9.4853 


3.2709 0.5147 


73 00 


10 


.2952 4700 


.9555 9802 


.3089 4898 


3.2371 5102 


50 


20 


.2979 4741 


.9546 9798 


.3121 4943 


3.2041 5057 


40 


30 


.3007 4781 


.9537 9794 


.3153 4987 


3.1716 5013 


30 


40 


.3035 4821 


.9528 9790 


.3185 5031 


3.1397 4969 


20 


50 


.3062 4861 


.9520 9786 


.3217 5075 


3.1084 4925 


10 


18 00 


.3090 9.4900 


.9511 9.9782 


.3249 9.5118 


3.0777 0.4882 


72 00 


10 


.3118 4939 


.9502 9778 


.3281 5161 


3.0475 4839 


50 


20 


.3145 4977 


.9492 9774 


.3314 5203 


3.0178 4797 


40 


30 


.3173 5015 


.9483 9770 


.3346 5245 


2.9887 4755 


30 


40 


.3201 5052 


.9474 9765 


.3378 5287 


2.9600 4713 


20 


50 


.3228 5090 


.9465 9761 


.3411 5329 


2.9319 4671 


10 


19 00 


.3256 9.5126 


.9455 9.9757 


.3443 9.5370 


2.9042 0.4630 


71 00 


10 


.3283 5163 


.9446 9752 


.3476 5411 


2.8770 4589 


50 


20 


.3311 5199 


.9436 9748 


.3508 5451 


2.8502 4549 


40 


30 


.3338 5235 


.9426 9743 


.3541 5491 


2.8239 4509 


30 


40 


.3365 5270 


.9417 9739 


.3574 5531 


2.7980 4469 


20 


50 


.3393 5306 


.9407 9734. 


.3607 5571 


2.7725 4429 


10 


20 00 


.3420 9.5341 


.9397 9.9730 


.3640 9.5611 


2.7475 0.4389 


70 00 


10 


.3448 5375 


.9387 9725 


.3673 5650 


2.7228 4350 


50 


20 


.3475 5409 


.9377 . 9721 


.3706 5689 


2.6985 4311 


40 


30 


.3502 5443 


.9367 9716 


.3739 5727 


2.6746 4273 


30 


40 


.3529 5477 


.9356 9711 


.3772 5766 


2.6511 4234 


20 


50 


.3557 5510 


.9346 9706 


.3805 5804 


2.6279 4196 


10 


21 00 

o » 


.3584 9.5543 


.9336 9.9702 


.3830 9.5842 


2.6051 0.4158 


69 00 

o / 


Nat. Log. 


Nat. Log. 


Nat. Log. 


Nat. Log. 


Angle 


Cosine 


Sine 


Cotangent 


Tangent 


Anglb^ 



Digitized by 



Google 



408^ 



REFERENCE TABLES 





► 


TABLE Xll.— Continued 






Anolb 


SiNB 


Cosine 


Tangent 


Cotangent 


Angle 


o / 

21 00 


Nat. Log. 


Nat. Log. 


Nat. Log. 


Nat. Log. 


o / 

69 00 


.3584 9.5643 


.9336 9.9702 


.3839 9.5842 


2.6051 0.4158 


10 


.3611 5576 


.9325 9697 


. 3872 6879 


2.5826 4121 


60 


20 


.3638 5609 


.9315 9692 


.3906 5917 


2.6605 4083 


40 


30 


.3665 5641 


.9304 9687 


.3939 5954 


2.6386 4046 


30 


40 


.3692 5673 


.9293 9682 


.3973 5991 


2.5172 4009 


20 


50 


.3719 5704 


.9283 9677 


.4006 6028 


2.4960 3972 


10 


22 00 


.3746 9.5736 


.9272 9.9672 


.4040 9.6064 


2.4761 0.3936 


68 00 


10 


.3773 5767 


.9261 9667 


.4074 6100 


2.4545 3900 


50 


20 


.3800 6798 


.9250 9661 


.4108 6136 


2.4342 3864 


40 


30 


.3827 5828 


.9239 9656 


.4142 6172 


2.4142 3828 


30 


40 


.3854 6859 


.9228 9651 


.4176 6208 


2.3945 3792 


20 


60 


.3881 6889 


.9216 9646 


.4210 6243 


2.3760 3767 


10 


23 00 


.3907 9.5919 


.9205 9.9640 


.4245 9.6279 


2.3659 0.3721 


67 00 


10 


.3934 5948 


.9194 9635 


.4279 6314 


2.3369 3686 


60 


20 


. 3961 5978 


.9182 9629 


.4314 6.348 


2.3183 3652 


40 


30 


.3987 6007 


.9171 9624 


.4348 6383 


2.2998 3617 


30 


40 


.4014 6036 


.9159 9618 


.4383 6417 


2.2817 3683 


20 


60 


.4041 6065 


.9147 9613 


.4417 6452 


2.2637 3548 


10 


24 00 


.4067 9.6093 


.9135 9.9607 


.4452 9.6486 


2.2460 0.3514 


66 00 


10 


.4094 6121 


.9124 9602 


.4487 6520 


2.2286 3480 


50 


20 


.4120 6149 


.9112 9596 


.4522 6553 


2.2113 3447 


40 


30 


.4147 6177 


.9100 9590 


.4667 6587 


2.1943 3413 


30 


40 


.4173 6205 


.9088 9584 


.4592 6620 


2.1775 3380 


20 


50 


.4200 6232 


.9076 9579 


.4628 6654 


2.1609 3346 


10 


25 00 


.4226 9.6259 


.9063 9.9573 


.4663 9.6687 


2.1445 0.3313 


65 00 


10 


.4253 6286 


.9051 9567 


.4699 6720 


2.1283 3280 


50 


20 


.4279 6313 


.9038 9561 


.4734 6752 


2.1123 3248 


40 


30 


.4305 6340 


.9026 9555 


.4770 6786 


2.0965 3215 


30 


40 


.4331 6366 


.9013 9549 


.4806 6817 


2.0809 3183 


20 


60 


.4358 6392 


.9001 9543 


.4841 6850 


2.0655 3160 


10 


26 00 


.4384 9.6418 


.8988 9.9537 


.4877 9.6882 


2.0503 0.3118 


64 00 


10 


.4410 6444 


.8975 9530 


.4913 6914 


2.0353 3086 


60 


20 


.4436 6470 


.8962 9624 


.4950 6946 


2.0204 3064 


40 


30 


.4462 6496 


.8949 9518 


.4986 6977 


2.0057 3023 


30 


40 


.4488 6521 


.8936 9512 


.6022 7009 


1.9912 2991 


20 


60 


.4514 6546 


.8923 9505 . 


.6059 7040 


1.9768 2960 


10 


27 00 


.4540 9.6570 


.8910 9.9499 


.5095 9.7072 


1.9626 0.2928 


63 00 


10 


.4566 6595 


.8897 9492 


.5132 7103 


1.9486 2897 


60 


20 


.4592 6620 


.8884 9486 


.6169 7134 


1.9347 2866 


40 


30 


.4617 6644 


.8870 9479 


.6206 7165 


1.9210 2836 


30 


40 


.4643 6668 


.8857 9473 


.6243 7196 


1.9074 2804 


20 


60 


.4669 6692 


.8843 9466 


.5280 7226 


1.8940 2774 


10 


28 00 
o / 


.4695 9.6716 


.8829 9.9459 


.6317 9.7257 


1.8807 0.2743 


62 00 

o / 


Nat. Log. 


Nat. Log. 


Nat. Log. 


Nat. Log. 


Anolb 


Cosine 


SiNB 


Cotangent 


Tangent 


Angle 



Digitized by 



Google 



EEFERENCE TABLES 



400 



TABLE Xll— Continued 



Angle 


Sine 


Cosine 


Tangent 


Cotangent 


Angle 


O f 

28 00 


Nat. Log. 


Nat. Log. 


Nat. Log. 


Nat. Log. 


62 00 


.4695 9.6716 


.8829 9.9469 


.6317 9.7257 


1.8807 0.2743 


10 


.4720 6740 


.8816 9453 ' 


.5354 7287 


1.8676 2713 


50 


20 


.4746 6763 


.8802 9446 


.53D2 7317 


1.8546 2683 


40 


30 


.4772 6787 


.8788 9439 


.5430 7348 


1.8418 2652 


30 


40 


.4797 6810 


.8774 9432 


.5467 7378 


1.8291 2622 


20 


60 


.4823 6833 


.8760 9425 


.5505 7408 


1.8165 2692 


10 


29 00 


.4848 9.6856 


.8746 9.9418 


.6643 9.7438 


1.8040 0.2562 


61 00 


10 


.4874 6878 


.8732 9411 


.6581 7467 


1.7917 25.33 


50 


20 


.4899 C901 


.8718 9404 


.6619 7497 


1.7796 2503 


40 


30 


.4924 6923 


.8704 9397 


.5658 7626 


1.7675 2474 


30 


40 


.4950 6946 


.8689 9390 


.5696 7556 


1.7556 2444 


20 


50 


.4975 6968 


.8675 9383 


.5735 7585 


1.7437 2415 


10 


30 00 


.5000 9.6990 


.8660 9.9376 


.5774 9.7614 


1.7321 0.2386 


60 00 


10 


.5025 7012 


.8646 9368 


.5812 7644 


1.7205 2356 


50 


20 


.5050 7033 


.8631 9361 


.5851 7673 


1.7090 2327 


40 


30 


.5075 7055 


.8016 9353 


.5890 7701 


1.6977 2299 


30 


40 


.5100 7076 


.8601 9346 


.5930 7730 


1.6864 2270 


20 


60 


.5125 7097 


.8587 9338 


.6969 7759 


1.6753 2241 


10 


31 00 


.5150 9.7118 


.8572 9.9331 


.6009 9.7788 


1.6643 0.2212 


59 00 


10 


.5175 7139 


.8557 9323 


.6048 7816- 


1.6534 2184 


50 


20 


.5200 7160 


.8542 9316 


.6088 7845 


1.6426 2155 


40 


30 


.5225 7181 


.8526 9308 


.6128 7873 


1.6319 2127 


30 


40 


.5250 7201 


.8511 9300 


.6168 7902 


1.6212 2098 


20 


50 


.5275 7222 


.8496 9292 


.6208 7930 


1.6107 2070 


10 


32 00 


.5299 9.7242 


.8480 9.9284 


.6249 9.7958 


1.6003 0.2042 


58 00 


10 


.5324 7262 


.8465 9276 


.6289 7986 


1.6900 2014 


50 


20 


.5348 7282 


.8450 9268 


.6330 8014 


1.5798 1986 


40 


30 


.5373 7302 


.8434 9260 


.6371 8042 


1.5697 1958 


30 


40 


.5398 7322 


.8418 9252 


.6412 8070 


1.5597 1930 


20 


50 


.6422 7342 


.8403 9244 


.6453 8097 


1.6497 1903 


10 


33 00 


.5446 9.7361 


.8387 9.9236 


.6494 9.8126 


1.5399 0.1875 


57 00 


10 


.5471 7380 


.8371 9228 


.6536 8153 


1.5301 1847 


50 


20 


.5495 7400 


.8355 9219 


.6577 8180 


1.5204 1820 


40 


30 


.5519 7419 


.8339 9211 


.6619 8208 


1.5108 1792 


30 


40 


.5544 7438 


.8323 9203 


.6661 8235 


1.6013 1765 


20 


50 


.5668 7457 


.8307 9194 


.6703 8263 


1.4919 1737 


10 


34 00 


.5592 9.7476 


.8290 9.9186 


.6745 9.8290 


1.4826 0.1710 


56 00 


10 


.5616 7494 


.8274 9177 


.6787 8317 


1.4733 1683 


60 


20 


.5640 7513 


.8258 9169 


.6830 8344 


1.4641 1656 


40 


30 


.5664 7531 


.8241 9160 


.6873 8371 


1.4550 1629 


30 


40 


.5688 7550 


.8225 9151 


.6916 8398 


1.4460 1602 


20 


60 


.5712 7568 


.8208 9142 


.6959 8425 


1.4370 1576 


10 


35 00 

o / 


.5736 9.7686 


.8192 9.9134 


.7002 9.8452 


1.4281 0.1648 


55 00 

o / 


Nat. Log. 


Nat. Log. 


Nat. Log. 


Nat. Log. 


Anqlb 


Cosine 


Sine 


Cotangent 


Tangent 


Angle 



Digitized by 



Google 



410 



REFERENCE TABLES 
TABLE XII.— Continued ' 



Anqud 


SiNB 


Cosine 


Tanqevt 


Cotangent 


Angle 


o t 

35 00 


Nat. Log. 


Nat. Log. 


Nat. Log. 


Nat. Log. 


o / 

55 00 


.5736 9.7586 


.8192 9.9134 


.7002 9.8462 


1.4281 0.1648 


10 


.5760 7604 


.8175 9125 


.7046 8479 


1.4193 1521 


50 


20 


.5783 7622 


.8168 9116 


.7089 8606 


1.4106 1494 


40 


30 


.5807 7640 


.8141 9107 


.7133 8533 


1.4019 1467 


30 


40 


.5831 7667 


.8124 9098 


.7177 8569 


1.3934 1441 


20 


60 


.5864 7675 


.8107 9089 


.7221 8686 


1.3848 1414 


10 


36 00 


.5878 9.7692 


.8090 9.9080 


.7266 9.8613 


1.3764 0.1387 


54 00 


10 


.5901 7710 


.8073 9070 


.7310 8639 


1.3680 1361 


50 


20 


.5925 7727 


.8066 9061 


.7355 8666 


1.3597 1334 


^40 


30 


.5948 7744 


.8039 9062 


.7400 8692 


1.3514 1308 


30 


40 


.6972 7761 


.8021 9042 


.7445 8718 


1.3432 1282 


20 


50 


.6995 7778 


.8004 9033 


.7490 8746 


1.3361 1256 


10 


37 00 


.6018 9.7795 


.7986 9.9023 


.7536 9.8771 


1.3270 0.1229 


53 00 


10 


.6041 7811 


.7969 9014 


.7681 8797 


1.3190 1203 


50 


20 


.6065 7828 


.7961 9004 


.7627 8824 


1.3111 1176 


40 


30 


.6088 7844 


.7934 8996 


.7673 8860 


1.3032 1160 


30 


40 


.6111 7861 


.7916 8985 


.7720 8876 


1.2954 1124 


20 


50 


.6134 7877 


.7898 8975 


.7766 8902 


1.2876 1098 


10 


38 00 


.6157 9.7893 


.7880 9.8965 


.7813 9.8928 


1.2799 0.1072 


52 00 


10 


.6180 7910 


.7862 8966 


.7860 8954 


1.2723 1046 


50 


20 


.6202 7926 


.7844 8946 


.7907 8980 


1.2647 1020 


40 


30 


.6225 7941 


.7826 8936 


.7954 9006 


1.2572 0994 


30 


40 


.6248 7967 


.7808 8926 


.8002 9032 


1.2497 0968 


20 


60 


.6271 7973 


.7790 8915 


.8060 9058 


1.2423 0942 


10 


39 00 


.6293 9.7989 


.7771 9.8905 


.8098 9.9084 


1.2349 0.0916 


51 00 


10 


.6316 8004 


.7753 8895 


.8146 9110 


1.2276 0890 


50 


20 


.6338 8020 


.7736 8884 


.8196 9135 


1.2203 0866 


40 


30 


.6361 8036 


.7716 8874 


.8243 9161 


1.2131 0839 


30 


40 


.6383 8050 


.7698 8864 


.8292 9187 


1.2069 0813 


20 


50 


.6406 8066 


.7679 8853 


.8342 9212 


1.1988 0788 


10 


40 00 


.6428 9.8081 


.7660 9.8843 


.8391 9.9238 


1.1918 0.0762 


50 00 


10 


.6460 8096 


.7642 8832 


.8441 9264 


1.1847 0736 


50 


20 


.6472 8111 


.7623 8821 


.8491 9289 


1.1778 0711 


40 


30 


.6494 8125 


.7604 8810 


.8641 9316 


1.1708 0685 


30 


40 


.6517 8140 


.7585 8800 


.8591 9341 


1 . 1040 0669 


20 


50 


.6539 8156 


.7566 8789 


.8642 9366 


1.1671 0634 


10 


41 00 


.6561 9.8169 


.7647 9.8778 


.8693 9.9392 


1.1504 0.0608 


49 00 


10 


.6583 8184 


.7628 8767 


.8744 9417 


1.1436 0683 


50 


20 


.6604 S198 


.7609 8756 


.8796 9443 


1.1369 0667 


40 


30 


.6626 8213 


.7490 8745 


.8847 9468 


1.1303 0532 


30 


40 


.6648 8227 


.7470 8733 


.8899 9494 


1.1237 0506 


20 


50 


.667Q 8241 


.7451 8722 


.8952 9519 


1.1171 0481 


10 


42 00 

o / 


.6691 9.8265 


.7431 9.8711 


.9004 9.9644 


1.1106 0.0456 


48 00 

o / 


Nat. Log. 


Nat. Log. 


Nat. Log. 


Nat. Log. 


Angle 


Cosine 


Sine 


Cotangent 


Tangent 


Angle 



Digitized by 



Google 



KEFERENCE TABLES 



411 







TABLE XIL— Con^inwed 






Angle 


Sine 


Cosine 


Tangent 


Cotangent 


Angle 


o / 

42 00 


Nat. Log. 


Nftt. Log. 


Nat. Log. 


Nat. Log. 


o / 

48 00 


.6691 9.8255 


.7431 9.8711 


.9004 9.9544 


1.1106 0.0456 


10 


.6713 8269 


.7412 8699 


.9057 9570 


1.1041 0430 


50 


20 


.6734 8283 


.7392 8688 


.9110 9695 


1.0977 0405 


40 


30 


.6756 8297 


.7373 8676 


.9163 9621 


1.0913 0379 


30 


40 


.6777 8311 


.7353 8665 


.9217 9646 


1.0850 0354 


20 


60 


.6799 8324 


.7333 8653 


.9271 9671 


1.0786 0329 


10 


43 00 


.6820 9.8338 


.7314 9.8641 


.9325 9.9697 


1.0724 0.0303 


47 00 


10 


.6841 8351 


.7294 8629 


.9380 9722 


1.0661 0278 


60 


20 


.6862 8365 


.7274 8618 


.9435 9747 


1.0599 0253 


40 


30 


.6884 8378 


.7254 8606 


.9490 9772 


1.0538 0228 


30 


40 


.6905 8301 


.7234 8594 


.9545 9798 


1.0477 0202 


20 


50 


.6926 8405 


.7214 8582 


.9601 9823 


1.0416 0177 


10 


44 00 


.6947 9.8418 


.7193 9.8569 


.9657 9.9848 


1.0355 0.0152 


46 00 


10 


.6967 8431 


.7173 8557 


.9713 9874 


1.0295 0126 


60 


20 


.6988 8444 


.7153 8545 


.9770 9899 


1.0235 0101 


40 


30 


.7009 8457 


.7133 8532 


.9827 9924 


1.0176 0076 


30 


40 


.7030 8469 


.7112 8520 


.9884 9949 


1.0117 0061 


20 


50 


.7050 8482 


.7092 8507 


.9942 9975 


1.0058 0025 


10 


45 00 

o r 


.7071 9.8495 


.7071 9.8495 


1.0000 0.0000 


1.0000 0.0000 


45 00 

o / 


Nat. Log. 


Nat. Log. 


Nat. Log. 


Nat. Log. 


Angle 


Cosine 


Sine 


Cotangent 


Tangent 


Angle 



Digitized by 



Google 



412 



KEFERENCE TABLES 



TABLE Xin.— THREE-PLACE NATURAL TRIGONOMETRIC 
FUNCTIONS 



Deg. 


sin 


cos 


tan 


Deg. 


sin 


cos 


tan 





.000 


1.000 


.000 


46 


.719 


.695 


1.04 


1 


.017 


.999 


.017 


47 


.731 


.682 


1.07 


2 


.035 


.999 


.035 


48 


.743 


.069 


1.11 


3* 


.052 


.999 


.052 


49 


.755 


.656 


1.15 


4 


.070 


.998 


.070 


50 


.766 


.643 


1.19 


5 


.087 


.996 


.087 


51 


.777 


.629 


1.23 


6 


.105 


.995 


.105 


52 


.788 


.616 


1.28 


7 


.122 


.993 


.123 


53 


.799 


.602 


1.33 


8 


.139 


.990 


.141 


54 


.809 


.588 


1.38 


9 


.156 


.988 


.158 


55 


.819 


.574 


1.43 


10 


.174 


.985 


.176 


56 


.829 


.559 


1.48 


11 


.191 


.982 


.194 


57 


.839 


.545 


1.54 


12 


.208 


.978 


.213 


58 


.848 


.530 


1.60 


13 


.225 


.974 


.231 


59 


.857 


.515 


1.66 


14 


.242 


.970 


.249 


60 


.866 


.500 


1.73 


15 


.259 


.966 


.268 


61 


.875 


.485 


1.80 


16 


.276 


.961 


.287 


62 


.883 


.469 


1.88 


17 


.292 


.956 


.306 


63 


.891 


.454 


1.96 


18 


.309 


.951 


.325 


64 


.899 


.438 


2.05 


19 


.326 


.946 


.344 


65 


.906 


.423 


2.14 


20 


.342 


.940 


.364 


66 


.914 


.407 


2.25 


21 


.358 


.934 


.384 


67 


.921 


.391 


2.36 


22 


.375 


.927 


.404 


68 


.927 


.375 


2.48 


23 


.391 


.921 


.424 


69 


.934 


.358 


2.61 


24 


.407 


.914 


.445 


70 


.940 


.342 


2.75 


25 


.423 


.906 


.466 


71 


.946 


.326 


2.90 


26 


.438 


.899 


.488 


72 


.951 


.309 


3.08 


27 


.454 


.891 


.510 


73 


.956 


.292 


3.27 


28 


.469 


.883 


.532 


74 


.961 


.276 


3.49 


29 


.485 


.875 


.554 


75 


.966 


.259 


3.73 


30 


.500 


.866 


.577 


76 


.970 


.242 


4.01 


31 


.515 


.857 


.601 


77 


.974 


.225 


4.33 


32 


.530 


.848 


.625 


78 


.978 


.208 


4.70 


33 


.545 


.839 


.649 


79 


.982 


.191 


5.14 


34 


.559 


.829 


.675 


80 


.985 


.174 


5.67 


35 


.574 


.819 


.700 


81 


.988 


.156 


6.31 


36 


.588 


.809 


.727 


82 


.990 


.139 


7.12 


37 


.602 


.799 


.754 


83 


.993 


.122 


8.14 


38 


.616 


.788 


.781 


84 


.995 


.105 


9.51 


39 


.629 


.777 


.810 


85 


.996 


.087 


11.4 


40 


.643 


.766 


.839 


86 


.998 


.070 


14.3 


41 


.656 


.755 


.869 


87 


.999 


.052 


19.1 


42 


.669 


.743 


.900 


88 


.999 


.035 


28.6 


43 


.682 


.731 


.933 


89 


.999 


.017 


57.3 


44 


.695 


.719 


.966 


90 


1.000 


,000 


Inf. 


45 


.707 


.707 


1.000 











Digitized by 



Google 



GREEK ALPHABET 



Aa 


Alpha 


I L 


Iota 


Pp 


Rho 


B^ 


Beta 


Kk 


Kappa 


Scr 


Sigma 


A^> 


Gamma 


AX 


Lambda 


Tr 


Tau 


AS 


Delta 


Mfi 


Mu 


Yv 


Upsilon ^ 


E e 


Epsilon 


Nv 


Nu 


^<l> 


Phi 


Zf 


Zeta 


Bi 


Xi 


xx 


Chi 


Hr, 


Eta 


O o 


Omicron 


^,A 


Psi 


ee 


Theta 


Htt 


Pi 


Q 0) 


Omega 



In the applications of mathematics, angles are commonly 
denoted by the small or lower case letters of the Greek alphabet. 
Those most generally used are a, 6, <^, and 8. 

413 



Digitized by 



Google 



Digitized by 



Google 



INDEX 



Absolute temperature, 86 
Accurate use of tables, 224 
Actual weight, 45 
Addition, 102-105 
Air, resistance, 97 

dry volume, 59 
Algebraic notation, 11-18 
Alphabet, 5 
Amperes, 35 
Anemometer, 90 
Angular velocity, 35, 88 
Area, circle, 29 

cone, 31 

cylinder, 31 

ellipse, 30 

on a globe, 89 

parallelogram, 28 

pjrramid, regular, 31 

regular polygon, 29 

ring, 30 

ring, cylindrical, 33 

sector, 30 

sphere, 32 

trapezium, 28 

trapezoid, 28 

triangle, 27, 28 
Areal velocity, 87 
Armor penetration, 85 
Atomic weights, 397 
Avoirdupois weight, 394 
Axes, ellipse, 30 

of reference, 63-64 
Axioms, 10 



B 

Barometer, 77 

Battery, multiple-series, 36 

parallel, 36 

series, 35 
Beam, deflection, 48, 96 

rupture, 97 

weight, 90 
Bending of bar, 89 
Binomial formula, 195 

theorem, 191-199 
Boyle's law, 88-89 
Briggs' system, 223 



Calorimeter, 66 
Center of gravity, 45 
Centrifugal force, 34, 89 
Charles' law, 86 
Check, 40 
Circle, arc of sector, 30 

area, 29 

area sector, 30 

ratio circumference to diameter, 
29 
Coefficient, 16 

of friction, 35 
Components, 63 
Compound motor, 53-54 
Cone, area, 31 

frustum, 32 

volume, 31 
Continued proportion, 38 

415 



Digitized by 



Google 



416 



INDEX 



Convention of signs, 64 

Copper wire, resistance, 50 

Cosine, 66 

Cotangent, definition, 72 

Coulomb's law, 98 

Cube, diagonal of, 99 

Cube root, polynomial, 205-206 

Cubic measure, 393 

Current, series battery, 35 

multiple battery, 36 

multiple-series battery, 36 

through shunted galvanometer, 
36 

variation, 98 
Cutting speed, 41 
Cylinder, area, 31 

lateral surface, 99 

slide-rule computation, 337-341 

volume, 31 

weight, 97 
Cylindrical ring, area, 33 



D 

Definition, absolute temperature, 
86,92 
addition, 102 
antilogarithm, 235 
areal velocity, 87 
checking, 40 
coefficient, 16 
complex fraction, 151 
compound ratio, 14 ^ 
cotangent, 72 i 

direct variation, 78 
division, 115 
equation, 14 
expansion, 192 
exponent, 16 

exponential equation, 252 
fraction, 144 

fractional, simple equation, 166 
factorial, 191 
factoring, 14, 128 
factors, 128 
formula, 350 
frustum, 32 



Definition, functions of an angle, 66 
grouping, 131 
"into," 35 
inverse variation, 78 
involution, 247 
"jointly," 90 
known quantity, 18 
LL scale, 342 

least common denominator, 40 
like terms, 101 
logarithm, 222 
members, 15 
modulus, 239 
molecular weight, 57 
multiplication, 113 
pitch of screw, 39 
polynomial, 139 
prime numbers, 144 
proportion, 37 

continued, 38 

extremes, 38 

means, 38 

simple, 38 

terms, 38 
pjrramid, regular, 31 
quadratic equation, 155 
radical, 207 
raciical equation, 219 
ratio, 12, 26-27 
rational quantity, 208 
rationalization, 218 
reduction, 208 
regular polygon, 29 
resultant, 74 
rise of a chord, 47 
sector, 30 

significant formula, 26 
similar radicals, 213 
simple equation, 18 

fraction, 151 
simplest form of, radical, 208 

result, 351 
simultaneous equations, 173 
subtraction, 105 
subtrahend, 105 
supplement of an angle, 69 
surd quantity, 208 
table of logarithms, 222 



Digitized by 



Google 



INDEX 



417 



Definition, terms, 15 
of a fraction, 144 

transformation, 360 

trapezium, 28 

trapezoid, 28 

unlike terms, 101 

work, 34 
Deflection of beam, 48 
De Morgan, 60 
Density of a gas, 89 
Diameter, long piston-rod, 51 

pulley arm, 43 

shaft, 43 
Diffusive power, 99 
Division, 115-118 
Dry air, volume, 59 
Dry Measure, 393 
Dynamo, resistance of shunt 
winding, 52 



E 

Element, percentage composition, 

57 
Efficiency, shunt motor, 52 
Electrical, efficiency, 36 

transmission, 46 
Elevation, outside rail, 48 
Ellipse, area, 30 

axes, 30 

perimeter, 30, 370 
Elongation, of a spring, 93 

of a wire, 95 
Equation, 14-16 

quadratic, 155-165 

simple, 18-25 

simple fractional, 166-172 
Equations, simultaneous simple, 
173-179 

trigonometric, 70-73 
Equilibrium on inclined plane, 49 

of liquids, 84 
Equivalents, U. S. and metric, 396 
Examples, addition, 103-105 

binomial formula, 197-199 

common factor, 129-130 

difference of cubes, 123 



Examples, division, 117-118 
exponents, 184-185, 187-188, 

190 
factoring, 129-130, 132-134, 

135-136, 139, 141-143 
factor law, 127 
fractions, 145-146, 148-151, 

153-154 
functions, acute angle, 66 

obtuse angle, 70 
grouping, 132-133 
logarithmic computation, 266- 

277 
logarithms, 226, 227, 231, 234, 

238, 239, 242, 244, 245-246, 

248-249, 251-252, 254, 268- 

274 
multiplication, 114-115 
powers, 201 

product sum and difference, 122 
projection, 62 

quadratic, 160-161, 163-165 
radicals, 210-217, 219-220 
removal of parenthesis, 109-112 
resolution, 64-65, 68 
resultant, 76 
roots, 203-206 
simple equations, 19-25 
simple fractional equation, 

167-172 
simultaneous simple equations, 

176-179 
shde-rule, 291-202, 294, 299, 

304-305, oC7, 311, 315-316, 

321-324, 326-328, 332, 335, 

338,341,346,349 
square 9f, any polynomial, 125 

sum of two numbers, 120- 
121 
subtraction, 106-108 
sum of cubes, 124 
trigonometric equations, 70-73 
Expansion, 192-197 
Exponent, 16-17 
Exponents, 180-190 
fractional, 185-188 
negative, 182-185 
zero, 181-182 



Digitized by 



Google 



418 



INDEX 



Factor, law, 126 

with negative exponent, 183 
Factorial symbol, 191 
Factoring, 128-143 

cases, 128 

common factor, 129-130 

difiFerence of two cubes, 134-135 

difference two squares, 133-134 

grouping, 131-133 

polynomial, 139-140 

special expressions, 140-143 
methods, 141-143 

sum of two cubes, 135-136 

trinomial, 136-139 
Factors, 14 
Falling body, average velocity, 33 

change in velocity, 33 

final velocity, 33 

space traversed, 33 
Flow, from orifice, 98 

of gas, 87 

through a pipe, 87 
Flux, 47 
Force, 27 

centrifugal, 34 
Forces, 63-76 
Formula, adiabatic expansion, 389 

area, trapezoid, 369 
triangle, 369 

armatures, winding, 361 

attracted disk electrometer, 388 

battery, electromotive force of, 
360 

beams, 356 

belt, effective pull, 373 

belting, 359 

bending moment, 356 

bending moment of wall-brack- 
et, 381 

binomial, 195 

boiler, pitch of, 359 
plate, 384 
stay, diameter, 359 
tube, thickness of, 373 

breaking strength, 383 

cable, location of fault in, 361 

cable testing, 367 



Formula, capacity, concentric 
cylinders, 386 

condensers, 378 

conical vessel, 363 
Camot's engine, 368 
catenary, 366, 385 
chimney, effective area, 380 

theoretical height, 380 
chord, length of, 370 
circular pitch, 49 
closed circuit, 386 
coefficient of, elasticity, 354 

linear expansion, 383 

self-induction, 378, 379 
columns, breadth of rectangular, 
373 

Rankine-Gordon formula, 368 
combustion, air required, 359 

rate of, 358 
condensers, 385, 387 

water required, 363 
cone, surface, 370 
connecting rod, 367, 387 
convex lens, focal length, 365, 
366 

magnifying power, 366 
Cotter, 384 

crank, strength of, 384 
crank-pin, tangential pressure, 

355 
current from battery, 362 
cut-off, point of, 372 

real and apparent, 353 
cylinders, 354 

exposed surface, 372 

ratio of, 355 

ratio of expansion, 367 
deflection, 385 

difference of magnetic poten- 
tial, 376 
double-riveted joint, 367 
dynamo, air-gap, 365 

fall of potential in, 365 
earth's magnetic force, 386 
economic coefficient, series dy- 
namo, 379 

shunt motor, 379 
economizers, 388 



Digitized by 



Google 



INDEX 



419 



Formiila, effective pull of belt, 

373 
efficiency, Carnot's engine, 368 

compound motor, 53-54 

motor, 380 

shunt motor, 52 
elastic resilience of bars, 369 
ellipee, from trammel, 374 

perimeter, 370 
equation of trajectory, 387 
equilibrium of couples, 376 
expansion, ratio of, 353 
factor of, evaporation, 358 

safety, 354 
fire-box plate, 358 
flange coupling, 381 
force of a charged sphere, 386 
forces on crank-pin, 385 
fly-wheel, 363 

centrifugal force, 383 

tension in arms, 383 
friction of, pivot, 384 

worm and wheel, 384 
gas, coefficient of expansion, 

362 
gear teeth, 382 
girder, stay, 378 

two-span, 367 
head of boiler shell, 358 
heat balance equation, 389 
heat, electrical equivalent, 355 

of vaporization, 352 
helix, 370 

hoops, shrinkage, 369 
horsepower, indicated, 353, 390 

of shunt motor, 51 
I-beam, thickness of web, 367 
inclined plane, 360 
inductance of transmission 

lines, 372 
internal diameter, hollow col- 
umn, 368 
joint-resistance, 46 
journal, dimensions, 381 

length, 384 
keys, proportion of, 357 
lap-joints, double-riveted, 358 
lathe, compound-geared, 361 



Formula, lever, law of, 367 
Leyden jar, charge of, 375 
Leyden jars, difference of elec- 
trical energy, 376 
linkages, centers of valve circles, 

372 
locknuts, 356 
loss, armature, 52 

shunt field, 52 

stray, 52 

through heating, 380 
magnetic field, earth's, 362 
magnetic,- force, 376 

needle, deflection of, 375 
M.E.P., 373, 390 
moisture in steam, 56 
moment, bending, 356 

of inertia, 356 
moments of horizontal stresses, 

356 
mutual induction, 380 
pantograph, 353 
parallel electrified plates, 

375 
pipe, diameter, 359 

discharge of water, 371 

pressure on, 354 

thickness, 359 
piston, 367 

mean pressure on, 389 

net pressure on, 353 
piston-rod, diameter, 382 
pitch of screw, 359 
potential, 386 

difference, 363 

of needle, 375 
power, 378 
pressure, 386 

and clearance, 371 
projectile, path of, 387 

velocity, 388 
pulley, acceleration, 351 
punch and shear frames, stress 

in, 358 
radius of gyration of a ship, 

367 
Rankine-Gordon, for columns, 

368 



Digitized by 



Google 



420 



INDEX 



Formula, ratio of, absolute tem- 
peratures, 389 
ratio of expansion, 367 
refraction of lines of force, 376 
resistance, by Wheatstone 

bridge, 360 

copper wire, 50 

effect of temperature, 367 

joint, 46 

measurement of, 355, 364 

nickel wire, 50 

shunt winding, 52 
resultant, 75 
riveted joint, efficiency, 358 

resistance of, 383 

strength of, 374 
rope, power transmitted, 382 
safety valve, 358, 361 
screw, 361 

segment of a circle, area of, 360 
series motor, eflficiency, 53 

losses, 53 
shaft, diameter, 371 

outside diameter, 381 

stability of, 383 

twisting moment, 380 
shearing, 382 
Siemens' method, 385 
solenoid, magnetic field, 355 
space traversed, 360 
specific heat of mercury, 50 
speed cones, 356 
sphere electric capacity, 375 
spherical mirror, focal dis- 
tance, 365 

radius of curvature, 364-365 
spiral, length of, 371 
spring-ring, 384 
stayed surface, 382 

strength of, 359 
steam, cylinder, 357 

per cent of moisture, 382 

pressure of a mixture, 355 

pressure of saturated, 372 

quality of, 367 

specific volume of saturated, 
372 
, strain, 353-354 



Formula, stress, 353, 388 

due to impact, 369 

from longitudinal tension, 366 

in rectangular plate. 366 
tangent galvanometer, 379 
teeth, strength of, 385 
temperature, coefficient, 361 

and heat, 377 

and pressure, 371 
tension in a bolt, 388 
Thompson's method, 364 
variation of resistance, 378 
velocity of sound, 364 
volume, dry air, 59 

frustum of a pyramid, 370 

prismoid, 370 

wedge, 370 
wear of wheels, 366 
weight, spur-gear blank, 49 
weighted pendulum governor, 

height of, 374 
Wheatstone bridge, resistance 

by, 360 
wire, diameter of, 363 
work, 353 

done by, an electric field, 377 
an engine, 371 

in adiabatic expansion, 389 
Formulas, 20-25, 352-390 
gearing, 21-22 
laws of logarithms, 254-255 
mensuration, 25-33 
transformation of, 350-390 
Formulation, 25-37 
Formulation and computation, 

37-59 
mathematical laws, 25-59 
Foster, 281 
Four-place logarithms of numbers, 

402-404 
trigonometric functions, 405- 

411 
Fractional exponent, 185-188 
Fractions, 144-154 
addition and subtraction, 146- 

149 
complex, 151-154 
law of signs, 69 



Digitized by 



Google 



INDEX 



421 



Fractions, multiplication and divi- 
sion, 149-151 

reduction, 144-146 
Friction, 91 
Frustum volume, cone, 32 

pyramid, 32 
Functions, obtuse angle, 68-70 
Fundamental operations, 101-119 

G 

Galvanometer, shunted, current 

through, 36 
Gear, leadscrew, 39 

radius, 44 

spindle, 39 

weight formula, 49 
Gears, standard, 41 
Generation of oxygen, 83 
Globe, area on, 89 
Graphical resolution of forces, 

63-65 
Gravity, 94 
Greek alphabet, 413 
Gunter's scale, 279-281 

H 

Hollow, column, 46, 368 

shaft, safe transmission, 51 
Horizontal component, 63 
Horsepower, 34 

shunt motor, 51 

steam engine, 43-44 

steamer, 96 

transmitted by rope, 98 
Hydraulic press, 84 
Hydrogen, generation of, 82-83 

volume, 92 
Hydrometer, Baum6, 57 

Twaddell, 57 
Hyperbolic logarithm, 238-242 
Hypotenuse, 28 



I-beam, 49 
Inclined plane, 49 



Illumination, 92 
Inertia, moment of, 34 
Instruments, 3 
Intensity of heat, 88 
"Into,** meaning of, 35 
Introduction, 11-59 



"Jointly," definition, 90 
Joint resistance, 46 



K 

Keys, taper of, 44 
Kilowatt, 34 
Kinetic energy, 35 
Knot, 391 



Lathe, compound geared, 54-55 

simple geared. 39 
Law, actual weight, 45 
addition, 103 
angular velocity, 88 
area, cone, 31 

cylinder, 31 

ellipse, 30 

parallelogram, 28 

regular pyramid, 31 

ring, 30 

ring, cylindrical, 33 

sector, 30 

sphere, 32 

trapezium, 28 

trapezoid, 28 

triangle, 27, 28 
battery, multiple, 36 

multiple-series, 36 

series, 35 
bending of bar, 89 
Boyle's, 88-89 • 
centrifugal force, 34, 89 
characteristic, 224, 234 
Charles^ 86 



Digitized by 



Google 



A22 



INDEX 



Law, circle, area, 29 

area sector, 30 

ratio circumference to diam- 
eter, 29 
coal consumption, 100 
coefficient any term in binomial 

expansion, 194 
coefficient of friction, 35 
common center of gravity, 45 
components, 67 
compound geared lathe, 54 
cone, area, 31 

volume, 31 
Coulomb's, 98 
cube root, 205 

current, shunted galvanometer, 
/ 36 

xjutting speed, 41 
cylinder, area, 30 

volume, 31 
deflection of beam, 48, 96 
density of a gas, 89 
diagonal of cube, 99 
diameter, piston-rod, 50 

shaft, 43 
diffusion, 99 
division, 115-116 
dry volume, 59 
efficiency of shunt motor, 52 
electrical efficiency, 36 

transmission, 46 
elevation outside rail, 48 
ellipse, area, 30 

perimeter, 30, 370 
elongation of, spring, 93 

a wire, 95 
equilibrium on inclined plane, 49 
factor, 126 
falling bodies, 33 
flow from orifice, 98 
flow of gas, 87 
flux, 47 
force, 27 

fractional division, 151 
friction, 91 
frustum volume, 32 
generation of, hydrogen, 82-83 

oxygen, 83 



Law, gravity, 94, 390 
heating effect of current, 96 
high resistance, 36 
horsepower, 34, 43^4 

indicated, 390 

of steamer, 96 

transmitted by rope, 98 
hydraulic press, 84 
illumination, 92 
inclined plane, 99 
intensity of heat, 88 
joint resistance, 46 
kilowatt, 34 
kinetic energy, 35 
lathe, compound geared, 39, 54 

simple geared, 39 
linear velocity, 88 
Mass, 27 

moment of inertia, 34 
moments, 35 
momentum, 35 
multiple battery, 36 
multiplication, 113 
offing at sea, 93 
Ohm's, 35 

Osmotic pressure, 86 
penetration of armor, 85 
percentage composition, 57 
perimeter ellipse, 30, 370 
period of pendulum, 36, 89 
pitch of rivets, 82 
power, 37 

pressure of a gas, 93 
projectile, 100 
proportion, 38 
pulley, arm diametei , 43 

velocities, 47 

velocity, 42 
pyramid, regular, area, 31 

volume, 31 
radius, from chord and rise, 47 

gear wheel, 44 

sphere, 46 
range of jet, 98 
reduction of a negative logarithm 

to tabular form, 258 
regular polygon, angle of, 29 

area, 29 



Digitized by 



Google 



INDEX 



423 



Law, regular '"polygon, central 

angle, 29 
resistance of air, 97 
revolution of planet, 99 
ring, cylindrical, area, 33 

volume, 32 
rupture of beam, 97 
safe load on I-beam, 49 
safe transmission by hollow 

shafts, 51 
shearing stress, 81 
sigas for fractions, 69 
specific gravity, 27 
specific gravity by, Baum6, 58 

pyknomet^r, 59 

Twaddell, 57 
sphere, area, 32 

volume, 32 
square hypotenuse, 28 

root, 203-204 
stiffness of shafting, 96 
strength of current, 85 
subtraction, 106 
subtrahend, 106 
summary fundamental opera- 
tions, 118-119 
taper of keys, 44 
tension in rope, 50 
tensional strength of shafting, 92 
theoretical weight, 45 
Thompson ammeter, 94 
time, 42 

torque of magnetic needle, 91 
torsion, 95-96 
variation of current, 98 
velocity sound, 100 
vibration of, hght, 88 

strings. 87 
visual angle, 94 
volume cone, 31, 32 

cylinder, 31 

hollow column, 46 

hydrogen, 92 

pyramid, 31, 32 

rectangular solid, 32 

ring, 46 

sphere, 32, 46 

spherical segment, 33, 47 



Law, wedge, 51 
weight, 26 
beam, 90 
cylinder, 97 
gaseous steam, 56 
gun, 86 

hollow column, 46 
pulley, 42 
work, 34 
Laws of number, 120-127 
difference of two cubes, 122 
factor law, 126 
product of sum and difference, 

122 
square of any polynomial, 125 
square of difference, 121 
square of sum, 120-121 
sum of two cubes, 124 
Leadscrew gear, 39, 55 
I^ever, law of moments, 35 
Linear measure, 391 

velocity, 88 
Liquid measure, 393 
Ix)ad on I-beam, 49 
Log-S 236 
Logarithmic computation, 266- 

277 
Logarithms, 221-277 
antilogarithms or log-i, 235- 

238 
base in higher mathematics, 238 
base of common and Naperian 

systems, 238 
Briggs', 238 
common, 238 

conversion factor, 239-240 
definition of a logarithm, 222 
division, 245-246, 256 
exponential equations, 252-254 
formulas for operations, 254-255 
how to reduce common to 

Naperian, 239 
how to take readings, 266 
hyperhohc, 238-242 
logarithm of a decimal, 232-235 
mantissa, 235 
meaning of hyp. 238 
meaning of log- ^ , 236 



Digitized by 



Google 



424 



INDEX 



Logarithms, methods of determin- 
ing a, power, 257-261 
root, 261-266 
miscellaneous examples, 266- 

277 
model solutions, exponential 
equation, 252-254 
division, 245, 256 
miscellaneous, 274-277 
multiplication, 243-244, 255- 

256, 
powers, 247-248, 256-261, 

267 
roots, 250-251, 261-266 
modulus of common system, 

239-240 
multiplication, 242-244, 255- 

256 
Naperian, 238-242 
Natural, 238 

negative characteristic, 234-235 
negative logarithm, 258 
number greater than unity, 

223-231 
number less than unity, 232-234 
powers, 247-249, 256-261, 

267-269 
proportional partes, 229-231 
reduction of a negative log- 
arithm to tabular form, 258 
roots, 249-252, 261-266,269-271 
summary of laws, 254-255 
systems of, 238-240 
table- of proportional parts, 

229-231 
tabular difference, 228-229 
Losses, series motor, 53 

M 

Mannheim, 281 

Marks of parenthesis, 17 

Mass, 27 

Materials of construction, 399 

Meaning of, log-i, 236 

s,237 
Measures, area, 392 

length, 391 



Measures, volume, 393 

weight, 394 
Mendel^eff, formula for volume of 
dry air, 59 

periodic table, 398 
Mercury, specific heat, 50 
Metric equivalents, 396 

linear measure, 392 

square measure, 392 
Moisture in steam, 56 
Molecular weight, 57 
Moment of inertia, 34 
Moments, law of, 35 
Momentum, 35 
Motor, compound, 53-54 

series, 53 

shunt, 52 
Multiple battery, 36 
Multiplication, 113-115 
Multiplier, literal, 16 

numerical, 15 

N 

Napier, 279 
Nautical measure, 391 
Negative exponent, 182-185 
Nickel wire, resistance, 50 
Notation, 11-18 

O 

Oak beam, 48 

Obtuse angle functions of, 68-70 

Offing at sea, 93 

Ohm's law, 35 

Orthogonal projection, 61 

Oscillation of pendulum, 86-87 

Osmotic pressure, 86 

Oughtred, 279-281 

Oxygen, generation of, 83 



Parallelogram, area, 28 
Parenthesis, 17 

minus, 109-112 
Pendulum, 36, 390 ^ 

Percentage composition, 57 



Digitized by 



Google 



INDEX 



425 



Perimeter ellipse, 30, 370 
Periodic table, 398 
Piston-rod, diameter, 51 
Pitch of rivets, 82 

screw, 39 
Polygon, regular, angle of, 29 
central angle, 29 
area, 29 
Powers, 200-201 

and roots, 200-206 
Pressure and parts conversion, 400 
Pressure of a gas, 93 
Problems, formulation and com- 
putation, 37-59 

formulation, 25-37 

area parallelogram, 373 

mean effective pressure, 390 

thickness of a boiler tube, 373 

variation, 80-100 
Product, 14 

sum and difference, 122 
Projection, GO-62 
Proportion, 37 
Pulley, arm diameter, 43 

ratio velocities, 47 

velocity, 42 

weight of, 42 
Pyknometer, 59 
Pyramid, area, 31 

frustum, 32 

volume, 31 

Q 

Quadratics, 155-105 

solution by, factoring, 157-161 
completing the square, 161-165 

R 

Radicals, 207-220 

addition and subtraction, 

213-214 
equations, 219-220 
multipUcation and division, 

214-216 
powers and roots, 217-218 
rationalization, 218-219 
reduction, 208-213 



Rail, elevation of, 48 
Radius, gear, 44 

sphere, 46 

vector, 65 
Range of a jet, 98 
Ratio, 12-14 
Rationalization, 218-219 
Ratios of a triangle, 65-66 
Reading of angles, 64 
Record sheet, 6-7 
Rectangular solid, volume, 32 
Resistance, 36 

air, 97 

copper wire, 50 

joint, 46 

nickel wire, 50 

shunt winding, 52 
Resolution and composition, 60-76 

by computation, 65 

graphical, 63-65 
Resultant, 74-76 

formula, 75 
Revolution, planet, 99 
Ring, area, 30 

cylindrical, area, 33 

volume, 32 
Rise of chord, 47 
Roots, 202-206 
Rope tension, 50 

transmission, 98 
Ruhmkorff coil, 95 

S 

Screw-jack, 45-46 

Second's pendulum, 100, 370 

Sector, area, 30 

length of arc, 30 
Segment, sphere, 33 
Series battery, 35 

motor, losses, 53 
Shaft, diameter, 43 

tensional strength, 92 

stiffness, 90 
Shunt dynamo, resistance of 
winding, 52 

motor, efficiency, 52 
horsepower, 51 



Digitized by 



Google 



426 



INDEX 



Signs of functions of obtuse 

angle, 69 
Simple proportion, 38 
Simplest form, 351-352 
sin* d-f cos* e, 71 
Sine, 66 
Slide-rule, 279-349 

accumulated errors, 311 
change of indexes, 317-318 
combined multiplication and 

division, 322-324 
cube, 329-332 
cube root, 332-335 
cylinders, 337-341 
description, 281 

diagrammatic setting, 296-299, 
303-305, 308-310, 312-313, 
317-318, 322, 329, 338, 
340-341, 344-345, 347 
division, 312-316 
duplex, 283 

examples, 291-292, 294, 299, 
304-305, 307, 311, 315-316, 
321-324, 326-328, 332, 335, 
338, 341, 346, 349 
gage-points, 336-341 
C and Ci, 338-341 
M, 337-338 
X, 336 

second and minute, 292-293 
graduation, A and B scales, 
283-285 

C and D scales, 288-289 
5 scale, 289-290 
historical, 279-281 
hyperbolic logarithms, 346-349 
integral figures, 301-302 

sine or tangent, 306-307 
K scale, 330-331 
L scale, 324-325 
logarithms, 324-326 

to any base, 346 
log log duplex, 283 
rule, 342-349 

hyperbolic logarithms, 

346-349 
logarithms to any base, 
346 



Slide-rule, log log rule, Naperian 
logarithms, 346-349 
powers, 345-346 
roots, 343-346 
Mannheim, 281 
multiplex, 283 
multiplication, 300-311 

sines and tangents, 305-307 
" off the rule," 317-319 
pointing off, cube, 331-332 

cube root, 333 

division, 314 

Naperian log, 346-^7 

product, 301-302 

seconds and minutes gage- 
point readings, 293 

sine, 290 

square root, 328 

tangent, 296 
powers and roots, 327-335 
principle of multiplication, 300- 

301 
product more than two factors, 

307-311 
proportion, 317-322 
quotient, 312-314 
ratio method for proportion, 

319-320 
scales, 281-283 

used in multiplication, 300 
sines, 289-294 

with reversed slide, 292 

with unreversed slide, 290-291 
square, 327-328 
square root, 328 
tangent, reading, 294-299 

of obtuse angle, 297-299 
tangents, 294-299 
triplex, 283 
Solution of equations, quadratic, 

155-165 
simple, 19-25 
simultaneous, 173-179 
trigonometric, 70-73 
Specific gravities, 399 
Specific gravity, 27 
by Baimi<^, 58 
bottle, 5Q 



Digitized by 



Google 



INDEX 



427 



Specific gravity, by pyknometer, 
59 

by Twaddell, 67 
Specific heat of mercury, 50 
Sphere area, 32 

radius, 46 

segment, volume, 33, 47 

volume, 32, 46 
Spindle gear, 39, 55 
Square, ditterence two numbers, 
121 

measure, 392 

root, polynomial, 203-204 
Standard gears, 41 
Steam cylinder, 43 

engine, horse-power, 34 

per cent of moisture, 56 

weight of gaseous, 56 
Subtraction, 105-112 
Surveyor's long measure, 391 

square measure, 392 



Table, periodic, 
avoirdupois weight, 394 
cubic measure, 393 
decimal equivalents, 395 
dry measure, 393 
mternational atomic weights, 

397 
linear measure, 391 
liquid measure, 393 
logarithms ot numbers, 402 
metric linear measure, 392 
metric square measure, 392 
nautical measure, 391 
Prerisure and parts conversion, 

400 
specific gravities and weights, 

399 
square measure, 392 
surveyor's long measure, 391 
surveyor's square measure, 392 
trigonometric functions, 405-411 
troy weight, 394 
volume and weight of water, 
394 



Table, wire-gage, 401 
Tangent, 66 

galvanometer, 85 
Taper of keys, 44 
Tension in a rope, 50 
Theoretical weight, 45 
Thompson ammeter, 94 
Time lomiula, 42 
Torque of magnetic needle, 91 
Torsion, 95-96 
Trammel, 374 
Transformation, definition, 350 

methods, 350-351 

of formulas, 350-390 
Trapezium, area, 28 
Trapezoid, 28 
Triangle, area, 27, 28 

functions of, 66 
Trigonometric equations, 70-73 
Troy weight, 394 
Twaddell hydrometer, 57 



Variation, 77-100 
Velocity, falling body, 33 

projectile, 100 

pulley, 42 

pulleys, 47 

sound, 100 
Vertical component, 63 
Vibration of, light, 88 

strings, 87 
Visual angle, 94 
Volts, 35 
Volume, cone, 31 

cone frustum, 32 

cylinder, 31 

dry air, 59 

frustum cone, 32 

frustum pyramid, 32 

gas, 82 

hollow column, 46 

hydrogen, 92 

pjrramid, 31 
frustum, 32 

rectangular solid, 32 

ring, 32 



Digitized by 



Google 



428 



INDEX 



Volume, segment of sphere, 33, 47 
sphere, 32, 46 
water, 394 
wedge, 51 

W 

Wedge volume, 51 
Weight, 26 

actual, 45 

atomic, 397 

avoirdupois, 394 

beam, 90 

concrete pier, 91 

cylinder, 97 

gaseous steam, 56 

gun, 86 

hollow column, 46 

materiab of construction, 390 

molecular, 57 

pulley, 42 



Weight, spur-gear blank, 49 

theoretical, 45 

troy, 394 

water, 394 
Wind pressure, 90 
Wire-gage table, 401 
Work, 34 
Work-book, 3-9 

collection, 7 

correction, 7 

distribution, 7 

inspection, 7 

instructions for, entries, 4r6 
record sheet, 6-7 

indication of results, 8 

index, 8-9 
Wrought-iron beam, 48 



Zero exponent, 181-182 



FEB 



Digitized by 



Google 



Digitized by 



Google 



Digitized by 



Google ' 



Digitized by 



Google 



«Si^%* 



UNIVERSITY OF MICHIfiAN 



3 9015 00292 3418 "^ Google 









'^vm 



'' J' M 



iiiliiliiiiifliiii 



iii 



.:i.!i 



;'!i:^l| 



'''i:MM